You are on page 1of 447

Williams

Obst et r ic s
st u d y Gu id e
NOTICE
Medicine is an ever-changing science. As new research and clinical experience broaden our knowl-
edge, changes in treatment and drug therapy are required. The authors and the publisher of this
work have checked with sources believed to be reliable in their efforts to provide information that is
complete and generally in accord with the standards accepted at the time of publication. However,
in view of the possibility of human error or changes in medical sciences, neither the authors nor
the publisher nor any other party who has been involved in the preparation or publication of this
work warrants that the information contained herein is in every respect accurate or complete, and
they disclaim all responsibility for any errors or omissions or for the results obtained from use of the
information contained in this work. Readers are encouraged to confirm the information contained
herein with other sources. For example and in particular, readers are advised to check the product
information sheet included in the package of each drug they plan to administer to be certain that the
information contained in this work is accurate and that changes have not been made in the recom-
mended dose or in the contraindications for administration. This recommendation is of particular
importance in connection with new or infrequently used drugs.
Williams
Obst et r ic s
st u d y Gu id e
24 h e on

Robyn Horsager, MD
Holder, Luis Leib, MD Professorship in Obstetrics and Gynecology
Chief, Obstetrics and Gynecology
University Hospital St. Paul
Professor
Department of Obstetrics and Gynecology
University of Texas Southwestern Medical Center at Dallas

Scott W. Roberts, MD
Medical Director, High-Risk Obstetrical Unit
Parkland Hospital
Professor
Department of Obstetrics and Gynecology
University of Texas Southwestern Medical Center at Dallas

Vanessa L. Rogers, MD
Director, Obstetrics and Gynecology Residency Program
Associate Professor
Department of Obstetrics and Gynecology
University of Texas Southwestern Medical Center at Dallas

Patricia C. Santiago-Muñoz, MD
Associate Professor
Department of Obstetrics and Gynecology
University of Texas Southwestern Medical Center at Dallas

Kevin C. Worley, MD
Associate Director, Obstetrics and Gynecology Residency Program
Assistant Professor
Department of Obstetrics and Gynecology
University of Texas Southwestern Medical Center at Dallas

Barbara L. Hoffman, MD
Associate Professor
Department of Obstetrics and Gynecology
University of Texas Southwestern Medical Center at Dallas

New York Athens Chicago San Francisco London Madrid Mexico City
Milan New Delhi Singapore Sydney Toronto
Copyright © 2015 by McGraw-Hill Education. All rights reserved. Except as permitted under the United States Copyright Act of 1976, no part of this publication
may be reproduced or distributed in any form or by any means, or stored in a database or retrieval system, without the prior written permission of the publisher.

ISBN: 978-0-07-179328-5

MHID: 0-07-179328-3

The material in this eBook also appears in the print version of this title: ISBN: 978-0-07-179327-8,
MHID: 0-07-179327-5.

eBook conversion by codeMantra


Version 1.0

All trademarks are trademarks of their respective owners. Rather than put a trademark symbol after every occurrence of a trademarked name, we use names in an
editorial fashion only, and to the bene t of the trademark owner, with no intention of infringement of the trademark. Where such designations appear in this book,
they have been printed with initial caps.

McGraw-Hill Education eBooks are available at special quantity discounts to use as premiums and sales promotions or for use in corporate training programs. To
contact a representative, please visit the Contact Us page at www.mhprofessional.com.

TERMS OF USE

This is a copyrighted work and McGraw-Hill Education and its licensors reserve all rights in and to the work. Use of this work is subject to these terms. Except as
permitted under the Copyright Act of 1976 and the right to store and retrieve one copy of the work, you may not decompile, disassemble, reverse engineer, reproduce,
modify, create derivative works based upon, transmit, distribute, disseminate, sell, publish or sublicense the work or any part of it without McGraw-Hill Education’s
prior consent. You may use the work for your own noncommercial and personal use; any other use of the work is strictly prohibited. Your right to use the work may
be terminated if you fail to comply with these terms.

THE WORK IS PROVIDED “AS IS.” McGRAW-HILL EDUCATION AND ITS LICENSORS MAKE NO GUARANTEES OR WARRANTIES AS TO THE
ACCURACY, ADEQUACY OR COMPLETENESS OF OR RESULTS TO BE OBTAINED FROM USING THE WORK, INCLUDING ANY INFORMATION
THAT CAN BE ACCESSED THROUGH THE WORK VIA HYPERLINK OR OTHERWISE, AND EXPRESSLY DISCLAIM ANY WARRANTY, EXPRESS
OR IMPLIED, INCLUDING BUT NOT LIMITED TO IMPLIED WARRANTIES OF MERCHANTABILITY OR FITNESS FOR A PARTICULAR PURPOSE.
McGraw-Hill Education and its licensors do not warrant or guarantee that the functions contained in the work will meet your requirements or that its operation
will be uninterrupted or error free. Neither McGraw-Hill Education nor its licensors shall be liable to you or anyone else for any inaccuracy, error or omission,
regardless of cause, in the work or for any damages resulting therefrom. McGraw-Hill Education has no responsibility for the content of any information accessed
through the work. Under no circumstances shall McGraw-Hill Education and/or its licensors be liable for any indirect, incidental, special, punitive, consequential or
similar damages that result from the use of or inability to use the work, even if any of them has been advised of the possibility of such damages. This limitation of
liability shall apply to any claim or cause whatsoever whether such claim or cause arises in contract, tort or otherwise.
DEDICATION
During our training and careers, we have had the privilege to learn from many of the great physicians in the fields of obstetrics
and reproductive medicine. These giants taught us the importance of basing clinical care on good scientific evidence—decades before
the phrases “evidence-based medicine” and “best practices” became part of the medical lexicon. They inspired us to be logical,
meticulous, curious, and courageous. With great admiration and appreciation, we dedicate this edition of our Williams Obstetrics
Study Guide to our heroes: Jack Pritchard, Paul McDonald, Peggy Whalley, Norman Gant, Ken Leveno, and Gary Cunningham.

Robyn Horsager
Scott W. Roberts
Vanessa L. Rogers
Patricia C. Santiago-Muñoz
Kevin C. Worley
Barbara L. Hoffman

v
This page intentionally left blank
CONTENTS

Prefa ce ........................................................................................................................... xi

Sec t io n 1
OVERVIEW

1 . O verview of O bstetrics.......................... 2

Sec t io n 2
MATERNALANATOMY AND PHYSIOLOGY

2 . Ma terna l Ana tomy .............................. 8 4 . Ma terna l Physiology ........................... 2 0


3 . Congenita l G enitourina ry
Abnorma lities .................................... 1 4

Sec t io n 3
PLACENTATION, EMBRYOGENESIS,
AND FETALDEVELOPMENT

5 . Impla nta tion a nd Pla centa l 7 . Embryogenesis a nd Feta l Morphologica l


Development ..................................... 3 0 Development ..................................... 4 2
6 . Pla centa l Abnorma lities ....................... 3 6

v
v Contents

Sec t io n 4
PRECONCEPTIONALAND PRENATALCARE

8 . Preconceptiona l Counseling ................. 5 0 9 . Prena ta l Ca re .................................... 5 6

Sec t io n 5
THE FETALPATIENT

1 0 . Feta l Ima ging .................................... 6 2 1 4 . Prena ta l Dia gnosis.............................. 8 6


1 1 . Amnionic Fluid ................................... 6 8 1 5 . Feta l Disorders ................................... 9 2
1 2 . Tera tology, Tera togens, a nd Fetotoxic 1 6 . Feta l Thera py .................................... 9 8
Agents ............................................. 7 5 1 7 . Feta l Assessment .............................. 1 0 4
1 3 . Genetics........................................... 8 1

Sec t io n 6
EARLY PREGNANCY COMPLICATIONS

1 8 . Abortion ......................................... 1 1 2 2 0 . Gesta tiona l Trophobla stic Disea se ....... 1 2 8


1 9 . Ectopic Pregna ncy............................ 1 1 8

Sec t io n 7
LABOR

2 1 . Physiology of La bor .......................... 1 3 8 2 5 . O bstetrica l Ana lgesia a nd


Anesthesia ..................................... 1 6 6
2 2 . N orma l La bor ................................. 1 4 4
2 6 . Induction a nd Augmenta tion of
2 3 . Abnorma l La bor .............................. 1 5 0
La bor ............................................ 1 7 3
2 4 . Intra pa rtum Assessment ..................... 1 5 9
Contents x

Sec t io n 8
DELIVERY

2 7 . Va gina l Delivery .............................. 1 8 0 3 0 . Cesa rea n Delivery a nd Peripa rtum


Hysterectomy .................................. 2 0 2
2 8 . Breech Delivery .............................. 1 8 8
3 1 . Prior Cesa rea n Delivery .................... 2 1 0
2 9 . O pera tive Va gina l Delivery ............... 1 9 4

Sec t io n 9
THE NEWBORN

3 2 . The N ewborn ................................. 2 1 8 3 4 . The Preterm N ewborn ...................... 2 2 9


3 3 . Disea ses a nd Injuries of the 3 5 . Stillbirth ......................................... 2 3 4
Term N ewborn ............................... 2 2 3

Sec t io n 10
THE PUERPERIUM

3 6 . The Puerperium ............................... 2 4 0 3 8 . Contra ception ................................ 2 5 3


3 7 . Puerpera l Complica tions ................... 2 4 6 3 9 . Steriliza tion .................................... 2 6 0

Sec t io n 11
OBSTETRICALCOMPLICATIONS

4 0 . Hypertensive Disorders ..................... 2 6 8 4 3 . Postterm Pregna ncy .......................... 2 8 7


4 1 . O bstetrica l Hemorrha ge ................... 2 7 4 4 4 . Feta l-G rowth Disorders ..................... 2 9 4
4 2 . Preterm La bor ................................. 2 8 1 4 5 . Multifeta l Pregna ncy ........................ 3 0 0
x Contents

Sec t io n 12
MEDICALAND SURGICALCOMPLICATIONS

4 6 . Genera l Considera tions a nd Ma terna l 5 6 . Hema tologica l Disorders .................. 3 6 6


Eva lua tion ...................................... 3 0 8
5 7 . Dia betes Mellitus ............................. 3 7 1
4 7 . Critica l Ca re a nd Tra uma ................. 3 1 4
5 8 . Endocrine Disorders ......................... 3 7 8
4 8 . O besity ......................................... 3 2 0
5 9 . Connective-Tissue Disorders .............. 3 8 3
4 9 . Ca rdiova scula r Disorders .................. 3 2 5
6 0 . N eurologica l Disorders .................... 3 8 9
5 0 . Chronic Hypertension ...................... 3 3 1
6 1 . Psychia tric Disorders ........................ 3 9 6
5 1 . Pulmona ry Disorders ........................ 3 3 7
6 2 . Derma tologica l Disorders .................. 4 0 0
5 2 . Thromboembolic Disorders ................ 3 4 4
6 3 . N eopla stic Disorders ....................... 4 0 6
5 3 . Rena l a nd Urina ry Tra ct Disorders ...... 3 5 0
6 4 . Infectious Disea ses .......................... 4 1 3
5 4 . Ga strointestina l Disorders .................. 3 5 6
6 5 . Sexua lly Tra nsmitted Infections ........... 4 2 0
5 5 . Hepa tic, Bilia ry, a nd
Pa ncrea tic Disorders ........................ 3 6 1

Index ......................................................................................................................... 4 2 7
Pr ef a c e

T e Williams Obstetrics 24th Edition Study Guide is designed to to the section of text that contains the answer. We hope that our
assess comprehension and retention of information presented clinical approach to this guide translates into a more accurate test
in Williams Obstetrics, 24th edition. T e questions for each sec- of important clinical knowledge.
tion have been selected to emphasize the key points from each
chapter. In total, nearly 2100 questions have been created from Robyn Horsager
the 65 chapters. Questions are in a multiple-choice format, and Scott W. Roberts
one single best answer should be chosen for each. With this edi- Vanessa L. Rogers
tion, we have also included more than 250 full-color images as Patricia C. Santiago-Muñoz
question material. In addition, clinical case questions have been Kevin C. Worley
added to test implementation of content learned. At the end of Barbara L. Ho man
each chapter, answers are found, and a page guide directs readers

xi
This page intentionally left blank
Se c t i o n 1

Ov er v iew
2

CHAPTer 1

O of Obst t cs

1–1. The field of obstetrics encompasses all EXCEPT 1–7. Which of the following is defined as the sum of
which of the following? stillbirths and neonatal deaths per 1000 total births?
a. Prenatal care a. Fetal death rate
b. Management of labor b. Neonatal mortality rate
c. Infertility treatments c. Perinatal mortality rate
d. Immediate newborn care d. None of the above

1–2. Registration of live births is currently assigned to 1–8. A patient presents at 22 weeks’ gestation with
which national agency? spontaneous rupture of membranes and delivers
a. Bureau of the Census a 489-g male infant who dies at 4 hours of life.
Her last menstrual period and early sonographic
b. National Institutes of Health
evaluation confirm her gestational dating. All
c. National Center for Health Statistics EXCEPT which of the following definitions
d. Department of Health and Human Services accurately apply to this delivery?
a. Abortus
1–3. How does the National Vital Statistics System, using
b. Preterm neonate
data from the National Center for Health Statistics,
define fetal death for its reports? c. Early neonatal death
a. Fetal weight > 350 g d. Extremely low birthweight
b. Fetal weight > 500 g
1–9. A death of a newborn at 5 days of life due to
c. Gestational age > 20 weeks congenital heart disease would be counted in which
d. Gestational age > 24 weeks of the following rates?
a. Infant mortality rate
1–4. The perinatal period starts after delivery at 20 weeks’
b. Perinatal mortality rate
gestation or older. When does it end?
c. Early neonatal death rate
a. 7 days after birth
d. All of the above
b. 1 year after birth
c. 28 days after birth 1–10. The fertility rate is the number of live births per
d. 1 calendar month after birth 1000 females of what age?
a. 9–39 years
1–5. Which of the following is synonymous with fetal
b. 11–55 years
death rate?
c. 15–44 years
a. Stillbirth rate
d. 18–49 years
b. Perinatal death rate
c. Spontaneous abortion rate 1–11. Delivery at what age divides preterm from term
d. Early neonatal death rate gestations?
a. 34 weeks
1–6. At the state level, which of the following is used to
b. 36 weeks
define fetal death?
c. 37 weeks
a. Fetal death > 20 weeks’ gestation
d. 38 weeks
b. Fetal death with a birthweight of ≥ 500 g
c. Any fetal death regardless of gestational age
d. Each has been used
O verview of O bstetrics 3

1–12. Which of the following is an example of an indirect 1–15. The death of the patient in Question 1–14 should
maternal death? also be classified as which of the following?
a. Septic shock following an abortion a. Maternal death

C
b. Aspiration following an eclamptic seizure b. Pregnancy-related death

H
A
c. Hemorrhage following a ruptured ectopic c. Pregnancy-associated death

P
T
pregnancy d. All of the above

e
r
d. Aortic rupture at 36 weeks’ gestation in a patient

1
with Marfan syndrome 1–16. Which of the following definitions most specifically
applies to the neonate in the previous clinical
1–13. A patient with no prenatal care presents to Labor scenario?
and Delivery with abdominal pain. Her fundal a. Low birthweight
height is 21 cm. She spontaneous delivers a 475-g
b. Growth restricted
female fetus with no heart rate. According to the
Centers for Disease Control, which of the following c. Very low birthweight
terminology correctly describes the death? d. Extremely low birthweight
a. Abortus
1–17. A 30-year-old multigravida presents with ruptured
b. Fetal death
membranes at term but without labor. Following
c. Neonatal death induction with misoprostol, her labor progresses
d. None of the above rapidly, and she spontaneously delivers a liveborn
3300-g neonate. Immediately after delivery,
1–14. A patient presents with severe preeclampsia at she complains of dyspnea. She becomes apneic
25 weeks’ gestation. Labor is induced and she and pulseless and is unable to be resuscitated.
spontaneously delivers a 692-g neonate. In the Photomicrographs from her autopsy reveal fetal
recovery room she complains of a severe headache squames (arrows) within the pulmonary vasculature.
and suddenly collapses. She is unable to be How would her death be classified?
resuscitated. An autopsy reveals the following
finding. How would her death be classified?

a. Perinatal death
b. Nonmaternal death
Reproduced with permission from Cunningham FG, Leveno KJ, Bloom SL, et al (eds):
c. Direct maternal death
Hypertensive disorders. In Williams Obstetrics, 24th ed. New York, McGraw-Hill, 2014, d. Indirect maternal death
Figure 40-11.

a. Perinatal death
b. Nonmaternal death
c. Direct maternal death
d. Indirect maternal death
4 O verview

1–18. A 24-year-old primigravida with no prior prenatal 1–21. What percentage of all pregnancies in the United
care presents with active preterm labor at 33 weeks’ States end in a live birth?
gestation. Following admission to Labor and Delivery, a. 30%
she complains of dyspnea, suddenly collapses, and
S
b. 50%
e
is unable to be resuscitated. Her fetus dies during
C
c. 65%
T
attempted maternal resuscitation. Autopsy of the
i
O
mother reveals marked right ventricular hypertrophy, d. 90%
N
and her peripheral pulmonary arteries microscopically
1
show marked hypertrophy of the tunica media. How 1–22. Which of the following is an accurate reflection of
would her death be classified? RV = right ventricle; fetal death rates between 20 and 28 weeks’ gestation?
LV = left ventricle. a. They have fallen significantly since 1990.
b. They have remained relatively stable since 1990.
c. The fetal mortality rate at 20–27 weeks’ gestation
approximates that at > 28 weeks’ gestation.
d. None of the above

1–23. Which of the following is the largest contributor to


the perinatal mortality rate?
a. Fetal deaths
b. Neonatal deaths
c. Spontaneous abortions < 16 weeks’ gestation
d. None of the above

1–24. Which of the following obstetrical complications


contributes the least to the pregnancy-related death
Used with permission from Dr. David Nelson. rate in the United States?
a. Hemorrhage
a. Perinatal death b. Thromboembolism
b. Nonmaternal death c. Ectopic pregnancy
c. Direct maternal death d. Anesthetic complications
d. Indirect maternal death
1–25. What is the most recent estimate of maternal
1–19. Which of the following is an accurate statement mortality in the United States?
regarding the birth rate in the United States? a. 4/10,000 live births
a. It is at an all-time low. b. 14/10,000 live births
b. The teenage birth rate has slowly increased in the c. 14/100,000 live births
past 20 years.
d. 41/100,000 live births
c. The greatest decrease in birth rate has been seen
in women older than 30 years. 1–26. All EXCEPT which of the following is an example
d. While the birth rate has fallen in some racial and of a “near miss?”
ethnic groups, it has increased in other groups. a. A postpartum patient who falls in shower without
injury
1–20. Which of the following makes the largest
contribution to infant death in the United States? b. A delay in sending the human immunodeficiency
virus (HIV) screening test of a laboring patient
a. Home births who ultimately has a negative test result
b. Multifetal gestations c. Failure to give Rh immunoglobulin to an
c. Congenital fetal anomalies Rh-negative postpartum patient who ultimately
d. Preterm birth < 32 weeks’ gestation has no change in antibody screen
d. High spinal anesthesia resulting in intubation,
admission to the intensive-care unit, and a
ventilator-associated pneumonia
O verview of O bstetrics 5

1–27. Which of the following is an accurate statement 1–30. For which of the following purposes would fetal
regarding current health care for women in the chromosomal microarray analysis be potentially
United States? beneficial?

C
a. Uninsured women with breast cancer have a 50% a. Evaluating a stillborn fetus

H
higher mortality rate than insured women. b. Screening the fetus of an advanced-age mother

A
P
b. The United States is ranked in the top 10 c. Evaluating the fetus with trisomy 21 and a

T
e
countries with the lowest neonatal mortality rates. double-outlet right ventricle

r
c. The Affordable Care Act mandates expanded

1
d. Screening the fetus at 12 weeks’ gestation whose
Medicaid coverage for poor women, improving mother personally carries a balanced translocation
availability to prenatal services.
d. The availability of Medicaid coverage for prenatal 1–31. Which of the following are reported physician
care has eliminated disparities in perinatal responses to the current liability environment in the
outcomes between insured and uninsured United States?
women. a. Higher cesarean delivery rates
b. Reduction in number of obstetric patients
1–28. How are programs supported by Title V Maternal
accepted for care
and Child Health Services Block Grants funded?
c. Refusal to care for women whose pregnancies are
a. States match federally provided funds.
considered high-risk
b. States generate revenue through property taxes.
d. All of the above
c. Private contributions support individual state
initiatives. 1–32. Which of the following is accurate regarding home
d. A percentage of Social Security revenue is births in the United States?
apportioned for Title V and Title X. a. Certified nurse midwives attend most home
births.
1–29. All EXCEPT which of the following contribute to b. They have a higher associated perinatal mortality
the increasing cesarean delivery rate? rate than births occurring in medical facilities.
28 c. Randomized trials suggest their outcomes are
27
26 equivalent to those of births occurring in medical
25 S ponta ne ous va gina l de live ry facilities.
24
23 d. None of the above
)
22
n
e
21
m
20
o
w
19 Epis iotomy
0
18
0
17
0
1
16
r
e
15
(p
14
s
13
e
t
12
ra
11
d
e
10 Ce s a re a n de live ry
t
s
9
u
j
d
8
a
7
-
e
6
g
A
5
4
3 Ope ra tive va gina l de live ry
2
1
0
1979 1993 2006
Time (in ye a rs )

Reproduced with permission from Cunningham FG, Leveno KJ, Bloom SL, et al (eds):
Overview of obstetrics. In Williams Obstetrics, 24th ed. New York, McGraw-Hill, 2014,
Figure 1-8.

a. Increasing rates of labor dystocia


b. Increasing rates of labor induction
c. Increasing rates of breech presentation
d. Decreasing rates of vaginal birth after cesarean
section
6 O verview

CHAPTer 1 ANSw er Key

Q uestion Letter Pa ge
S
number a nswer cited Hea der cited
e
C
T
1–1 c p. 2 Introduction
i
O
1–2 c p. 2 Vita l Sta tistics
N
1–3 c p. 3 Definitions
1
1–4 c p. 3 Definitions
1–5 a p. 3 Definitions
1–6 d p. 3 Definitions
1–7 c p. 3 Definitions
1–8 a p. 3 Definitions
1–9 d p. 3 Definitions
1–10 c p. 3 Definitions
1–11 c p. 3 Definitions
1–12 d p. 3 Definitions
1–13 a p. 3 Definitions
1–14 c p. 3 Definitions
1–15 d p. 3 Definitions
1–16 d p. 3 Definitions
1–17 c p. 3 Definitions
1–18 d p. 3 Definitions
1–19 a p. 4 Pregna ncy Ra tes
1–20 d p. 4 Infa nt Dea ths
1–21 c p. 4 Ta ble 1 -2
1–22 b p. 5 Figure 1 -3
1–23 a p. 4 Perina ta l Morta lity
1–24 d p. 6 Ta ble 1 -3
1–25 c p. 5 Ma terna l Morta lity
1–26 d p. 6 Severe Ma terna l Morbidity
1–27 a p. 7 Hea lth Ca re for W omen a nd Their Infa nts
1–28 a p. 7 Hea lth Ca re for W omen a nd Their Infa nts
1–29 c p. 9 Rising Cesa rea n Delivery Ra te
1–30 a p. 9 G enomic Technology
1–31 d p. 1 0 Medica l Lia bility
1–32 b p. 1 1 Home Births
Se c t i o n 2

MATERNALANATOMY AND
PHYSIOLOGY
8

CHAPTER 2

Maternal Anatomy

2–1. The emoral artery gives rise to all EXCEPT which 2–3. Which statement accurately describes the location o
o the ollowing vessels? the in erior epigastric artery above the arcuate line?
a. External pudendal artery a. Ventral to the anterior rectus sheath
b. In erior epigastric artery b. Dorsal to the posterior rectus sheath
c. Super icial epigastric artery c. Ventral to the external oblique muscle
d. Super icial circum lex iliac artery aponeurosis
d. Ventral to the transversus abdominis muscle
2–2. The in erior epigastric artery arises rom which o aponeurosis
the ollowing?
2–4. The anterior abdominal wall is innervated by all
EXCEPT which o the ollowing?
a. Subcostal nerve
b. Internal pudendal nerve
c. Intercostal nerves (T 7 –T 11)
d. Iliohypogastric nerve (L1)

2–5. The labia minora lack all EXCEPT which o the


ollowing?
a. Eccrine glands
b. Hair ollicles
c. Apocrine glands
d. Sebaceous glands

2–6. The internal pudendal artery supplies which o the


ollowing?
a. Bladder trigone
b. Proximal vagina
c. Distal vaginal walls
d. Posterior vaginal wall

Reproduced with permission rom Corton MM: Anatomy. In Hof man BL, Schorge JO,
Schaf er JI, et al (eds): Williams Gynecology, 2nd ed. New York, McGraw-Hill, 2012,
Figure 38-3.

a. Aorta
b. Femoral artery
c. Hypogastric artery
d. External iliac artery
Ma terna l Ana tomy 9

2–7. The vagina and its investing musculature are supplied 2–11. The cervix contains little o which o the ollow
by all EXCEPT which o the ollowing arteries? components?
a. Elastin

C
b. Collagen

H
A
c. Smooth muscle

P
T
d. Proteoglycans

E
R
2
2–12. Concerning the endometrium, which o the
ollowing is true?

Epithe lium
Ca pilla rie s
Ve nous s inus
Endome tria l gla nd

Reproduced with permission rom Cunningham FG, Leveno KJ, Bloom SL, et al (eds): Ra dia l a rte ry
Maternal anatomy. In Williams Obstetrics, 23rd ed. New York, McGraw-Hill, 2010,
Figure 2-5. Arcua te a rte ry
Ute rine a rte ry
a. Perineal artery
b. Pudendal artery Reproduced with permission rom Hof man BL: Abnormal uterine bleeding. In Hof man
c. In erior rectal artery BL, Schorge JO, Schaf er JI, et al (eds): Williams Gynecology, 2nd ed. New York,
McGraw-Hill, 2012, Figure 8-3.
d. Posterior labial artery
a. The basal artery comes directly rom the arcuate
2–8. Which o the ollowing is true concerning the anal artery.
sphincters?
b. Spiral arteries extend directly rom radial arteries.
a. The external anal sphincter measures 3 to 4 cm in
length. c. The spiral arteries extend directly rom the
arcuate artery.
b. The external sphincter remains in a state o
constant relaxation. d. Functionalis layer contains spiral arteries and
radial arteries.
c. The internal anal sphincter contributes the bulk
o anal canal resting pressure.
2–13. During postpartum tubal sterilization, which o the
d. The external anal sphincter receives blood supply ollowing correct anatomical in ormation may assist
rom the superior rectal artery. you?
a. The round ligament lies anterior to the allopian
2–9. The perineal body is ormed partly by which o the tube.
ollowing muscles?
b. The allopian tube lies anterior to the round
a. Levator ani muscle ligament.
b. Gluteus maximus muscle c. The uteroovarian ligament lies anterior to the
c. Bulbocavernosus muscle round ligament.
d. Ischiocavernosus muscle d. The allopian tube lies posterior to the
uteroovarian ligament.
2–10. The vestibule is an almond shaped area bound by
which o the ollowing?
a. Laterally by the Hart line
b. Anteriorly by the ourchette
c. Laterally by the labia minora
d. Laterally by the external sur ace o hymen
10 Ma terna l Ana tomy a nd Physiology

2–14. Which o the ollowing arteries is marked by the arrow? 2–15. Which o the ollowing is true regarding the external
anal sphincter?
a. Is bound anteriorly by the perineal body
S
b. Is bound anteriorly by the posterior vagina
E
C
c. Contains involuntarily innervated smooth muscle
T
I
O
d. Is supplied by the superior and middle rectal
N
arteries
2
2–16. Re erring to the drawing, which o the ollowing is
marked by the letter A?
a Ureter
b. Uterine artery
c. Uteroovarian ligament
d. In undibulopelvic ligament

Reproduced with permission rom Cunningham FG, Leveno KJ, Bloom SL, et al (eds): Mater-
nal anatomy. In Williams Obstetrics, 23rd ed. New York, McGraw-Hill, 2010, Figure 2-15.

a. Sampson artery
b. Uterine artery
c. Obturator artery
d. Internal iliac artery

Reproduced with permission rom Corton MM: Anatomy. In Hof man BL, Schorge JO, Schaf er JI, et al (eds): Williams Gynecology, 2nd ed. New York, McGraw-
Hill, 2012, Figure 38-14.
Ma terna l Ana tomy 11

2–17. Re erring to the drawing in Question 2–16, which 2–21. The uterine artery is a main branch o which o the
o the ollowing is marked by the letter B? ollowing vessels?
a. Ureter a. Iliolumbar artery

C
b. Uterine artery b. Common iliac artery

H
A
c. Uteroovarian ligament c. External iliac artery

P
T
d. In undibulopelvic ligament d. Internal iliac artery

E
R
2
2–18. Re erring to the drawing in Question 2–16, which 2–22. From proximal (uterus) to distal ( imbriae), the
o the ollowing is marked by the letter C? correct progression o allopian tube anatomy is
a. Ureter which o the ollowing?
b. Uterine artery
c. Uteroovarian ligament
d. In undibulopelvic ligament

2–19. Re erring to the drawing in Question 2–16, which


o the ollowing is marked by the letter D?
a. Ureter
b. Uterine artery
c. Uteroovarian ligament
d. In undibulopelvic ligament

2–20. The common iliac artery arises directly rom which


o the ollowing?

Reproduced with permission rom Hnat MD: Parkland tubal ligation at the time o cesar-
ean section (update). In Cunningham FG, Leveno KL, Bloom SL, et al (eds): Williams
Obstetrics, 22nd ed. Online. New York, McGraw-Hill, 2006, http://www.accessmedicine.
com. Figure 1.

a. Isthmus, in undibulum, ampulla


Common ilia c b. Isthmus, ampulla, in undibulum
c. In undibulum, ampulla, isthmus
d. Ampulla, in undibulum, isthmus

2–23. The pelvis is ormed by which o the ollowing bone(s)?


a. Sacrum
b. Coccyx
Ute rine a rte ry c. Innominate
Inte rna l d. All o the above
pude nda l a rte ry
2–24. Which o the ollowing is true regarding relaxation
o the pelvic joints at term in pregnancy?
a. Is permanent and not accentuated in subsequent
Reproduced with permission rom Cunningham FG, Leveno KJ, Bloom SL, et al (eds): pregnancies
Maternal anatomy. In Williams Obstetrics, 23rd ed. New York, McGraw-Hill, 2010,
Figure 2-16. b. Allows or an increase in the transverse diameter
o the midpelvis
a. Aorta c. Results in marked mobility o the pelvis at term
b. External iliac artery because o a downward gliding movement o the
sacroiliac (SI) joint
c. Internal iliac artery
d. Displacement o the SI joint increases outlet
d. None o the above diameters by 1.5 to 2.0 cm in dorsal lithotomy
position
12 Ma terna l Ana tomy a nd Physiology

2–25. Which o the ollowing statements best describes the 2–29. The lower uterine segment incised at the time
origin o the internal branch o the common iliac o cesarean delivery is ormed by which o the
artery? ollowing?
a. Proximal to the iliolumbar artery a. Cornu
S
E
C
b. Distal to the lateral sacral artery b. Cervix
T
I
c. Distal to the superior rectal artery c. Isthmus
O
N
d. Proximal to where the ureters cross the pelvic brim d. Uterine corpus
2
2–26. The clinical evaluation o the pelvic inlet requires 2–30. Hegar sign re ers to which o the ollowing?
manual measurement o which diameter? a. Cervical so tening
a. True conjugate b. Bluish tint to ectocervix
b. Diagonal conjugate c. Replacement o collagen by smooth muscle in the
c. Obstetric conjugate cervix
d. Pelvic inlet transverse diameter d. Replacement o ectocervix by endocervix
commonly seen during pregnancy
2–27. Engagement occurs when the biparietal diameter o
the etal head descends below the level o which o 2–31. Which o the ollowing statements best describes the
the ollowing? pelvic outlet?
a. Midpelvis a. The base o the posterior triangle is the coccyx.
b. Pelvic inlet b. The angle o the pubic arch is usually < 90
c. Pelvic loor degrees.
d. Ischial tuberosities c. The lateral boundaries o the posterior triangle
are the descending in erior rami o the pubic
2–28. In this diagram, which o the ollowing is bones.
demonstrated? d. The common base o the two triangles is
ormed by a line drawn between the two ischial
tuberosities.

2–32. The posterior division o the internal iliac artery


contains which o the ollowing?
a. Iliolumbar artery
b. Middle rectal artery
Tra ns ve rs e of inle t = 13.5 cm
c. Superior rectal artery
Inte rs pinous = 10 cm d. Superior vesical artery
m
c
5
l
.
a
0
c
1
i
r
t
=
e
j
t
n
s
b
o
O
C
Reproduced with permission rom Cunningham FG, Leveno KJ, Bloom SL, et al.
Williams Obstetrics. 23rd ed. New York: McGraw-Hill Pro essional, 2010.

a. Ischial tuberosities
b. Midpelvis ischial spines
c. Important pelvic outlet diameters
d. An inadequate obstetrical conjugate
Ma terna l Ana tomy 13

CHAPTER 2 ANSw ER KEY

Q uestion Letter Pa ge

C
number a nswer cited Hea der cited

H
A
2–1 b p. 16 Blood Supply

P
T
2–2 d p. 17 Blood Supply; Figure 2 -1

E
R
2–3 d p. 18 Skin, Subcuta neous la yer, a nd Fa scia ; Figure 2 -2

2
2–4 b p. 17 Innerva tion; Figure 2 -1
2–5 d p. 18 Mons Pubis, La bia , a nd Clitoris
2–6 c p. 20 Va gina a nd Hymen
2–7 c p. 20 Va gina a nd Hymen; Figure 2 -8
2–8 c p. 24 Ana l Sphincter Complex
2–9 c p. 21 Perineum
2–10 a p. 20 Vestibule
2–11 c p. 26 Cervix
2–12 b p. 28 Blood Supply
2–13 a p. 27 Liga ments
2–14 a p. 27 Liga ments
2–15 a p. 24 Ana l Sphincter Complex; Figure 2 -4
2–16 b p. 26 Figure 2 -1 0
2–17 c p. 26 Figure 2 -1 0
2–18 d p. 26 Figure 2 -1 0
2–19 a p. 26 Figure 2 -1 0
2–20 a p. 28 Blood Supply
2–21 d p. 28 Blood Supply
2–22 b p. 30 Fa llopia n Tube
2–23 d p. 31 Pelvic Bones
2–24 d p. 32 Pelvic Joints
2–25 a p. 29 Figure 2 -1 3
2–26 b p. 32 Pelvic Inlet
2–27 b p. 32 Pelvic Inlet
2–28 b p. 33 Midpelvis a nd Pelvic O utlet
2–29 c p. 25 Uterus
2–30 a p. 26 Cervix
2–31 d p. 33 Midpelvis a nd Pelvic O utlet
2–32 a p. 29 Figure 2 -1 3
14

CHAPTER 3

Congenital Genitourinar Abnormalities

3–1. Which o the ollowing is not derived rom the 3–4. In this image, which o the ollowing ultimately
müllerian ducts? develops into the uterus?
a. Ovary
b. Uterus
c. Proximal vagina
d. All derive rom the müllerian ducts A

3–2. In emales, what does the metanephros ultimately


orm? B
Bla dde r
a. Uterus
b. Kidney
c. Embryonic remnants
D
d. None o the above
C

3–3. The urogenital sinus gives rise to which o the ollowing? Cloa ca
a. Bladder Modi ed with permission rom Bradshaw KB: Anatomic disorders. In Schorge JO,
b. Urethra Schaf er JI, Halvorson LM, et al (eds): Williams Gynecology. New York, McGraw-Hill,
2008, Figure 18-1C.
c. Distal vagina
d. All o the above a. A
b. B
c. C
d. D

3–5. Compared with the general population, women with


müllerian anomalies are at increased risk or which
o the ollowing?
a. Urinary tract anomalies
b. Premature ovarian ailure
c. Gastrointestinal anomalies
d. All o the above
Congenita l G enitourina ry Abnorma lities 15

3–6. I a müllerian anomaly is identi ied during pregnancy, 3–9. Cloacal exstrophy, bladder exstrophy, and epispadias
which o the ollowing modalities may be pre erred to all originate rom premature embryological rupture
initially search or an associated renal anomaly? o which o the ollowing?

C
a. Renal sonography a. Yolk sac

H
b. Computed tomography b. Sinovaginal bulb

A
P
c. Intravenous pyelography c. Cloacal membrane

T
E
R
d. Magnetic resonance imaging d. Hymeneal membrane

3
3–7. Your patient presents with vaginal spotting in the 3–10. In this sagittal image o the early etal pelvis,
irst trimester. During transvaginal 2-dimensional incomplete resorption at the point marked (*) leads
(2-D) sonographic evaluation, a live singleton etus is to which anomaly?
seen, and a uterine müllerian anomaly is suspected.
Three-dimensional (3-D) sonography is per ormed
at the same visit and shows this banana-shaped
uterus containing a gestational sac. What is the next Bla dde r
clinically prudent step during this pregnancy?

S ymphys is

P ha llus
* Anus

Modi ed with permission rom Bradshaw KD: Anatomic disorders. In Schorge JO,
Schaf er JI, Halvorson LM, et al (eds): Williams Gynecology. New York, McGraw-Hill,
2008, Figure 18-4B.

a. Vaginal adenosis
b. Gartner duct cyst
c. Imper orate hymen
d. Ambiguous genitalia
Reproduced with permission rom Moschos E, Twickler DM: Techniques used or
imaging in gynecology. In Hof man BL, Schorge JO, Schaf er JI, et al (eds): Williams
Gynecology, 2nd ed. New York, McGraw-Hill, 2012, Figure 2-23.
3–11. Imper orate hymen typically irst presents in which
age group and with which symptoms?
a. Schedule renal sonographic examination a. Perimenarche with amenorrhea
b. Schedule computed tomography with contrast b. Fetal period with polyhydramnios
c. Per orm prophylactic cervical cerclage at 14 c. Neonatal period with urinary retention
weeks’ gestation d. Reproductive age with primary in ertility
d. Recommend pregnancy termination due to the
high rate o uterine horn rupture 3–12. Classically, the pathogenesis o a müllerian de ect
involves which o the ollowing?
3–8. Mesonephric duct remnants may lead to which o a. Agenesis o one mesonephric duct
the ollowing?
b. Duplication o one paramesonephric duct
a. Skene gland cyst c. Faulty usion o the two mesonephric ducts
b. Gartner duct cyst d. None o the above
c. Urethral diverticulum
d. Bartholin gland duct cyst
16 Ma terna l Ana tomy a nd Physiology

3–13. Mayer-Rokitansky-Küster-Hauser (MRKH) 3–17. For diagnosing müllerian anomalies, which o the
syndrome is characterized by upper vaginal agenesis ollowing tools is the most accurate?
that is typically associated with uterine hypoplasia a. Hysterosalpingography
or agenesis. Other systems that may also be a ected
S
b. Saline in usion sonography
E
include all EXCEPT which o the ollowing?
C
c. Magnetic resonance imaging
T
a. Renal
I
O
d. Two-dimensional transvaginal sonography
b. Skeletal
N
2
c. Auditory 3–18. This hysterosalpingogram depicts a unicornuate
d. Gastrointestinal uterus. For diagnosing müllerian anomalies in
nonpregnant women, which o the ollowing are
3–14. A 19-year-old presents at 14 weeks’ gestation with disadvantages o hysterosalpingography?
this vaginal anomaly, which extends the ull vaginal
length. Additional evaluation reveals no associated
uterine de ect. You counsel her that this anomaly is
typically associated with a greater risk or which o
the ollowing peripartum complications?

a. Dye will not ill noncavitary horns.


b. No outer uterine undal contour is seen.
c. Dye will not ill noncommunicating horns.

Used with permission rom Dr. Alison Brooks.


d. All o the above

3–19. For diagnosing müllerian anomalies, which o


a. Urinary retention
the ollowing are advantages to 3-dimensional
b. Face presentation sonography?
c. Breech presentation a. Displays the contour o the endometrium
d. None o the above b. Less expensive than magnetic resonance imaging
c. Displays the contour o the outer uterine undus
3–15. During labor, a transverse vaginal septum may be
managed appropriately with all EXCEPT which o d. All o the above
the ollowing strategies?
a. Permit normal labor
b. Avoid labor augmentation
c. Per orm cesarean delivery
d. Cruciate incision o the septum once cervical
dilatation is complete

3–16. Which o the ollowing is the more common uterine


müllerian anomaly?
a. Uterine agenesis
b. Bicornuate uterus
c. Uterine didelphys
d. Unicornuate uterus
Congenita l G enitourina ry Abnorma lities 17

3–20. With magnetic resonance imaging, a septate uterus is 3–24. A longitudinal vaginal septum is LEAST commonly
displayed here. For diagnosing müllerian anomalies, seen with which o the ollowing müllerian
which o the ollowing are advantages to this anomalies?
modality?

C
a. Septate uterus

H
b. Unicornuate uterus

A
P
c. Bicornuate uterus

T
E
R
d. Uterine didelphys

3
3–25. Which uterine müllerian anomaly is seen in this
hysterosalpingogram?

Reproduced with permission rom Moschos E, Twickler DM: Techniques used or


imaging in gynecology. In Schorge JO, Schaf er JI, Halvorson LM et al (eds): Williams
Gynecology. New York, McGraw-Hill, 2008, Figure 2-27.

a. Is nearly 100-percent accurate


b. Displays undal, myometrial, and endometrial
contours
c. Permits identi ication o concurrent skeletal or Reproduced with permission rom Halvorson LM: Evaluation o the in ertile couple. In
Schorge JO, Schaf er JI, Halvorson LM, et al (eds): Williams Gynecology. New York,
renal anomalies
McGraw-Hill, 2008, Figure 19-7C.
d. All o the above
a. Arcuate uterus
3–21. The pathogenesis o poor pregnancy outcomes with
b. Septate uterus
a unicornuate uterus is thought to be related to all
EXCEPT which o the ollowing primary actors? c. Uterine didelphys
a. Cervical incompetence d. Bicornuate uterus
b. Reduced uterine capacity
3–26. With magnetic resonance imaging, a bicornuate
c. Poor implantation into endometrium uterus is most reliably di erentiated rom a septate
d. Anomalous distribution o the uterine artery uterus by which o the ollowing characteristics?
a. Intra undal cle t < 1 cm deep
3–22. Rates or all EXCEPT which o the ollowing
b. Intra undal cle t > 1 cm deep
obstetrical complications are increased in women
with uterine müllerian anomalies? c. Two distinct endometrial cavities
a. Twinning d. Partition running the ull uterine cavity length
b. Miscarriage
3–27. Reparative excision is most easible and easiest or
c. Malpresentation which o the ollowing uterine müllerian anomalies?
d. Preterm delivery a. Septate uterus
b. Uterine didelphys
3–23. Which category o unicornuate uterus poses the
greatest risk or ectopic pregnancy? c. Bicornuate uterus
a. Agenesis o one horn d. Unicornuate uterus with a communicating
cavitary rudimentary horn
b. Communicating noncavitary rudimentary horn
c. Noncommunicating cavitary rudimentary horn
d. Noncommunicating noncavitary rudimentary
horn
18 Ma terna l Ana tomy a nd Physiology

3–28. The highest miscarriage rate is associated with which 3–30. For which o the ollowing müllerian anomalies
o the ollowing uterine müllerian anomalies? should prophylactic cervical cerclage be
a. Septate uterus recommended in most cases?
a. Bicornuate uterus
S
b. Bicornuate uterus
E
C
c. Uterine didelphys b. Uterine didelphys
T
I
c. Unicornuate uterus
O
d. Unicornuate uterus with a noncommunicating
N
noncavitary rudimentary horn d. None o the above
2
3–29. A 22-year-old G1P0 presents to your o ice as a 3–31. All EXCEPT which o the ollowing are common
new patient or prenatal care. During transvaginal symptoms associated with a retro lexed incarcerated
2-dimensional (2-D) sonographic evaluation, a live uterus?
singleton etus is seen, but a müllerian anomaly is a. Abdominal pain
suspected. Three-dimensional (3-D) sonography b. Pelvic pressure
is subsequently per ormed, and this image shows
an arcuate uterus containing a gestational sac. The c. Vaginal bleeding
outer uterine contour rounds slightly outward. The d. Urinary retention
endometrial contour indents only slightly inward.
Which o the ollowing untoward outcomes has been 3–32. Which o the ollowing is the most common
consistently associated with this particular inding? complication encountered during cesarean delivery
or anterior or posterior uterine sacculation?
a. Placenta previa
b. Placenta accreta
c. Urinary retention
d. Distorted anatomy

a. Miscarriage
b. Preterm delivery
c. Incompetent cervix
d. None o the above
Congenita l G enitourina ry Abnorma lities 19

CHAPTER 3 ANSw ER KEy

Q uestion Letter Pa ge

C
number a nswer cited Hea der cited

H
A
3–1 a p. 36 Embryology of the Urina ry System

P
T
3–2 b p. 36 Embryology of the Urina ry System

E
R
3–3 d p. 36 Embryology of the Urina ry System

3
3–4 a p. 37 Embryology of the G enita l Tra ct
3–5 a p. 37 Embryology of the G enita l Tra ct
3–6 a p. 37 Embryology of the G enita l Tra ct
3–7 a p. 37 Embryology of the G enita l Tra ct
3–8 b p. 37 Mesonephric Remna nts
3–9 c p. 38 Bla dder a nd Perinea l Anoma lies
3–10 c p. 38 Defects of the Hymen
3–11 a p. 38 Defects of the Hymen
3–12 d p. 38 Mülleria n Abnorma lities
3–13 d p. 39 Va gina l Abnorma lities
3–14 d p. 39 Va gina l Abnorma lities
3–15 b p. 39 Va gina l Abnorma lities
3–16 b p. 40 Uterine Abnorma lities
3–17 c p. 40 Uterine Abnorma lities
3–18 d p. 40 Uterine Abnorma lities
3–19 d p. 40 Uterine Abnorma lities
3–20 d p. 40 Uterine Abnorma lities
3–21 c p. 40 Unicornua te Uterus (Cla ss II)
3–22 a p. 40 Unicornua te Uterus (Cla ss II)
3–23 c p. 40 Unicornua te Uterus (Cla ss II)
3–24 b p. 41 Uterine Didelphys (Cla ss III); Bicornua te Uterus (Cla ss IV);
Septa te Uterus (Cla ss V)
3–25 c p. 41 Uterine Didelphys (Cla ss III)
3–26 b p. 41 Bicornua te Uterus (Cla ss IV)
3–27 a p. 42 Septa te Uterus (Cla ss V)
3–28 a p. 42 Septa te Uterus (Cla ss V)
3–29 d p. 42 Arcua te Uterus (Cla ss VI)
3–30 d p. 42 Trea tment with Cercla ge
3–31 c p. 42 Retroflexion
3–32 d p. 43 Sa ccula tion
20

CHAPTER 4

Maternal Ph siolog

4–1. Changes in maternal blood volume and cardiac output 4–4. In this photograph, cervical eversion is
in pregnancy may mimic which following disease states? demonstrated. As shown, what kind of epithelium
a. Hypertension makes up most of the visible portion of the cervix?
b. Thyrotoxicosis
c. Diabetes insipidus
d. Chronic renal disease

4–2. Regarding Braxton Hicks contractions, which of the


following is true?
a. Their intensity varies between 20 and 40 mm Hg.
b. They occur early in pregnancy and may be
palpated in the second trimester.
c. Late in pregnancy, these contractions become
more regular and may cause discomfort.
d. B and C

4–3. Uterine blood flow near term most closely


approximates which of the following?
a. 150 mL/min
b. 350 mL/min
c. 550 mL/min
d. 850 mL/min Reproduced with permission from Cunningham FG, Leveno KJ, Bloom SL, et al
(eds): Maternal physiology. In Williams Obstetrics, 24th ed. New York, McGraw-Hill, 2014,
Figure 4-1.

a. Serous
b. Columnar
c. Squamous
d. Transitional
Ma terna l Physiology 21

4–5. This pattern of cervical mucus is typically seen in 4–7. The graphic below illustrates which of the following
which of the following clinical settings? pregnancy-related concepts? LMP = last menstrual
period; MP = menstrual period.

C
H
300

A
P
T
296

E
R
4
292

)
kg
/
l
o
288

m
s
O
m
284

(
m
s
o
P
280

276

272

MP MP LMP 4 8 12 16
We e ks of pre gna ncy
Reproduced with permission from Cunningham FG, Leveno KJ, Bloom SL, et al (eds): Maternal
physiology. In Williams Obstetrics, 24th ed. New York, McGraw-Hill, 2014, Figure 4-2. Reproduced with permission from Cunningham FG, Leveno KJ, Bloom SL, et al (eds):
Maternal physiology. In Williams Obstetrics, 24th ed. New York, McGraw-Hill, 2014,
Figure 4-4.
a. Ovulation
b. Uncomplicated pregnancy a. Maternal plasma osmolality decreases early in
c. Pregnancy with amnionic fluid leakage pregnancy.
d. A and C b. Maternal plasma osmolality increases throughout
pregnancy.
4–6. Your pregnant patient in her second trimester c. Maternal plasma osmolality does not change
presents with breasts that have enlarged during the during pregnancy.
past few months. This photograph illustrates which d. Maternal plasma osmolality is affected most by
of the following? increases in sodium.

Used with permission from Dr. Mary Jane Pearson.

a. Gigantomastia
b. Inflammatory breast carcinoma
c. Pathologic enlargement that may ultimately
require surgery.
d. A and C
22 Ma terna l Ana tomy a nd Physiology

4–8. This graphic concerning insulin and glucose levels 4–11. As illustrated by this graphic, which of the following
during pregnancy suggests which of the following? occurs during pregnancy?

140 50 Blood volume


S
Glucos e Nonpre gna nt (n = 8)
E
P la s ma volume
C
Norma l pre gna nt
120 (n = 8) Re d ce ll volume
T
40
I
O
L
s
l
d
N
e
100
/
v
g
s
e
m
e
l
2
g
t
30

n
n
a
a
80

n
h
g
c
e
t
r
n
p
e
n
60 20

c
o
r
n
e
P
m
o
r
f
Ins ulin Nonpre gna nt (n = 8) 10
250 Norma l pre gna nt
(n = 8)

200
10 20 30 40 De live ry 6 we e ks
pos tpa rtum
L
150 We e ks of ge s ta tion
m
/
U
Reproduced with permission from Cunningham FG, Leveno KJ, Bloom SL, et al (eds):
µ
100
Maternal physiology. In Williams Obstetrics, 24th ed. New York, McGraw-Hill, 2014,
Figure 4-6.
50
a. The hematocrit increases.
0
8 AM 1 PM 6 PM 12 M 8 AM b. Total blood volume increases 40% by term.
MEALS : c. Red cell volume does not increase until 20 weeks.
Reproduced with permission from Cunningham FG, Leveno KJ, Bloom SL, et al (eds): d. The hematocrit increases due to an increased red
Maternal physiology. In Williams Obstetrics, 24th ed. New York, McGraw-Hill, 2014,
Figure 4-5.
cell volume relative to plasma volume.

a. Hypoinsulinemia
4–12. This graphic suggests which of the following?
b. Hyperinsulinemia µg/dL mg/dL ng/dL

c. Postprandial hypoglycemia 100 500

d. Mild fasting hyperglycemia


80 400 80

4–9. The total increase in protein during pregnancy Tra ns fe rrin


approximates which of following?
n
i
n
n
t
60 300 60
i
i
o
r
r
r
r
r
i
a. 500 g
e
e
f
f
m
S e rum iron
s
m
u
n
r
u
a
e
b. 1000 g
r
r
T
S
e
S
40 200 40

c. 500 g for contractile protein in the uterus


d. B and C 20 100 20
S e rum fe rritin

4–10. Related to calcium metabolism, which of the 0


0 0
following occurs during pregnancy? Non- 1s t 2nd 3rd De live ry P os t-
pre gna nt trime s te r pa rtum
a. Serum magnesium levels increase.
Reproduced with permission from Cunningham FG, Leveno KJ, Bloom SL, et al (eds):
b. Total serum calcium levels decline. Maternal physiology. In Williams Obstetrics, 23rd ed. New York, McGraw-Hill, 2014,
c. The fetal skeleton accrues 70 g of calcium by Figure 5-6.
term.
d. Maximums of 500 mEq of sodium and 200 mEq a. Serum iron is decreased in the first trimester.
of potassium are retained. b. Serum ferritin is increased in the second
trimester.
c. Serum ferritin is increased by the end of
pregnancy.
d. Serum transferrin is increased by the end of
pregnancy.
Ma terna l Physiology 23

4–13. Average blood loss for a vaginal delivery is which the 4–17. During pregnancy, the venous pressure does which
following? of the following?
a. 500 mL a. Decreases when the woman is lying in the lateral

C
b. 1000 mL position

H
b. Declines from 24 mm Hg to 8 mm Hg at term in

A
c. Half of that lost during cesarean delivery of twins

P
the lower extremities

T
d. A and C

E
c. Is responsible for dependent edema in the lower

R
4–14. Regarding immunological function during extremities

4
pregnancy, which of the following statements is true? d. A and C
a. Th1 response is suppressed.
b. Th2 cells are downregulated. 4–18. Which of the following are true regarding infused
angiotensin II and its vascular effects during
c. There is up regulation of T-cytotoxic cells.
pregnancy?
d. All of the above
a. Hypertensive patients become and then remain
4–15. Regarding the coagulation system in pregnancy, refractory.
which of the following statements is true? b. The vascular response is believed to be
a. Mean platelet count is 250,000/µL. progesterone related.
b. Fibrinolytic activity is usually reduced. c. Normotensive nulliparas near term are responsive
to the effects of angiotensin II.
c. Fibrinogen levels are increased to a median of
250 mg/dL. d. Increased vessel refractoriness to angiotensin II
results primarily from altered renin-angiotensin
d. Decreases in platelet concentration are solely due
secretion.
to hemodilution.

4–16. The graphic below demonstrates which of the 4–19. Concerning acid-base equilibrium during pregnancy,
following points? which of the following statement is true?
a. Plasma bicarbonate concentration decreases from
S upine 26 to approximately 22 mmol/L.
S ide
t
u
b. The maternal oxygen-disassociation curve is
p
S itting
t
u
)
6
o
shifted to the right.
n
i
c
m
a
/
i
c. Physiological dyspnea results from slightly
L
d
(
r
a
decreased tidal volume that lowers CO 2 levels.
C
4
100 d. Estrogen acts centrally, where it lowers the
threshold and increases the sensitivity of the
)
n
e
i
t
chemoreflex response to CO 2.
m
a
r
/
80
t
s
r
t
a
a
e
e
H
b
(
60
e
m
90
u
l
vo
)
L
m
ke
70
(
o
r
t
S
50
20–24 28 – 32 38 –40 6– 8 we e ks ’
We e ks ’ ge s ta tion PP
Reproduced with permission from Cunningham FG, Leveno KJ, Bloom SL, et al (eds): Maternal
physiology. In Williams Obstetrics, 24th ed. New York, McGraw-Hill, 2014, Figure 4-7.

a. Cardiac output increases between 20 and 40 weeks’


gestation.
b. Heart rate increases when pregnant women are
sitting compared with lying supine.
c. Cardiac output increases when postpartum
women are sitting compared with lying supine.
d. Stroke volume increases when pregnant women
are supine compared with lying on their sides.
24 Ma terna l Ana tomy a nd Physiology

4–20. Regarding lung volumes during pregnancy, which of


the following is true?

6 6
S
Not P re gna nt
E
C
pre gna nt (7–9 mos .)
T
I
5 5
O
N
IRV
2
4 IC IRV 4
IC
)
V
L
o
FVC
(
l
u
VT
e
3 FVC 3

m
m
e
VT
u
TLC TLC
l
(
o
L
V
)
2 ERV 2
ERV
FRC
1 FRC 1
RV RV
RV RV
0 0
Expiratory reserve volume (ERV); Functional residual capacity (FRC); Forced vital capacity (FVC); Inspiratory capacity (IC); Inspiratory reserve volume (IRV);
Residual volume (RV); Tidal volume (VT); Total lung capacity (TLC). Reproduced with permission from Cunningham FG, Leveno KJ, Bloom SL, et al (eds):
Maternal physiology. In Williams Obstetrics, 24th ed. New York, McGraw-Hill, 2014, Figure 4-13.

a. Tidal volume is increased. 4–24. Which of the following is true regarding the
b. Reserve volume is increased. gallbladder during pregnancy?
c. Total lung capacity increases by approximately a. It empties more completely.
5% by term. b. It has reduced contractility caused by
d. A and C progesterone.
c. Stasis leads to formation of pyruvate-containing
4–21. The maternal arteriovenous oxygen difference stones.
decreases during pregnancy because of which of the d. Cholestasis is linked to high circulating levels of
following? progesterone.
a. Increased tidal volume
b. Increased cardiac output 4–25. Regarding the pituitary gland during pregnancy,
c. Increased hemoglobin mass which of the following is true?
d. All of the above a. Growth of microadenomas is likely.
b. Serum prolactin levels remain unchanged
4–22. Regarding the bladder during pregnancy, which of throughout pregnancy.
the following is true? c. It enlarges due to estrogen-stimulated
a. There is increased bladder capacity. hypertrophy and hyperplasia of the lactotrophs.
b. Absolute and functional urethral length increases. d. A and C
c. Bladder pressure decreases from 15 to 8 cm H 2O
by term.
d. Approximately three fourths of all pregnant
women experience incontinence during pregnancy.

4–23. Regarding the gastrointestinal tract during


pregnancy, which of the following is true?
a. Gastric emptying time is shortened during labor.
b. Gastric emptying time is lengthened in each
trimester.
c. Pyrosis is caused by reflux of acidic secretions into
the lower esophagus.
d. Epulis gravidarum is a highly vascular swelling
that may affect any mucosal membrane.
Ma terna l Physiology 25

4–26. Regarding the thyroid gland during pregnancy, 4–27. Regarding iodine during pregnancy, which of the
which of the following is true? following is true?
a. Maternal requirements remain the same during
Mothe r

C
normal pregnancy.

H
b. Fetal neurodevelopment is dependent on

A
P
adequate thyroid hormone exposure.

T
E
TBG c. Iodine deficiency is the most preventable cause of

R
fetal neurological deficiency.

4
Tota l T4
d. In women with low or marginal intake, deficiency
may manifest as decreased thyroid-stimulating
hormone (TSH) levels.
hCG
Fre e T4 4–28. Regarding fetal skeletal mineralization, which of the
Thyrotropin following is true?
a. The calcium required by the fetus represents only
3% of maternal stores.
b. A total of 3 g of calcium is required, primarily
during the third trimester.
Fe tus c. Calcium absorption reaches approximately
100 mg/day in the third trimester.
TBG d. A and B

4–29. This graphic illustrates which of the following?


Tota l T4
50
500
400
Thyrotropin 40
300
200
Fre e T4

)
)
Tota l T3 80
30
70

(
(
Fre e T3
60

)
)
L
L
d
m
20

/
50

/
g
10 20 30 40

g
µ
p
(
(
We e k of pre gna ncy 40

l
H
o
s
T
i
t
30
C
r
Reproduced with permission from Cunningham FG, Leveno KJ, Bloom SL, et al (eds):
o
10
A
C
Maternal physiology. In Williams Obstetrics, 24th ed. New York, McGraw-Hill, 2014,
20
Figure 4-17.
10
a. It undergoes enlargement through hypertrophy. 0
b. Total T 4 concentration increases during 10 20 30 40
pregnancy. We e ks ’ ge s ta tion
c. Free T 4 concentration increases its mean value by Reproduced with permission from Cunningham FG, Leveno KJ, Bloom SL, et al (eds):
term. Maternal physiology. In Williams Obstetrics, 24th ed. New York, McGraw-Hill, 2014,
Figure 4-19.
d. Human chorionic gonadotropin (hCG), which
mimics thyroid-stimulating hormone, has
a. Maximum secretion of cortisol is reached by
declining levels beginning at approximately
20 weeks.
20 weeks.
b. Serum cortisol secretion decreases throughout
pregnancy.
c. Serum adrenocorticotropic hormone (ACTH)
secretion increases across pregnancy.
d. None of the above
26 Ma terna l Ana tomy a nd Physiology

4–30. Regarding aldosterone during pregnancy, which of 4–31. Regarding the central nervous system during
the following is true? pregnancy, which of the following is true?
a. It is the principal mineralocorticoid. a. Memory decline is typically limited to the third
S
b. Secretion is decreased by sodium restriction. trimester.
E
C
c. Increased aldosterone levels protect against the b. Pregnancy significantly alters cerebrovascular
T
autoregulation.
I
antinatriuretic effect of progesterone and atrial
O
N
natriuretic peptide. c. Attention and memory are typically decreased in
2
d. A and B women receiving magnesium sulfate.
d. Mean blood flow in the middle cerebral artery
(MCA) and posterior cerebral artery (PCA)
increases in the third trimester.
Ma terna l Physiology 27

CHAPTER 4 ANSw ER KEy

Q uestion Letter Pa ge

C
number a nswer cited Hea der cited

H
A
4–1 b p. 46 Introduction

P
T
4–2 d p. 47 Uterine Contra ctility

E
R
4–3 c p. 47 Uteropla centa l Blood Flow

4
4–4 b p. 48 Cervix
4–5 d p. 48 Cervix
4–6 d p. 50 Brea sts
4–7 a p. 51 W a ter Meta bolism
4–8 b p. 53 Ca rbohydra te Meta bolism
4–9 d p. 53 Protein Meta bolism
4–10 b p. 54 Electrolyte a nd Minera l Meta bolism
4–11 b p. 55 Blood Volume
4–12 d p. 55 Iron Meta bolism
4–13 a p. 56 The Puerperium
4–14 a p. 56 Immunologica l Functions
4–15 b p. 57 Coa gula tion a nd Fibrinolysis
4–16 b p. 58 Ca rdiova scula r System
4–17 d p. 60 Circula tion a nd Blood Pressure
4–18 b p. 61 Renin, Angiotensin II, a nd Pla sma Volume
4–19 a p. 63 Acid–Ba se Equilibrium
4–20 a p. 62 Pulmona ry Function
4–21 d p. 63 O xygen Delivery
4–22 b p. 66 Bla dder
4–23 c p. 66 G a strointestina l Tra ct
4–24 b p. 67 G a llbla dder
4–25 c p. 67 Pituita ry G la nd
4–26 b p. 68 Thyroid G la nd
4–27 b p. 69 Iodine Sta tus
4–28 a p. 70 Pa ra thyroid Hormone
4–29 c p. 70 Adrena l G la nd
4–30 a p. 70 Aldosterone
4–31 a p. 72 Memory
This page intentionally left blank
Se c t i o n 3

PLACENTATION, EMBRYOGENESIS,
AND FETALDEVELOPMENT
30

CHAPTER 5

Implantation and Placental Development

5–1. What is the average duration o a normal menstrual 5–6. What is the approximate peak production o ovarian
cycle? progesterone during midluteal phase?
a. 14–28 days a. 10–20 mg/day
b. 25–32 days b. 25–50 mg/day
c. 28–55 days c. 60–80 mg/day
d. 40–50 days d. 75–100 mg/day

5–2. O 2 million oocytes in the human ovary present at 5–7. Which o the ollowing is the most biologically
birth, how many are present at the onset o puberty? potent naturally occurring estrogen?
a. 200,000 a. Estriol
b. 300,000 b. Estrone
c. 400,000 c. Estetrol
d. 500,000 d. 17β -Estradiol

5–3. Which hormone is required or the late-stage devel- 5–8. Which o the ollowing is the endometrial layer that
opment o antral ollicles? is shed with every menstrual cycle?
a. Estradiol a. Basalis layer
b. Androstenedione b. Decidual layer
c. Luteinizing hormone c. Luteinized layer
d. Follicle-stimulating hormone d. Functionalis layer

5–4. Which cells o the dominant ollicle are responsible 5–9. Within the glandular epithelium o the endo-
or estrogen production during the ollicular phase o metrium, what is the irst histological sign o
the menstrual cycle? ovulation?
a. Theca a. Cessation o glandular cell mitosis
b. Decidual b. Vacuoles at the apical portion o the secretory
nonciliated cells
c. Granulosa
d. Endometrial c. Secretory nonciliated cells devoid o glycoprotein
and mucopolysaccharide
5–5. What name is given to the process through which d. Subnuclear vacuoles and pseudostrati ication in
the corpus luteum develops rom the remains o the the basal portion o the glandular epithelium
Graa ian ollicle?
a. Luteinization
b. Thecalization
c. Decidualization
d. Graa ian trans ormation
Impla nta tion a nd Pla centa l Development 31

5–10. In the ollowing image, which letter identi ies the


spiral arteries?

C
Ute rine

H
lume n

A
P
Epithe lium

T
E
Ca pilla rie s

R
5
Ve nous s inus
Functiona lis
Endome tria l gla nd la ye r

Arte ry A

Arte ry B
Ba s a lis la ye r

Arte ry C
Myome trium
Arte ry D

Ute rine a rte ry

Modif ed with permission rom Cunningham FG, Leveno KJ, Bloom SL, et al (eds): Implantation and placental development.
In Williams Obstetrics, 24th ed. New York, McGraw-Hill, 2014, Figure 5-4.

a. A 5–12. In sequence rom letters A to C, please identi y the


b. B three types o deciduas in the igure:
c. C
d. D Yolk s a c
De cidua A
5–11. Which prostaglandin plays a role in vasoconstriction Embryo in a mnionic s a c
Chorionic
o the spiral arteries, leading to menstruation? villi Chorionic villi
a. Prostaglandin E1 De cidua B De cidua C
b. Prostaglandin E2
c. Prostaglandin D 2 Exocoe lomic
cavity
d. Prostaglandin F2α
Ce rvica l
Ute rine cavity ca na l

Modif ed with permission rom Cunningham FG, Leveno KJ, Bloom SL, et al (eds):
Implantation and placental development. In Williams Obstetrics, 24th ed. New York,
McGraw-Hill, 2014, Figure 5-6.

a. Basalis, capsularis, parietalis


b. Capsularis, basalis, parietalis
c. Parietalis, basalis, capsularis
d. Parietalis, capsularis, basalis
32 Pla centa tion, Embryogenesis, a nd Feta l Development

5–13. What is the Nitabuch layer? 5–17. In this drawing o implantation, which o the ollow-
a. A layer o the decidua made o large, distended ing labeled structures will eventually become the etus?
glands
S
b. An area o the decidua with large, closely packed
E
A
C
epithelioid, polygonal cells
T
c. A zone o ibrinoid degeneration where the
I
O
invading trophoblast and decidua meet
N
3
d. An area o super icial ibrin deposition at the
bottom o the intervillous space and surrounding B
the anchoring villi C

5–14. Which o the ollowing unctions does the placenta


not per orm or the etus?
a. Renal
b. Hepatic
D
c. Adrenal
d. Pulmonary Modif ed with permission rom Cunningham FG, Leveno KJ, Bloom SL, et al (eds):
Implantation and placental development. In Williams Obstetrics, 24th ed. New York,
McGraw-Hill, 2014, Figure 5-9B.
5–15. At 5 days post ertilization, the blastocyst is released
and hatched rom which surrounding structure?
a. A
a. Morula
b. B
b. Chorion laeve
c. C
c. Trophectoderm
d. D
d. Zona pellucida
5–18. Which o the ollowing statements is accurate
5–16. Which o the ollowing gives rise to the chorionic
regarding the chorion rondosum?
structures that transport oxygen and nutrients
between etus and mother? a. It is the same as the chorion laeve.
a. Villous trophoblast b. It is the maternal component o the placenta.
b. Interstitial trophoblast c. It is the area o villi in contact with the decidua
basalis.
c. Extravillous trophoblast
d. It is the avascular area that abuts the decidua
d. Endovascular trophoblast parietalis.

5–19. Maternal regulation o trophoblast invasion and


vascular growth is mainly controlled by which o the
ollowing?
a. CD4 T cells
b. Progesterone
c. Cellular adhesion molecules
d. Decidual natural killer (DNK) cells

5–20. Remodeling o maternal spiral arteries by invad-


ing trophoblasts is completed by which week(s) o
pregnancy?
a. 8th week
b. 12th week
c. 12–16 weeks
d. 16–18 weeks
Impla nta tion a nd Pla centa l Development 33

5–21. End-diastolic blood low can be identi ied in the 5–26. As shown in this igure, blood coming rom the pla-
etal umbilical artery by the end o which week o centa to the etus travels irst rom the umbilical vein
pregnancy? into which o the ollowing structures?

C
a. 10th week

H
Ductus
b. 14th week

A
a rte rios us

P
c. 18th week

T
E
R
d. 22nd week S upe rior ve na cava

5
5–22. Regarding the orientation o spiral blood vessels in LA
relationship to the uterus, which o the ollowing is Fora me n ova le
true?
RA
a. Both arteries and veins are parallel to the uterine
wall. LV
RV
b. Both arteries and veins are perpendicular to the
uterine wall.
c. Arteries are perpendicular and veins are parallel to
the uterine wall.
d. Veins are perpendicular and arteries are parallel to
Porta l
the uterine wall. s inus Aorta

5–23. What is the name given to the phenomenon that


describes how etal cells can become engra ted in
Umbilica l a a .
the mother during pregnancy and then be identi ied
decades later?
Umbilica l v.
a. Microchimerism Hypoga s tric
aa.
b. Histocompatibility
Oxyge na te d
c. Hemochorial invasion
d. Immunological neutrality Mixe d

P la ce nta De oxyge na te d
5–24. Which o the ollowing is a component o the
amnion? Modif ed with permission rom Cunningham FG, Leveno KJ, Bloom SL, et al (eds):
Embryogenesis and etal morphological development. In Williams Obstetrics, 24th ed.
a. Nerves New York, McGraw-Hill, 2014, Figure 7-8.
b. Lymphatics
c. Blood vessels a. The portal vein
d. Acellular zona spongiosa b. The hepatic vein
c. The ductus venosus
5–25. Which o the ollowing is accurate regarding a
d. The in erior vena cava
Meckel diverticulum?
a. It is an allantoic duct remnant. 5–27. The amino acid sequence o the α -subunit o human
b. It is a ailure o the right umbilical vein to chorionic gonadotropin (hCG) is identical in all
involute. EXCEPT which o the ollowing?
c. It is a portion o one umbilical artery that a. Luteinizing hormone
remains patent postnatally. b. Thyroid-stimulating hormone
d. It is a ailure o the intraabdominal portion o the c. Follicle-stimulating hormone
umbilical vesicle to atrophy.
d. Corticotropin-releasing hormone
34 Pla centa tion, Embryogenesis, a nd Feta l Development

5–28. Abnormally low levels o human chorionic gonado- 5–32. Which o the ollowing is true o bilateral oophorec-
tropin (hCG) may be ound most typically with tomy at 9 weeks’ gestation?
which o the ollowing? a. It will cause a miscarriage.
a. Down syndrome
S
b. It will cause a signi icant drop in progesterone.
E
C
b. Ectopic pregnancy c. It will not alter the maternal excretion o urinary
T
I
c. Erythroblastosis etalis pregnanediol.
O
N
d. Gestational trophoblastic disease d. None o the above
3
5–29. Known biological actions o human chorionic 5–33. Ichthyosis, an X-linked disorder that a ects male
gonadotropin (hCG) include all EXCEPT which o etuses, is associated with which o the ollowing?
the ollowing? a. Fetal adrenal hypoplasia
a. Maternal thyroid stimulation b. Fetal adrenal hyperplasia
b. Inhibition o relaxin secretion c. Fetal placental sul atase de iciency
c. Sexual di erentiation o the male etus d. Fetal placental aromatase de iciency
d. Rescue and maintenance o the corpus luteum
5–34. Which o the ollowing conditions is associated with
5–30. Which o the ollowing has the greatest production increased estrogen levels in pregnancy?
rate o any known human hormone? a. Fetal demise
a. Progesterone b. Down syndrome
b. Human placental lactogen c. Fetal anencephaly
c. Human chorionic gonadotropin d. Erythroblastosis etalis
d. Chorionic adrenocorticotropin

5–31. Among placental peptide hormones, which has


shown a correlation with birthweight?
a. Leptin
b. Activin
c. Inhibin
d. Neuropeptide Y
Impla nta tion a nd Pla centa l Development 35

CHAPTER 5 ANSw ER KEY

Q uestion Letter Pa ge

C
number a nswer cited Hea der cited

H
A
5–1 b p. 8 0 The O va ria n Endometria l Cycle

P
T
5–2 c p. 8 0 Follicula r a nd Preovula tory O va ria n Pha se

E
R
5–3 d p. 8 0 Follicula r a nd Preovula tory O va ria n Pha se

5
5–4 c p. 8 0 Follicula r a nd Preovula tory O va ria n Pha se
5–5 a p. 8 2 Lutea l or Postovula tory O va ria n Pha se
5–6 b p. 8 2 Lutea l or Postovula tory O va ria n Pha se
5–7 d p. 8 3 Estrogen a nd Progesterone Action
5–8 d p. 8 4 The Endometria l Cycle; Prolifera tive a nd Preovula tory Endometria l Pha se
5–9 d p. 8 5 Secretory or Postovula tory Endometria l Pha se
5–10 a p. 8 6 Figure 5 -4
5–11 d p. 8 6 Prosta gla ndins a nd Menstrua tion
5–12 d p. 8 7 Decidua l Structure; Figure 5 -6
5–13 c p. 8 8 Decidua l Histology
5–14 c p. 8 9 Impla nta tion a nd Ea rly Trophobla st Inva sion
5–15 d p. 9 0 The Bla stocyst
5–16 a p. 9 0 Trophobla st Differentia tion
5–17 b p. 9 2 Ea rly Trophobla st Inva sion; Figure 5 -9
5–18 c p. 9 3 Chorion a nd Decidua Development
5–19 d p. 9 3 Ma terna l Regula tion of Trophobla st Inva sion a nd Va scula r G rowth
5–20 c p. 9 3 Inva sion of Spira l Arteries
5–21 a p. 9 5 Feta l Circula tion
5–22 c p. 9 6 Ma terna l Circula tion
5–23 a p. 9 7 Brea ks in the Pla centa l “Ba rrier”
5–24 d p. 9 8 The Amnion
5–25 d p. 1 0 0 Cord Development
5–26 c p. 1 0 1 Cord Function
5–27 d p. 1 0 1 hCG ; Chemica l Cha ra cteristics
5–28 b p. 1 0 3 Abnorma lly High or Low hCG Levels
5–29 b p. 1 0 3 Biologica l Functions of hCG
5–30 b p. 1 0 3 Huma n Pla centa l La ctogen; Chemica l Cha ra cteristics
5–31 a p. 1 0 6 Leptin
5–32 c p. 1 0 6 Pla centa l Progesterone Production
5–33 c p. 1 1 0 Feta l Pla centa l Sulfa ta se Deficiency
5–34 d p. 1 0 9 Feta l Conditions Tha t Affect Estrogen Production
36

CHAPTER 6

Placental Abnormalities

6–1. For which situation is pathological examination o 6–4. This is which type o placental variant?
the placenta considered most in ormative and cost
e ective?
a. Multi etal gestation
b. Cholestasis o pregnancy
c. Maternal seizure disorder
d. All obstetrical deliveries

6–2. At term, which o the ollowing most closely


approximates typical placental disk measurements?
a. 200 g weight, 10 cm diameter, 15 mm thickness
b. 500 g weight, 20 cm diameter, 25 mm
thickness
c. 1000 g weight, 15 cm diameter, 35 mm thickness
d. 1500 g weight, 25 cm diameter, 45 mm thickness Used with permission rom Dr. Jaya George.

6–3. Which o the ollowing terms describes this sur ace a. Bilobate placenta
o the placenta?
b. Succenturiate lobe
c. Placenta enestrata
d. Circumvallate placenta

6–5. Which placental variant is marked by arrowheads in


this image?

a. Basal plate
b. Amniochorion
c. Placental bed
d. Chorionic plate Used with permission rom Dr. Heather Lytle.

a. Bilobate placenta
b. Succenturiate lobe
c. Placenta enestrata
d. Circumvallate placenta
Pla centa l Abnorma lities 37

6–6. Compared with a normally shaped placenta, which 6–9. Pregnancies with this type o extrachorial placentation
complication o third-stage labor is more common are at increased risk or which o the ollowing
with an undiagnosed succenturiate lobe? complications?

C
a. Cord avulsion

H
b. Chorioamnionitis

A
P
c. Uterine inversion

T
E
R
d. Retained cotyledon

6
6–7. The placenta in this image meets sonographic
criteria or placentomegaly. Common causes o an
increased placental thickness include all EXCEPT
which o the ollowing?

a. Stillbirth
b. Placental abruption
c. Intrapartum etal acidosis
d. None o the above

6–10. Which term best describes the small opaque plaque


(arrow) seen on the etal sur ace o this placenta?

Reproduced with permission rom Hof man BL, Ziadie MS, Dashe JS, et al: Placental
Chorioangioma (update). In Cunningham FG, Leveno KL, Bloom SL, et al (eds):
Williams Obstetrics, 23rd ed. Online. Accessmedicine.com. New York, McGraw-Hill,
2009, Figure 15.

a. Syphilis
b. Diabetes mellitus
c. Fatty liver o pregnancy
d. Gestational trophoblastic neoplasia

6–8. Extrachorial placentation describes which o the


ollowing structural abnormalities?
a. Amnion rupture
b. Total chorion sur ace area signi icantly exceeds
that o the amnion a. Subamnionic hematoma
c. Excessive olds o amnion are present at the cord b. Maternal loor in arction
insertion site
c. Fetal thrombotic vasculopathy
d. Placental basal plate sur ace area signi icantly
exceeds that o the chorionic plate d. Subchorionic ibrin deposition

6–11. Which o the ollowing is most consistently associated


with poor etal outcomes such as miscarriage, growth
restriction, preterm birth, and stillbirth?
a. Subamnionic hematoma
b. Maternal loor in arction
c. Perivillous ibrin deposition
d. Subchorionic ibrin deposition
38 Pla centa tion, Embryogenesis, a nd Feta l Development

6–12. Which o the ollowing is most consistently 6–16. Which o the ollowing cancers most requently
associated with placental abruption? metastasizes to the placenta?
a. Subamnionic hematoma a. Colon
S
b. Subchorial thrombosis b. Gastric
E
C
c. Retroplacental hematoma c. Ovarian
T
I
O
d. Perivillous ibrin deposition d. Melanoma
N
3
6–13. The ollowing placental tumor was ound during 6–17. Which route o bacterial inoculation causes most
routine etal anatomic survey. The di erential diagnosis cases o chorioamnionitis?
includes chorioangioma, placental hematoma, partial a. Hematogenous spread rom maternal blood
hydatidi orm mole, teratoma, tumor metastasis, and
b. Direct spread through the allopian tubes
leiomyoma. What is most commonly irst employed to
help distinguish among these? c. Ascension rom the lower reproductive tract
d. Needle inoculation during intraamnionic procedures

6–18. Which o the ollowing risk actors is most


commonly associated with chorioamnionitis?
a. Maternal drug abuse
b. Poor maternal hygiene
c. Prior cesarean delivery
d. Prolonged rupture o membranes

6–19. With chorioamnionitis, etal contact with bacteria


through which o the ollowing routes may lead to
etal in ection?
a. Aspiration
b. Swallowing
c. Hematogenous
Reproduced with permission rom Hof man BL, Dashe JS: Placental chorioangioma d. All o the above
(update). In Cunningham FG, Leveno KL, Bloom SL, et al (eds): Williams Obstetrics,
22nd ed. Online. Accessmedicine.com. New York, McGraw-Hill, 2009, Figure 9.
6–20. The multiple, small, raised lesions o amnion
a. Color Doppler imaging nodosum are most commonly associated with which
o the ollowing?
b. Chorionic villus sampling
a. Oligohydramnios
c. Magnetic resonance imaging
b. Chorioamnionitis
d. Three-dimensional sonography
c. Meconium staining
6–14. Which o the ollowing is true regarding calcium d. Placental abruption
deposits within the placenta?
a. These most commonly orm just beneath the 6–21. Sonographically, an amnionic sheet or band may
chorionic plate. re lect several di erent clinical conditions. Which o
the ollowing poses the greatest etal risk?
b. Increasing calcium deposits should prompt
delivery at 39 weeks’ gestation. a. Amnionic band
c. Deposits are associated with advancing gestation, b. Amniochorion o a vanishing twin
nulliparity, and smoking. c. Amniochorion o a circumvallate placenta
d. Grannum grade 3 lesions are consistently linked with d. Amniochorion o a pregnancy in one horn o a
intrapartum etal acidosis and low Apgar scores. partial bicornuate uterus

6–15. Which o the ollowing is most consistently 6–22. A short umbilical cord may be associated with which
associated with large chorioangiomas? o the ollowing perinatal outcomes?
a. Fetal hydrops a. Intrapartum distress
b. Pulmonary embolism b. Fetal-growth restriction
c. Severe preeclampsia c. Congenital mal ormations
d. Gestational diabetes d. All o the above
Pla centa l Abnorma lities 39

6–23. A long umbilical cord may be more commonly 6–27. A single umbilical cord cyst is ound during a
associated with which o the ollowing? irst-trimester scan per ormed or vaginal bleeding.
a. Cord prolapse What is the most reasonable next step?

C
b. Cord alse knots a. Chorionic villus sampling

H
b. Ultrasound-guided percutaneous cyst drainage

A
c. Cord pseudocysts

P
c. Repeated sonographic evaluation in the second

T
d. Velamentous insertion

E
trimester

R
6
6–24. The number o complete coils per centimeter o cord d. Counsel regarding pregnancy termination o this
length has been termed the umbilical coiling index. anomalous pregnancy
Which o the ollowing is true o cord coiling?
a. A normal coiling index in a postpartum cord 6–28. This cord insertion variant is most commonly
approximates 1.2. associated with a higher rate o which o the ollowing?
b. Hypocoiling has not been associated with adverse
etal outcomes.
c. Hypercoiling has been associated with greater
rates o intrapartum etal acidosis.
d. All o the above

6–25. A transverse-plane sonographic image o the lower


abdomen rom a 17-week etus is shown here. What
is the most reasonable next step?

a. Cord avulsion
b. Fetal anomalies
c. Uterine inversion
d. Single umbilical artery

6–29. In which o the ollowing clinical settings is the cord


insertion variant seen here most likely to develop?

a. Fetal karyotyping
b. Continued routine prenatal care only
c. Detailed sonographic anatomic survey
d. Radio requency ablation o this anomaly

6–26. Sectioning o a term umbilical cord a ter delivery


may reveal embryonic remnants. Which o the
ollowing would not be possibly ound?
a. Wol ian duct
b. Vitelline duct
c. Allantoic duct
d. Second umbilical vein Used with permission rom Dr. David Nelson.

a. Multi etal gestation


b. Fetus with trisomy 21
c. Concurrent cocaine substance abuse
d. Poorly controlled chronic hypertension prior to
conception
40 Pla centa tion, Embryogenesis, a nd Feta l Development

6–30. The cord insertion abnormality seen in Question 6–32. A true knot in the umbilical cord is associated with
6-28 may more commonly be associated with which an increased risk o which o the ollowing etal
o the ollowing complications? complications?
a. Funisitis a. Stillbirth
S
E
C
b. Vasa previa b. Cerebral palsy
T
I
c. Placenta percreta c. Chromosomal anomalies
O
N
d. Maternal loor in arction d. Amnionic band sequence
3
6–31. At 28 weeks’ gestation, this inding is noted during 6–33. This umbilical cord inding is more commonly
subsequent sonographic evaluation o a placenta associated with which o the ollowing etal
previa that was identi ied earlier at 18 weeks. complications?
The patient is asymptomatic. What is the most
reasonable next step?

a. Stillbirth
Used with permission rom Dr. Jodi Dashe. b. Trisomy 13
c. Cardiac anomalies
a. Immediate cesarean delivery d. None o the above
b. Cesarean delivery at 39 weeks’ gestation
c. Cesarean delivery at 34 to 35 weeks’ gestation
d. Administer corticosteroids to promote lung
maturation and per orm cesarean delivery
24 hours later
Pla centa l Abnorma lities 41

CHAPTER 6 ANSw ER KEy

Q uestion Letter Pa ge

C
number a nswer cited Hea der cited

H
A
6–1 a p. 117 Ta ble 6 -1

P
T
6–2 b p. 116 N orma l Pla centa

E
R
6–3 a p. 116 N orma l Pla centa

6
6–4 a p. 117 Sha pe a nd Size
6–5 b p. 117 Sha pe a nd Size
6–6 d p. 117 Sha pe a nd Size
6–7 c p. 117 Sha pe a nd Size
6–8 d p. 118 Extra choria l Pla centa tion
6–9 d p. 118 Extra choria l Pla centa tion
6–10 d p. 119 Subchorionic Fibrin Deposition
6–11 b p. 119 Ma terna l Floor Infa rction
6–12 c p. 119 Hema toma
6–13 a p. 120 Suba mnionic Hema toma
6–14 c p. 120 Pla centa l Ca lcifica tion
6–15 a p. 120 Chorioa ngioma
6–16 d p. 121 Tumors Meta sta tic to the Pla centa
6–17 c p. 121 Chorioa mnionitis
6–18 d p. 121 Chorioa mnionitis
6–19 d p. 121 Chorioa mnionitis
6–20 a p. 121 O ther Membra ne Abnorma lities
6–21 a p. 121 O ther Membra ne Abnorma lities
6–22 d p. 121 Length
6–23 a p. 121 Length
6–24 c p. 121 Coiling
6–25 c p. 122 Vessel N umber
6–26 a p. 122 Remna nts a nd Cysts
6–27 c p. 122 Remna nts a nd Cysts
6–28 a p. 122 Insertion
6–29 a p. 122 Insertion
6–30 b p. 122 Insertion
6–31 c p. 123 Va sa Previa
6–32 a p. 123 Knots, Strictures, a nd Loops
6–33 d p. 123 Knots, Strictures, a nd Loops
42

CHAPTER 7

Embr ogenesis and Fetal Morphological Development

7–1. A patient rep rts that the irst day her last 7–7. C rresp nding with midbrain maturati n, when d
menstrual peri d was September 19th. Based n eye m vements begin?
Naegele rule, her due date is whi h the ll wing? a. 10–12 weeks
a. July 10th b. 12–14 weeks
b. June 14th c. 14–16 weeks
c. June 26th d. 16–18 weeks
d. De ember 12th
7–8. At 28 weeks’ gestati n, what is the han e survival
7–2. Pregnan y an be divided int three units r with ut physi al r neur l gi al impairment?
trimesters ea h lasting h w many weeks? a. 10%
a. 12 b. 25%
b. 13 c. 50%
c. 15 d. 90%
d. 16
7–9. All EXCEPT whi h the ll wing pass thr ugh
7–3. At the beginning whi h week ll wing pla ental tissue by simple di usi n?
ertilizati n is a n eptus termed an embry ? a. IgG
a. 1st week b. Water
b. 2nd week c. Oxygen
c. 3rd week d. Anestheti gases
d. 4th week
7–10. What is the average xygen saturati n intervill us
7–4. H w many weeks d es the embry ni peri d last? spa e bl d?
a. 4 a. 10–20%
b. 8 b. 30–35%
c. 10 c. 65–75%
d. 12 d. 90–95%

7–5. During whi h week devel pment is the primitive 7–11. Whi h the ll wing statements regarding the
heart partiti ned? trans er arb n di xide a r ss the pla enta is true?
a. 4th week a. Carb n di xide traverses the h ri ni villus
b. 6th week m re sl wly than xygen.
c. 8th week b. Fetal bl d has m re a inity r arb n di xide
than maternal bl d.
d. 10th week
c. he partial pressure arb n di xide in the
7–6. At 10 weeks’ gestati n based n the last menstrual umbili al arteries averages 50 mm Hg.
peri d, all EXCEPT whi h the ll wing are true? d. Mild maternal hyp ventilati n results in a all
a. Arms bend at the elb ws. in Pco 2 levels, av ring a trans er arb n
di xide r m the etal mpartment t maternal
b. Cr wn-rump length is 7 m.
bl d.
c. he upper lip is mplete.
d. Heart is mpletely rmed.
Embryogenesis a nd Feta l Morphologica l Development 43

7–12. Whi h the ll wing is und in higher 7–18. A ter birth, the intraabd minal remnants the
n entrati ns in the m ther than the etus? umbili al vein rm whi h the ll wing?
a. Ir n a. Ligamentum teres

C
b. Zin b. Ligamentum ven sum

H
A
c. As rbi a id c. Ligament reitz

P
T
d. Human pla ental la t gen d. Umbili al ligaments

E
R
7
7–13. Whi h the ll wing maternal immun gl bulins 7–19. he rder in whi h hem p iesis is seen in the
(Ig) rea h the etus? embry / etus r m earliest t latest is whi h the
a. Only IgA ll wing?
b. Only IgG a. Liver, y lk sa , b ne marr w
c. Only IgM b. Y lk sa , liver, b ne marr w
d. IgA and IgG c. B ne marr w, liver, y lk sa
d. Y lk sa , b ne marr w, liver
7–14. H w is i dide transp rted a r ss the pla enta?
a. End yt sis 7–20. Whi h the ll wing has the l west erythr yte
mean ell v lume (MCV)?
b. Simple di usi n
a. Embry
c. Carrier-mediated pr ess
b. erm etus
d. I dide d es n t r ss the pla enta
c. Aneupl id etus
7–15. In early pregnan y, amni ni luid is mp sed d. he MCV remains the same thr ugh ut gestati n
whi h the ll wing?
a. Fetal urine 7–21. A etus weighs 3000 g at term. What is the expe ted
et pla ental bl d v lume?
b. Fetal pulm nary luid
a. 125 mL
c. Ultra iltrate maternal plasma
b. 250 mL
d. Extra ellular luid that di uses thr ugh etal skin
c. 375 mL
7–16. Amni ni luid v lume peaks at what gestati nal d. 500 mL
age?
a. 24 weeks 7–22. Where is etal hem gl bin F pr du ed?
b. 28 weeks a. Liver
c. 34 weeks b. Y lk sa
d. 38 weeks c. B ne marr w
d. Y lk sa and liver
7–17. Whi h the ll wing statements regarding the
etal ardi vas ular system is true? 7–23. In ants attain adult levels IgM at what age?
a. Fetal heart hambers w rk in series. a. 3 m nths
b. he p rtal sinus is the maj r bran h the b. 6 m nths
umbili al vein. c. 9 m nths
c. Oxygen is delivered r m the pla enta by the d. 12 m nths
umbili al artery.
d. he du tus ven sus traverses the liver t enter the
in eri r vena ava dire tly.
44 Pla centa tion, Embryogenesis, a nd Feta l Development

7–24. Whi h the ll wing sequen es rre tly identi ies 7–26. Whi h the ll wing sequen es rre tly identi ies
the b nes labeled in the image? the dimensi ns labeled in this image?

C
S
E
C
T
I
O
N
B B
3
B

A A

C
M dif ed with permissi n r m Cunningham FG, Leven KJ, Bl m SL, et al (eds):
Embry genesis and Fetal M rph l gi al Devel pment. In Williams Obstetri s, 24th ed.
New Y rk, M Graw-Hill, 2014, Figure 7-11.
M dif ed with permissi n r m Cunningham FG, Leven KJ, Bl m SL, et al (eds): Fetal
Gr wth and Devel pment. In Williams Obstetri s, 22nd ed. New Y rk, M Graw-Hill,
a. A is the r ntal b ne, B is the parietal b ne, C is 2005, Figure 4-9.
the ipital b ne.
b. A is the parietal b ne, B is the ipital b ne, C a. A is sub ipit bregmati , B is ipit r ntal,
is the r ntal b ne. C is ipit mental.
c. A is the temp ral b ne, B is the r ntal b ne, C is b. A is ipit r ntal, B is sub ipit bregmati ,
the ipital b ne. C is ipit mental.
d. A is the r ntal b ne, B is the temp ral b ne, C is c. A is ipit mental, B is ipit r ntal, C is
the parietal b ne. sub ipit bregmati .
d. A is ipit r ntal, B is ipit mental, C is
7–25. In the igure Questi n 7-24, the suture b rdered sub ipit bregmati .
by the b nes labeled A and B is whi h the
ll wing? 7–27. Whi h the statements regarding etal swall wing
a. C r nal suture is true?
b. Lambd id suture a. Swall wing begins at 20 weeks’ gestati n.
c. Fr ntal suture b. erm etuses swall w between 200 and 760 mL
per day.
d. Sagittal suture
c. I swall wing is inhibited in late pregnan y,
lig hydramni s will ur.
d. Swall wing greatly a e ts amni ni luid v lume,
parti ularly in early pregnan y.

7–28. he limits etal viability are determined by whi h


the ll wing pr esses?
a. Pulm nary gr wth
b. Kidney rmati n
c. Hepati devel pment
d. Fetal immun mpeten e
Embryogenesis a nd Feta l Morphologica l Development 45

7–29. A 25-year- ld G1P0 presents with rupture 7–31. he hild bel w has a webbed ne k, streak g nads,
membranes at 19 weeks. She subsequently delivers. and genital in antilism. Her m ther rep rts that
On hist l gi al evaluati n the lungs the when pregnant with her, her ultras und was
n nviable etus, y u w uld expe t whi h the abn rmal, with luid lle ting n the ba k the

C
H
ll wing? hild’s ne k. he m st likely diagn sis is whi h

A
the ll wing?

P
a. Mature alve li

T
E
b. N artilage devel pment

R
c. Presen e terminal sa s

7
d. N rmal br n hial bran hing

7–30. Y u deliver an in ant with ambigu us genitalia.


During examinati n, y u n te a small phallus that
y u suspe t is lit ral hypertr phy. A ph t graph is
pr vided bel w. Y u unsel the m ther and rder
kary typing. he kary type is 46,XY. he m st
likely diagn sis is whi h the ll wing?

Repr du ed with permissi n r m Fuster V, Walsh RA, Harringt n RA (eds): Hurst’s T e


Heart, 13th ed. New Y rk, M Graw-Hill, 2011, Figure 14-14.

Repr du ed with permissi n r m Cunningham FG, Leven KJ, Bl m SL, et al (eds):


a. G nadal dysgenesis
Fetal gr wth and devel pment. In Williams Obstetri s, 23rd ed. New Y rk, M Graw-Hill, b. rue hermaphr ditism
Figure 4-20.
c. Rei enstein syndr me
a. rue hermaphr ditism d. Andr gen insensitivity syndr me
b. Fetal ar matase de i ien y
c. Male pseud hermaphr ditism
d. C ngenital adrenal hyperplasia
46 Pla centa tion, Embryogenesis, a nd Feta l Development

7–32. A w man presents t lab r and delivery a ter


having a vaginal birth at h me. She has had n
prenatal are. A ter attending t her, y u examine
the newb rn. A ph t graph is pr vided. Y u n te
S
E
a hemis r tum am ng ther things. he likely
C
T
diagn sis is whi h the ll wing?
I
O
N
3
Repr du ed with permissi n r m Cunningham FG, Leven KJ, Bl m SL, et al (eds):
Fetal gr wth and devel pment. In Williams Obstetri s, 23rd ed. New Y rk, M Graw-
Hill, Figure 4-21B.

a. rue hermaphr ditism


b. Fetal ar matase de i ien y
c. Male pseud hermaphr ditism
d. C ngenital adrenal hyperplasia
Embryogenesis a nd Feta l Morphologica l Development 47

CHAPTER 7 ANSw ER KEy

Q uestion Letter Pa ge

C
number a nswer cited Hea der cited

H
A
7–1 c p. 127 G esta tiona l Age Va riously Defined

P
T
7–2 b p. 127 G esta tiona l Age Va riously Defined

E
R
7–3 c p. 128 Embryonic Period

7
7–4 c p. 128 Embryonic Period
7–5 a p. 128 Embryonic Period
7–6 b p. 128 Embryonic Period
7–7 d p. 129 1 6 G esta tiona l W eeks
7–8 d p. 129 2 8 G esta tiona l W eeks
7–9 a p. 132 Mecha nisms of Tra nsfer
7–10 c p. 132 Tra nsfer of O xygen a nd Ca rbon Dioxide
7–11 c p. 132 Tra nsfer of O xygen a nd Ca rbon Dioxide
7–12 d p. 133 G lucose a nd Feta l G rowth
7–13 b p. 134 Proteins
7–14 c p. 134 Ions a nd Tra ce Meta ls
7–15 c p. 135 Amnionic Fluid Forma tion
7–16 c p. 135 Amnionic Fluid Forma tion
7–17 d p. 135 Ca rdiova scula r System
7–18 a p. 137 Circula tory Cha nges a t Birth
7–19 b p. 137 Hemopoiesis
7–20 b p. 137 Hemopoiesis
7–21 c p. 137 Fetopla centa l Blood Volume
7–22 a p. 137 Feta l Hemoglobin
7–23 c p. 139 Immunoglobulin M a nd A
7–24 a p. 139 Skull
7–25 a p. 139 Skull
7–26 a p. 139 Skull
7–27 b p. 140 G a strointestina l System
7–28 a p. 142 Ana tomica l Ma tura tion
7–29 d p. 142 Ana tomica l Ma tura tion
7–30 c p. 148 G enita l Ambiguity of the N ewborn
7–31 a p. 148 G enita l Ambiguity of the N ewborn
7–32 a p. 148 G enita l Ambiguity of the N ewborn
This page intentionally left blank
Se c t i o n 4

PRECONCEPTIONALAND
PRENATALCARE
50

CHAPTER 8

Preconceptional Counseling

8–1. Preconceptional counseling involves collection o 8–5. A 23-year-old patient tells her gynecologist that she
in ormation regarding previous pregnancy outcomes, wants to have a child in the next ew years. She is
medical conditions, and amily history. What is the concerned as she has a seizure disorder and takes
optimal method o collecting this in ormation? valproic acid. It has been 1 year since her last seizure,
a. Nurse visit and she has heard that seizures are dangerous or the
etus. What can you tell her regarding her seizure
b. Paper intake orm
risk during pregnancy?
c. Online questionnaire
a. Because o the length o time she has been
d. Combined questionnaire plus interview seizure- ree, it is reduced 50%.
b. I she delays pregnancy or an additional
8–2. A 30-year-old woman with no known medical
12 months, her risk will be reduced by 50%.
conditions is noted to have 3+ glucosuria at her irst
prenatal visit. A asting blood glucose level is c. here is an inverse relationship between the time
144 mg/dL. How should she be counseled regarding she has been seizure- ree be ore pregnancy and
her risk or etal anomalies? her risk during pregnancy.
a. Her risk is the same as other 30-year-old women. d. None o the above
b. Her risk is twice as high as other 30-year-old
8–6. For the patient in Question 8-5, what supplement
women.
should she begin prior to attempting conception?
c. Her risk is our old higher than other 30-year-old
a. Iron
women.
b. Folate
d. Her risk is 10 times higher than other 30-year-old
women. c. Niacin
d. Vitamin D
8–3. Preconceptional evaluation o a woman with diabetes
mellitus should include all EXCEPT which o the 8–7. Which o the ollowing antiseizure medications,
ollowing? when taken as monotherapy, is associated with the
a. Hemoglobin A1c highest rate o major congenital mal ormations?
b. Retinal examination a. Phenytoin
c. Bone density testing b. Valproic acid
d. 24-hour urine collection c. Phenobarbital
d. Carbamazepine
8–4. All EXCEPT which o the ollowing can be
expected ollowing preconceptional counseling and 8–8. Which o the ollowing is not an example o a killed
its implementation in women with pregestational bacterial or viral vaccine and there ore should not be
diabetes? administered during pregnancy?
a. Decreased perinatal death rate a. Rabies
b. Reduced congenital anomaly rate b. etanus
c. Improved preconceptional olic acid use c. Meningococcus
d. Decreased need or antihypertensive therapy d. Varicella-zoster
Preconceptiona l Counseling 51

8–9. A woman presents or a screening sonographic 8–12. he incidence o etal cardiac abnormalities is
evaluation at 21 weeks’ gestation. A etal leg is increased in all EXCEPT which o the ollowing
imaged below. She and her partner ask about the maternal conditions?
requency o birth de ects in the general population.

C
a. Lead exposure

H
What is the correct response to their question?
b. Phenylketonuria

A
P
c. Diabetes mellitus

T
E
R
d. Methylene tetrahydro olate reductase mutation

8
carrier

8–13. What percent o etuses with the ollowing condition


are born to women at low risk or the anomaly?

a. 0.1%
b. 1%
c. 3%
d. 11%

8–10. Birth de ects are responsible or what percentage o Used with permission rom Dr. if any Woodus.
in ant mortality?
a. 2% a. 10%
b. 5% b. 50%
c. 10% c. 75%
d. 20% d. 90%

8–11. What is the signi icance o the individual identi ied 8–14. Preconception supplementation and orti ication o
by the arrow in the ollowing image? some ood products with olate has had what impact
on pregnancy outcomes?
1 2 a. Reduced the rate o preeclampsia
I
b. Reduced the incidence o childhood seizures
c. Reduced the incidence o neural-tube de ects
1 2 3
II d. Reduced the incidence o spontaneous abortions

Reproduced with permission rom Cunningham FG, Leveno KJ, Bloom SL, et al (eds):
Preconceptional counseling. In Williams Obstetrics, 24th ed. New York, McGraw-Hill,
2014, Fig. 8-2.

a. She is one o a triplet gestation.


b. She is a ected by the condition in question.
c. She is being evaluated or a suspected condition.
d. She is the only one in her amily una ected by
the condition in question.
52 Preconceptiona l a nd Prena ta l Ca re

8–15. A woman with phenylketonuria presents to her 8–18. A 33-year-old multipara presents to Labor and
primary care physician’s o ice 1 week a ter her Delivery complaining o decreased etal movement
missed menses. A home urine pregnancy test was or 2 days at 33 weeks’ gestation. Sonographic
positive. he pregnancy was unplanned, and her evaluation con irms the diagnosis o etal demise.
S
E
blood phenylalanine level is 1012 µmol/L, which No other abnormalities are noted. What percentage
C
T
is essentially unchanged rom her baseline levels o stillbirths who undergo standard karyotyping will
I
O
over the past several months. How should she be have a chromosomal abnormality detected?
N
counseled regarding about her pregnancy outcomes? a. 0.1%
4
a. Her risk or congenital anomalies is not increased. b. 1%
b. here is a 50% chance or neurological c. 13%
impairment in her etus.
d. 21%
c. he risk or congenital cardiac abnormalities in
her etus is 12%. 8–19. What bene it does chromosomal microarray analysis
d. Her risk or a spontaneous abortion in the irst have over standard karyotyping in the evaluation o
trimester is doubled. stillbirth?
a. Can be per ormed on maternal blood
8–16. A 23-year-old woman presents to the hospital in
b. Can be per ormed on nonviable tissue
active labor and delivers the in ant seen below. She
had received no prenatal care and upon questioning c. Detects lower levels o tissue mosaicism
admits she has some medical problems. Which o d. None o the above
the ollowing conditions is most likely responsible
or the in ant’s condition? 8–20. All EXCEPT which o the ollowing obstetrical
complications is increased in adolescent pregnancies
compared with women aged 20 to 35 years?
a. Anemia
b. Preeclampsia
c. Preterm labor
d. Postpartum hemorrhage

8–21. For women older than 40 years, pregnancy-related


mortality rates are increase by what magnitude
compared with women in their twenties?
a. wo old
b. Five old
c. Eight old
d. en old

8–22. Which o the ollowing is the most common


a. Gaucher disease pregnancy complication in women older than
b. Phenylketonuria 35 years?
c. Diabetes mellitus a. Diabetes
d. None o the above b. Hypertension
c. Preterm birth
8–17. What are the most common single-gene disorders d. Low birthweight
worldwide?
a. Hemophilias
b. halassemias
c. Cystic ibrosis mutations
d. Glycogen storage diseases
Preconceptiona l Counseling 53

8–23. In 2005, what percentage o all multi etal gestations 8–28. What percent o women giving birth in the United
were due to assisted reproductive technologies? States are smokers?
a. 20% a. 2%

C
b. 30% b. 6%

H
A
c. 40% c. 14%

P
T
d. 50% d. 24%

E
R
8
8–24. Which o the ollowing assisted reproductive 8–29. Pregnant women should limit their weekly dietary
technologies is associated with a higher rate o intake o canned tuna to 12 ounces and should
congenital abnormalities in the etus? eliminate consumption o certain kinds o ish such
a. Cryopreservation as mackerel and sword ish to minimize exposure to
which etal neurotoxin?
b. Ovulation induction
c. In vitro ertilization a. Cadmium
d. Intracytoplasmic sperm injection b. Methyl mercury
c. Organic phosphate
8–25. An in ant with achondroplasia is born to a 41-year- d. Algae-related toxin
old diabetic woman and her 51-year-old husband
who uses testosterone supplementation. Which o 8–30. A 19-year-old G3P2 at 28 weeks’ gestation is
the actors in their backgrounds is most likely related embarrassed to admit she has been craving and
to the in ant’s condition? eating ice and dirt. She thinks she must be “going
a. Maternal age crazy.” What test is likely to be abnormal?
b. Paternal age a. oxicology screen
c. Maternal diabetes b. Liver unction tests
d. Paternal sex steroid use c. Complete blood count
d. Electroencephalogram
8–26. Which o the ollowing obstetrical complications is
not increased in obese patients?
a. Hypertension
b. Fetal anomalies
c. Cesarean delivery
d. Group B streptococcal carrier requency

8–27. A 23-year-old primigravida who is seeking pregnancy


presents or her annual examination, and you
ascertain that she smokes approximately one pack
o cigarettes daily. She is considering pregnancy
and is concerned about the e ect o tobacco use on
pregnancy outcomes. Which o the ollowing is an
accurate statement?
a. Her risk o preterm delivery continues to all
during the irst 6 months she abstains rom
cigarette use.
b. Her risk or pulmonary edema complicating
preeclampsia remains higher than that o the
nonsmoking population.
c. he risk o etal-growth restriction should be no
higher that the general population i she quits
be ore she conceives.
d. None o the above
54 Preconceptiona l a nd Prena ta l Ca re

8–31. What diagnostic test should be per ormed either 8–33. You have recently taken the practice rom a retiring
prior to pregnancy or early in prenatal care to assess obstetrician and are meeting a patient or her irst
the risk to the etus and the pregnancy rom the prenatal appointment. In reviewing her old chart,
disease that is typi ied by the blood smear shown in you see the ollowing photo and notes that state her
S
E
this image? husband was responsible. Why are you concerned or
C
T
this patient?
I
O
N
4
Reproduced with permission rom Longo DL: Atlas o hematology and analysis o peripheral
blood smears. In Longo DL, Fauci AS, Kasper DL, et al (eds): Harrison’s Principles o
Internal Medicine, 18th ed. New York, McGraw-Hill, 2012, Figure e17-12.

a. Serum erritin
b. Antiglobulin test
c. Indirect Coombs test
d. Hemoglobin electrophoresis

8–32. Ashkenazi Jewish individuals should be o ered


preconceptional screening or all EXCEPT which o
the ollowing?
a. Canavan disease
b. Cystic ibrosis
c. Beta thalassemia
d. ay-Sachs disease
Reproduced with permission rom Knoop KJ, Stack LB, Storrow AB, et al (eds): T e Atlas
o Emergency Medicine, 3rd ed. New York, McGraw-Hill, 2010, Figure 1-3.

a. Rate o success ul breast eeding is reduced.


b. Risk or violence against your patient is higher
during pregnancy.
c. here is reduced bonding between the ather and
in ant in the setting o domestic violence.
d. None o the above
Preconceptiona l Counseling 55

CHAPTER 8 ANSw ER KEy

Q uestion Letter Pa ge

C
number a nswer cited Hea der cited

H
A
8–1 d p. 156 Counseling Session

P
T
8–2 c p. 157 Dia betes Mellitus

E
R
8–3 c p. 163 Ta ble 8 -4

8
8–4 d p. 157 Dia betes Mellitus
8–5 a p. 158 Epilepsy
8–6 b p. 158 Epilepsy
8–7 b p. 158 Ta ble 8 -2
8–8 d p. 158 Immuniza tions
8–9 c p. 159 G enetic Disea se
8–10 d p. 159 G enetic Disea ses
8–11 b p. 160 Figure 8 -2
8–12 a p. 159 N eura l-Tube Defects
8–13 d p. 159 N eura l-Tube Defects
8–14 c p. 159 N eura l-Tube Defects
8–15 a p. 159 Ta ble 8 -3
8–16 b p. 159 Ta ble 8 -3
8–17 b p. 160 Tha la ssemia s
8–18 c p. 161 Reproductive History
8–19 b p. 161 Reproductive History
8–20 d p. 161 Ma terna l Age
8–21 b p. 161 Ma terna l Age
8–22 a p. 161 Figure 8 -3
8–23 c p. 161 Assisted Reproductive Technologies
8–24 d p. 161 Assisted Reproductive Technologies
8–25 b p. 162 Pa terna l Age
8–26 d p. 162 Diet
8–27 c p. 162 Recrea tiona l Drugs a nd Smoking
8–28 c p. 162 Recrea tiona l Drugs a nd Smoking
8–29 b p. 163 Ta ble 8 -4
8–30 c p. 162 Diet
8–31 d p. 163 Ta ble 8 -4
8–32 c p. 161 Individua ls of Ea stern Europea n Jewish Descent
8–33 b p. 163 Intima te Pa rtner Violence
56

CHAPTER 9

Prenatal Care

9–1. Which of the following statements accurately 9–5. A 29-year-old primigravida with an unknown last
describes racial differences in prenatal care usage? menstrual period presents complaining of vaginal
a. Hispanic women are least likely to obtain care. spotting and cramping. Transvaginal sonographic
examination is performed as part of her evaluation.
b. Ten percent of African American women receive
One image is shown here. This finding would
no prenatal care.
represent a pregnancy of approximately what
c. Minority women now access prenatal care as gestational age?
readily as nonminority women.
d. Minority women have made the largest gains
in timely access to prenatal care during the past
2 decades.

9–2. Common reasons cited by women as barriers to


enrolling in prenatal care include which of the
following?
a. Lack of funding
b. Late identification of pregnancy
c. Inability to obtain an appointment
d. All of the above

9–3. Fetal movements are typically first perceived by the


mother at approximately what gestational age?
a. 8 weeks
Used with permission from Dr. Elysia Moschos.
b. 14 weeks
c. 16 weeks
d. 22 weeks a. 3 weeks
b. 5 weeks
9–4. A false-positive human chorionic gonadotropin c. 7 weeks
(hCG) test result due to circulating heterophilic
antibodies is most likely to occur in which
d. 9 weeks
individuals?
9–6. Referring to the patient and image described in
a. Women carrying a twin gestation Question 9-5, which of the following increases the
b. Women with a history of a molar pregnancy certainty that this finding represents an intrauterine
c. Women who have worked closely with animals pregnancy?
d. Women with autoimmune conditions such as a. An echogenic rim along one side of the decidua
systemic lupus erythematosus b. Two concentric echogenic rings surrounding the
sac
c. A sac positioned eccentrically within the
endometrium
d. All of the above
Prena ta l Ca re 57

9–7. Referring to the patient described in Question 9–12. Approximately how often will delivery dates be
9-5, an examination was performed and identified estimated incorrectly using the device shown in the
a closed cervical os and no bleeding. Bimanual image below?
examination was benign. What is the most

C
H
appropriate management plan?

A
P
a. Dilation and curettage

T
E
b. Methotrexate administration

R
c. Obtain serial serum β -hCG levels

9
d. Repeat sonographic examination in 48 hours

9–8. Provided it occurs in the first trimester, all EXCEPT


which of the following are typically performed or
obtained at the initial prenatal care evaluation?
a. Urine culture
b. Neural-tube defect screening
c. Complete physical examination
d. Blood type and antibody screen

9–9. Which of the following women could be classified as


a nulligravida?
a. A 30-year-old who has never been pregnant
before
b. A 23-year-old who is pregnant for the first time a. 10%
at 22 weeks’ gestation b. 30%
c. A 25-year-old who is 6 weeks postpartum after c. 50%
her first term delivery d. 70%
d. A 34-year-old who has two previous pregnancies
that ended in miscarriages at 8 weeks’ gestation 9–13. The accuracy of gestational age dating using the
last menstrual period is affected by which of the
9–10. How should a woman who has had 4 pregnancies following?
delivered at term, one of which was a twin a. Anovulatory bleeding
pregnancy, be designated?
b. Menstrual cycle length
a. Gravida 5 para 4
c. Oral contraceptive use
b. Gravida 4 para 5
d. All of the above
c. Gravida 4 para 4
d. Gravida 5 para 5 9–14. Which groups of women should receive psychosocial
screening as a part of their prenatal care?
9–11. Adding 7 days to the first day of the last menstrual a. All women
period and counting back 3 months to estimate the
b. Women who use illicit substances
day of delivery is termed what?
c. Women at risk for domestic violence
a. Hegar rule
d. Women of minority race or ethnicity
b. Naegele rule
c. Kessner rule 9–15. All EXCEPT which of the following adverse
d. Chadwick rule outcomes have been linked to smoking in
pregnancy?
a. Preeclampsia
b. Preterm birth
c. Placental abruption
d. Sudden infant death syndrome
58 Preconceptiona l a nd Prena ta l Ca re

9–16. According to the Centers for Disease Control and 9–20. At least how much elemental iron should be given
Prevention, which of the following are characteristics daily as a supplement to pregnant women?
of women who are most likely to use alcohol in a. 15 mg
pregnancy?
S
b. 27 mg
E
a. Teenagers
C
c. 42 mg
T
b. Unemployed
I
O
d. 60 mg
N
c. College educated
4
d. African American race 9–21. Which of the following strategies may decrease side
effects from iron supplementation?
9–17. Intimate partner violence has been associated with a. Ingestion at bedtime
all EXCEPT which of the following untoward
b. Taking it on an empty stomach
pregnancy outcomes?
c. Avoiding it in the first trimester
a. Perinatal death
d. All of the above
b. Preterm delivery
c. Gestational hypertension 9–22. Which women who are planning a pregnancy should
d. Fetal-growth restriction receive 0.4–0.8 mg of supplemental folate?
a. All women
9–18. As demonstrated in the image below, fundal height
b. Those with proven folate deficiency
measurements in centimeters correlate closely
with gestational age between 20 and 34 weeks. All c. Those with a previous child affected by a neural-
EXCEPT which of the following can introduce error tube defect
into this measurement? d. Those who do not receive adequate dietary intake
of grains

9–23. A 29-year-old G1P1 was just delivered of a newborn


with a large posterior encephalocele as demonstrated
in the image. Assuming she takes the appropriate
dose of folate in the periconceptional period of her
next pregnancy, the risk for recurrence is decreased
by approximately what percentage?

Used with permission from Dr. Heather Lytle.

a. Obesity
b. An empty bladder
c. Uterine leiomyomas
d. Multifetal gestation

9–19. Which of the following statements is true regarding


the current recommendations for weight gain by the Used with permission from Dr. Heather Lytle.
Institute of Medicine?
a. Recommended weight gain differs by ethnicity. a. 20%
b. Adolescents should gain slightly more weight b. 40%
than adults.
c. 70%
c. Women of normal weight should gain between d. 90%
15 to 25 pounds.
d. Recommendations are stratified based on
prepregnancy body mass index.
Prena ta l Ca re 59

9–24. Doses of vitamin A in excess of what amount have 9–30. Air travel in pregnancy is not recommended after
been associated with congenital malformations? what gestational age?
a. > 1,000 IU per day a. 12 weeks

C
b. > 10,000 IU per day b. 22 weeks

H
A
c. > 100,000 IU per day c. 30 weeks

P
T
d. > 1,000,000 IU per day d. 36 weeks

E
R
9
9–25. Which of the following vitamins, when combined 9–31. Which of the following statements regarding
with the antihistamine doxylamine, has been found vaccinations in pregnancy is true?
to be helpful in cases of nausea and vomiting? a. Tdap should be given to all pregnant women
a. Vitamin C between 16 and 20 weeks.
b. Vitamin D b. All pregnant women should be offered influenza
c. Vitamin B6 vaccine during the appropriate season.
d. Vitamin B12 c. Varicella vaccine should be offered to all women
who have a chicken pox exposure in pregnancy.
9–26. Maternal deficiency of vitamin D has been associated d. Measles-mumps-rubella (MMR) vaccine should
with which of the following complications in the be given to all pregnant women who are rubella
offspring? nonimmune.
a. Anemia
9–32. The condition shown in the image is a common
b. Jaundice
complaint in pregnancy that results from increased
c. Seizures pelvic venous pressure. Treatment of this condition
d. Congenital rickets is typically conservative, but surgery can be required
in what situation?
9–27. Which of the following conditions would be
considered absolute contraindications to exercise in
pregnancy?
a. Mild hypertension
b. Restrictive lung disease
c. Placenta previa at 16 weeks’ gestation
d. All of the above

9–28. Which of the following types of fish should be


avoided in pregnancy due to potentially high
methylmercury levels?
a. Tuna
b. Salmon
c. Flounder
d. Swordfish a. If they are multiple

9–29. Which of the following is a blood lead concentration b. If they are thrombosed
threshold, above which would indicate lead c. If they are associated with vulvar varicosities
poisoning that requires treatment? d. None of the above
a. > 5 µg/dL
b. > 20 µg/dL
c. > 45 µg/dL
d. > 75 µg/dL
60 Preconceptiona l a nd Prena ta l Ca re

CHAPTER 9 ANSw ER KEy

Q uestion Letter Pa ge
S
number a nswer cited Hea der cited
E
C
T
9–1 d p. 167 Prena ta l Ca re in the United Sta tes
I
O
9–2 d p. 167 Assessing Prena ta l Ca re Adequa cy
N
9–3 c p. 169 Feta l Movement
4
9–4 c p. 169 Mea surement of hCG
9–5 b p. 170 Sonogra phic Recognition of Pregna ncy
9–6 d p. 170 Sonogra phic Recognition of Pregna ncy
9–7 c p. 170 Sonogra phic Recognition of Pregna ncy
9–8 b p. 171 Ta ble 9 -2
9–9 a p. 170 Definitions
9–10 c p. 170 Definitions
9–11 b p. 172 N orma l Pregna ncy Dura tion
9–12 c p. 172 N orma l Pregna ncy Dura tion
9–13 d p. 172 Previous a nd Current Hea lth Sta tus
9–14 a p. 172 Psychosocia l Screening
9–15 a p. 172 Ciga rette Smoking
9–16 c p. 173 Alcohol
9–17 c p. 174 Intima te Pa rtner Violence
9–18 b p. 176 Funda l Height
9–19 d p. 177 W eight G a in Recommenda tions; Ta ble 9 -5
9–20 b p. 179 Iron
9–21 d p. 179 Iron
9–22 a p. 181 Folic Acid
9–23 c p. 181 Folic Acid
9–24 b p. 181 Vita min A
9–25 c p. 181 Vita min B6—Pyridoxine
9–26 d p. 181 Vita min D
9–27 b p. 182 Ta ble 9 -7
9–28 d p. 183 Sea food Consumption
9–29 c p. 183 Lea d Screening
9–30 d p. 183 Automobile a nd Air Tra vel
9–31 b p. 184 Immuniza tion a nd Ta ble 9 -9
9–32 b p. 188 Va ricosities a nd Hemorrhoids
Se c t i o n 5

Th e FeTa l Pa Tien T
62

Ch a PTeR 10

F t im g g

10–1. Which of the following statements accurately 10–6. An anembryonic pregnancy may be accurately
describes the relationship between tissue penetration diagnosed at what mean gestational sac diameter
and image resolution in ultrasound? when using transvaginal ultrasound?
. Higher-frequency transducers yield better image . 5 mm
resolution. b. 10 mm
b. Lower-frequency transducers yield better image c. 15 mm
resolution.
d. 20 mm
c. Higher-frequency transducers penetrate tissue
more effectively. 10–7. A 41-year-old G3P2 presents at 12 weeks’ gestation
d. None of the above for a first-trimester sonographic evaluation. The
nuchal translucency is measured as shown in the
10–2. Although sonography is generally considered safe image and is noted to be increased at 4.6 mm. She
in human pregnancy, the potential for temperature subsequently undergoes chorionic villus sampling,
elevation is increased in which of the following and the fetal karyotype is 46,XY. Her fetus still
situations? needs to be evaluated in the second trimester for
. Third trimester which of the following?
b. Longer examination time
c. Near soft tissue rather than bone
d. All of the above

10–3. Documentation of embryonic or fetal cardiac activity


should be accomplished with which of the following
types of ultrasound?
. M-mode
b. L-mode
c. Color Doppler
d. Pulsed Doppler

10–4. What is the single most accurate biometric predictor


of gestational age?
. Crown-rump length
b. Head circumference
c. Abdominal circumference . Aneuploidy
d. Gestational sac mean diameter b. Cardiac defects
10–5. Sonographic evaluation of all EXCEPT which of the c. Duodenal atresia
following are best achieved in the first trimester? d. Cleft lip and palate
. Adnexa
b. Cervical length
c. Ectopic pregnancy
d. Chorionicity of twins
Feta l Ima ging 63

10–8. The fetal head circumference should be measured in 10–12. Which of the following fetal anomalies has few
which of the following views? sonographic findings and is typically not diagnosed
. Transatrial view antenatally?

C
b. Transthalamic view . Anencephaly

h
a
c. Transcerebellar view b. Hydrocephalus

P
T
d. Any of the above are acceptable c. Gastroschisis

e
R
d. Choanal atresia

1
10–9. Which of the biometric parameters measured in

0
the second trimester has the greatest variation for 10–13. Absence of the cavum septum pellucidum, as shown
gestational age estimation? in this ultrasound image, may be associated with all
. Femur length EXCEPT which of the following conditions?
b. Head circumference
c. Biparietal diameter
d. Abdominal circumference

10–10. A 39-year-old multipara with chronic hypertension


is noted to have lagging fundal growth at 29 weeks’
gestation. Sonographic evaluation is completed,
and the fetal weight estimate is less than the 3rd
percentile for this gestational age. According to
recommendations from the American Institute
of Ultrasound in Medicine, when would it be
appropriate to repeat sonographic evaluation of
interval fetal growth?
. 2 days
b. 1 week
c. 3 weeks Reproduced with permission from Cunningham FG, Leveno KJ, Bloom SL, et al (eds): Fetal
imaging. In Williams Obstetrics, 24th ed. New York, McGraw-Hill, 2014, eFigure 10-4.
d. 5 weeks

10–11. The following sonographic image is taken from a . Porencephaly


pregnancy in which polyhydramnios is suspected. b. Septo-optic dysplasia
The distance between the two calipers must exceed c. Lobar holoprosencephaly
what value to confirm this diagnosis? d. Agenesis of the corpus callosum

10–14. What is the most common class of fetal


malformations?
. Cardiac
b. Oral cleft
c. Neural tube
d. Ventral wall

10–15. In fetuses with spina bifida defects, the associated


cranial abnormalities include all EXCEPT which of
the following?
. Ventriculomegaly
b. Dandy-Walker malformation
Reproduced with permission from Cunningham FG, Leveno KJ, Bloom SL, et al (eds): Fetal
c. Scalloping of the frontal bones
imaging. In Williams Obstetrics, 23rd ed. New York, McGraw-Hill, 2010, Figure 16-5. d. Effacement of the cisterna magna

. 6 cm
b. 8 cm
c. 10 cm
d. 12 cm
64 The Feta l Pa tient

10–16. A 19-year-old primigravida presents at 20 weeks’ 10–18. Shown in the image below are the intracranial
gestation for a dating sonographic examination. findings of alobar holoprosencephaly. Fetal
The following fetal head image is obtained in the karyotyping is most likely to identify which
transatrial view. Overt or severe ventriculomegaly is aneuploidy? V= ventricle, Th = thalami.
S
e
diagnosed when the lateral ventricular atrial width
C
T
exceeds what measurement threshold?
i
O
n
5
Reproduced with permission from Cunningham FG, Leveno KJ, Bloom SL, et al (eds): Fetal
imaging. In Williams Obstetrics, 24th ed. New York, McGraw-Hill, 2014, Figure 10-10A.

Reproduced with permission from Cunningham FG, Leveno KJ, Bloom SL, et al (eds): Fetal
imaging. In Williams Obstetrics, 24th ed. New York, McGraw-Hill, 2014, Figure 10-8. . Trisomy 13
b. Trisomy 18
. 5 mm
c. Trisomy 21
b. 10 mm
d. Monosomy X
c. 12 mm
d. 15 mm 10–19. Caudal regression sequence, which is characterized
by absence of the sacral spine, is associated with
10–17. Again referring to the patient in Question 10-16, which of the following maternal conditions?
which of the following are appropriate as a part of . Epilepsy
the subsequent evaluation?
b. Diabetes mellitus
. Fetal karyotyping
c. Sickle-cell anemia
b. Tests for congenital infections
d. Systemic lupus erythematosus
c. Fetal magnetic resonance imaging
d. All of the above 10–20. Which of the following cleft abnormalities are most
associated with aneuploidy?
. Unilateral cleft lip
b. Bilateral cleft lip and palate
c. Unilateral cleft lip and palate
d. All are equally associated with aneuploidy
Feta l Ima ging 65

10–21. The following image depicts a 15-week fetus with 10–24. The following ultrasound image demonstrates a
massive cystic hygromas. When associated with normal four-chamber view of the fetal heart. Which
aneuploidy, which of the following fetal karyotypes of the following cardiac malformations may not be

C
is most likely? detected when only this view is obtained? LV = Left

h
ventricle, RV = right ventricle, LA = left atrium,

a
P
RA = right atrium, Ao = aorta.

T
e
R
1
0
Reproduced with permission from Cunningham FG, Leveno KJ, Bloom SL, et al (eds): Fetal
imaging. In Williams Obstetrics, 24th ed. New York, McGraw-Hill, 2014, Figure 10-16B.

Reproduced with permission from Cunningham FG, Leveno KJ, Bloom SL, et al (eds): Fetal
. 45,X imaging. In Williams Obstetrics, 24th ed. New York, McGraw-Hill, 2014, Figure 10-20B.

b. 47,XY,+ 21
. Ebstein anomaly
c. 47,XX,+ 18
b. Hypoplastic left heart
d. 47,XX,+ 16
c. Transposition of the great vessels
10–22. Where do most congenital diaphragmatic hernia d. Atrioventricular septal defect (endocardial
defects protrude into the fetal thorax? cushion defect)
. Midline
10–25. Endocardial cushion defects are associated with
b. Left side
which of the following conditions?
c. Bilateral
. Trisomy 21
d. Right side
b. Heterotaxy syndromes
10–23. The vascular supply to an extralobar pulmonary c. Third-degree atrioventricular block
sequestration originates from which of the following d. All of the above
blood vessels?
. Aorta 10–26. Approximately 50 percent of cardiac rhabdomyomas
are associated with which of the following genetic
b. Vena cava
conditions?
c. Pulmonary artery
. Tay-Sachs
d. None of the above
b. Cystic fibrosis
c. Tuberous sclerosis
d. Infantile polycystic kidney disease
66 The Feta l Pa tient

10–27. Maternal risk factors for fetal gastroschisis defects 10–30. What is the most common abnormality associated
include which of the following? with renal pelvis dilatation?
. Young age . Bladder outlet obstruction
S
b. Pregestational diabetes b. Ureteropelvic junction obstruction
e
C
c. Phenytoin administration c. Duplicated renal collecting system
T
i
O
d. All of the above d. Ureterovesical junction obstruction
n
5
10–28. Gastrointestinal atresia in which of the following 10–31. Only which of the following types of polycystic
portions of the bowel is most likely to be associated kidney disease (PKD) may be reliably diagnosed
with polyhydramnios? prenatally?
. Ileum . X-linked PKD
b. Jejunum b. Autosomal dominant PKD
c. Sigmoid colon c. Autosomal recessive PKD
d. Transverse colon d. None of the above

10–29. Amnionic fluid production is largely from the 10–32. What is the most common nonlethal skeletal
placenta and membranes until the fetal kidney dysplasia?
assumes this role at what gestational age? . Hypophosphatasia
. 12 weeks b. Thanatophoric dysplasia
b. 14 weeks c. Heterozygous achondroplasia
c. 18 weeks d. Type IIa osteogenesis imperfecta
d. 22 weeks
Feta l Ima ging 67

Ch a PTeR 10 a n Sw eR Key

Q uestion Letter Pa ge

C
number a nswer cited Hea der cited

h
a
P
10–1 p. 194 Technology a nd Sa fety

T
e
10–2 b p. 195 Feta l Sa fety

R
1
10–3 p. 195 Feta l Sa fety

0
10–4 p. 195 First Trimester Sonogra phy
10–5 b p. 195 First Trimester Sonogra phy
10–6 d p. 195 First Trimester Sonogra phy
10–7 b p. 196 N ucha l Tra nslucency
10–8 b p. 198 Feta l Biometry
10–9 d p. 198 Feta l Biometry
10–10 c p. 198 Feta l Biometry
10–11 b p. 199 Amnionic Fluid
10–12 d p. 199 Second-Trimester Feta l Anoma ly Detection
10–13 p. 200 Bra in a nd Spine
10–14 p. 201 N eura l Tube Defects
10–15 b p. 201 N eura l Tube Defects
10–16 d p. 202 Ventriculomega ly
10–17 d p. 202 Ventriculomega ly
10–18 p. 203 Holoprosencepha ly
10–19 b p. 204 Ca uda l Regression Sequence—Sa cra l Agenesis
10–20 b p. 205 Fa cia l Clefts
10–21 p. 205 Cystic Hygroma s
10–22 b p. 206 Congenita l Dia phra gma tic Hernia
10–23 p. 208 Extra loba r Pulmona ry Sequestra tion
10–24 c p. 208 Ba sic Ca rdia c Exa mina tion
10–25 d p. 209 Endoca rdia l Cushion Defects
10–26 c p. 211 Ca rdia c Rha bdomyoma s
10–27 p. 212 G a stroschisis
10–28 b p. 213 G a strointestina l Atresia
10–29 c p. 214 Kidneys a nd Urina ry Tra ct
10–30 b p. 215 Ureteropelvic Junction O bstruction
10–31 c p. 216 Polycystic Kidney Disea se
10–32 c p. 217 Skeleta l Dyspla sia s
68

CHAPTER 11

Amnionic Fluid

11–1. Which o the ollowing conditions is not related to 11–6. Amnionic luid volume is a balance between
an absence or diminution o amnionic luid volume production and resorption. What is the primary
during etal development? mechanism o luid resorption?
a. Contractures a. Fetal breathing
b. Pulmonary hypoplasia b. Fetal swallowing
c. Abdominal wall de ects c. Absorption across etal skin
d. Gastrointestinal tract development d. Absorption and iltration by etal kidneys

11–2. What is the normal amnionic luid volume at term? 11–7. All EXCEPT which o the ollowing are acceptable
a. 300 mL methods o sonographic amnionic luid volume
evaluation?
b. 800 mL
a. Subjective estimate
c. 1200 mL
b. Amnionic luid index
d. 1500 mL
c. Dye-dilution measurement
11–3. Which o the ollowing is NOT a signi icant source d. Two-dimension single-pocket measurement
or luid in the amnionic cavity in the irst trimester?
a. Fetal skin 11–8. Which o the ollowing is associated with the single
deepest pocket measurement seen below?
b. Fetal urine
c. Flow across amnion
d. Flow across etal vessels

11–4. In a normal etus at term, what is the daily volume


o etal urine that contributes to the amount o
amnionic luid present?
a. 250 mL
b. 500 mL
c. 750 mL
d. 1000 mL

11–5. A 28-year-old primigravida presents with a 3-day


history o ever, vomiting, and diarrhea at 28 weeks’
gestation. Several amily members are also sick at
home with similar complaints. During sonographic a. Increased perinatal mortality rate
evaluation, her etus is appropriately grown, but her
amnionic luid index is below the 10th percentile b. Increased rate o bronchopulmonary dysplasia
or the gestational age. What is the most likely c. Increased rate o operative vaginal delivery
explanation or this inding? d. Decreased rate o nonreassuring etal heart rate
a. Increased etal swallowing tracings
b. Decreased etal serum osmolality
c. Increased maternal serum osmolality
d. Probable premature rupture o membranes
Amnionic Fluid 69

11–9. Oligohydramnios is de ined as which o the ollowing? 11–12. Which o the ollowing is a clinical sign o
a. Amnionic luid index < 5 cm polyhydramnios?
b. Single deepest pocket < 2 cm a. Tense uterus

C
b. Increase in undal height measurement

H
c. Amnionic luid index < 90th percentile

A
c. Inability to palpate etal small parts

P
d. All o the above

T
d. All o the above

E
R
11–10. What technique or amnionic luid evaluation in

1
multi etal gestations is used in the image below? 11–13. Using the technique demonstrated in this igure,

1
what is the lower threshold or diagnosing
hydramnios?

A
a. Amnionic luid index
b. Single deepest pocket
c. Subjective evaluation
d. Two-dimension single pocket measurement

11–11. Concurrent use o this imaging technique with


amnionic luid index measurements leads to which
o the ollowing?

a. 18 cm
b. 20 cm
c. 24 cm
d. 28 cm

a. Improved etal outcomes


b. Overdiagnosis o hydramnios
c. Overdiagnosis o oligohydramnios
d. More accurate estimation o amnionic luid volume
70 The Feta l Pa tient

11–14. How would the amnionic luid be categorized based 11–16. Which o the ollowing laboratory studies is NOT
on the ollowing image o a single deepest pocket? currently indicated in evaluation o the patient in
Question 11–15?
a. Creatinine
S
E
C
b. Indirect Coombs
T
I
c. Cytomegalovirus IgM and IgG titers
O
N
d. Venereal Disease Research Laboratory (VDRL)
5
a. Normal
b. Mild polyhydramnios
c. Severe polyhydramnios
d. Moderate polyhydramnios

11–15. A new patient presents or her irst prenatal visit at


26 weeks’ gestation. She has no complaints other
than rapid abdominal growth. Sonographic indings
include a 26-week etus with these indings and
a pleural e usion. Potential associated maternal
complications may include all EXCEPT which o
the ollowing?

A B

a. Dyspnea
b. Oliguria
c. Seizures
d. Vulvar edema
Amnionic Fluid 71

11–17. What is the etiology o hydramnios in the condition 11–19. What placental abnormality, seen in the ollowing
depicted in the ollowing image? Arrows point to the image, is associated with polyhydramnios?
etal eye and nose.

C
H
A
P
T
E
R
1
1
Reproduced with permission rom Hof man BL, Dashe JS: Placental chorioangioma
(update) in Cunningham FG, Leveno KL, Bloom SL, et al (eds): Williams Obstetrics, 22nd
ed. Online. New York, McGraw-Hill, 2009. http://www.accessmedicine.com. Figure 10.
Reproduced with permission rom Cunningham FG, Leveno KJ, Bloom SL, et al (eds): Fetal
imaging. In Williams Obstetrics, 23rd ed. New York, McGraw-Hill, 2010, Figure 16-8.
a. Chorioangioma
a. Reduced etal swallowing b. Choriocarcinoma
b. Increased maternal glucose levels c. Placenta previa
c. Increased production o etal urine d. Placenta accreta
d. High requency o associated tracheal-esophageal
istula 11–20. A 30-year-old patient had an sonographic
evaluation or a uterine size-date discrepancy. The
11–18. Which o the ollowing congenital anomalies is amnionic luid index was 36 cm. Without any
NOT associated with polyhydramnios? other in ormation, what is the risk o congenital
mal ormation in this patient’s etus?
a. Pierre Robin sequence
b. In antile polycystic kidney a. 1%
c. Congenital diaphragmatic hernia b. 5%
d. Ureteropelvic junction obstruction c. 10%
d. 25%
72 The Feta l Pa tient

11–21. The ollowing image depicts the etal abdomen 11–24. Which o the ollowing is NOT a recognized
seen during sonographic evaluation o the patient maternal complication associated with hydramnios?
in Question 11–20. Which o the ollowing is a. Postpartum atony
appropriate in the evaluation o this etus?
S
b. Placental abruption
E
C
c. Ureteral obstruction
T
I
O
d. Gestational hypertension
N
5
11–25. Fetal-growth restriction and polyhydramnios are
associated with which o the ollowing chromosomal
abnormalities?
a. Triploidy
b. Trisomy 18
c. Trisomy 21
d. Turner syndrome (Monosomy X)

11–26. Use o the amnionic luid index rather than single


deepest pocket or de ining oligohydramnios is
associated with which o the ollowing?
a. Improved pregnancy outcomes
a. Glucose tolerance test b. Increased diagnosis o oligohydramnios
b. Fetal magnetic resonance imaging c. Improved detection o congenital anomalies
c. Amniocentesis with etal karyotype d. Increased detection o etal-growth restriction
d. None o the above
11–27. Second-trimester oligohydramnios may be attributed
11–22. A patient with a known monozygotic twin gestation to which o the ollowing conditions?
presents at 26 weeks’ gestation or sonographic a. Poor placental per usion
evaluation o etal growth. Twin A has an estimated
etal weight o 804 g, whereas twin B’s estimated b. Rupture o etal membranes
etal weight is 643 g. The largest pocket o amnionic c. Fetal bladder outlet obstruction
luid around twin A is 9.6 cm and 2.2 cm or twin d. All o the above
B. Which o the ollowing conditions most likely
explains these indings?
a. Gestational diabetes
b. Congenital anomaly in twin A
c. Twin-twin trans usion syndrome
d. Twin B with premature membrane rupture

11–23. Idiopathic hydramnios is associated with which o


the ollowing conditions?
a. Congenital in ection
b. Birthweight > 4000 g
c. Neonatal diabetes mellitus
d. Increased perinatal mortality rate
Amnionic Fluid 73

11–28. An obstetric patient presents at 35 weeks’ gestation 11–29. The evaluation o the patient in Question 11–28 is
with a complaint o decreased etal movement. normal. What is the most appropriate step in the
Variable decelerations are present on a nonstress test, management o her pregnancy?

C
so an amnionic luid index (AFI) is per ormed. The a. Immediate cesarean delivery

H
result is seen below. What subsequent evaluation is

A
b. Induction o labor in 1 week

P
recommended?

T
c. Administration o antenatal corticosteroids

E
R
d. Expectant management with etal surveillance

1
1
11–30. Which o the ollowing medications is associated
with oligohydramnios when taken in the latter hal
o pregnancy?
a. Hydralazine
b. Beta blockers
c. Calcium-channel blockers
d. Angiotensin-receptor blockers
A
11–31. Oligohydramnios is NOT associated with which o
the ollowing pregnancy complications?
a. Stillbirth
b. Neonatal sepsis
c. Congenital mal ormations
d. Meconium aspiration syndrome

11–32. A borderline amnionic luid index (AFI), de ined


as an AFI between 5 and 8 cm, is associated
with increased rates o all EXCEPT which o the
ollowing?
a. Preterm birth
b. Neonatal mortality
B c. Fetal-growth restriction
d. Cesarean delivery or nonreassuring etal heart
a. Sterile speculum examination rate pattern
b. Umbilical artery Doppler studies
c. Sonographic measurement o etal growth
d. All o the above
74 The Feta l Pa tient

CHAPTER 11 ANSw ER KEy

Q uestion Letter Pa ge
S
number a nswer cited Hea der cited
E
C
T
11–1 c p. 231 Introduction
I
O
11–2 b p. 231 N orma l Amnionic Fluid Volume
N
11–3 b p. 231
5
Physiology
11–4 d p. 231 Physiology
11–5 c p. 231 Physiology
11–6 b p. 231 Physiology
11–7 c p. 232 Sonogra phic Assessment
11–8 a p. 232 Single Deepest Pocket
11–9 d p. 231 Single Deepest Pocket
11–10 b p. 232 Single Deepest Pocket
11–11 c p. 233 Amnionic Fluid Index (AFI)
11–12 d p. 233 Hydra mnios
11–13 c p. 233 N orma l AFI
11–14 b p. 233 Hydra mnios
11–15 c p. 235 Complica tions
11–16 a p. 236 Etiology
11–17 a p. 237 Congenita l Anoma lies
11–18 b p. 237 Congenita l Anoma lies
11–19 a p. 237 Congenita l Anoma lies
11–20 c p. 237 Congenita l Anoma lies
11–21 c p. 237 Congenita l Anoma lies
11–22 c p. 235 Multifeta l G esta tion
11–23 b p. 235 Idiopa thic Hydra mnios
11–24 d p. 235 Complica tions
11–25 b p. 235 Pregna ncy O utcomes
11–26 b p. 236 O ligohydra mnios
11–27 d p. 236 Etiology
11–28 d p. 236 O ligohydra mnios a fter Midpregna ncy
11–29 d p. 238 Ma na gement
11–30 d p. 237 Medica tion
11–31 b p. 238 Pregna ncy O utcomes
11–32 b p. 238 “Borderline” O ligohydra mnios
75

CHAPTER 12

Teratolog , Teratogens, and Fetotoxic Agents

12–1. What percentage of all birth defects is caused by 12–6. If discordant among fetuses, which of the following
exposure to medications during pregnancy? can render certain fetuses more susceptible to a
a. 1% teratogen?
b. 7% a. Fetal genome
c. 11% b. Folic acid pathway disturbances
d. 17% c. Paternal exposures to certain drugs
d. All of the above
12–2. What is the strict definition of a trophogen?
a. An agent that alters growth 12–7. What is the leading cause of preventable birth
defects in the United States?
b. An agent that interferes with normal function of
an organ a. Maternal smoking
c. An agent that interferes with normal maturation b. Maternal alcohol consumption
of an organ c. Maternal anticonvulsant treatment
d. All of the above d. Continued inadvertent use of birth control pills
during early pregnancy
12–3. Which of the following drawbacks is typical of case-
control studies when studying potential teratogens? 12–8. In addition to having dysmorphic facial features and
a. Recall bias postnatal growth restriction, which of the following
would have to be present for a diagnosis of fetal
b. Lack of a control group
alcohol syndrome?
c. Only causality can be established
a. Scoliosis
d. All of the above
b. Dysplastic kidney
12–4. What is the background rate of major congenital c. Ventricular septal defect
anomalies diagnosed at birth? d. Head size < 10th percentile
a. 0.5%
12–9. Increased rates of which complication have been
b. 3%
linked to binge drinking during pregnancy?
c. 6%
a. Stillbirth
d. 9%
b. Preterm birth
12–5. Which of the following criteria is not required to c. Postpartum depression
prove teratogenicity of a particular agent? d. Fetal-growth restriction
a. he agent must cross the placenta.
12–10. A 32-week fetus is growth restricted and has
b. Exposure to the agent must occur during
oligohydramnios and an abnormal calvarium. Which
organogenesis.
antihypertensive agent taken by the mother may
c. he association with the teratogen must be have caused this problem?
biologically plausible.
a. Verapamil
d. wo or more high-quality epidemiological studies
b. Nifedipine
must report similar findings.
c. Lisinopril
d. Methyldopa
76 The Feta l Pa tient

12–11. Which of the following associations regarding 12–15. Prenatal exposure to which of the following agents is
anticonvulsants and their risk of birth defects has not associated with the sonographic finding shown here?
been reported?
a. Hydantoin exposure can cause midfacial
S
E
hypoplasia.
C
T
b. Valproic acid exposure can cause neural-tube
I
O
defects.
N
c. opiramate exposure increases the risk of
5
orofacial clefts.
d. Valproic acid exposure increases the risk of
abdominal wall defects.

12–12. What fetal complication is associated with the


nonsteroidal antiinflammatory agent indomethacin?
a. Hydramnios
b. Pulmonary valve atresia
c. Bronchopulmonary dysplasia
d. Premature closure of the ductus arteriosus
a. amoxifen
12–13. Considered Category X in pregnancy, which b. opiramate
antiviral causes skull, palate, jaw, eye, limb, and c. rastuzumab
gastrointestinal anomalies in rodent models? d. Methotrexate
a. Ribavirin
b. Efavirenz 12–16. Which of the following genitourinary anomalies is
not typically associated with prenatal exposure to
c. Zidovudine
diethylstilbestrol?
d. Nevirapine
a. Microphallus
12–14. Which of the following associations between first- b. Hooded cervix
trimester antibiotic exposure and the given birth c. Bicornuate uterus
defect is true? d. esticular hypoplasia
a. Aminoglycosides may cause ototoxicity.
b. Chloramphenicol may cause ashen-gray skin 12–17. Which of the following statements is true regarding
coloration. corticosteroids and the risk of birth defects?
c. etracyclines may cause deciduous teeth a. hey may cause orofacial fetal clefts.
discoloration. b. hey may cause clitoromegaly in the female fetus.
d. Nitrofurantoin may cause hypoplastic left heart c. hey may cause phallic enlargement in the male
syndrome. fetus.
d. hey may cause labioscrotal fusion in the female
fetus.

12–18. For fetuses exposed to mycophenolate mofetil during


pregnancy, which of the following is the most likely
outcome?
a. Spontaneous abortion
b. Born at term with ear abnormalities
c. Born at term without evidence of abnormalities
d. Born prematurely without evidence of
abnormalities
Tera tology, Tera togens, a nd Fetotoxic Agents 77

12–19. Which prenatal exposure is associated with an 12–22. Which of the following selective serotonin-reuptake
increased risk of childhood thyroid cancer? inhibitors (SSRIs) is most strongly associated with
a. Lead the cardiac defect shown in this sonogram?

C
b. Lithium

H
A
c. Mercury

P
T
d. Radioiodine

E
R
1
12–20. Which drug is associated with the rare cardiac

2
anomaly shown in this fetal sonogram?

a. Paroxetine
b. Fluoxetine
c. Sertraline
d. Escitalopram

12–23. he severe form of the neonatal behavioral syndrome


associated with prenatal exposure to serotonin-
a. Lithium reuptake inhibitors includes all EXCEPT which of
b. Leflunomide the following?
c. Indomethacin a. Seizures
d. Cyclophosphamide b. Hyperpyrexia
c. Respiratory failure
12–21. Prenatal exposure near term to which of the
d. Persistent pulmonary hypertension
following agents can lead to neonatal toxicity that
manifests as hypothyroidism, diabetes insipidus,
cardiomegaly, bradycardia, and hypotonia?
a. Lithium
b. Fluoxetine
c. Paroxetine
d. Escitalopram
78 The Feta l Pa tient

12–24. Prenatal exposure to which of the following 12–25. Severe malformations may be seen with use of all
agents is most likely responsible for the congenital EXCEPT which of the following vitamin A-derived
malformations seen in these photographs? compounds?
a. Acitretin
S
E
C
b. Bexarotene
T
I
c. Isotretinoin
O
N
d. Beta carotene
5
12–26. With the upper limb defect seen in this photograph,
when in the first trimester of pregnancy was this
infant most likely exposed to the causative drug?

Reproduced with permission from Walsh Ra, O’Rourke RA, Shaver JA: T e history,
physical examination, and cardiac auscultation. In Fuster V, Walsh RA, Harrington RA,
et al (eds): Hurst’s T e Heart, 13th ed. New York, McGraw-Hill, 2011, Figure 14-6.

a. Days 27–30
b. Days 30–33
c. Days 40–47
d. Days 42–43
B

Reproduced with permission from Cunningham FG, Leveno KJ, Bloom SL, et al (eds):
eratology, teratogens, and fetotoxic agents. In Williams Obstetrics, 24th ed. New York,
McGraw-Hill, 2014, Figure 12-4.

a. Alcohol
b. Warfarin
c. Isotretinoin
d. Valproic acid
Tera tology, Tera togens, a nd Fetotoxic Agents 79

12–27. he nasal hypoplasia seen in this prenatal sonogram 12–29. Which of the following herbal remedies is associated
is consistent with exposure to which of the following with increased risk of bleeding by inhibiting
agents? cyclooxygenase?

C
a. Garlic

H
A
b. Ginger

P
T
c. Ginseng

E
R
d. Gingko biloba

1
2
12–30. Among recreational drugs, which one has been
linked to cleft palate, cardiovascular abnormalities,
and urinary tract abnormalities?
a. Heroin
b. Cocaine
c. Methadone
d. Methamphetamine

12–31. Cigarette smoking in pregnancy has been associated


with an increased risk for all EXCEPT which of the
Reproduced with permission from Cunningham FG, Leveno KJ, Bloom SL, et al (eds):
eratology, teratogens, and fetotoxic agents. In Williams Obstetrics, 24th ed. New York,
following?
McGraw-Hill, 2014, Figure 12-5B. a. Microcephaly
b. Cleft lip and palate
a. Warfarin
c. Neonatal hypoglycemia
b. Corticosteroids
d. Congenital heart disease
c. Diethylstilbestrol
d. Mycophenolate mofetil 12–32. Which of the following agents has not been shown
to be a human teratogen?
12–28. When used in the second and third trimesters, which a. oluene
drug may lead to central nervous system defects such
as agenesis of the corpus callosum, Dandy-Walker
b. Marijuana
malformation, and midline cerebellar malformations? c. Methadone
a. Heroin d. Phencyclidine
b. Cocaine
c. Warfarin
d. Isotretinoin
80 The Feta l Pa tient

CHAPTER 12 ANSw ER KEy

Q uestion Letter Pa ge
S
number a nswer cited Hea der cited
E
C
T
12–1 a p. 240 Introduction
I
O
12–2 a p. 240 Tera tology
N
12–3 a p. 241 Ca se-Control Studies
5
12–4 b p. 241 Criteria for Determining Tera togencity
12–5 b p. 241 Criteria for Determining Tera togencity
12–6 d p. 244 G enetic a nd Physiologica l Susceptibility to Tera togens
12–7 b p. 245 Alcohol
12–8 d p. 245 Alcohol; Ta ble 1 2 -4
12–9 a p. 245 Alcohol
12–10 c p. 247 Angiotensin-Converting Enzyme Inhibitors a nd Angiotensin-Receptor
Blocking Drugs
12–11 d p. 246 Anticonvulsa nt Medica tions
12–12 d p. 247 N onsteroida l Antiinfla mma tory Drugs
12–13 a p. 249 Riba virin
12–14 d p. 248 N itrofura ntoin
12–15 b p. 246 Anticonvulsa nt Medica tions
12–16 c p. 249 Diethylstilbestrol
12–17 a p. 250 Corticosteroids
12–18 a p. 250 Mycophenola te Mofetil
12–19 d p. 250 Ra dioiodine
12–20 a p. 250 Lithium
12–21 a p. 250 Lithium
12–22 a p. 250 Selective Serotonin- a nd N orepinephrine- Reupta ke Inhibitors
12–23 d p. 250 Selective Serotonin- a nd N orepinephrine- Reupta ke Inhibitors
12–24 c p. 251 Isotretinoin; Figure 1 2 -4
12–25 d p. 251 Retinoids
12–26 a p. 252 Tha lidomide a nd Lena lidomide
12–27 a p. 252 W a rfa rin
12–28 c p. 252 W a rfa rin
12–29 b p. 253 Herba l Remedies; Ta ble 1 2 -5
12–30 b p. 253 Coca ine
12–31 c p. 254 Miscella neous Drugs
12–32 b p. 255 Toba cco
81

CHAPTER 13

Genetics

13–1. What percentage of individuals will experience a 13–3. Which of the following karyotypes accurately
disease with a genetic component during the course describes a female with a deletion on the long arm of
of their lifetime? chromosome 17 at band q3?
a. 10% a. 46,XY,del(17)(q3)
b. 33% b. 46,del(17)(q3)
c. 66% c. 46,XX,del(17)(q3)
d. 85% d. XX,46,del(17)(q3)

13–2. When correctly reporting a karyotype, where should 13–4. What is the most common cause of the abnormality
the sex chromosomes be listed? depicted in the karyotype below?
a. First, before the total number of chromosomes a. Dispermy
b. Second, after the total number of chromosomes b. Mitotic nondisjunction
c. Last, after any description of structural c. Meiotic nondisjunction
abnormalities d. Unbalanced translocation
d. None of the above

Reproduced with permission from Cunningham FG, Leveno KJ, Bloom SL, et al (eds): Genetics. In Williams Obstetrics, 24th ed.
New York, McGraw-Hill, 2014, Figure 13-3.
82 The Feta l Pa tient

13–5. Trisomy of which of the following autosomal 13–9. This sonographic image depicts a fetal head in the
chromosomes is LEAST likely to result in a term midtrimester of pregnancy. Cysts are noted within
pregnancy? the choroid plexus. This finding, when associated
a. 8 with other fetal abnormalities, raises suspicion for
S
E
which genetic condition?
C
b. 13
T
I
c. 18
O
N
d. 21
5
13–6. Which of the following generalizations is true
regarding the reproductive capacity of adults with
Down syndrome?
a. Both males and females are sterile.
b. Both males and females are fertile.
c. Males are fertile, and females are sterile.
d. Males are sterile, and females are fertile.

13–7. Characteristic features of infants with Down Reproduced with permission from Cunningham FG, Leveno KJ, Bloom SL, et al (eds):
syndrome, as depicted in this image, include all Genetics. In Williams Obstetrics, 24th ed. New York, McGraw-Hill, 2014, Figure 13-5A.
EXCEPT which of the following?
a. Down syndrome
b. Patau syndrome
c. Edwards syndrome
d. Turner syndrome

13–10. A 22-year-old G2P1 undergoes a routine


sonographic evaluation of fetal anatomy at 18
weeks’ gestation. Alobar holoprosencephaly is noted
and shown in the image below. Which of the
following genetic conditions is frequently associated
with this finding?

Reproduced with permission from Cunningham FG, Leveno KJ, Bloom SL, et al (eds):
Genetics. In Williams Obstetrics, 24th ed. New York, McGraw-Hill, 2014, Figure 13-4A.

a. Brachycephaly
b. Epicanthal folds
c. Flat nasal bridge
d. Down-slanting palpebral fissures

13–8. Which of the following is not an acrocentric


Used with permission from Dr. Jodi Dashe.
chromosome?
a. 13 a. Trisomy 13
b. 14 b. Trisomy 16
c. 18 c. Trisomy 21
d. 22 d. Monosomy X
G enetics 83

13–11. Amniocentesis is elected by the patient described in 13–17. All EXCEPT which of the following statements
Question 13-10 and confirms the suspected genetic regarding Robertsonian translocations are correct?
abnormality. What is the likelihood of survival of a. They are not a major cause of miscarriage.

C
her infant at 1 year of age? b. The most common of them is between

H
a. 95%

A
chromosomes 13 and 14.

P
b. 50%

T
c. Offspring are more likely to be abnormal if the

E
R
c. 25% father is the translocation carrier.

1
d. Less than 5% d. When a child is found to have one, both parents

3
should be offered karyotype testing.
13–12. The patient described in Question 13-10 would be
at an increased risk for which particular pregnancy- 13–18. The most common isochromosome involves the long
related complication? arm of what chromosome?
a. Preeclampsia a. 6
b. Gestational diabetes b. X
c. Postpartum hemorrhage c. Y
d. Acute fatty liver of pregnancy d. 21

13–13. The overwhelming majority of monosomy X 13–19. What term is used to refer to two or more
conceptions result in what outcome? cytogenetically distinct cell lines that are derived
from a single zygote?
a. Early first-trimester spontaneous abortion
b. Small cystic hygromas and live birth at term a. Chimerism
c. Large cystic hygromas, hydrops fetalis, and b. Mosaicism
second-trimester fetal death c. Polygenic
d. None of the above d. Genetic blending

13–14. All EXCEPT which of the following are clinical 13–20. Gonadal mosaicism may explain what type of de
features associated with Turner syndrome? novo mutations in offspring of normal parents?
a. Hypothyroidism a. Mitochondrial
b. Mental retardation b. Autosomal recessive
c. Renal abnormalities c. Autosomal dominant
d. Coarctation of the aorta d. None of the above

13–15. What is the most common sex chromosome 13–21. What term is used to describe the degree to which
abnormality? an individual with an autosomal dominant condition
demonstrates the phenotype?
a. 45,X
b. 47,XXX a. Penetrance
c. 47,XYY b. Variability
d. 47,XXY c. Concordance
d. Expressivity
13–16. Carriers of Robertsonian translocations involving
which of the following chromosomes could produce 13–22. Which of the following genetic disorders would be
only unbalanced gametes? expected to occur more frequently in the setting of
advanced paternal age?
a. 13 and 14
b. 14 and 21 a. Turner syndrome
c. 21 and 22 b. Cystic fibrosis
d. 21 and 21 c. Sickle-cell anemia
d. Tuberous sclerosis
84 The Feta l Pa tient

13–23. A 22-year-old G1P0 at 12 weeks’ gestation who has 13–29. All EXCEPT which of the following conditions are
sickle-cell anemia requests genetic counseling to learn considered to have multifactorial inheritance?
about the risk of transmission to her fetus. If her a. Beta-thalassemia
partner is a heterozygous carrier for this condition,
S
b. Diabetes mellitus
E
what is the risk that their offspring will be affected?
C
c. Neural-tube defects
T
a. 25%
I
O
d. Coronary heart disease
b. 50%
N
5
c. 75% 13–30. Hyperthermia has been associated with what specific
d. 100% type of neural-tube defect?
a. Anencephaly
13–24. Consanguinity increases the risk for what types of
b. Lumbar defects
genetic syndromes?
c. Sacral defects
a. X-linked dominant
d. Cervical defects
b. Autosomal dominant
c. X-linked recessive 13–31. The image below represents amnionic fluid analyzed
d. Autosomal recessive with fluorescence in situ hybridization (FISH). If the
green probe is directed at the X chromosome and
13–25. All EXCEPT which of the following statements the red probe is directed at chromosome 21, which
regarding X-linked diseases are true? karyotype is correct?
a. Most are X-linked recessive.
b. X-linked dominant disorders mainly affect
females.
c. Female carriers of X-linked recessive conditions
may demonstrate some clinical features.
d. All statements are true.

13–26. A couple is referred for genetic counseling because


the woman has several relatives with the same
unusual genetic condition. A pedigree of her family
reveals that both males and females are equally Reproduced with permission from Hassold TJ, Schwartz S: Chromosome disorders. In
affected but transmission occurs only though Fauci AS, Braunwald E, Kasper DL (eds): Harrison’s Principles of Internal Medicine, 17th ed.
New York, McGraw-Hill, 2008, Figure 63-2B.
females. Which pattern of inheritance is suggested?
a. Mitochondrial a. 46,XX,+ 21
b. Multifactorial b. 47,XX,+ 21
c. X-linked dominant c. 46,XY,+ 21
d. Autosomal dominant d. 47,XY,+ 21
13–27. According to the American College of Obstetricians 13–32. Chromosomal microarray analysis can identify DNA
and Gynecologists, how many CGG triplet repeats deletions and duplications as small as how many
are required to have the premutation for fragile-X kilobases?
syndrome?
a. 1
a. < 45
b. 3
b. 45–54
c. 5
c. 55–200
d. 10
d. > 200

13–28. Females who are premutation carriers for fragile-X


syndrome are at increased risk for which of the
following conditions?
a. Breast cancer
b. Endometriosis
c. Hypothyroidism
d. Primary ovarian failure
G enetics 85

CHAPTER 13 ANSw ER KEy

Q uestion Letter Pa ge

C
number a nswer cited Hea der cited

H
A
P
13–1 c p. 259 Introduction

T
E
13–2 b p. 260 Sta nda rd N omencla ture

R
1
13–3 c p. 260 Sta nda rd N omencla ture

3
13–4 c p. 260 Autosoma l Trisomies
13–5 a p. 260 Autosoma l Trisomies
13–6 d p. 261 Trisomy 2 1 —Down Syndrome
13–7 d p. 261 Trisomy 2 1 —Down Syndrome; Clinica l Findings
13–8 c p. 263 Trisomy 1 8 —Edwa rds Syndrome
13–9 c p. 263 Trisomy 1 8 —Edwa rds Syndrome
13–10 a p. 263 Trisomy 1 3 —Pa ta u Syndrome
13–11 d p. 263 Trisomy 1 3 —Pa ta u Syndrome
13–12 a p. 263 Trisomy 1 3 —Pa ta u Syndrome
13–13 a p. 264 4 5 ,X—Turner Syndrome
13–14 b p. 264 4 5 ,X—Turner Syndrome
13–15 d p. 265 4 7 ,XXY—Klinefelter Syndrome
13–16 d p. 267 Robertsonia n Tra nsloca tions
13–17 c p. 267 Robertsonia n Tra nsloca tions
13–18 b p. 268 Isochromosomes
13–19 b p. 269 Chromosoma l Mosa icism
13–20 c p. 269 G ona da l Mosa icism
13–21 d p. 270 Expressivity
13–22 d p. 270 Adva nced Pa terna l Age
13–23 b p. 271 Autosoma l Recessive Inherita nce
13–24 d p. 271 Consa nguinity
13–25 d p. 272 X-Linked a nd Y-Linked Inherita nce
13–26 a p. 272 Mitochondria l Inherita nce
13–27 c p. 272 Fra gile X Syndrome
13–28 d p. 272 Fra gile X Syndrome
13–29 a p. 274 Multifa ctoria l Inherita nce
13–30 a p. 275 N eura l-Tube Defects
13–31 b p. 276 Fluorescence in-Situ Hybridiza tion
13–32 a p. 277 Chromosoma l Microa rra y Ana lysis
86

CHAPTER 14

Prenatal Diagnosis

14–1. A 36-year-old primigravida at 20 weeks’ gestation 14–3. The infant shown below was also born with a cleft
presents to her obstetrician’s office with a complaint palate. These findings are consistent with which of
of leaking fluid. Sonographic examination performed the following processes?
confirms markedly decreased fluid, and midtrimester
rupture of membranes is suspected. The patient
elects to continue her pregnancy, and minimal
amnionic fluid is present around the fetus. At term,
her fetus is born with a right-sided clubbed foot.
This is an example of which of the following?
a. Sequence
b. Disruption
c. Deformation
d. Malformation

14–2. The finding seen below was identified prenatally


during sonographic examination and is an example
of which of the following?

Used with permission from Valorie Butler.

a. Syndrome
b. Sequence
c. Association
d. Chromosome abnormality

14–4. A pregnant 25-year-old postdoctoral student presents


for genetic counseling following a multiple marker
screen that revealed an increased risk for an open
neural-tube defect and trisomy 18. She is from
Used with permission from Dr. Dina Chamsy. France and her husband is from Great Britain. Her
medical history is significant for a seizure disorder
a. Syndrome well controlled on phenytoin. Which of the following
b. Disruption is NOT an expected possible contributing factor in
her elevated risk for an open neural-tube defect?
c. Association
a. Ethnicity
d. Malformation
b. Medication exposure
c. Chromosome abnormality
d. None of the above
Prena ta l Dia gnosis 87

14–5. What is the recurrence risk for an open neural-tube 14–9. During routine sonographic examination, the
defect after a couple has had one child born with abnormality seen below was detected. This condition
anencephaly? is associated with which of the following conditions?

C
a. 3% to 5%

H
A
b. 10%

P
T
c. 25%

E
R
d. Unknown

1
4
14–6. Four-milligram folic acid supplementation before
conception and in the first trimester of pregnancy
would be most indicated in which of the following
scenarios?
a. Maternal pregestational diabetes
b. A personal history of open neural-tube defect
c. Maternal valproic acid use for seizure disorder
d. Maternal paroxetine use for depression

14–7. Which of the following maternal factors does not


affect the maternal serum alpha fetoprotein (AFP) a. Twin gestation
multiples of the median calculation? b. Low maternal inhibin level
a. Race c. Maternal pregestational diabetes
b. Parity d. Elevated maternal serum AFP level
c. Weight
d. Gestational age 14–10. Your patient has a 1:100 risk for a fetal open neural-
tube defect based on serum screening at 18 weeks’
gestation. She undergoes targeted sonographic
14–8. Which of the following is NOT an indication for
examination, which documents a singleton fetus and
sonographic evaluation of an elevated maternal
a marginal placenta previa. No fetal abnormalities
serum AFP level result?
are detected. Following this examination, how
a. Determination of fetal sex should she be counseled regarding her fetus’s risk for
b. Estimation of gestational age having an open neural-tube defect?
c. Determination of fetal number a. Reduced by 25%
d. Documentation of fetal viability b. Reduced by 50%
c. Reduced by 95%
d. Unchanged from the 1% risk

14–11. Which of the following obstetric complications is the


patient in Question 14–10 NOT at increased risk
for?
a. Fetal death
b. Fetal macrosomia
c. Placenta abruption
d. Preterm rupture of membranes
88 The Feta l Pa tient

14–12. Multiple screening strategies exist to detect Down 14–15. The American College of Obstetricians and
syndrome during pregnancy. Which of the following Gynecologists recommends a strategy using first- and
tests has the highest detection rate for Down second-trimester screening for what reason?
syndrome? a. Earlier results
S
E
a. Maternal serum AFP
C
b. Improved Down syndrome detection rates
T
b. Integrated screening
I
c. Reduced false-positive rate compared with second
O
N
c. Quadruple marker test trimester screening
5
d. Combined first-trimester screening d. None of the above

14–13. A 38-year-old woman presents for first-trimester 14–16. Testing for the most common trisomies that
screening for Down syndrome at a gestational age complicate pregnancies can be accomplished by
of 12 weeks and 1 day. The ultrasound image below isolating which of the following substances from
was seen. What is the next step in evaluating this maternal blood?
finding? a. Fetal RNA
b. Free fetal cells
c. Mitochondrial DNA
d. Cell-free fetal DNA

14–17. The congenital anomaly seen in the ultrasound


image below was discovered during routine
evaluation at 20 weeks’ gestation and was an isolated
finding. What should your patient be told regarding
the risk of a chromosome abnormality in her fetus?

a. Offer diagnostic prenatal testing


b. Repeat the ultrasound measurement in 1 week
c. Complete the first-trimester screen and wait for
her numeric risk assessment
d. Offer a sequential test as it has a high sensitivity
for Down syndrome detection

14–14. Which of the following correctly identifies the


second-trimester analyte level abnormalities in a
pregnancy at increased risk for Down syndrome? Reproduced with permission from Cunningham FG, Leveno KJ, Bloom SL, et al (eds):
Fetal imaging. In Williams Obstetrics, 24th ed. New York, McGraw-Hill, 2014, eF10-22A.
a. Decreased MSAFP, increased unconjugated
estriol, increased inhibin, increased beta hCG
a. 10%
b. Decreased MSAFP, decreased unconjugated
estriol, increased inhibin, increased beta hCG b. 22%
c. Increased MSAFP, increased unconjugated estriol, c. 30%
decreased inhibin, decreased beta hCG d. No increase in risk
d. Decreased MSAFP, decreased unconjugated
estriol, decreased inhibin, increased beta hCG
Prena ta l Dia gnosis 89

14–18. A 25-year-old primigravida from China has a Down 14–22. A white couple presents for preconceptional
syndrome risk of 1:5000 based on her first-trimester counseling regarding cystic fibrosis. She has a prior
screening results. The finding shown is noted during child affected with the disease; he has two unaffected

C
a routine sonographic examination performed at children. What is their risk of having an affected

H
17 weeks’ gestation. How should she be counseled fetus?

A
P
regarding this finding? a. 1:50

T
E
b. 1:100

R
1
c. 1:200

4
d. 1:2500

14–23. What is the appropriate screening test for


hemoglobinopathies in patients of African descent?
a. Complete blood count
b. Peripheral blood smear
c. Hemoglobin electrophoresis
d. Hemoglobin S mutation analysis

14–24. An Asian patient presents for prenatal care. Her


complete blood count reveals a microcytic anemia.
What is the most appropriate next step in the
a. Schedule a fetal echocardiogram at 22 weeks’ evaluation of her anemia?
gestation
a. Iron studies
b. Offer an amniocentesis since she is now
considered “high-risk” b. Hemoglobin electrophoresis
c. Inform her that this finding is seen in up to 30% c. Alpha thalassemia molecular genetic testing
of fetuses of Asian descent d. None of the above
d. Inform her that her Down syndrome risk has
now increased from 1 in 5000 to 1 in 500 14–25. Hexosaminidase A activity levels should be used in
testing for carrier status for Tay-Sachs disease in
14–19. Which of the following fetal conditions or events is which of the following samples?
NOT associated with the finding of echogenic bowel a. Blood from a pregnant woman
during a second-trimester sonographic examination? b. Blood from a male of Indian descent
a. Down syndrome c. Blood from a male of Ashkenazi Jewish descent
b. Cystic fibrosis d. Amnionic fluid from a suspected affected fetus
c. Toxoplasmosis infection
d. Intraamniotic hemorrhage 14–26. Which of the following is NOT a reason for the
decreased prevalence of fetal blood sampling in the
14–20. Which of the following skeletal findings during last decade?
sonographic examination suggest an increased fetal a. Increase in procedure-related loss rate with fetal
risk for Down syndrome? blood sampling
a. Observed:expected femur ratio ≤ .90 b. Increased availability of DNA-based tests for
b. Observed:expected humerus ratio ≤ .90 amnionic fluid
c. Femur length:abdominal circumference ratio < .20 c. Increased use of fluorescence in situ hybridization
(FISH) on amnionic fluid samples
d. Observed:expected biparietal diameter ratio < .89
d. Increased use of middle cerebral artery Doppler
studies in the evaluation of suspected fetal anemia
14–21. What chromosome is the cystic fibrosis conductance
transmembrane regulator (CFTR) gene located on?
a. 7
b. 14
c. 17
d. None of the above
90 The Feta l Pa tient

14–27. There is an increased pregnancy loss rate following 14–32. Fetal blood sampling performed at the placental
amniocentesis in all EXCEPT which of the insertion site is associated with which of the
following conditions? following?
a. Twin gestation a. Shorter procedure duration
S
E
C
b. Maternal BMI ≥ 40 kg/m2 b. Increased pregnancy loss rate
T
I
c. Transplacental puncture with needle c. Increased procedure success rate
O
N
d. All of the above d. Decreased maternal blood contamination
5
14–28. A 40-year-old infertility patient underwent an 14–33. Which of the following statements correctly
amniocentesis at 17 weeks’ gestation. She calls describes polar body analysis when used for
1 day later and reports that she is leaking amnionic preimplantation genetic testing?
fluid. What should she be told about this a. It involves sampling one cell of the embryo on
postamniocentesis complication? day 3.
a. Fetal survival is > 90%. b. It is associated with decreased pregnancy success
b. The risk of fetal death is 25%. rates.
c. The risk for chorioamnionitis is 2%. c. It can be used to determine paternally inherited
genetic disorders.
d. Fluid leakage occurs in approximately 10% of
patients. d. None of the above

14–29. Early amniocentesis is defined as amniocentesis 14–34. Preimplantation genetic diagnosis may be used for
that is performed during which of the following which of the following scenarios?
gestational age windows? a. Determine fetal gender
a. 9–11 weeks b. Diagnose single gene mutations
b. 11–14 weeks c. Human leukocyte antigen (HLA) typing
c. 12–15 weeks d. All of the above
d. 14–16 weeks
14–35. Which of the following is NOT a limitation of
14–30. A woman undergoes a chorionic villus sampling preimplantation genetic screening using fluorescence
(CVS) at 11 weeks’ gestation. The result shows two in situ hybridization?
cell lines—46,XY and 47,XY,+ 21. What is the a. The result may not reflect the embryonic
appropriate next step? karyotype.
a. Repeat CVS at 13 weeks’ gestation b. Genomic hybridization arrays have a high failure
b. Plan no further evaluation or treatment rate.
c. Offer amniocentesis for clarification of results c. Mosaicism is common in cleavage-stage embryo
blastomeres.
d. Provide the patient with appropriate information
regarding Down syndrome d. Pregnancy rates are lower following
preimplantation genetic screening.
14–31. Chorionic villus sampling has been associated with
limb reduction defects under what condition? 14–36. What is the most common cause of infant death in
the United States?
a. Multiple needle passes are made.
a. Preterm birth
b. Performed at a gestational age < 10 weeks
b. Motor vehicle accidents
c. Performed using a transabdominal approach
c. Major congenital anomalies
d. Larger volumes of chorionic villi are sampled.
d. Complications from maternal hypertension
Prena ta l Dia gnosis 91

CHAPTER 14 ANSw ER KEy

Q uestion Letter Pa ge

C
number a nswer cited Hea der cited

H
A
P
14–1 c p. 283 Introduction

T
E
14–2 b p. 283 Introduction

R
1
14–3 b p. 283 Introduction

4
14–4 b p. 284 Ta ble 1 4 -1
14–5 a p. 284 Risk Fa ctors
14–6 b p. 284 Prevention
14–7 b p. 284 Ma terna l Serum Alpha -Fetoprotein Screening
14–8 a p. 285 MSAFP Eleva tion
14–9 d p. 287 Ta ble 1 4 -2
14–10 c p. 285 Ta rgeted Sonogra phy
14–11 b p. 287 Unexpla ined Ma terna l Serum AFP Level Eva lua tion
14–12 b p. 291 Combined First- a nd Second-Trimester Screening
14–13 a p. 289 N ucha l Tra nslucency
14–14 b p. 290 Second-Trimester Screening
14–15 b p. 291 Combined First- a nd Second-Trimester Screening
14–16 d p. 291 Cell-Free Feta l DN A Screening
14–17 d p. 292 Ta ble 1 4 -6
14–18 c p. 292 Second-Trimester Sonogra phic Ma rkers—”Soft Signs”
14–19 c p. 292 Second-Trimester Sonogra phic Ma rkers—”Soft Signs”
14–20 a p. 292 Second-Trimester Sonogra phic Ma rkers—”Soft Signs”
14–21 a p. 295 Cystic Fibrosis
14–22 c p. 295 Cystic Fibrosis
14–23 c p. 296 Sickle Hemoglobinopa thies
14–24 a p. 296 Alpha -Tha la ssemia
14–25 b p. 296 Ta y-Sa chs Disea se
14–26 a p. 297 Prena ta l a nd Preimpla nta tion Dia gnostic Testing
14–27 c p. 299 Complica tions
14–28 a p. 299 Complica tions
14–29 b p. 299 Ea rly Amniocentesis
14–30 c p. 300 Complica tions
14–31 b p. 300 Complica tions
14–32 a p. 300 Complica tions
14–33 d p. 301 Preimpla nta tion G enetic Testing
14–34 d p. 301 Preimpla nta tion G enetic Dia gnosis (PGD)
14–35 b p. 302 Preimpla nta tion G enetic Screening (PGS)
14–36 c p. 283 Introduction
92

CHAPTER 15

Fetal Disorders

15–1. What is the most common cause of fetal anemia? 15–6. A mother whose blood type is A negative labors
a. Alpha thalassemia and delivers a B-positive child at term. She does
not receive anti-D immune globulin postpartum.
b. Fetomaternal hemorrhage
What are the chances that the patient will become
c. Parvovirus B19 infection alloimmunized?
d. Red cell alloimmunization a. 2%
b. 8%
15–2. What is the approximate prevalence of red cell
alloimmunization in pregnancy? c. 20%
a. 0.1% d. 33%
b. 1% 15–7. All EXCEPT which of the following antibodies are
c. 5% harmless in pregnancy?
d. 8% a. Anti-I
b. Anti-Lua
15–3. Most cases of severe fetal anemia secondary to
alloimmunization are due to all EXCEPT which of c. Anti-Fyb
the following antibodies? d. Anti-Lewis
a. Anti-c
15–8. What percentage of Kell-sensitized cases is due to
b. Anti-d prior transfusion?
c. Anti-D a. 60%
d. Anti-Kell b. 70%
15–4. What is the critical titer for anti-D antibody c. 80%
alloimmunization? d. 90%
a. 1:4
15–9. What is the most common cause of hemolytic
b. 1:8 disease in newborns?
c. 1:16 a. Alpha-thalassemia
d. 1:32 b. ABO incompatibility
15–5. What is the minimum amount of fetal erythrocytes c. Rh alloimmunization
needed in the maternal circulation to provoke d. Glucose-6-phosphate dehydrogenase (G6PD)
sensitization to D antigen? deficiency
a. 0.01 mL
b. 0.1 mL
c. 1 mL
d. 5 mL
Feta l Disorders 93

15–10. Why can ABO incompatibility manifest in firstborn 15–15. The patient from Question 15–13 declines invasive
children, even though there has been no prior testing. Which of the following can you offer to
exposure to pregnancy? serially assess the fetus’s risk for anemia?

C
a. Fetal red cells have more antigenic sites than a. Maternal anti-D titers

H
A
adult cells. b. Peak systolic velocity of the umbilical artery

P
b. Anti-A and anti-B antibodies can cross the

T
c. Peak systolic velocity of the middle cerebral artery

E
placenta early in the first trimester.

R
d. All of the above

1
c. Most group O women have been previously

5
exposed to bacteria possessing A- or B-like 15–16. The patient from Question 15–13 returns for
antigens. sonographic surveillance. The peak systolic
d. None of the above velocity of the middle cerebral artery is 52 cm/sec
at 23 weeks. Based on this graph by Oepkes and
15–11. What percentage of fetuses from Rh D colleagues (2006), what is your next management
alloimmunized pregnancies will develop mild to step?
moderate anemia?

)
140
a. 5%

c
Fe tus without a ne mia or with mild a ne mia

e
e
h
s
120 Fe tus with s eve re a ne mia

/
b. 15%

t
m
n
i
c
(
c. 25%
y
100
t
y
i
r
c
e
o
d. 50% t
l
r
e
a
v
l
80
60
c
a
i
r
l
15–12. Of these, what percentage will progress to hydrops if
b
o
t
e
40
s
r
left untreated?
y
e
s
c
k
20
e
a. 0.1%
a
l
d
e
d
P
b. 5% 0
i
m
0 16 18 20 22 24 26 28 30 32 34 36 38 40
c. 15% Ge s ta tiona l a ge (we e k)
d. 25%
Reproduced with permission from Cunningham FG, Leveno KJ, Bloom SL, et al (eds):
Fetal disorders. In Williams Obstetrics, 24th ed. New York, McGraw-Hill, 2014, Figure
15–13. A 42-year-old G3P2 presents for prenatal care at 12 15-1.
weeks’ gestation. She is Rh D-negative and has a
positive antibody screen with anti-D antibodies. She a. Perform fetal blood transfusion
has a firstborn child who needed a blood transfusion
b. Perform fetal blood sampling to assess fetal
at birth for mild anemia. What management strategy
hematocrit
would you implement next?
c. Perform amniocentesis to measure amnionic fluid
a. Test the patient for titers of anti-D antibodies
bilirubin concentration
b. Test the father of the baby for red-cell antigens
d. Continue serial measurement of the peak systolic
c. Perform amniocentesis to assess fetal antigen type velocity of the middle cerebral artery
d. Perform fetal blood sampling to assess fetal
hematocrit 15–17. What is the main reason that peak systolic velocity
of the middle cerebral artery increases as fetal anemia
15–14. The patient from Question 15–13 confirms worsens?
paternity, and the patient’s husband is a heterozygote a. Cardiac output increases.
at the D locus. Which of the following is an
b. Blood viscosity decreases.
appropriate next step?
c. Fetus shunts blood preferentially to the brain.
a. Expectant management
d. All of the above
b. Amniocentesis to assess fetal antigen type
c. Serial measurement of maternal anti-D titers
d. Fetal blood sampling to assess fetal hematocrit
94 The Feta l Pa tient

15–18. What should be the next management step if fetal 15–22. What is the overall survival rate after fetal blood
anemia is suspected by noninvasive methods at transfusion for fetuses of alloimmunized pregnancies
36 weeks’ gestation? that are similar to the one seen in these sonograms?
a. Proceed with delivery
S
E
C
b. Perform fetal blood transfusion
T
I
c. Administer corticosteroids for fetal lung
O
N
maturation
5
d. Continue serial measurement of the peak systolic
velocity of the middle cerebral artery

15–19. Compared with Doppler measurement of the peak


systolic velocity of the middle cerebral artery (MCA),
which of the following is true regarding the Liley
curve for evaluation of fetal anemia?
a. It is a less invasive assessment tool.
b. It leads to lower pregnancy loss rates.
c. It is less sensitive to identify fetal anemia.
d. It leads to lower rates of further
alloimmunization. A

15–20. Red cells used for fetal transfusion should have


which of the following characteristics?
a. Irradiated
b. Leukocyte enriched
c. Same ABO group as the mother
d. An approximate hematocrit of 50%

15–21. What is the risk of death after fetal blood transfusion


for the fetus of an alloimmunized pregnancy?
a. 0.1%
b. 3%
c. 11%
d. 22%
B

a. 15%
b. 40%
c. 75%
d. 90%

15–23. Which of the following statements is true regarding


a 300-µg dose of anti-D immunoglobulin?
a. It has a half-life of 6 weeks.
b. It reduces the risk of alloimmunization to less
than 2%.
c. It will worsen alloimmunization if given to a
previously D-sensitized patient.
d. It will provide protection from a fetomaternal
hemorrhage of approximately 30 mL of fetal red
cells.
Feta l Disorders 95

15–24. An Rh D-negative pregnant patient most likely may 15–27. The image here shows a test used to assess
become alloimmunized after all EXCEPT which of fetomaternal hemorrhage. Which of the following is
the following situations? n o T a limitation of this test?

C
a. Amniocentesis

H
A
b. Abdominal trauma

P
T
c. Platelet transfusion

E
R
d. Administration of anti-D immune globulin

1
5
15–25. Which test should be performed to quantify the
appropriate dose of anti-D immune globulin needed
to treat an at-risk patient?
a. Rosette test
b. Indirect coombs
c. Kleihauer-Betke test
d. All of the above

15–26. A patient at 29 weeks’ gestation presents with heavy Reproduced with permission from Cunningham FG, Leveno KJ, Bloom SL, et al (eds):
vaginal bleeding after a motor vehicle accident. The Fetal disorders. In Williams Obstetrics, 24th ed. New York, McGraw-Hill, 2014, Figure
15-4.
fetal heart rate tracing is shown here. Ultimately,
what procedure could be helpful in the management
of this patient? a. It is labor intensive.
b. It is only qualitative.
c. It is less accurate at term.
d. It is less accurate in the setting of maternal
hemoglobinopathy.

15–28. What is the most common cause of severe neonatal


thrombocytopenia?
a. Neonatal viral infection
b. Alloimmune thrombocytopenia
c. Idiopathic thrombocytopenia
d. Gestational thrombocytopenia

15–29. Which human platelet antigen (HPA) is the


most common in cases of neonatal alloimmune
thrombocytopenia?
Reproduced with permission from Cunningham FG, Leveno KJ, Bloom SL, et al (eds): a. HPA-1a
Fetal assessment. In Williams Obstetrics, 24th ed. New York, McGraw-Hill, 2014, Figure b. HPA-1b
24-10.
c. HPA-3a
d. HPA-5b
a. Kleihauer-Betke test
b. Fetal blood transfusion 15–30. What is the approximate risk of fetal or neonatal
c. Sonographic measurement of the peak systolic intracranial hemorrhage in the setting of alloimmune
velocity of the middle cerebral artery thrombocytopenia?
d. All of the above a. 0.5%
b. 7%
c. 18%
d. 33%
96 The Feta l Pa tient

15–31. Your pregnant patient has a firstborn child affected 15–34. An Rh D-positive patient is referred for evaluation
by neonatal alloimmune thrombocytopenia and because of the sonographic fetal finding shown here.
associated intracranial hemorrhage. The management If this finding is isolated, which of the following
of her pregnancy should involve all EXCEPT which statements is true?
S
E
of the following interventions?
C
T
a. Prednisone
I
O
b. Cesarean delivery
N
c. Intravenous immunoglobulins
5
d. Maternal platelet transfusion

15–32. All EXCEPT which of the following are treatable


causes of nonimmune hydrops?
a. Monosomy X
b. Chylothorax
c. Tachyarrhythmia
d. Parvovirus B19 infection

15–33. What is the aneuploidy risk for a hydropic fetus


detected in the first trimester?
a. 25% a. The risk of aneuploidy is decreased.
b. 33% b. The fetus can be considered hydropic.
c. 50% c. The condition may resolve with treatment.
d. 66% d. None of the above

Reference
Oepkes D, Seaward PG, Vandenbussche FP, et al: Doppler ultraso-
nography versus amniocentesis to predict fetal anemia. N Engl J Med
355:156, 2006
Feta l Disorders 97

CHAPTER 15 ANSw ER KEy

Q uestion Letter Pa ge

C
number a nswer cited Hea der cited

H
A
P
15–1 d p. 306 Feta l Anemia

T
E
15–2 b p. 306 Red Cell Alloimmuniza tion

R
1
15–3 b p. 306 Red Cell Alloimmuniza tion

5
15–4 c p. 307 Alloimmuniza tion Detection
15–5 b p. 307 CDE (Rh) Blood G roup Incompa tibility
15–6 a p. 307 CDE (Rh) Blood G roup Incompa tibility
15–7 b p. 308 Ta ble 1 5 -1
15–8 d p. 308 Kell Alloimmuniza tion
15–9 b p. 308 ABO Blood G roup Incompa tibility
15–10 c p. 308 ABO Blood G roup Incompa tibility
15–11 c p. 309 Ma na gement of the Alloimmunized Pregna ncy
15–12 d p. 309 Ma na gement of the Alloimmunized Pregna ncy
15–13 b p. 309 Determining Feta l Risk
15–14 b p. 309 Determining Feta l Risk
15–15 c p. 309 Determining Feta l Risk
15–16 b p. 310 Middle Cerebra l Artery Doppler Velocimetry; Figure 1 5 -1
15–17 d p. 310 Middle Cerebra l Artery Doppler Velocimetry
15–18 a p. 310 Feta l Blood Tra nsfusion
15–19 c p. 310 Amniotic Fluid Spectra l Ana lysis
15–20 a p. 310 Feta l Blood Tra nsfusion
15–21 b p. 311 O utcomes
15–22 c p. 311 O utcomes
15–23 b p. 311 Prevention of Rh D Alloimmuniza tion
15–24 d p. 311 Prevention of Rh D Alloimmuniza tion; Ta ble 1 5 -2
15–25 c p. 311 Prevention of Rh D Alloimmuniza tion
15–26 d p. 312 Fetoma terna l Hemorrha ge
15–27 b p. 313 Tests for Fetoma terna l Hemorrha ge
15–28 b p. 313 Alloimmune Thrombocytopenia
15–29 a p. 313 Alloimmune Thrombocytopenia
15–30 b p. 313 Alloimmune Thrombocytopenia
15–31 d p. 313 Alloimmune Thrombocytopenia
15–32 a p. 315 N onimmune Hydrops
15–33 c p. 315 N onimmune Hydrops
15–34 c p. 316 Isola ted Effusion or Edema
98

CHAPTER 16

Fetal Therap

16–1. All EXCEPT which of the following are examples of 16–3. What is the most common fetal arrhythmia?
fetal conditions that may be amendable to medical a. Atrial flutter
therapy delivered transplacentally?
b. Congenital heart block
a. Dandy-Walker malformation
c. Supraventricular tachycardia
b. Supraventricular tachycardia
d. Premature atrial contraction
c. Congenital adrenal hyperplasia
d. Congenital cystic adenomatoid malformation 16–4. A 16-year-old primigravida is referred for
sonographic examination because of a suspected fetal
16–2. When a fetal rhythm disturbance is suspected, arrhythmia. The following image is obtained and
what type of sonography should be performed to demonstrates a premature atrial contraction (blue
determine the relationship between the atrial and arrow). What is the most appropriate treatment for
ventricular rates? this condition?
a. M-mode a. Digoxin
b. Power Doppler b. Amiodarone
c. 3-dimensional c. Corticosteroids
d. Color Doppler d. None of the above

Reproduced with permission from Cunningham FG, Leveno KJ, Bloom SL, et al (eds): Fetal imaging. In Williams
Obstetrics, 24th ed. New York, McGraw-Hill, 2014, Figure 10-24.
Feta l Thera py 99

16–5. For the patient in Question 16-4, you counsel her 16–8. A fetal tachyarrhythmia is defined as sustained if it is
that this condition can progress to supraventricular present for what percentage of the time?
tachycardia in approximately what percentage of a. 10%

C
fetuses?
b. 25%

H
a. 2%

A
c. 50%

P
b. 5%

T
d. 75%

E
R
c. 15%

1
d. 35% 16–9. A 28-year-old G2P1 at 25 weeks’ gestation is

6
referred for fetal echocardiography for a suspected
16–6. All EXCEPT which of the following are tachyarrhythmia. The image below is obtained
characteristic of fetal supraventricular tachycardia? confirming atrial flutter with a 2:1 atrioventricular
a. Can be caused by an ectopic focus block. What is typically used as first-line treatment
in this situation? A = atrial contractions, V =
b. Displays a varying degree of atrioventricular block
ventricular contractions.
c. Has a ventricular rate of 180 to 300 beats per
a. Digoxin
minute
b. Flecainide
d. All are characteristic
c. Amiodarone
16–7. All EXCEPT which of the following conditions can d. Procainamide
cause fetal sinus tachycardia?
a. Fetal anemia
b. Maternal fever
c. Fetal accessory pathway
d. Maternal thyrotoxicosis

Reproduced with permission from Cunningham FG, Leveno KJ, Bloom SL, et al (eds): Fetal therapy. In Williams
Obstetrics, 24th ed. New York, McGraw-Hill, 2014, Figure 16-1.
100 The Feta l Pa tient

16–10. For the patient in Question 16-9, what additional 16–15. The couple in Question 16-14 plan to conceive soon
sonographic finding in the fetus would make the and understand that dexamethasone has been used in
treatment of this condition less effective? pregnant women to prevent virilization of a female
a. Hydramnios fetus. You inform them that treatment, if elected,
S
E
needs to be initiated by what gestational age?
C
b. Hydrops fetalis
T
a. 6 weeks
I
c. An associated atrial-septal defect
O
b. 9 weeks
N
d. A ventricular rate of 220 beats per minute
5
c. 12 weeks
16–11. What is the most common cause of congenital heart d. 15 weeks
block in a structurally normal fetal heart?
a. Congenital syphilis 16–16. A 22-year-old primigravida presents at 29 weeks’
gestation for prenatal care enrollment. During
b. Maternal diabetes mellitus
routine sonographic examination, the following fetal
c. Maternal anti-SSA or anti-SSB antibodies chest mass (C) is identified. You suspect a congenital
d. Maternal administration of beta-blocking cystic adenomatoid malformation. You counsel her
medications that if these masses become large enough, the fetus is
at risk for which of the following?
16–12. Third-degree heart block in a fetus can be
successfully reversed with maternal administration of
which of the following medications?
a. Sotalol
b. Dexamethasone
c. Betamethasone
d. None of the above

16–13. More than 90 percent of cases of congenital adrenal


hyperplasia are caused by a deficiency of which of
the following enzymes?
a. 21-Hydroxylase
b. 11β -Hydroxylase
c. 17α -Hydroxylase
d. 3β -Hydroxysteroid dehydrogenase

16–14. A 28-year-old G1P1 presents with her husband for


preconceptional counseling since her daughter was Reproduced with permission from Cunningham FG, Leveno KJ, Bloom SL, et al (eds):
Fetal imaging. In Williams Obstetrics, 24th ed. New York, McGraw-Hill, 2014, Figure
born with nonclassic congenital adrenal hyperplasia 10-18A.
(CAH) and evidence of genital virilization. Her
husband was the father of that child also. You a. Hydrops fetalis
discuss the inheritance pattern of this condition and
inform her that approximately what proportion of b. Mediastinal shift
their offspring would be at risk for CAH? c. Depressed cardiac output
a. ½ d. All of the above
b. ¼
16–17. For the patient in Question 16-16, you calculate
c. ⅛ a congenital cystic adenomatoid malformation-
d. 1 16
volume-ratio (CVR) of 1.9. What is the next most
appropriate step in management based on this
calculation?
a. Observation
b. Open fetal surgery
c. Medical therapy in pregnancy
d. Percutaneous thoraco-amnionic shunt placement
Feta l Thera py 101

16–18. Once a fetal goiter is identified, it is important to 16–23. A 28-year-old G2P1 at 19 weeks’ gestation is referred
guide therapy to determine whether this finding is for a targeted sonographic evaluation because of
associated with hyper- or hypothyroidism. Sampling an elevated maternal serum alpha-fetoprotein level.

C
which of the following is the preferred method to The following spinal defect is appreciated and is

H
make this determination? consistent with a myelomeningocele (arrowheads

A
P
a. Placenta indicate nerve roots; arrow marks the cephalad junction

T
of skin and bulging meningeal sac). She inquires about

E
b. Fetal blood

R
possible fetal surgery. You counsel her that infants

1
c. Maternal blood who underwent prenatal surgery in the Management

6
d. Amnionic fluid of Myelomeningocele Study (MOMS) had which of
the following outcomes compared with those that
16–19. What is the clinical goal of treating fetal thyroid had postnatal surgery?
disorders associated with a goiter?
a. Prevent hydramnios
b. Decrease the risk of labor dystocia
c. Correct the underlying physiological abnormality
d. All of the above

16–20. Which of the following conditions has


been successfully treated with fetal stem-cell
transplantation?
a. Beta thalassemia
b. Sickle-cell anemia
c. Maple-syrup urine disease
d. Severe combined immunodeficiency syndrome

16–21. Consideration for fetal surgery should be limited to


those situations in which all EXCEPT which of the Reproduced with permission from Cunningham FG, Leveno KJ, Bloom SL, et al (eds): Fetal
imaging. In Williams Obstetrics, 24th ed. New York, McGraw-Hill, 2014, Figure 10-7.
following criteria are met?
a. The defect is isolated. a. They experienced lower hindbrain herniation
b. The defect results in a high likelihood of death. rates.
c. The safety and efficacy of the procedure is b. They were more likely to walk independently at
proven. 30 months.
d. There is an animal model for the defect and c. They were less likely to require
procedure. ventriculoperitoneal shunting by 1 year of age.
d. All of the above
16–22. What is the first nonlethal birth defect for which
fetal surgery has been offered? 16–24. The patient in Question 16–23 asks about the
a. Spina bifida risks of fetal surgery. You counsel her that in the
b. Sacrococcygeal teratoma MOMS trial, all EXCEPT which of the following
c. Extralobar pulmonary sequestration morbidities occurred more frequently in the prenatal
surgery group compared with the postnatal surgery
d. Congenital cystic adenomatoid malformation
group?
a. Preterm delivery
b. Placental abruption
c. Maternal hypertension
d. Maternal pulmonary edema
102 The Feta l Pa tient

16–25. Shown in the figure is a large sacrococcygeal 16–28. What is the most significant risk factor for mortality
teratoma. Hydrops fetalis can occur in these in fetuses with isolated congenital diaphragmatic
situations as a result of which of the following? hernia?
a. Bowel herniation
S
E
C
b. Liver herniation
T
I
c. Mediastinal shift
O
N
d. Low residual lung volumes
5
16–29. The lung-to-head ratio is a measurement used to
predict survival in fetuses with isolated left-sided
congenital diaphragmatic hernia. This ratio is
calculated by dividing the right lung area by which
of the following?
a. Brain volume
b. Head circumference
c. Biparietal diameter
d. Occipitofrontal diameter

16–30. What is the most common etiology of a primary


pulmonary effusion in the fetus?
Used with permission from Dr. Michael Zaretsky.
a. Aneuploidy
b. Chylothorax
a. Lymphatic obstruction c. Viral infection
b. Congestive heart failure d. Extralobar pulmonary sequestration
c. High-output heart failure
d. None of the above 16–31. What is the most common cause of bladder outlet
obstruction?
16–26. In the United States, which of the following twin a. Urethral cysts
pregnancies would be candidates for fetoscopic laser b. Urethral atresia
ablation therapy for twin-twin transfusion syndrome c. Prune-belly syndrome
(TTTS)?
d. Posterior urethral valves
a. Monochorionic diamnionic twins at 23 weeks’
gestation with stage I TTTS 16–32. All EXCEPT which of the following would generally
b. Dichorionic diamnionic twins at 19 weeks’ be considered contraindications to vesicoamnionic
gestation with stage II TTTS shunt placement in fetuses with bladder outlet
c. Monochorionic diamnionic twins at 15 weeks’ obstruction?
gestation with stage IV TTTS a. Female sex
d. Monochorionic diamnionic twins at 21 weeks’ b. Aneuploidy
gestation with stage III TTTS c. Presence of renal cysts
16–27. Reasonable expectations of laser ablation for severe d. Urinary sodium of 80 mmol/L
twin-twin transfusion syndrome include what
anticipated fetal mortality rate following therapy?
a. 5–10%
b. 10–25%
c. 30–50%
d. 60–80%
Feta l Thera py 103

CHAPTER 16 ANSw ER KEy

Q uestion Letter Pa ge

C
number a nswer cited Hea der cited

H
A
P
16–1 a p. 321 Medica l Thera py

T
E
16–2 a p. 321 Arrhythmia s

R
1
16–3 d p. 321 Prema ture Atria l Contra ctions

6
16–4 d p. 321 Prema ture Atria l Contra ctions
16–5 a p. 321 Prema ture Atria l Contra ctions
16–6 b p. 321 Ta chya rrhythmia s
16–7 c p. 321 Ta chya rrhythmia s
16–8 c p. 321 Ta chya rrhythmia s
16–9 a p. 321 Ta chya rrhythmia s
16–10 b p. 321 Ta chya rrhythmia s
16–11 c p. 322 Bra dya rrhythmia
16–12 d p. 322 Bra dya rrhythmia
16–13 a p. 323 Congenita l Adrena l Hyperpla sia
16–14 c p. 323 Congenita l Adrena l Hyperpla sia
16–15 b p. 323 Congenita l Adrena l Hyperpla sia
16–16 d p. 323 Congenita l Cystic Adenoma toid Ma lforma tion
16–17 c p. 323 Congenita l Cystic Adenoma toid Ma lforma tion
16–18 b p. 324 Thyroid Disea se
16–19 d p. 324 Thyroid Disea se
16–20 d p. 324 Feta l Stem Cell Tra nspla nta tion
16–21 c p. 324 Surgica l Thera py; Ta ble 1 6 -1
16–22 a p. 325 Myelomeningocele Surgery
16–23 d p. 325 Myelomeningocele Surgery
16–24 c p. 325 Myelomeningocele Surgery; Ta ble 1 6 -3
16–25 c p. 327 Sa crococcygea l Tera toma
16–26 d p. 327 Twin-Twin Tra nsfusion Syndrome
16–27 c p. 327 Twin-Twin Tra nsfusion Syndrome; Complica tions
16–28 b p. 328 Congenita l Dia phra gma tic Hernia
16–29 b p. 328 Lung-Hea d Ra tio
16–30 b p. 329 Thora cic Shunts
16–31 d p. 330 Urina ry Shunts
16–32 d p. 330 Urina ry Shunts; Ta ble 1 6 -4
104

CHAPTER 17

Fetal Assessment

17–1. What are the goals of antepartum fetal surveillance? 17–4. Common fetal movement counting protocols
a. Prevent fetal death include which of the following?
b. Indicate timing of intervention a. 10 movements in 1 hour
c. Improve negative predictive values for antepartum b. 10 movements in 2 hours
testing c. Informal maternal perceptions of fetal activity are
d. All of the above clinically meaningless.
d. Monitor 2 hours daily; an accepted count equals
17–2. Concerning antepartum testing, positive predictive or exceeds a previously established baseline count.
values for true-positive abnormal test results
approximate which of the following? 17–5. The figure below illustrates which of the following?
a. 5–10% a. Minimal counts of fetal movement occur at term.
b. 10–40% b. Maximal counts of fetal movement occur at
c. 40–60% 32 weeks.
d. 60–80% c. Daily movement counts reach approximately
16 at 29 weeks.
17–3. Regarding sleep cyclicity (sleep-wake cycles), which d. Declining amnionic fluid volume and space
of the following is true? account for decreased fetal movements at
a. It varies from 5 to 120 minutes. 32 weeks.
b. Mean length of inactive state for term fetuses is
23 minutes.
c. In normal term pregnancies, the longest period of
inactivity is 68 minutes.
d. None of the above

700
)
.
M
r
h
.
600 (11) (15)
E
2
(9)
.
1
S
(16)
/
(25)
g
±
(16) (15) (16)
n
500 (26)
i
n
(8) (9)
d
a
(29)
r
(17)
o
e
(17)
m
c
400 (3)
e
(6)
(
10
r
)
(3) 12
s
e
t
g
n
300 (3)
a
e
r
m
e
v
e
a
v
200
o
y
m
l
k
e
l
100
a
e
t
w
e
F
a
(
0
18 20 22 24 26 28 30 32 34 36 38 40
We e ks of pre gna ncy

Reproduced with permission from Cunningham FG, Leveno KJ, Bloom SL, et al (eds): Antepartum assessment.
In Williams Obstetrics, 23rd ed. New York, McGraw-Hill, 2010, Figure 15-2.
Feta l Assessment 105

17–6. Which of the following is true regarding fetal 17–9. The type of breathing displayed below has been
breathing motions measured in this figure? called which of the following?

C
50
)
t
H
n
e
A
c
r
P
e
40

T
p
(
E
g
R
n
i
h
1
t
30

7
a
e
r
b
t
n
e
20 Fe ta l Fe ta l
p
s
che s t a bdome n
e
m
i
Ins pira tion
T
10 A
8 am Noon 4 pm 8 pm Mid- 4 am 8 am
night
Time of day
Reproduced with permission from Cunningham FG, Leveno KJ, Bloom SL, et al (eds):
Antepartum assessment. In Williams Obstetrics, 24th ed. New York, McGraw-Hill, 2014,
Figure 17-4.

a. There is clear diurnal variation.


b. They occur randomly during 24 hours.
c. There is a sharp increase in the early morning Fe ta l tion
Expira Fe ta l
hours while the mother sleeps. che s t a bdome n

d. B and C B Expira tion

17–7. Concerning contraction stress testing, which of the Reproduced with permission from Cunningham FG, Leveno KJ, Bloom SL, et al (eds):
Antepartum assessment. In Williams Obstetrics, 24th ed. New York, McGraw-Hill, 2014,
following is true? Figure 17-3.
a. Is positive if late decelerations follow at least 40%
of contractions. a. Anatomical
b. Is unsatisfactory if there are fewer than three b. Paradoxical
contractions in 10 minutes. c. Diaphragmatic
c. If done with oxytocin, uses a dilute intravenous
d. Late term fetal breathing
infusion initiated at 1 mU/min.
d. Is reactive if two fetal heart rate accelerations are
noted in 10 minutes.

17–8. Using nipple stimulation for contraction stress


testing has which of the following advantages?
a. No side effects
b. Shorter duration of testing time
c. Cheaper than oxytocin contraction stress testing
d. B and C
106 The Feta l Pa tient

17–10. Which of the following describes the fetal heart rate 17–12. In one study at Parkland Hospital, when
tracing in panel A? accelerations were absent for 80 minutes on a
a. Has good variability continuous fetal heart rate tracing, which finding(s)
had a > 50 percent prevalence?
S
b. Has a baseline of 150
E
a. Fetal acidemia
C
c. Shows good accelerations
T
b. Oligohydramnios
I
d. Does not contain late decelerations
O
N
c. Fetal-growth restriction
5
d. B and C

240 240
A
210 210

180 180

150 150

120 120

90 90

60 60

30 30

100 100

80 80

60 60

40 40

20 20

0 0

240 240
B
210 210

180 180

150 150

120 120

90 90

60 60

30 30

Reproduced with permission from Cunningham FG, Leveno KJ, Bloom SL, et al (eds): Antepartum assessment. In
Williams Obstetrics, 24th ed. New York, McGraw-Hill, 2014, Figure 17-8.

17–11. A “terminal cardiotocogram” includes which of the 17–13. Biophysical profiles are composed of all EXCEPT
following? which of the following?
a. Occasional accelerations a. Fetal tone
b. Spontaneous decelerations b. Fetal breathing
c. Late decelerations with spontaneous contractions c. Contraction stress test
d. Baseline oscillation greater than 5 beats per d. Amnionic fluid volume measurement
minute
Feta l Assessment 107

17–14. This graphic illustrates which of the following 17–17. Which of the following is true of Doppler
concepts regarding the biophysical profile? velocimetry, as applied to fetal surveillance?
a. Characterizes downstream impedance

C
7.40
b. Is superior to other modes of fetal surveillance

H
A
c. Focuses primarily on the systolic waveform of

P
T
7.35 blood flow

E
H
R
* d. Is recommended by the American College of
p
1
s
Obstetricians and Gynecologists in the presence

7
u
7.30
o
of other abnormal fetal tests
n
e
v
*
l
a
17–18. Of these three umbilical artery velocimetry
c
i
7.25
l
i
b
waveforms, which is normal?
m
u
m
7.20
u
*
t
r
a
p
e
t
n
A
7.10

*
7.05
10 8 6 4 2 0
Fe ta l biophys ica l profile s core

Reproduced with permission from Cunningham FG, Leveno KJ, Bloom SL, et al (eds): A
Antepartum assessment. In Williams Obstetrics, 24th ed. New York, McGraw-Hill, 2014,
Figure 17-9.

a. Score of 2/10 is associated with acidemia.


b. Score of 6/10 is indication to proceed with
delivery.
c. Score of 4/10 is similar to 6/10 in regard to
umbilical venous pH.
d. Score of 8/10 is associated with an umbilical
venous pH ≤ 7.30.

17–15. Which of the following is true of a modified


biophysical profile? B

a. Is superior to other forms of fetal surveillance


b. Combines nonstress testing (NST) with fetal
breathing assessment
c. Is normal if the amnionic fluid index (AFI) =
5.1 cm and NST is reactive
d. Has a false-negative rate of 4.8 per 1000 and a
false-positive rate of 1.5%

17–16. Which of the following statements is true regarding


the modified biophysical profile? C
a. It is associated with a false-negative rate of 4.8 per
Reproduced with permission from Cunningham FG, Leveno KJ, Bloom SL, et al (eds):
1000. Fetal imaging. In Williams Obstetrics, 24th ed. New York, McGraw-Hill, 2014, Figure
b. It is associated with a false-positive rate of 1.5 per 10-39.
1000.
c. The American College of Obstetricians and a. A
Gynecologists agrees that it is as good as any b. B
other fetal test of well-being. c. C
d. None of the above d. None of the above
108 The Feta l Pa tient

17–19. In the image in Question 17-18, which waveform 17–23. Concerning antenatal fetal testing, which of the
demonstrates reversed end-diastolic flow? following is true?
a. A a. It is clearly beneficial.
S
b. B b. It should be considered experimental.
E
C
c. C c. The American College of Obstetricians and
T
I
d. None of the above Gynecologists recommends contraction stress
O
N
testing as the best test to evaluate fetal well-being.
17–20. Concerning middle cerebral artery Doppler
5
d. Despite its widespread use, the nonstress test is
velocimetry, which of the following is true? inferior to uterine artery Doppler velocimetry at
a. It is useful for detection and management of fetal predicting marginally poor cognitive outcomes.
anemia of any cause.
b. It is superior to the modified biophysical profile 17–24. During acoustic stimulation testing, which fetal
in forecasting pregnancy outcomes. response is measured?
c. In those with brain sparing, decreased blood a. Breathing
flow from reduced cerebrovascular impedance is b. Heart rate
detected. c. Eye movement
d. It was found to be inferior to amniocentesis and d. Body movement
amnionic fluid spectral analysis for predicting
fetal anemia. 17–25. What controls fetal heart rate accelerations?
17–21. Doppler interrogation of which of the following is a. Aortic baroreceptor reflexes
the best predictor of perinatal outcome at 26 to b. Carotid baroreceptor reflexes
33 weeks’ gestation in growth-restricted fetuses? c. Autonomic function at the brainstem level
a. Ductus venosus d. Humoral factors such as atrial natriuretic peptide
b. Uterine artery
c. Umbilical artery 17–26. Your patient is a 24-year-old G1P0 at 33 weeks’
gestation who comes to Labor and Delivery
d. Middle cerebral artery obtunded with the following fetal hear rate tracing.
17–22. Concerning uterine artery Doppler velocimetry, After obtaining intravenous access and ordering
which of the following is true? laboratory tests, you obtain the following initial
results: glucose 400 mg/dL, hematocrit 28 volume
a. Vascular resistance decreases in the first half of percent. Which of the following best explains this
pregnancy. tracing?
b. Low-resistance patterns are associated with
a. Abruption
preeclampsia.
c. Low-resistance patterns have been linked to b. Fetal immaturity
various pregnancy complications. c. Maternal acidosis
d. It is not helpful in assessing pregnancies at high d. Sinusoidal pattern from fetal anemia
risk due to uteroplacental insufficiency.
240 240

210 210

180 180

150 150

120 120

90 90

60 60

30 30

100 100

80 80

60 60

40 40

20 20

0 0
Reproduced with permission from Cunningham FG, Leveno KJ, Bloom SL, et al (eds): Antepartum assess-
ment. In Williams Obstetrics, 24th ed. New York, McGraw-Hill, 2014, Figure 17-8A.
Feta l Assessment 109

17–27. What should your clinical response be to these 17–30. The following week, umbilical artery Doppler
findings? studies appear as shown here for the patient in
a. Initiate management of diabetic ketoacidosis Question 17-28. The nonstress test now has no

C
variability and no accelerations. The best course of
b. Transfuse packed red blood cells for maternal

H
action is which of the following?

A
anemia

P
T
c. Induce labor for nonreassuring fetal heart rate

E
tracing

R
1
d. Perform cesarean delivery for nonreassuring fetal

7
heart rate tracing

17–28. Your patient is a 23-year-old African American


woman at 27 weeks’ gestation. Sonographic
evaluation reveals no structural abnormalities,
normal amnionic fluid index, and an estimated fetal
weight measuring below the 3rd centile. During Reproduced with permission from Cunningham FG, Leveno KJ, Bloom SL, et al (eds):
Antepartum assessment. In Williams Obstetrics, 23rd ed. New York, McGraw-Hill, 2010,
umbilical artery Doppler interrogation, the following Figure 15-11C.
waveform is obtained. What pattern is displayed
here? a. Move toward delivery
b. Continue ward rest with intensified surveillance
c. Neuroprophylaxis with magnesium sulfate and
corticosteroids for fetal maturity if not previously
given
d. B and C

17–31. The risk of fetal death with elevated resistance in


uterine artery Doppler studies is increased when
Reproduced with permission from Cunningham FG, Leveno KJ, Bloom SL, et al (eds): associated with all EXCEPT which of the following?
Antepartum assessment. In Williams Obstetrics, 23rd ed. New York, McGraw-Hill, 2010,
Figure 15-11B.
a. Preeclampsia
b. Placental abruption
a. Absent end-diastolic flow c. Chronic hypertension
b. Reversed end-diastolic flow d. Fetal-growth restriction
c. Elevated systolic/diastolic (S/D) ratio
d. None of the above 17–32. The most important consideration when deciding
to begin antepartum fetal testing is which of the
following?
17–29. For the patient in Question 17-28, a nonstress test is
obtained. It demonstrates 10 × 10 beat-per-minute a. Age of fetus
accelerations and minimal variability. Your next best b. Severity of maternal condition
step is which of the following? c. Prognosis for neonatal survival
a. Emergency cesarean delivery d. Associated complication, such preeclampsia or
b. Neuroprophylaxis with magnesium sulfate fetal-growth restriction
c. Ward rest with fetal surveillance that includes at
least weekly umbilical artery Doppler evaluations 17–33. Which of the following statements is true regarding
antepartum fetal testing?
d. None of the above
a. Has evolved continually, exposing an inability to
find a “best” test
b. Definitely associated with decreased incidence of
cerebral palsy
c. Has been used to forecast fetal “wellness” rather
than fetal “illness”
d. A and C
110 The Feta l Pa tient

CHAPTER 17 ANSw ER KEy

Q uestion Letter Pa ge
S
number a nswer cited Hea der cited
E
C
T
17–1 a p. 335 Introduction
I
O
17–2 b p. 335 Introduction
N
17–3 b p. 335
5
Feta l Movements
17–4 b p. 336 Clinica l Applica tion
17–5 c p. 335 Feta l Movements
17–6 d p. 337 Feta l Brea thing
17–7 b p. 338 Contra ction Stress Testing
17–8 d p. 338 Contra ction Stress Testing
17–9 b p. 337 Feta l Brea thing
17–10 b p. 340 Abnorma l N onstress Tests
17–11 c p. 340 Abnorma l N onstress Tests
17–12 d p. 340 Abnorma l N onstress Tests
17–13 c p. 341 Biophysica l Profile
17–14 a p. 341 Biophysica l Profile
17–15 c p. 341 Biophysica l Profile
17–16 c p. 343 Modified Biophysica l Profile
17–17 a p. 344 Umbilica l Artery Velocimetry
17–18 a p. 344 Umbilica l Artery Velocimetry
17–19 c p. 344 Umbilica l Artery Velocimetry
17–20 a p. 344 Middle Cerebra l Artery
17–21 a p. 345 Ductus Venosus
17–22 a p. 345 Uterine Artery
17–23 b p. 345 Significa nce of Feta l Testing
17–24 b p. 341 Acoustic Stimula tion Tests
17–25 c p. 339 Feta l Hea rt Ra te Accelera tion
17–26 c p. 340 Abnorma l N onstress Tests
17–27 a p. 340 Abnorma l N onstress Tests
17–28 a p. 344 Umbilica l Artery Velocimetry
17–29 c p. 339 Feta l Hea rt Ra te Accelera tion
17–30 d p. 344 Umbilica l Artery Velocimetry
17–31 c p. 345 Uterine Artery
17–32 c p. 345 Current Antena ta l Testing Recommenda tions
17–33 d p. 345 Significa nce of Feta l Testing
Se c t i o n 6

Ea r l y Pr Eg n a n c y c o mPl ic a t io n s
112

c Ha Pt Er 18

ab

18–1. Which o the ollowing gestational ages and weights 18–5. A 24-year-old G3P0A2 presents with 8 weeks o
are typically used to de ine abortion? amenorrhea and vaginal bleeding. Her serum beta
. Less than 12 weeks, less than 100 g human chorionic gonadotropin (β -hCG) level is
68,000 mIU/mL, and her internal cervical os is
b. Less than 12 weeks, less than 250 g
closed. The sonographic uterine indings are shown
. Less than 16 weeks, less than 500 g below, and etal heart motion is noted. Normal
d. Less than 20 weeks, less than 500 g adnexal anatomy is seen. Your diagnosis is which o
the ollowing?
18–2. Approximately what percentage o spontaneous
miscarriages occur in the irst 12 weeks o
pregnancy?
. 20%
b. 40%
. 60%
d. 80%

18–3. Which o the ollowing is the most common cause


o irst-trimester pregnancy loss?
. Uterine anomalies
b. Incompetent cervix
. Intrauterine in ection
d. Fetal chromosomal abnormalities
Reproduced with permission rom Werner CL, Moschos E, Gri th WF, et al (eds): Abor-
tion. In Williams Gynecology, 2nd Edition Study Guide. New York, McGraw-Hill, 2012,
18–4. Which o the ollowing actors is least likely to be Q6–10.
linked with higher irst-trimester miscarriage rates?
. Obesity . Missed abortion
b. Diabetes mellitus b. Incomplete abortion
. Parvovirus in ection . Threatened abortion
d. Maternal age older than 40 years d. Inevitable abortion

18–6. E ective prevention o miscarriage in women with


threatened abortion includes which o the ollowing?
. Bed rest
b. McDonald cerclage
. Human chorionic gonadotropin injection
d. None o the above
Abortion 113

18–7. Anti-D immunoglobulin should be considered 18–9. While in your emergency department, the patient
or Rh-negative women in which o the ollowing in Question 18–8 passes tissue, shown here in a cup
settings? illed with ormalin. Her bleeding and pain have

c
. Threatened abortion now subsided signi icantly. Her cervical os is closed.

H
Your diagnosis is which o the ollowing?

a
b. Following complete hydatidi orm mole

P
t
evacuation

E
r
. A ter irst-trimester elective pregnancy

1
termination

8
d. All o the above

18–8. The same patient in Question 18–5 represents


2 weeks later with light bleeding and strong, pain ul
cramps. Her blood pressure is 132/78, pulse is 72,
and she is a ebrile. Her cervical os is closed, and
hematocrit is 40 volume percent. Transvaginal
sonography reveals the sagittal uterine image below.
Appropriate management includes which o the
ollowing?

Used with permission rom Dr. Heather Lytle.

. Missed abortion
b. Complete abortion
. Threatened abortion
d. None o the above

18–10. Appropriate management o the patient rom


Question 18–9 now includes which o the ollowing?
. Diagnostic laparoscopy
b. Transvaginal sonography
. Dilatation and curettage
Reproduced with permission rom Werner CL, Moschos E, Gri th WF, et al (eds): Abor- d. Administration o Rho [D] immunoglobulin, i
tion. In Williams Gynecology, 2nd Edition Study Guide. New York, McGraw-Hill, 2012, the patient is Rh negative
Q6–11.

18–11. An 18-year-old G1P0 presents with 12 weeks o


. Per orm rescue cerclage amenorrhea and heavy vaginal bleeding. Her urine
b. Await spontaneous miscarriage pregnancy test is positive. Tissue with the appearance
. Excision o cesarean scar pregnancy o placenta is seen through an open cervical os. Your
d. Administer intramuscular injection o diagnosis and management plan include which o
methotrexate the ollowing?
. Threatened abortion, plan bed rest
b. Incomplete abortion, plan dilatation and
curettage
. Ectopic tubal pregnancy, plan laparoscopic
resection
d. Complete abortion, plan subsequent β -hCG
testing in 48 hours
114 Ea rly Pregna ncy Complica tions

18–12. Incomplete abortion may be treated success ully with 18–16. For the treatment o septic abortion, which o the
which o the ollowing? ollowing antibiotic coverages is most suitable?
. Observation . Gram-positive
s
b. Dilatation and curettage b. Gram-negative
E
c
. Prostaglandin E1 administration . Broad-spectrum
t
i
o
d. All o the above d. Anaerobic, gram-positive
n
6
18–13. A 40-year-old G3P2 presents with 10 weeks o 18–17. With recurrent abortion, which o the ollowing
amenorrhea. Her serum β -hCG level is 25,000 is a commonly ound parental chromosomal
mIU/mL, and her internal cervical os is closed. The abnormality?
ollowing uterine sonographic indings are noted, . X chromosome mosaicism
and the anechoic intrauterine area measures 40 mm.
b. Robertsonian translocations
Normal adnexal anatomy is seen. Your diagnosis is
which o the ollowing? . Balanced reciprocal translocations
d. All o the above

18–18. As shown in this 3-dimensional sonogram, which


o the ollowing müllerian anomalies has been
associated with the highest rates o recurrent
miscarriage?

. Missed abortion
b. Complete abortion
. Threatened abortion
d. Complete hydatidi orm mole

18–14. Missed abortion may be treated success ully with


which o the ollowing? . Septate uterus
. Observation b. Arcuate uterus
b. Dilatation and curettage . Uterine didelphys
. Prostaglandin E1 administration d. Longitudinal vaginal septum
d. All o the above
18–19. Which o the ollowing tests would be the most
18–15. Your patient with a pregnancy at 16 weeks’ gestation e ective in identi ying an underlying cause o
presents with ever (38.5°C) and lower abdominal recurrent miscarriage?
pain, but without bleeding. She describes a small . Antithrombin III assay
leakage o vaginal luid yesterday. Appropriate b. Serum progesterone level
primary management includes intravenous . Lupus anticoagulant assay
antibiotics and which o the ollowing?
d. Luteinizing hormone assay
. Labor induction
b. Bed rest and observation
. Tocolytic administration
d. Hysterotomy and evacuation
Abortion 115

18–20. All EXCEPT which o the ollowing statements is 18–24. You place several laminaria in preparation or
true regarding cervical incompetence? pregnancy termination at 16 week’ gestation. An
. Prior cervical conization is a risk actor. example is seen on the le t, below. The next day

c
your patient chooses NOT to proceed with the
b. It is characterized by painless, second-trimester

H
abortion. You remove the swollen laminaria (one

a
cervical dilatation.

P
example is on the right, below) and counsel her

t
. Rupture o membranes is not a contraindication which o the ollowing?

E
r
to rescue cerclage placement.

1
d. It may be suspected sonographically by

8
membranes unneling past the internal os and
shortening o the cervix to < 25 mm.

18–21. Which o the ollowing statements is true regarding


cerclage?
. The Shirodkar procedure is most o ten selected.
b. Broad-spectrum antibiotic prophylaxis is required.
. It is ideally per ormed at 10 to 12 weeks’
gestation.
d. Prolapsing membranes may be reduced with
Trendelenburg positioning and bladder illing
prior to cerclage placement.

18–22. Your patient underwent McDonald cerclage Reproduced with permission rom Word L, Hof man BL. Surgeries or benign gynecologic
placement 3 days ago at 13 weeks’ gestation and disorders. In Hof man BL, Schorge JO, Schaf er JI, et al (eds): Williams Gynecology, 2nd
ed. New York, McGraw-Hill, 2012, Figure 41-16.1.
presents with complaints o strong cramps. Her
temperature is 38.6°C, pulse 118, and blood pressure
98/66. Speculum examination reveals no pooling
. Observation is recommended.
luid, and her cerclage is in place. Her uterus is b. Abortion will spontaneously occur in most cases.
tender, and etal heart rate is 160 beats per minute. . Oral antimicrobials are required to prevent
Laboratory and physical examination exclude in ection.
urinary, respiratory, or gastrointestinal sources o d. Cerclage placement is required to sustain the
ever. In addition to broad-spectrum antibiotics, pregnancy.
antipyretics, and intravenous luids, how is this
patient best managed? 18–25. During D&C, i the uterine undus is per orated
. Bed rest with one o the instruments shown here, which
b. Tocolysis o the ollowing is the most appropriate primary
. Hysterotomy or uterine evacuation management?
d. Cerclage removal and uterine evacuation

18–23. Which o the ollowing may lower complications


associated with dilatation and curettage?
. Perioperative oral antibiotics
b. Preoperative cervical laminaria
. Preoperative bimanual examination
d. All o the above Reproduced with permission rom Hof man BL: Surgeries or benign gynecologic condi-
tions. In Schorge JO, Schaf er JI, Halvorson LM, et al (eds): Williams Gynecology. New
York, McGraw-Hill, 2008, Figure 41-17.3.

. Observation
b. Hysterectomy
. Abdominal exploration
d. Uterine artery embolization
116 Ea rly Pregna ncy Complica tions

18–26. E ective and sa e combinations used or pregnancy 18–30. Accepted options or second-trimester abortion
termination include all EXCEPT which o the include which o the ollowing?
ollowing? . Intravenous oxytocin
. Misoprostol alone
s
b. Dilatation and evacuation
E
c
b. Mi epristone and misoprostol . Intravaginal prostaglandin E2
t
i
. Methotrexate and misoprostol
o
d. All o the above
n
d. Mi epristone and methotrexate
6
18–31. A common side e ect o prostaglandin E2 includes
18–27. The American College o Obstetricians and which o the ollowing?
Gynecologists supports outpatient medical abortion . Fever
as an acceptable alternative to surgical abortion or
pregnancies below what menstrual age threshold? b. Dysuria
. 5 weeks . Arthralgias
b. 7 weeks d. Somnolence
. 9 weeks 18–32. Postoperatively, elective abortion is associated with
d. 11 weeks subsequent increased rates o which o the ollowing?
. In ertility
18–28. In most cases o elective pregnancy termination,
all EXCEPT which o the ollowing is true b. Mental illness
regarding medical regimens compared with surgical . Ectopic pregnancy
techniques? d. None o the above
. Avoids anesthesia
b. Requires one visit
. Comparable success rate
d. Avoids invasive procedure

18–29. All EXCEPT which o the ollowing are


contraindications to medical abortion?
. Anticoagulant use
b. Type 2 diabetes mellitus
. In situ intrauterine device
d. Concurrent glucocorticoid use
Abortion 117

c Ha Pt Er 18 a n sw Er KEy

Q uestion Letter Pa ge

c
number a nswer cited Hea der cited

H
a
P
18–1 d p. 350 N omencla ture

t
E
18–2 d p. 351 Pa thogenesis

r
1
18–3 d p. 351 Figure 1 8 -1

8
18–4 p. 352 Ma terna l Fa ctors
18–5 p. 354 Threa tened Abortion
18–6 d p. 355 Ma na gement
18–7 d p. 355 Anti-D Immunoglobulin
18–8 b p. 356 Complete Abortion
18–9 b p. 356 Complete Abortion
18–10 d p. 356 Complete Abortion
18–11 b p. 356 Incomplete Abortion
18–12 d p. 356 Incomplete Abortion
18–13 p. 356 Missed Abortion
18–14 d p. 357 Ta ble 1 8 -3
18–15 p. 356 Septic Abortion
18–16 p. 356 Septic Abortion
18–17 d p. 358 Pa renta l Chromosoma l Abnorma lities
18–18 p. 359 Ta ble 1 8 -5
18–19 p. 359 Immunologica l Fa ctors
18–20 p. 360 Cervica l Insufficiency
18–21 d p. 361 Eva lua tion a nd Trea tment; Cercla ge Procedures
18–22 d p. 363 Complica tions
18–23 d p. 364 Techniques for Abortion
18–24 p. 365 Cervica l Prepa ra tion
18–25 p. 367 Complica tions
18–26 d p. 369 Ta ble 1 8 -9
18–27 b p. 368 Medica l Abortion
18–28 b p. 368 Medica l Abortion
18–29 b p. 368 Contra indica tions
18–30 d p. 369 Ta ble 1 8 -1 0
18–31 p. 370 Prosta gla ndins E2 (PG E2 ) a nd E1 (PG E1 )
18–32 d p. 370 Hea lth a nd Future Pregna ncies
118

CHAPTER 19

Ectopic Pregnanc

19–1. Which o the ollowing most accurately de ines 19–3. Which o the ollowing de ines heterotopic
ectopic pregnancy? pregnancy?
a. Composed solely o cytotrophoblast a. One tubal and one abdominal pregnancy
b. Implantation within the allopian tube b. One ectopic and one intrauterine pregnancy
c. Implantation outside the uterine cavity c. Two pregnancies, one in each allopian tube
d. Abnormally rising maternal serum β -human d. Two ectopic pregnancies in one allopian tube
chorionic gonadotropin (β -hCG) level
19–4. Which o the ollowing is least likely to increase the
19–2. As shown here, what is the most common site o risk or ectopic pregnancy?
tubal pregnancy implantation? a. Prior pelvic in ection
b. Prior hydatidi orm mole
c. Prior ectopic pregnancy
d. Salpingitis isthmica nodosum

19–5. With contraceptive ailure, which method has a


relative increased risk o ectopic pregnancy?
a. Condoms
b. Vasectomy
c. Tubal sterilization
d. All o the above

19–6. Which o the ollowing clinical outcomes o tubal


pregnancy is has the lowest potential or maternal
morbidity?

Used with permission rom Dr. Kevin Doody. Reproduced with permission rom Werner
a. Tubal rupture
CL, Moschos E, Gri th WF (eds): Williams Gynecology, 2nd ed. Study Guide. New b. Tubal abortion
York, McGraw-Hill, 2012, p 39.
c. Chronic persistence
a. Cornua d. Pregnancy resorption
b. Fimbria
c. Isthmus
d. Ampulla
Ectopic Pregna ncy 119

19–7. Your patient is a 21-year-old nulligravida currently 19–8. Regarding the patient in Question 19-7, her abdomen
desiring pregnancy and attempting conception. Her is so t and nontender. The cervical os is closed,
last menstrual period was 6 weeks ago. She presents scant blood is seen in the vagina, and uterine size

C
with complaints o vaginal spotting and right lower approximates that o a lemon. What is the next best

H
quadrant pain. She also provides you with tissue clinical step?

A
P
she passed just prior to coming to the emergency a. Obtain urine β -hCG assay

T
department. Without histological evaluation, what

E
b. Per orm transvaginal sonography

R
clinical term best describes this tissue that is shown

1
in photograph A and opened in B? c. Per orm dilatation and curettage

9
d. Discharge her home a ter assessing hematocrit
and Rh status

19–9. For the patient rom Question 19-7, her urine


β -hCG assay is positive, vital signs are normal,
hematocrit is 36 volume percent, and blood type is
O negative. What is the next best clinical step?
a. Per orm transvaginal sonography
b. Administer Rho(D) immunoglobulin
c. Per orm dilatation and curettage
d. Discharge home with plans to reevaluate in
48 hours

a. Decidual cast
b. Blighted ovum
c. Molar pregnancy
d. Products o conception
120 Ea rly Pregna ncy Complica tions

19–10. For the patient rom Question 19-7, during 19–12. For the patient rom Question 19-7, during her
transvaginal sonographic examination, uterine sonographic examination, in addition to the
images are obtained. In general, which o the trilaminar endometrial lining seen in Figure 19-10C,
ollowing sagittal uterine images is most suggestive o other indings include normal myometrium, a
S
E
an intrauterine pregnancy? cul-de-sac without ree luid, and normal adnexa.
C
T
a. A Her serum β -hCG level is 1400 mIU/mL. What is
I
O
her diagnosis?
b. B
N
c. C a. Ectopic pregnancy
6
d. Both A and B b. Completed abortion
c. Intrauterine pregnancy
d. Pregnancy o unknown location

A B

19–11. In Figure 19-10A, the hypoechoic, tear-shaped 19–13. For the patient rom Question 19-7, with this
structure measured by calipers most likely represents diagnosis, which o the ollowing is an appropriate
which o the ollowing? next clinical step?
a. Gestational sac a. Administer methotrexate
b. Intradecidual sign b. Per orm diagnostic laparoscopy
c. Double decidual sign c. Obtain second serum β -hCG level in 48 hours
d. Pseudogestational sac d. Discharge her home with ollow-up appointment
in 2 weeks
Ectopic Pregna ncy 121

19–14. The patient rom Question 19-7 returns in 48 19–16. O the our images above, which is most commonly
hours with minimal right lower quadrant pain. Her seen sonographically at the time ectopic pregnancy is
vital signs are normal, and spotting has abated. Her diagnosed?

C
clinical and laboratory indings are unchanged except a. A

H
or a serum β -hCG level now measuring 2100 mIU/

A
b. B

P
mL and a progesterone level o 15 ng/mL. What is

T
the next best clinical step or this patient? c. C

E
R
a. Administer methotrexate d. D

1
9
b. Repeat transvaginal sonography
c. Per orm dilatation and curettage
d. Schedule repeat serum β -hCG level in 48 hours

19–15. For the patient rom Question 19-7, her second


transvaginal sonographic evaluation reveals a
trilaminar endometrial stripe, no cul-de-sac luid,
and normal le t adnexa. However, a paraovarian
mass is seen on the right. Which o the ollowing
sonographic adnexal images would be most
diagnostic o ectopic pregnancy?
a. A
b. B
c. C
d. D

A B

C D
122 Ea rly Pregna ncy Complica tions

19–17. For the patient rom Question 19-7, color Doppler 19–18. For the patient rom Question 19-7, her sonographic
is applied, and the ollowing image is seen. This images are seen below. Which o the ollowing is the
“ring o ire” may indicate ectopic pregnancy but next best clinical step?
may also be seen with which o the ollowing? a. Per orm dilatation and curettage
S
E
C
b. Obtain second serum β -hCG level in 48 hours
T
I
c. Discuss management o ectopic pregnancy with
O
N
her
6
d. Discuss management o completed abortion with
her

a. Corpus luteum
b. Ovarian endometrioma
c. Ovarian serous cystadenoma
d. Prior Filshie clip application

A B

C D
Ectopic Pregna ncy 123

19–19. Which o the ollowing would now be reasonable to 19–25. Your patient rom Question 19-7 returns on day 4 o
o er the patient rom Question 19-7. methotrexate therapy or β-hCG level surveillance.
a. Salpingectomy Her serum β -hCG level is 2300 mIU/mL, and Hct

C
is 35 volume percent. She notes aching right lower
b. Salpingostomy

H
quadrant pain without vaginal bleeding. During

A
c. Methotrexate administration

P
abdominal and gentle bimanual examination, you

T
d. All the above note no peritoneal signs and minimal tenderness. Her

E
R
vital signs are normal. What is the next clinical step

1
19–20. Contraindications or methotrexate therapy include or this rising serum β -hCG level?

9
all EXCEPT which o the ollowing? a. Plan laparoscopic salpingectomy
a. Breast eeding b. Per orm dilatation and curettage
b. Thrombocytopenia c. Administer second methotrexate dose
c. Migraine headache d. Schedule her day 7 serum β -hCG level blood
d. Intraabdominal hemorrhage draw

19–21. Which o the ollowing would be most closely 19–26. The patient rom Question 19-7 returns on day
associated with methotrexate therapy ailure during 7 and her serum β -hCG level is 2500 mIU/
ectopic pregnancy treatment? mL. Although you counsel her that a second
a. Increased parity methotrexate dose is an option, she now pre ers
de initive surgical intervention. In counseling, which
b. Ectopic size o 2.5 cm o the ollowing is true regarding salpingostomy?
c. Prior ectopic pregnancy a. It poses a higher risk o persistent trophoblastic
d. Serum β -hCG level o 9000 mIU/mL tissue compared with salpingectomy.
b. Laparotomy is the pre erred route due to higher
19–22. Which o the ollowing is NOT typically seen as a associated subsequent pregnancy rates.
component o methotrexate embryopathy?
c. Procedurally, it requires suture closure o the
a. Omphalocele tubal incision with delayed-absorbable suture.
b. Skeletal abnormalities d. It leads to lower uture ertility rates in
c. Fetal-growth restriction subsequent pregnancies compared with
d. Cranio acial abnormalities salpingectomy.

19–23. Which o the ollowing should be avoided during 19–27. Persistent trophoblast ollowing surgical treatment
methotrexate therapy or ectopic pregnancy o ectopic pregnancy is unlikely i the serum
treatment? β -hCG level alls by what minimum percentage on
a. Folic acid postoperative day 1?
b. Vaginal intercourse a. 10
c. Nonsteroidal antiin lammatory drugs b. 25
d. All o the above c. 50
d. 75
19–24. Your patient rom Question 19-7 elects to use
methotrexate. Which o the ollowing is NOT an
expected possible complication o methotrexate
therapy or ectopic pregnancy?
a. Stomatitis
b. Endocarditis
c. Liver toxicity
d. Gastroenteritis
124 Ea rly Pregna ncy Complica tions

19–28. Criteria that may aid the sonographic diagnosis o 19–30. Sonographic indings that may suggest abdominal
interstitial ectopic pregnancy include which o the pregnancy include which o the ollowing?
ollowing? a. Oligohydramnios
S
b. Fetus outside and separate rom the uterus
E
C
c. Absent myometrium between the etus and
T
I
maternal anterior abdominal wall
O
N
d. All o the above
6
19–31. As shown here, what adjunctive imaging modality
may aid the diagnosis o abdominal pregnancy?

a. Empty endometrial cavity


b. Gestational sac separate rom the endometrium
c. Thin myometrial mantle surrounding the
gestational sac
d. All o the above

19–29. As shown in Figure 19-29B, during cornuectomy


or interstitial ectopic pregnancy, what is the most a. Computed tomography
compelling reason to resect the ipsilateral allopian b. Magnetic resonance imaging
tube?
c. Saline in usion sonography
a. Lower her ovarian cancer risk
d. 3-dimensional sonography
b. Avoid hydrosalpinx ormation in the retained
ipsilateral allopian tube
c. Avoid ectopic pregnancy in the retained ipsilateral
allopian tube
d. None o the above A

A B

Used with permission rom Dr. Jenni er Muller.


Ectopic Pregna ncy 125

19–32. What is the most commonly used approach to treat 19–36. Which o the ollowing most signi icantly increases
an abdominal pregnancy at 16 weeks’ gestation? the risk o cervical pregnancy?
a. Intragestational sac methotrexate a. Advanced maternal age

C
b. Laparotomy with delivery o the etus b. In vitro ertilization

H
A
c. Expectant management until etal viability c. Increased cesarean delivery rate

P
T
d. Uterine artery embolization, then await etal and d. Increased incidence o cervical neoplasia

E
R
placental resorption

1
19–37. What is the pre erred treatment or cervical

9
19–33. Criteria proposed by Spiegelberg or the diagnosis o pregnancy in a hemodynamically stable patient?
ovarian pregnancy include all EXCEPT which o the a. Methotrexate
ollowing?
b. Hysterectomy
a. The ectopic pregnancy occupies the a ected
c. Trachelectomy
ovary.
b. Trophoblast are seen histologically amid ovarian d. Cerclage ollowed by dilatation and curettage
stroma.
19–38. This represents which o the ollowing ectopic
c. The pregnancy is attached to the uterus via the pregnancy types?
round ligament.
d. An intact ipsilateral allopian tube is seen distinct
rom the ovary.

19–34. Which o the ollowing is an appropriate treatment


o ovarian ectopic pregnancy?
a. Cystectomy
b. Oophorectomy
c. Ovarian wedge resection
d. All o the above

19–35. As shown by this sonogram image, which o the


ollowing is NOT a sonographic criterion or cervical
pregnancy?

a. Isthmic
b. Cervical
c. Interstitial
d. Cesarean scar

Reproduced with permission rom Moschos E, Hof man BL: Cervical ectopic pregnancy
(update). In Cunningham FG, Leveno KL, Bloom SL, et al (eds): Williams Obstetrics,
22nd ed. Online. Accessmedicine.com. New York, McGraw-Hill, 2007, Figure 1.

a. Hourglass uterine shape


b. Ballooned cervical canal
c. Anterior uterine isthmic mass
d. Hyperechoic thin endometrial stripe
126 Ea rly Pregna ncy Complica tions

19–39. As shown, which o the ollowing is NOT a 19–40. Which o the ollowing may be appropriate
sonographic criterion or cesarean scar pregnancy? treatment o cesarean scar pregnancy in a
hemodynamically stable patient?
a. Methotrexate
S
E
C
b. Hysterectomy
T
I
c. Anterior uterine isthmus resection
O
N
d. All o the above
6
a. Empty cervical canal
b. Hourglass uterine shape
c. Anterior uterine isthmic mass
d. Hyperechoic thin endometrial stripe
Ectopic Pregna ncy 127

CHAPTER 19 ANSw ER KEy

Q uestion Letter Pa ge

C
number a nswer cited Hea der cited

H
A
P
19–1 c p. 377 Introduction

T
E
19–2 d p. 377 Cla ssifica tion

R
1
19–3 b p. 377 Cla ssifica tion

9
19–4 b p. 377 Risks
19–5 c p. 377 Risks
19–6 d p. 378 Evolution a nd Potentia l O utcomes
19–7 a p. 379 Clinica l Ma nifesta tions
19–8 a p. 381 Beta Huma n Chorionic G ona dotropin
19–9 a p. 380 Figure 1 9 -4
19–10 b p. 382 Endometria l Findings
19–11 d p. 382 Endometria l Findings
19–12 d p. 381 Beta Huma n Chorionic G ona dotropin
19–13 c p. 381 Levels below the Discrimina tory Zone
19–14 b p. 381 Levels below the Discrimina tory Zone
19–15 b p. 383 Adnexa l Findings
19–16 a p. 383 Adnexa l Findings
19–17 a p. 383 Adnexa l Findings
19–18 c p. 383 Adnexa l Findings
19–19 d p. 384 Trea tment O ptions
19–20 c p. 384 Ta ble 1 9 -2
19–21 d p. 385 Pa tient Selection
19–22 a p. 384 Regimen O ptions
19–23 d p. 384 Regimen O ptions
19–24 b p. 385 Trea tment Side Effects
19–25 d p. 385 Monitoring Thera py Effica cy
19–26 a p. 385 Surgica l Ma na gement
19–27 c p. 386 Persistent Trophobla st
19–28 d p. 387 Interstitia l Pregna ncy
19–29 c p. 387 Interstitia l Pregna ncy
19–30 d p. 388 Abdomina l Pregna ncy
19–31 b p. 388 Abdomina l Pregna ncy
19–32 b p. 389 Ma na gement
19–33 c p. 390 O va ria n Pregna ncy
19–34 d p. 390 O va ria n Pregna ncy
19–35 c p. 391 Figure 1 9 -1 3
19–36 b p. 390 Cervica l Pregna ncy
19–37 a p. 390 Ma na gement
19–38 d p. 391 Cesa rea n Sca r Pregna ncy
19–39 b p. 391 Cesa rea n Sca r Pregna ncy
19–40 d p. 391 Cesa rea n Sca r Pregna ncy
128

CHAPTER 20

Gestational Trophoblastic Disease

20–1. As a group, gestational trophoblastic disease is 20–3. Gestational trophoblastic neoplasia includes all
typi ied by which o the ollowing? EXCEPT which o the ollowing?
a. Scant cytotrophoblast a. Invasive mole
b. Perivillous ibrin deposition b. Choriocarcinoma
c. Villous mesenchymal hyperplasia c. Partial hydatidi orm mole
d. Abnormal trophoblast proli eration d. Placental site trophoblastic tumor

20–2. As illustrated by di erences seen here between


invasive mole (A) and choriocarcinoma (B),
hydatidi orm moles as a group are di erentiated
histologically rom other nonmolar neoplasms by the
presence o which o the ollowing?
a. Villi
b. Cytotrophoblast
c. Syncytiotrophoblast
A
d. Marked angiogenesis

A B

A. Used with permission rom Dr. Ona Faye-Peterson. B. Reproduced with permission rom Schorge JO: Gestational trophoblastic disease. In Hof man BL, Schorge JO, Schaf er JI,
et al (eds): Williams Gynecology, 2nd ed. New York, McGraw-Hill, 2012, Figure 37-8.

B
G esta tiona l Trophobla stic Disea se 129

20–4. Which o the ollowing histological changes, as 20–7. This molar pregnancy lacked a etal component. All
shown here, are characteristic o hydatidi orm moles? EXCEPT which o the ollowing eatures are also
characteristic o this type o hydatidi orm mole?

C
H
A
P
T
E
R
2
0
Used with permission rom Dr. Y. Erika Fong. Reproduced with permission rom Schorge
JO: Gestational trophoblastic disease. In Hof man BL, Schorge JO, Schaf er JI, et al (eds): Used with permission rom Dr. Sasha Andrews. Reproduced with permission rom
Williams Gynecology, 2nd ed. New York, McGraw-Hill, 2012, Figure 37-1B. Schorge JO: Gestational trophoblastic disease. In Hof man BL, Schorge JO, Schaf er JI,
et al (eds): Williams Gynecology, 2nd ed. New York, McGraw-Hill, 2012, Figure 37-3.

a. Chronic villitis and villous inclusion bodies


a. Diploid karyotype
b. Villous mesenchymal hyperplasia and acute villitis
b. Focal villous edema
c. Villous lymphocytic in iltrates and syncytial knots
c. Theca-lutein ovarian cysts are requently
d. Trophoblast proli eration and villous stromal
associated
edema
d. Approximate 15% risk o subsequent gestational
20–5. A predominant maternal risk actor or molar trophoblastic neoplasia
pregnancy includes which o the ollowing?
20–8. All EXCEPT which o the ollowing eatures are
a. Advanced maternal age characteristic o partial hydatidi orm mole?
b. Prior cesarean delivery a. Triploid karyotype
c. Type 2 diabetes mellitus b. Focal villous edema
d. A rican American ethnicity c. Fetal tissue present
20–6. Your patient has completed treatment or a complete d. Approximate 15% risk o subsequent gestational
hydatidi orm mole. Compared with women without trophoblastic neoplasia
a prior molar pregnancy, those with one prior mole
have which o the ollowing risks o developing this 20–9. With regard to molar pregnancies, what does the
condition again in a subsequent pregnancy? term “androgenesis” re er to?
a. 2% a. Increased placental androgen production that
promotes villous edema
b. 13%
b. Development o a zygote that contains only
c. 26% maternal chromosomes
d. 42% c. Increased placental androgen production that
leads to maternal virilization
d. None o the above
130 Ea rly Pregna ncy Complica tions

20–10. The pathogenesis o which o the ollowing is shown 20–12. Patients with complete hydatidi orm molar
in this diagram? pregnancy requently present with all EXCEPT
a. Partial mole which o the ollowing clinical indings?
a. Vaginal bleeding
S
b. Complete mole
E
C
c. Mature cystic teratoma b. Multiple simple ovarian cysts
T
I
c. Increased thyroid-stimulating hormone levels
O
d. Complete mole with coexistent twin
N
d. Greater than expected serum β -human chorionic
6
gonadotropin (hCG) levels
69,XXY

23,Y

23,Y Triploid 69, XXY Ce lls


23,X 69,XXY Ma te rna l a nd Pa te rna l
23,X 23,X
Chromos ome s
23,X
Dis pe rmy

69,XXY

Reproduced with permission rom Schorge JO: Gestational trophoblastic disease. In Schorge JO, Schaf er
JI, Halvorson LM, et al (eds): Williams Gynecology. New York, McGraw-Hill, 2008, Figure 37-1B.

20–11. Your patient is a 39-year-old G2P1 with one prior 20–13. Your patient is diagnosed with a complete
uncomplicated pregnancy and vaginal delivery. Her hydatidi orm mole. Sonographic examination o the
current twin pregnancy is made up o a complete adnexa reveals the indings below. The underlying
mole and a coexistent karyotypically normal etus. etiology stems rom increased placental production
Magnetic resonance imaging was completed, and one o which o the ollowing?
view is presented below. This cross-sectional image
shows the complete mole (asterisk), a normal placenta
above the mole, and a cross section o the normal
etus’s abdomen on the le t. Complications that
may be reasonably anticipated during this pregnancy
include all EXCEPT which o the ollowing?

a. Estrogen
b. Thyroxine
c. Progesterone
d. β -Human chorionic gonadotropin

20–14. The condition shown in Question 20–13 is best


Used with permission rom Dr. April Bleich.
managed by which o the ollowing?
a. Oophoropexy
a. Preeclampsia
b. Oophorectomy
b. Fetal demise
c. Ovarian cystectomy
c. Preterm delivery
d. Molar pregnancy uterine evacuation
d. Placenta accreta
G esta tiona l Trophobla stic Disea se 131

20–15. Increased serum ree thyroxine levels in women with 20–19. What is the treatment o choice or a 20-week size
hydatidi orm moles stem rom increases in which o complete mole in a 28-year-old G2P1?
the ollowing? a. Hysterectomy

C
a. Maternal estrogen levels b. Hysterotomy and evacuation

H
A
b. Fetal thyroxine production c. Dilatation and suction curettage

P
T
c. Maternal progesterone levels d. Intramuscular systemic methotrexate

E
R
d. Maternal β -human chorionic gonadotropin levels

2
20–20. Steps during dilatation and curettage that may

0
20–16. A 24-year-old G3P2 presents with vaginal bleeding, hasten evacuation and lessen intraoperative blood
a β -human chorionic gonadotropin (β -hCG) level loss include which o the ollowing?
o 300,000 mIU/mL, uterine size consistent with a. Preoperative laminaria
a 12-week gestation, B negative blood type, and
b. Large-bore suction cannula
the sonographic indings below. What is the most
appropriate management? c. Uterotonic administration during curettage
d. All o the above

20–21. Which o the ollowing uterotonics are


contraindicated in the setting o molar pregnancy
evacuation?
a. Misoprostol
b. Synthetic oxytocin
c. Carboprost tromethamine
d. None o the above

20–22. In the United States, routine postevacuation


treatment o molar pregnancy typically includes
which o the ollowing?
a. Methotrexate chemotherapy
b. Intrauterine device insertion
a. Plan or hysterectomy
c. Rhogam administration to Rh-negative women
b. Rhogam administration and bed rest
d. All o the above
c. Plan or dilatation and curettage
d. Repeat a serum β -hCG level in 48 hours 20–23. In the United States, a reasonable alternative to
dilatation and curettage or the management o
20–17. Prior to surgical intervention or a hydatidi orm complete hydatidi orm mole includes which o the
mole, all EXCEPT which o the ollowing are ollowing?
typically completed? a. Hysterectomy
a. Type and screen b. Hysterotomy and uterine evacuation
b. Complete blood count c. Misoprostol labor induction ollowing laminaria
c. Chest computed tomography placement
d. Serum testing o liver, renal, and thyroid unction d. All o the above

20–18. Prior to molar pregnancy evacuation, a preoperative


chest radiograph is typically obtained to exclude
which o the ollowing associated conditions?
a. Cardiomegaly
b. Pleural e usion
c. Hilar lymphadenopathy
d. Trophoblastic deportation
132 Ea rly Pregna ncy Complica tions

20–24. Your patient, who is pregnant with an estimated 20–27. During surveillance, all EXCEPT which o the
gestational age o 8 weeks by last menstrual period, ollowing portend a greater risk or development o
presents to the emergency department with heavy gestational trophoblastic neoplasia?
vaginal bleeding and passage o tissue. Sonographic a. Maternal age > 40 years
S
E
examination reveals an endometrial cavity illed
C
b. 8-cm theca lutein cysts
T
with blood and tissue exhibiting inhomogeneous
I
c. Rapidly declining β -human chorionic
O
echoes. You per orm a dilatation and curettage
N
with no complications. A week later, you receive gonadotropin level
6
the pathology report or the evacuated products o d. β -Human chorionic gonadotropin level >
conception: 100,000 mIU/mL prior to uterine evacuation
Specimen: uterine contents 20–28. Your patient is a 32-year-old G1P0A1 who has
DNA interpretation by image cytometry: diploid undergone molar pregnancy evacuation and is
Immunostaining: p57KIP2 positive using combination oral contraceptive pills. During
postevacuation surveillance, her serum β -human
These histological indings are consistent with which chorionic gonadotropin levels had previously
o the ollowing diagnoses? dropped to an undetectable level. Today, as part o
a. Partial mole her monthly surveillance, her value is 900 mIU/mL.
b. Complete mole Appropriate initial management includes which o
c. Spontaneous abortion the ollowing?
d. None o the above a. Preparation or dilatation and curettage
b. Initiation o intramuscular methotrexate therapy
20–25. Which o the statements below are true regarding c. Repeat β -human chorionic gonadotropin level in
surveillance practices ollowing evacuation o a molar 48 hours
pregnancy? d. International Federation o Gynecology and
a. Endometrial biopsy and chest radiograph should Obstetrics (FIGO) staging
be per ormed every 3 months or 1 year.
b. Endometrial biopsy, chest radiographs, and 20–29. The patient rom Question 20–28 presents
β -human chorionic gonadotropin levels are again in 48 hours and has a β -human chorionic
obtained serially, but each at di erent intervals. gonadotropin level o 6000 mIU/mL. What is the
c. Serum β -human chorionic gonadotropin levels next most appropriate step in her care?
should be monitored every 1 to 2 weeks until a. Transvaginal sonography
undetectable, a ter which monthly levels are b. Preparation or dilatation and curettage
drawn or the next 6 months. c. Initiation o intramuscular methotrexate therapy
d. None o the above d. Chest and abdominopelvic computed
tomography (CT) imaging and brain magnetic
20–26. Which o the ollowing statements is true regarding resonance imaging
contraceptive practices a ter evacuation o a molar
pregnancy? 20–30. The patient rom Question 20–28 undergoes
a. Intrauterine devices should not be inserted until transvaginal sonography, which reveals no
the β -human chorionic gonadotropin (β -hCG) intrauterine or adnexal gestation. Appropriate
level is undetectable. management includes which o the ollowing?
b. Pregnancies that occur during the monitoring a. Hysterectomy
period increase the risk o progression to b. Initiation o intravenous dactinomycin therapy
gestational trophoblastic neoplasia.
c. Initiation o intramuscular methotrexate therapy
c. Hormonal contraception, such as
oral contraceptive pills and injectable d. International Federation o Gynecology and
medroxyprogesterone acetate, should not be Obstetrics (FIGO) staging
initiated until the β -hCG level is undetectable.
d. All o the above
G esta tiona l Trophobla stic Disea se 133

20–31. In practice, the diagnosis o gestational trophoblastic 20–37. Following dilatation and curettage or a complete
neoplasia typically is determined by which o the mole, your patient is surveilled with serial β -human
ollowing? chorionic gonadotropin (β -hCG) levels. For the

C
a. Histologic tissue evaluation past 3 weeks, the β -hCG values have plateaued.

H
Diagnostic evaluation reveals a metastatic lesion in

A
b. Physical examination indings

P
the liver (shown here). Given this extent o disease,

T
c. Computed tomography (CT) imaging what is the International Federation o Gynecology

E
R
d. Serum β -human chorionic gonadotropin levels and Obstetrics (FIGO) stage?

2
0
20–32. Criteria or the diagnosis o gestational trophoblastic
neoplasia includes which o the ollowing?
a. Rising β -human chorionic gonadotropin levels
b. Plateaued β -human chorionic gonadotropin levels
c. Persistent β -human chorionic gonadotropin levels
d. All o the above

20–33. Gestational trophoblastic neoplasia may develop


a ter which o the ollowing?
a. Evacuation o a partial mole
b. Delivery o a normal term pregnancy
c. Miscarriage o a genetically normal abortus
d. All o the above

20–34. The hallmark sign o gestational trophoblastic


neoplasia is which o the ollowing? Used with permission rom Dr. John Schorge.

a. Seizures
a. Stage I
b. Hemoptysis
b. Stage II
c. Uterine bleeding
c. Stage III
d. Pelvic vein thrombosis
d. Stage IV
20–35. Evaluation o abnormal bleeding or more than
6 weeks ollowing any pregnancy may include which 20–38. According to the World Health Organization
o the ollowing? (WHO) modi ied prognostic scoring system that
was adapted by the International Federation o
a. Transvaginal sonography Gynecology and Obstetrics (FIGO), patients with
b. Serum β -human chorionic gonadotropin level scores below which o the ollowing thresholds are
c. Endometrial sampling to exclude placental site assigned to the low-risk gestational trophoblastic
trophoblastic tumor or epithelioid trophoblastic neoplasia group?
tumor a. ≤ 4
d. All o the above b. ≤ 6
c. ≤ 8
20–36. According to the World Health Organization
(WHO) modi ied prognostic scoring system that d. ≤ 10
was adapted by the International Federation o
Gynecology and Obstetrics (FIGO), which o the 20–39. Which o the ollowing characteristics are most
ollowing is assessed and assigned a rating score typical o invasive moles?
during staging o gestational trophoblastic neoplasia? a. Follows a term pregnancy
a. Parity b. Penetrates deeply into the myometrium
b. Severity o thyrotoxicosis c. Displays minimal trophoblastic growth
c. Number o months rom the antecedent d. Is almost invariably associated with widespread
pregnancy pulmonary metastasis
d. Presence and diameter o largest theca-lutein cyst
134 Ea rly Pregna ncy Complica tions

20–40. Metastatic disease, such as that shown here, is most 20–43. Your patient has International Federation o
commonly due to which o the ollowing? Gynecology and Obstetrics (FIGO) stage I
gestational trophoblastic neoplasia. Pre erred and
e ective treatment includes methotrexate or which
S
E
o the ollowing?
C
T
a. Radical hysterectomy
I
O
b. Combination chemotherapy
N
6
c. External beam pelvic radiation
d. Actinomycin-D single-agent chemotherapy

20–44. Your patient has International Federation o


Gynecology and Obstetrics (FIGO) stage III
gestational trophoblastic neoplasia. Which o the
ollowing is considered typical treatment?
a. Radical hysterectomy
b. Combination chemotherapy
Reproduced with permission rom Schorge JO: Gestational trophoblastic disease. In c. Radical hysterectomy plus adjuvant methotrexate
Schorge JO, Schaf er JI, Halvorson LM, et al (eds): Williams Gynecology. New York,
McGraw-Hill, 2008, Figure 37-8. d. External beam pelvic radiation plus adjuvant
methotrexate
a. Invasive mole
20–45. Chemotherapeutic agents in the EMA-CO regimen
b. Choriocarcinoma include all EXCEPT which o the ollowing?
c. Epithelioid trophoblastic tumor a. Cisplatin
d. Placental site trophoblastic tumor b. Etoposide
20–41. Metastatic spread o choriocarcinoma is most c. Methotrexate
commonly by which o the ollowing routes? d. Actinomycin-D
a. Lymphatic
20–46. True evidenced-based risks or uture pregnancy
b. Hematogenous ollowing treatment o gestational trophoblastic
c. Peritoneal luid disease include which o the ollowing?
d. Cerebrospinal luid a. Decreased ertility
b. Increased risk o preterm labor
20–42. What is the most common site o metastatic spread
o choriocarcinoma? c. Increased risk o placenta accreta
a. Brain d. Increased risk o a second molar pregnancy
b. Liver
c. Lungs
d. Spleen
G esta tiona l Trophobla stic Disea se 135

CHAPTER 20 ANSw ER KEy

Q uestion Letter Pa ge

C
number a nswer cited Hea der cited

H
A
P
20–1 d p. 396 Introduction

T
20–2 a p. 396 Introduction

E
R
20–3 c p. 396 Introduction

2
0
20–4 d p. 396 Hyda tidiform Mole—Mola r Pregna ncy
20–5 a p. 396 Epidemiology a nd Risk Fa ctors
20–6 a p. 397 Epidemiology a nd Risk Fa ctors
20–7 b p. 397 Ta ble 2 0 -1
20–8 d p. 397 Ta ble 2 0 -1
20–9 d p. 397 Pa thogenesis
20–10 a p. 398 Figure 2 0 -2 B
20–11 d p. 398 Twin Pregna ncy Comprising a N orma l Fetus a nd Coexistent Complete Mole
20–12 c p. 398 Clinica l Findings
20–13 d p. 398 Clinica l Findings
20–14 d p. 398 Clinica l Findings
20–15 d p. 398 Clinica l Findings
20–16 c p. 399 Sonogra phy
20–17 c p. 400 Ta ble 2 0 -2
20–18 d p. 400 Termina tion of Mola r Pregna ncy
20–19 c p. 400 Termina tion of Mola r Pregna ncy
20–20 d p. 400 Termina tion of Mola r Pregna ncy
20–21 d p. 400 Ta ble 2 0 -2
20–22 c p. 400 Termina tion of Mola r Pregna ncy
20–23 a p. 400 Termina tion of Mola r Pregna ncy
20–24 c p. 400 Pa thologica l Dia gnosis
20–25 c p. 401 Posteva cua tion Surveilla nce
20–26 a p. 401 Posteva cua tion Surveilla nce
20–27 c p. 401 Posteva cua tion Surveilla nce
20–28 c p. 401 Posteva cua tion Surveilla nce
20–29 a p. 401 Posteva cua tion Surveilla nce
20–30 d p. 401 Posteva cua tion Surveilla nce
20–31 d p. 401 G esta tiona l trophobla stic N eopla sia
20–32 d p. 402 Ta ble 2 0 -3
20–33 d p. 401 G esta tiona l Trophobla stic N eopla sia
20–34 c p. 401 Clinica l Findings
20–35 d p. 402 Dia gnosis, Sta ging, a nd Prognostic Scoring
20–36 c p. 402 Ta ble 2 0 -4
20–37 d p. 402 Ta ble 2 0 -4
20–38 b p. 402 Dia gnosis, Sta ging, a nd Prognostic Scoring
20–39 b p. 402 Inva sive Mole
20–40 b p. 403 G esta tiona l Chorioca rcinoma
20–41 b p. 403 G esta tiona l Chorioca rcinoma
20–42 c p. 403 G esta tiona l Chorioca rcinoma
20–43 d p. 403 Trea tment
20–44 b p. 403 Trea tment
20–45 a p. 403 Trea tment
20–46 d p. 404 Subsequent Pregna ncy
This page intentionally left blank
Se c t i o n 7

LABOR
138

CHAPTER 21

Ph siolog of Labor

21–1. O the our phases o parturition, phase 2 is 21–6. Contraction-associated proteins (CAPs) within
characterized by which o the ollowing? uterine smooth muscle prepare it to contract during
a. Uterine activation, cervical ripening labor. CAP concentrations increase during phase 2
o parturition and include all EXCEPT which o the
b. Uterine quiescence, cervical so tening
ollowing proteins?
c. Uterine involution, cervical remodeling
a. Connexin 43
d. Uterine contraction, cervical dilatation
b. Oxytocin receptor
21–2. Which phase o parturition corresponds to the c. Progesterone receptor A
clinical stages o labor? d. Prostaglandin F receptor
a. Phase 1
21–7. Compared with the uterine body, the cervix has
b. Phase 2
a signi icantly lower percentage o which o the
c. Phase 3 ollowing?
d. Phase 4 a. Collagen
b. Proteoglycans
21–3. During which o the stages o labor is the etus
delivered? c. Smooth muscle
a. Stage 1 d. Glycosaminoglycans
b. Stage 2
21–8. All EXCEPT which o the ollowing mechanisms
c. Stage 3 lead in part to the cervical ripening that is
d. Stage 4 characteristic o phase 2 o parturition?
a. Decreased collagen ibril diameter
21–4. All EXCEPT which o the ollowing cervical
b. Increased spacing between collagen ibrils
unctions and cervical events take place during phase
1 o parturition? c. Altered levels o decorin and biglycan, which are
proteoglycans
a. Maintenance o cervical competence despite
growing uterine weight d. Increased expression o the enzymes responsible
or synthesis o hyaluronan, which is a
b. Maintenance o barrier between uterine contents
glycosaminoglycan
and vaginal bacteria
c. Alterations in extracellular matrix to gradually
increase cervical tissue compliance
d. Alteration o cervical collagen to sti en the cervix

21–5. Cervical so tening in phase 1 o parturition results in


part rom which o the ollowing?
a. Stromal atrophy
b. Increased stromal vascularity
c. Increased collagen monomer cross-linking
d. All o the above
Physiology of La bor 139

21–9. Your primigravid patient at 41 weeks’ gestation 21–12. During cesarean delivery, the hysterotomy incision
presents or her weekly prenatal care visit with is ideally made in the lower uterine segment, shown
complaints o decreased etal movement. Clinically, here prior to bladder lap creation. Which o the

C
you are unable to ballot the etal head, and the ollowing aids development o this uterine segment

H
amnionic luid index re lects oligohydramnios. Her during phase 3 o parturition?

A
P
Bishop score is only 4. The vaginal insert shown

T
here is one o which class o agents used clinically to

E
R
e ect cervical ripening?

2
1
a. Progressive thickening o the upper uterine
segment with labor progression
b. Smooth muscle ibers o the undus relax to their
Reproduced with permission rom Cunningham FG, Leveno KJ, Bloom SL, et al (eds):
Labor induction. In Williams Obstetrics, 24th ed. New York, McGraw-Hill, 2014, Figure original length a ter each contraction
26-1. c. Smooth muscle cell ibers o the lower uterine
segment relax to their original length a ter each
a. Oxytocin contraction
b. Beta mimetics d. All o the above
c. Prostaglandins
d. Nonsteroidal antiin lammatory drugs 21–13. Extreme development o both upper and lower
uterine segments may be seen with obstructed labor
21–10. Which o the ollowing are considered plausible and clinically may be re lected by which o the
causes o uterine contraction pain? ollowing?
a. Myometrial hypoxia a. Hegar sign
b. Uterine peritoneum stretching b. Bandl ring
c. Compression o nerve ganglia in the cervix c. Bloody show
d. All o the above d. Chadwick sign

21–11. Which o the ollowing best de ines the Ferguson 21–14. As a result o contraction orces, the cervix e aces
re lex? and dilates by mechanisms that include all EXCEPT
which o the ollowing?
a. Mechanical stretch o the cervix enhances uterine
activity a. Contraction orces create lateral pull against the
cervix to open its canal.
b. Maternal ambulation augments contraction
intensity and requency
b. Contraction orces are trans erred directly
through the presenting part to the cervix to dilate
c. Fetal scalp stimulation leads to etal heart rate its canal.
acceleration
c. Contraction orces pull smooth muscle ibers at
d. Maternal shi ting to the le t lateral recumbent the internal os up into the adjacent upper uterine
position increases venous return segment to e ace the cervix.
d. Contraction orces are translated into hydrostatic
pressure within the amnionic sac, which presses
against the cervix to dilate the cervical canal.
140 La bor

21–15. This image depicts stresses on the pelvic loor caused 21–18. Which o the ollowing can bring about myometrial
by etal head delivery during phase 3 o parturition. contractions?
All EXCEPT which o the ollowing is true at this a. Extracellular magnesium
point in delivery?
S
b. Actin-tubulin protein pairs
E
C
c. G-protein-coupled receptors
T
I
O
d. Gap junctions composed o decorin subunits
N
7
21–19. Once bound to its receptor, oxytocin promotes
contraction through which o the ollowing
mechanisms?
a. Opens calcium channels
b. Generates nitric oxide
c. Degrades 15-hydroxyprostaglandin dehydrogenase
d. Activates the gene promoter region o the myosin
light-chain kinase gene

21–20. In many mammals, suspension o the quiescence


seen in phase 2 o parturition is due to which o the
ollowing?
a. Cortisol withdrawal
b. Progesterone withdrawal
c. In lammatory cell activation
d. Increased oxytocin receptor concentration
Reproduced with permission rom Cunningham FG, Leveno KJ, Bloom SL, et al (eds):
Maternal anatomy. In Williams Obstetrics, 23rd ed. New York, McGraw-Hill, 2010, 21–21. Your patient presents with 6 weeks o amenorrhea.
Figure 2-26. Transvaginal sonography reveals an intrauterine
pregnancy. She elects to proceed with a medically
a. The anus may dilate up to 3 cm. induced abortion. For this, regimens o mi epristone
b. The perineal body becomes attenuated. and misoprostol are suitable. Mi epristone promotes
c. The puborectalis muscle is markedly stretched. cervical ripening and increased uterotonin sensitivity
through which o the ollowing mechanisms?
d. The coccygeus muscles receive the bulk o
expulsive orces. a. Progesterone antagonism
b. Calcium-channel blockade
21–16. Which o the ollowing is most important or initial c. Adenyl cyclase activation
placental separation rom its uterine implantation
d. Opening o maxi-K channels
site?
a. Maternal pushing 21–22. For women with a prior preterm birth delivered at
b. Gentle cord traction < 37 weeks’ gestation, 17α -hydroxyprogesterone
c. Uterine smooth muscle contraction caproate may be used to prevent recurrent preterm
birth. One mechanism by which progesterone
d. Hematoma ormation between the uterine wall
maintains uterine quiescence is its ability to decrease
and placenta
expression o which o the ollowing?
21–17. Compared with skeletal muscle, uterine smooth a. Adenyl cyclase
muscle o ers which o the ollowing advantages? b. Progesterone receptor A
a. Generates orces along one axis c. Progesterone receptor B
b. Has greater muscle iber shortening d. Contraction-associated proteins
c. Displays more structured alignment o muscle
ibers
d. All o the above
Physiology of La bor 141

21–23. A decline in progesterone’s relative activity may be 21–26. Indomethacin, a nonsteroidal antiin lammatory drug
important or initiation o phase 2 o parturition (NSAID), has some tocolytic actions. As a group,
in humans. This decline may be achieved through NSAIDs target which enzyme in prostaglandin

C
which o the ollowing mechanisms? production?

H
a. Increased expression o progesterone receptor a. Cyclooxygenase-1

A
P
corepressors b. Phospholipase A2

T
E
b. Posttranslational modi ications o the

R
c. Prostaglandin isomerase

2
progesterone receptor
d. Prostaglandin dehydrogenase

1
c. Changes in the expression o di erent
progesterone iso orms 21–27. During phases 1 and 2 o parturition, uterine
d. All o the above quiescence is maintained in part through inhibition
o smooth muscle’s response to oxytocin. Which
21–24. Your patient presents at 39 weeks’ gestation with a o the ollowing is a primary regulator o oxytocin
breech-presenting etus. A ter a discussion o the risks receptor expression?
and bene its, she agrees to an external cephalic version a. Calcium
attempt. Prior to initiation, you administer 0.25 mg o b. Progesterone
terbutaline subcutaneously. This drug binds to beta-
adrenergic receptors to create which o the ollowing c. Prostaglandin dehydrogenase
cellular responses to cause uterine relaxation? d. Corticotropin-releasing hormone
a. Increased extracellular Mg2+ levels
21–28. Stretch is believed important or uterine activation
b. Increased intracellular Ca2+ levels in phase 1 o parturition. One result o myometrial
c. Increased cyclic adenosine monophosphate stretch includes increased expression o which o the
(cAMP) levels ollowing?
d. Decreased cyclic guanosine monophosphate a. Actin and myosin
(cGMP) levels b. Contraction-associate proteins
21–25. Human chorionic gonadotropin (hCG) shares the c. Corticotropin-releasing hormone
exact same receptor with which o the ollowing d. None o the above
hormones? Because o this, the high hCG levels
seen with complete hydatidi orm moles may lead 21–29. Corticotropin-releasing hormone (CRH) is suggested
to ovarian stimulation and ormation o the theca- to promote parturition progression. Which o the
lutein cysts seen here. ollowing is the main contributor to CRH levels in
pregnancy?
a. Placenta
b. Fetal adrenal
c. Fetal hypothalamus
d. Maternal hypothalamus

Used with permission rom Dr. Sarah White.

a. Luteinizing hormone (LH)


b. Thyroid-stimulating hormone (TSH)
c. Follicle-stimulating hormone (FSH)
d. None o the above
142 La bor

21–30. Which o the ollowing abnormalities o normal 21–31. Which group o agents is theorized to initiate phase
parturition has been associated with this neural-tube 3 o parturition?
de ect? a. Uterotonins
S
b. Sex steroids
E
C
c. Beta mimetics
T
I
O
d. Calcium-channel blockers
N
7
21–32. Prostaglandins play a critical role in phase 3 o
parturition. Levels o these are altered by which o
the ollowing?
a. Amnion
b. Decidua
c. Chorion
d. All o the above

Reproduced with permission rom Hof man BL, Dashe JS, Snell K: Update in Cunning-
ham FG, Leveno KL, Bloom SL, et al (eds): Williams Obstetrics, 23rd ed. Online. New
York, McGraw-Hill, 2013. http://www.accessmedicine.com.

a. Preterm labor
b. Prolonged gestation
c. Uterine tachysystole
d. None o the above
Physiology of La bor 143

CHAPTER 21 ANSw ER KEy

Q uestion Letter Pa ge

C
number a nswer cited Hea der cited

H
A
P
21–1 a p. 410 Pha se 2 of Pa rturition: Prepa ra tion for La bor

T
E
21–2 c p. 408 Pha ses of Pa rturition

R
2
21–3 b p. 409 Figure 2 1 -2

1
21–4 d p. 409 Cervica l Softening
21–5 b p. 409 Structura l Cha nges with Softening
21–6 c p. 410 Myometria l Cha nges
21–7 c p. 410 Cervica l Ripening During Pha se 2
21–8 a p. 410 Cervica l Connective Tissue
21–9 c p. 412 Induction a nd Prevention of Cervica l Ripening
21–10 d p. 412 Uterine La bor Contra ctions
21–11 a p. 412 Uterine La bor Contra ctions
21–12 a p. 412 Distinct Lower a nd Upper Uterine Segments
21–13 b p. 412 Distinct Lower a nd Upper Uterine Segments
21–14 c p. 414 Cervica l Cha nges
21–15 d p. 415 Pelvic Floor Cha nges During La bor
21–16 c p. 416 Third Sta ge of La bor: Delivery of Pla centa a nd Membra nes
21–17 b p. 417 Ana tomica l a nd Physiologica l Considera tions
21–18 c p. 417 Regula tion of Myometria l Contra ction a nd Rela xa tion
21–19 a p. 417 Intra cellula r Ca lcium
21–20 b p. 419 Progesterone a nd Estrogen Contributions
21–21 a p. 419 Progesterone a nd Estrogen Contributions
21–22 d p. 421 Myometria l Cell-to-Cell Communica tion
21–23 d p. 423 Functiona l Progesterone W ithdra wa l in Huma n Pa rturition
21–24 c p. 421 Beta -a drenoreceptors
21–25 a p. 421 Luteinizing Hormone (LH) a nd Huma n Chorionic G ona dotropin (hCG )
Receptors
21–26 a p. 422 Prosta gla ndins
21–27 b p. 423 O xytocin Receptor
21–28 b p. 424 Uterine Stretch a nd Pa rturition
21–29 a p. 424 Feta l Endocrine Ca sca des Lea ding to Pa rturition
21–30 b p. 426 Feta l Anoma lies a nd Dela yed Pa rturition
21–31 a p. 426 Pha se 3 : Uterine Stimula tion
21–32 d p. 428 Contributions of Intra uterine Tissues to Pa rturition
144

CHAPTER 22

Normal Labor

22–1. The relation of the fetal long axis to that of the 22–6. This drawing shows a fetal head in which position?
mother is termed which of the following?
a. Fetal lie
b. Fetal angle
c. Fetal position
d. Fetal polarity

22–2. Which of the following is not a predisposing factor


for transverse fetal lie?
a. Multiparity
b. Oligohydramnios
c. Placenta previa
d. Uterine anomalies

22–3. Which of the following fetal presentations is the least


Reproduced with permission from Cunningham FG, Leveno KJ, Bloom SL, et al (eds):
common? Normal labor. In Williams Obstetrics, 24th ed. New York, McGraw-Hill, 2014, Figure
a. Breech 22-3A.

b. Cephalic
a. Left occiput anterior (LOA)
c. Compound
b. Left occiput posterior (LOP)
d. Transverse lie
c. Right occiput anterior (ROA)
22–4. What percentage of fetuses are breech at 28 weeks’ d. Right occiput posterior (ROP)
gestation?
a. 1%
b. 10%
c. 25%
d. 50%

22–5. When the anterior fontanel is the presenting part,


which term is used?
a. Brow
b. Face
c. Vertex
d. Sinciput
N orma l La bor 145

22–7. The face presentation in this drawing is described as 22–8. The fetus in this drawing is breech. His position is
which of the following? best described as which of the following?

C
H
A
P
T
E
R
2
2
Reproduced with permission from Cunningham FG, Leveno KJ, Bloom SL, et al (eds):
Normal labor. In Williams Obstetrics, 24th ed. New York, McGraw-Hill, 2014, Figure
22-5A.

a. Left mento-anterior
b. Left mento-posterior
c. Right mento-anterior
d. Right mento-posterior

Reproduced with permission from Cunningham FG, Leveno KJ, Bloom SL, et al (eds):
Breech delivery. In Williams Obstetrics, 24th ed. New York, McGraw-Hill, 2014, Figure
28-2.

a. Left sacrum anterior


b. Left sacrum posterior
c. Right sacrum anterior
d. Right sacrum posterior
146 La bor

22–9. The fetus in this drawing has a transverse lie. The 22–13. Regarding engagement of the fetal head, which of
position is best described as which of the following? the following statements is true?
a. It does not occur until labor commences.
S
b. Engagement prior to the onset of labor does not
E
C
affect vaginal delivery rates.
T
I
c. It is the mechanism by which the biparietal
O
N
diameter passes through the pelvic outlet.
7
d. A normal-sized head usually engages with its
sagittal suture directed anteroposteriorly.

22–14. On palpation of the fetal head during vaginal


examination, you note that the sagittal suture is
transverse and close to the pubic symphysis. The
posterior ear can be easily palpated. Which of the
following best describes this orientation?
a. Anterior asynclitism
b. Posterior asynclitism
c. Mento-anterior position
d. Mento-posterior position
Reproduced with permission from Cunningham FG, Leveno KJ, Bloom SL, et al (eds):
Normal labor. In Williams Obstetrics, 24th ed. New York, McGraw-Hill, 2014, Figure
22-7.
22–15. Of the cardinal movements of labor, internal
rotation achieves what goal?
a. Left acromidorsoanterior (LADA) a. Flexes the fetal neck
b. Left acromidorsoposterior (LADP) b. Brings the occiput to an anterior position
c. Right acromidorsoanterior (RADA) c. Brings the anterior fontanel through the pelvic
inlet
d. Right acromidorsoposterior (RADP)
d. None of the above
22–10. In shoulder presentations, the portion of the fetus
chosen for orientation with the maternal pelvis is 22–16. In what percentage of labors does the fetus enter the
which of the following? pelvis in an occiput posterior position?
a. Head a. 0.5%
b. Breech b. 5%
c. Scapula c. 20%
d. Umbilicus d. 33%

22–11. Which of the following could inhibit performance of 22–17. Which of the following is not a risk factor for
Leopold maneuvers? incomplete rotation of the posterior occiput?
a. Oligohydramnios a. Macrosomia
b. Maternal obesity b. Poor contractions
c. Posterior placenta c. Lack of analgesia
d. Supine maternal positioning d. Inadequate head flexion

22–12. Which of the following is the correct order for the


cardinal movements of labor?
a. Descent, engagement, internal fixation, flexion,
extension, external rotation, expulsion
b. Descent, flexion, engagement, external fixation,
extension, internal rotation, expulsion
c. Engagement, descent, flexion, internal rotation,
extension, external rotation, and expulsion
d. Engagement, flexion, descent, internal rotation,
straightening, extension, and expulsion
N orma l La bor 147

22–18. This photograph demonstrates which of the 22–22. According to Friedman, the minimum normal rate
following? of active-phase labor in a multipara is which of the
following?

C
a. 1 cm/hr

H
A
b. 1.2 cm/hr

P
T
c. 1.5 cm/hr

E
R
d. 3.4 cm/hr

2
2
22–23. Which stage of labor begins with complete cervical
dilatation and ends with delivery of the fetus?
a. First stage
b. Second stage
c. Third stage
d. Fourth stage

22–24. A 24-year-old G1P0 at 27 weeks’ gestation presents


in active preterm labor to a hospital without
delivery services or a neonatal intensive care unit.
a. Hydrocephalus
The physician in the emergency department
b. Plagiocephaly evaluates the patient. He determines that her cervix
c. Craniosynostosis is approximately 4 cm dilated and membranes are
d. Caput and molding intact. He would like to transfer her to you because
you are at the nearest hospital with obstetric and
22–19. Which of the following statements regarding the neonatal services qualified to handle this patient’s
preparatory division of labor is true? complications. According to the Emergency Medical
Treatment and Labor Act (EMTALA), which of the
a. The cervix dilates very little.
following is true?
b. Connective tissue components of the cervix
a. A woman complaining of contractions is not
change considerably.
considered an emergency.
c. Sedation and conduction analgesia are capable of
b. A screening examination is not required because
arresting this labor division.
it will unreasonably slow the transfer of the
d. All of the above patient.
c. The patient cannot be transferred because a
22–20. When does the latent phase of labor end for most
woman in true labor is considered “unstable” for
women?
interhospital transfer.
a. 1–2 cm
d. This patient can be transferred if the physician
b. 2–3 cm certifies that the benefits of treatment at your
c. 3–5 cm facility outweigh the transfer risks.
d. 7–8 cm
22–25. When evaluating a pregnant woman for rupture
22–21. A 20-year-old G1P0 at 39 weeks’ gestation presents of membranes, which of the following has been
complaining of strong contractions. Her cervix is associated with a false-positive nitrazine test result?
dilated 1 cm. She is given sedation, and 4 hours a. Blood
later, her contractions have stopped. Her cervix is b. Semen
still 1 cm dilated. Which of the following is the most
c. Bacterial vaginosis
likely diagnosis?
a. False labor d. All of the above
b. Prolonged latent phase of labor
c. Arrest of the latent phase of labor
d. Arrest of the active phase of labor
148 La bor

22–26. When performing a bimanual examination on 22–30. What is the median duration of the second-stage
a pregnant woman, the position of the cervix is labor in multiparas without conduction analgesia?
determined by the relationship of the cervical os to a. 20 minutes
which of the following?
S
b. 40 minutes
E
a. Rectum
C
c. 50 minutes
T
b. Uterus
I
O
d. 90 minutes
N
c. Fetal head
7
d. Pubic symphysis 22–31. A 25-year-old G1P0 at 39 weeks’ gestation presents
in active labor. Her cervix is dilated 4 cm and is
22–27. Station describes the relationship between which of completely effaced, and the presenting fetal part
the following? has reached 0 station. Membranes are intact. With
a. The biparietal diameter and the pelvic outlet examination 2 hours later, you note that the cervix
is still 4 cm dilated. At this point, which of the
b. The biparietal diameter and the ischial spines
following is the best management?
c. The lowermost portion of the presenting fetal
a. Cesarean delivery
part and the pelvic inlet
b. Rupture of membranes
d. The lowermost portion of the presenting fetal
part and the ischial spines c. Insertion of a bladder catheter to assist fetal head
descent
22–28. A 20-year-old G2P1 presents in active labor at term. d. Rupture of membranes, placement of internal
The patient requires augmentation with oxytocin monitors, and oxytocin augmentation
during her labor course. She has a forceps-assisted
vaginal delivery and sustains a second-degree 22–32. A 19-year-old G1P0 at term presents in active labor.
laceration. Which of the following is not a risk factor Her cervix is 5 cm dilated, and fluid is leaking from
for urinary retention in this patient? spontaneously ruptured membranes. You examine
a. Multiparity her 2 hours later, and the cervix is still 5 cm dilated.
At this point, which of the following is the best
b. Perineal laceration
management?
c. Oxytocin-augmented labor
a. Cesarean delivery
d. Operative vaginal delivery
b. Placement of internal monitors and reassessment
in 2 hours
22–29. What is the median duration of second-stage labor
in nulliparas without conduction analgesia? c. Placement of internal monitors, oxytocin
augmentation, and reassessment in 2 hours
a. 20 minutes
d. Placement of internal monitors, oxytocin
b. 40 minutes
augmentation, antibiotics for prolonged rupture
c. 50 minutes of membranes, and reassessment in 2 hours
d. 90 minutes
N orma l La bor 149

CHAPTER 22 ANSw ER KEy

Q uestion Letter Pa ge

C
number a nswer cited Hea der cited

H
A
P
22–1 a p. 433 Feta l Lie

T
E
22–2 b p. 433 Feta l Lie

R
2
22–3 c p. 434 Ta ble 2 2 -1

2
22–4 c p. 433 Cepha lic Presenta tion
22–5 d p. 434 Figure 2 2 -1
22–6 d p. 435 Figure 2 2 -3
22–7 a p. 436 Figure 2 2 -5
22–8 b p. 436 Figure 2 2 -6
22–9 d p. 437 Figure 2 2 -7
22–10 c p. 437 Va rieties of Presenta tions a nd Positions
22–11 b p. 437 Abdomina l Pa lpa tion—Leopold Ma neuvers
22–12 c p. 438 O cciput Anterior Presenta tion
22–13 b p. 439 Enga gement
22–14 b p. 439 Asynclitism
22–15 b p. 442 Interna l Rota tion
22–16 c p. 443 O cciput Posterior Presenta tion
22–17 c p. 443 O cciput Posterior Presenta tion
22–18 d p. 444 Figure 2 2 -1 8
22–19 d p. 445 First Sta ge of La bor
22–20 c p. 445 La tent Pha se
22–21 a p. 446 Prolonged La tent Pha se
22–22 c p. 446 Active La bor
22–23 b p. 447 Second Sta ge of La bor
22–24 d p. 448 Emergency Medica l Trea tment a nd La bor Act—EMTALA
22–25 d p. 448 Ruptured Membra nes
22–26 c p. 449 Cervica l Assessment
22–27 d p. 449 Cervica l Assessment
22–28 a p. 451 Urina ry Bla dder Function
22–29 c p. 451 Ma na gement of the Second Sta ge of La bor
22–30 a p. 451 Ma na gement of the Second Sta ge of La bor
22–31 b p. 452 Figure 2 2 -2 5
22–32 b p. 452 Figure 2 2 -2 5
150

CHAPTER 23

Abnormal Labor

23–1. Which of the following may be responsible for 23–3. In this diagram below, what represents the biggest
dystocia in labor? obstacle to labor and delivery?
a. Bony-pelvis abnormalities a. Prominent coccyx
b. Inadequate expulsive forces b. Contraction band in the lower uterine segment
c. Soft-tissue abnormalities of the reproductive tract c. Decreased anteroposterior diameter of the pelvic
d. All of the above inlet
d. A and C
23–2. Which of the following is true regarding
cephalopelvic disproportion?
a. It currently is responsible for 34% of dystocia
cases.
b. It is a term that originated in the 1960s to
describe abnormal bony pelves.
c. It was defined during a time when dystocia
developed secondary to vitamin D deficiency or
rickets, which is now rare in developed countries.
d. B and C

Ac
tiv
e

Pa
ss
iv
.
.R

e
C

.
.R s
C t.o
s

In
o

s
s
t.

.o
o
In

xt
t.

E
x
E

A B

Reproduced with permission from Cunningham FG, Leveno KJ, Bloom SL, et al (eds): Abnormal labor. In Williams Obstetrics, 24th ed. New York, McGraw-Hill, 2014, Figure 23-1.
C. R. = contraction ring, Ext. = external, Int. = internal.
Abnorma l La bor 151

23–4. The National Institute of Child Health and Human 23–7. Where are contraction forces the greatest during
Development (NICHD) and American College normal labor?
of Obstetricians and Gynecologists (ACOG) have a. Fundus

C
made recommendations concerning the diagnosis
b. Lower uterine segment

H
of arrested second-stage labor. Which of the

A
c. Midzone of the posterior uterine wall

P
following statements are consistent with their

T
recommendations? d. Forces are equal throughout the uterus

E
R
a. Arrested labor in the second stage should not be

2
23–8. The Montevideo group concluded that which of

3
diagnosed until adequate time has elapsed.
the following was the lowest contraction pressure
b. Before this diagnosis is given, nulliparas without
necessary to cause cervical dilation?
epidural anesthesia should be allowed 2 hours
without progress. a. 15 mm Hg
c. Before this diagnosis is given, nulliparas without b. 25 mm Hg
epidural anesthesia should be allowed 3 hours c. 35 mm Hg
without progress. d. 45 mm Hg
d. A and C
23–9. Terms to describe specific active-phase abnormalities
23–5. At Parkland Hospital, neonates delivered from include which of the following?
parturients whose second-stage labor lasted > 3 a. Arrest disorders
hours had which of the following adverse outcomes
b. Saltatory disorders
compared with neonates of mothers with shorter
second-stage labor? c. Protraction disorders
a. Neonates from each group of parturients had d. A and C
equivalent rates of perinatal morbidity.
b. The lowest prevalence of 5-minute Apgar scores
≤ 3 was noted in the group of parturients with
longer second-stage labor.
c. The percentage of neonates requiring resuscitative
efforts was higher in the group of parturients with
longer second-stage labor.
d. B and C

23–6. Which of the following is among the advances in


labor dysfunction management?
a. Use of oxytocin
b. Reliance on midforceps deliveries for transverse
arrest
c. Realization that undue prolongation of labor
leads to increased perinatal morbidity
d. A and C
152 La bor

23–10. What is the total number of Montevideo units 23–14. Which of the following is true regarding epidural
shown in this monitor strip? anesthesia during labor?
a. 235 a. It slows the first stage of labor.
S
b. 242 b. It slows the second stage of labor.
E
C
c. 196 c. It has no effect on the length of labor.
T
I
d. None of the above d. A and B
O
N
7
FHR 240 bpm FHR 240 bpm FHR 240 bpm
210 210 210
10 min
180 180 180

150 150 150

120 120 120

90 90 90

60 60 60

30 30 30

100 100 100

1 75 75 75
2 3 4 5

50 50 50

25 25 25

mmHg UA mmHg UA mmHg UA mmHg


0 0 0
52 mm Hg 50 mm Hg 47 mm Hg 44 mm Hg 49 mm Hg

Reproduced with permission from Cunningham FG, Leveno KJ, Bloom SL, et al (eds): Abnormal labor. In Williams Obstetrics, 24th ed. New York, McGraw-Hill, 2014, Figure 23-3.

23–11. According to data from Menticoglou (1995a,b), 23–15. Which of the following is true regarding
approximately what percentage of parturients chorioamnionitis and its effects on labor?
achieved spontaneous vaginal delivery in the a. Infection in early labor is a cause of labor
subsequent hour once second-stage labor reached dysfunction.
5 hours?
b. Infection in late second-stage labor is a
a. 0.5% by-product of dysfunctional labor.
b. 5% c. Chorioamnionitis is most often associated with
c. 15% precipitous labor.
d. 20% d. A and C

23–12. Which of the following is true regarding coached 23–16. For low-risk parturients, walking in the first stage of
maternal pushing efforts during second-stage labor? labor has which of the following effects?
a. It has no effect on second-stage length. a. Has no effect on labor length
b. It significantly shortens the second stage. b. Decreases second-stage labor length
c. It shortens the second stage but has no effect on c. Decreases the neonatal 5-minute Apgar score
maternal or neonatal morbidity rates. d. Increases the length of the latent phase of labor
d. B and C
23–17. Compared with recumbent positioning, upright
23–13. In laboring nulliparas, fetal station above 0 is positions during second-stage labor are associated
associated with which of the following? with which of the following?
a. A 25% cesarean rate a. Less pain
b. A 50% cesarean rate b. Slightly shorter labor duration
c. A higher cesarean rate than if the head is engaged c. Higher rates of blood loss exceeding 500 mL
d. A and C d. All of the above
Abnorma l La bor 153

23–18. Laboring in a birthing tub is associated with higher 23–23. The computed tomographic image shown here
rates of which adverse neonatal outcome? demonstrates which of the following?
a. Waterborne infection

C
b. Neonatal hypocalcemia

H
A
c. Neonatal intensive care admission

P
T
d. A and C

E
R
2
23–19. According to research by Hannah (1996) and Peleg

3
(1999), which of the following is true regarding
premature rupture of membranes at term?
a. Cesarean delivery rates were lowest in those
managed expectantly.
b. Oxytocin induction led to the lowest rates of
chorioamnionitis.
c. Prophylactic antibiotics significantly lowered rates
of chorioamnionitis.
d. A and C

23–20. Which of the following is true regarding precipitous Reproduced with permission from Cunningham FG, Leveno KJ, Bloom SL, et al (eds):
labor? Abnormal labor. In Williams Obstetrics, 24th ed. New York, McGraw-Hill, 2014, Figure
23-4C.
a. Defined as delivery within 3 hours of labor onset
b. May result from diminished pelvic soft-tissue
a. Obstetrical conjugate
resistance
b. Intertuberous diameter
c. May result from a decreased sensation and
awareness of active labor c. Transverse diameter of the midpelvis
d. All of the above d. Transverse diameter of the pelvic inlet

23–21. Which of the following is an associated complication 23–24. Which interischial tuberous diameter measurement
of precipitous labor and delivery? serves as the threshold to define pelvic outlet
contraction?
a. Uterine atony
a. 7 cm
b. Chorioamnionitis
b. 8 cm
c. Shoulder dystocia
c. 9 cm
d. A and B
d. 10 cm
23–22. In obstetrics, which of the following defines a
contracted pelvic inlet? 23–25. Your patient has a history of a prior pelvic fracture.
Which of the following is true regarding this
a. A transverse diameter < 12 cm
condition?
b. A diagonal conjugate < 11.5 cm
a. Most cases are caused by a fall.
c. An anteroposterior diameter < 10 cm
b. It is a contraindication to vaginal delivery.
d. All of the above
c. Bony anatomy must be reviewed with pelvimetry
prior to allowing vaginal delivery.
d. A and C
154 La bor

23–26. The graphic below demonstrates the prevalence 23–27. This image illustrates which fetal presentation?
of cesarean deliveries after a failed forceps delivery
attempt plotted against fetal birthweight. Which of
the following is true regarding these data?
S
E
C
T
I
O
25
N
7
20
t
n
e
15
c
r
e
P
10

5
9
9

9
9

0
9

4
9


9

0
9

0
9

6
9

4
7

4
2

4

3
0

3
0

3
5

2
2

Birthwe ight

Reproduced with permission from Cunningham FG, Leveno KJ, Bloom SL, et al (eds):
Abnormal labor. In Williams Obstetrics, 24th ed. New York, McGraw-Hill, 2014, Figure
23-5.

a. Most cesarean deliveries occurred in women with Reproduced with permission from Cunningham FG, Leveno KJ, Bloom SL, et al (eds):
Abnormal labor. In Williams Obstetrics, 24th ed. New York, McGraw-Hill, 2014, Figure
macrosomic babies. 23-6.
b. Fetal size appears to be the significant contributor
to failed forceps deliveries. a. Brow presentation
c. Nearly 20% of cesarean deliveries occurred in b. Face presentation
women whose newborns weighed < 3100 g.
c. Occiput presentation
d. None of the above
d. Synciput presentation

23–28. Which of the following is a risk factor for face


presentation?
a. Prematurity
b. Multiparity
c. Anencephaly
d. All of the above
Abnorma l La bor 155

23–29. This image illustrates which fetal presentation? 23–30. This vigorous newborn most likely presented how
during labor?

C
H
A
P
T
E
R
2
3
Reproduced with permission from Cunningham FG, Leveno KJ, Bloom SL, et al (eds):
Abnormal labor. In Williams Obstetrics, 24th ed. New York, McGraw-Hill, 2014, Figure a. Brow presentation
23-8.
b. Face presentation
a. Brow presentation c. Occiput presentation
b. Face presentation d. Synciput presentation
c. Occiput presentation
d. Synciput presentation
156 La bor

23–31. Which of the following describes the position of the 23–33. Which of the following complications may follow
fetus in this drawing? vaginal delivery with the presentation shown here?
S
E
C
T
I
O
N
7
Reproduced with permission from Cunningham FG, Leveno KJ, Bloom SL, et al (eds):
Abnormal labor. In Williams Obstetrics, 24th ed. New York, McGraw-Hill, 2014, Figure
23-11.

Reproduced with permission from Cunningham FG, Leveno KJ, Bloom SL, et al (eds): a. Klumpke palsy
Abnormal labor. In Williams Obstetrics, 24th ed. New York, McGraw-Hill, 2014, Figure
23-9C. b. Cookie-cutter scalp laceration
c. Significant brachial plexus injury
a. Left acromidorsoanterior d. Ischemic necrosis of the presenting forearm
b. Left acromidorsoposterior
c. Right acromidorsoanterior 23–34. The following adverse outcomes are associated with
dystocia?
d. Right acromidorsoposterior
a. Chorioamnionitis
23–32. Common causes of transverse lie include which of b. Retained placenta
the following? c. Puerperal endometritis
a. Nulliparity d. A and C
b. Prolonged labor
c. Placenta previa 23–35. In which of the following clinical scenarios is
prolonged labor associated with uterine rupture?
d. Oligohydramnios
a. High parity
b. Previous cesarean delivery
c. 32-week fetus in a transverse lie
d. All of the above

23–36. Prolonged labor can result in which of the following


maternal complications?
a. Uterine rupture
b. Fistula formation
c. Symphyseal necrosis
d. A and B
Abnorma l La bor 157

23–37. Which of the following nerves is more commonly References


injured during vaginal delivery due to poor patient
Hannah M, Ohlsson A, Farine D, et al: International Term PROM
positioning? Trial: a RCT of induction of labor for prelabor rupture of mem-

C
a. Femoral nerve branes at term. Am J Obstet Gynecol 174:303, 1996

H
A
b. Ilioinguinal nerve Menticoglou SM, Manning F, Harman C, et al: Perinatal outcomes in

P
relation to second-stage duration. Am J Obstet Gynecol 173:906,

T
c. Genitofemoral nerve

E
1995a

R
d. Common fibular nerve (formerly common Menticoglou SM, Perlman M, Manning FA: High cervical spinal cord

2
peroneal nerve) injury in neonates delivered with forceps: report of 15 cases. Obstet

3
Gynecol 86:589, 1995b
Peleg D, Hannah ME, Hodnett ED, et al: Predictors of cesarean deliv-
ery after prelabor rupture of membranes at term. Obstet Gynecol
93:1031, 1999
158 La bor

CHAPTER 23 ANSw ER KEy

Q uestion Letter Pa ge
S
number a nswer cited Hea der cited
E
C
T
23–1 d p. 455 Dystocia
I
O
23–2 c p. 455 Dystocia Descriptors
N
23–3 b p. 456 Mecha nisms of Dystocia
7
23–4 d p. 457 Ta ble 2 3 -3
23–5 c p. 459 Ta ble 2 3 -5
23–6 d p. 458 Abnorma lities of the Expulsive Forces
23–7 a p. 458 Types of Uterine Dysfunction
23–8 a p. 458 Types of Uterine Dysfunction
23–9 d p. 459 Active-Pha se Disorders
23–10 b p. 459 Active-Pha se Disorders
23–11 c p. 459 Second-Sta ge Disorders
23–12 c p. 461 Ma terna l Pushing Efforts
23–13 c p. 461 Feta l Sta tion a t O nset of La bor
23–14 d p. 461 Epidura l Anesthesia
23–15 b p. 461 Chorioa mnionitis
23–16 a p. 461 Ma terna l Position During La bor
23–17 d p. 462 Birthing Position in Second-Sta ge La bor
23–18 d p. 462 W a ter Immersion
23–19 b p. 462 Prema ture Ruptured Membra nes a t Term
23–20 d p. 462 Precipitous La bor a nd Delivery
23–21 a p. 462 Ma terna l Effects
23–22 d p. 463 Contra cted Inlet
23–23 c p. 463 Contra cted Midpelvis
23–24 b p. 463 Contra cted O utlet
23–25 c p. 464 Pelvic Fra ctures
23–26 c p. 464 Feta l Dimensions in Fetopelvic Disproportion
23–27 b p. 466 Fa ce Presenta tion
23–28 d p. 466 Etiology
23–29 a p. 467 Brow Presenta tion
23–30 b p. 466 Fa ce Presenta tion
23–31 c p. 468 Tra nsverse Lie
23–32 c p. 468 Etiology
23–33 d p. 469 Ma na gement a nd Prognosis
23–34 d p. 470 Ma terna l Complica tions
23–35 d p. 470 Uterine Rupture
23–36 d p. 471 Fistula Forma tion
23–37 d p. 471 Postpa rtum Lower Extremity N erve Injury
159

CHAPTER 24

Intrapartum Assessment

24–1. Which of the following is currently the most 24–4. In cases of fetal demise, external Doppler ultrasound
prevalent obstetrical procedure performed in the will most likely detect signals from which of the
United States? following?
a. Episiotomy
b. Fetal monitoring
c. Cesarean delivery
d. Operative vaginal delivery

24–2. Continuous R-to-R wave fetal heart rate


computation is reflected clinically as which of the
following?
a. Periodic change
b. Episodic change
c. Beat-to-beat variability
d. Shift in the heart rate baseline

24–3. Which of the following are scaling factors for fetal


heart rate monitoring recommended by a 1997
National Institute of Child Health and Human
Development (NICHD) workshop?
Reproduced with permission from Cunningham FG, Leveno KJ, Bloom SL, et al (eds):
a. 3 cm/min chart recorder paper speed Intrapartum assessment. In Williams Obstetrics, 24th ed. New York, McGraw-Hill, 2014,
Figure 24-5.
b. 1 cm/min chart recorder paper speed
c. 60 beats per minute per vertical centimeter a. Placenta
d. None of the above b. Maternal heart
c. Maternal aorta
d. None of the above

24–5. According to the National Institute of Child Health


and Human Development (NICHD), a fetal
heart rate acceleration is defined by which of the
following?
a. Accelerations lasting > 5 minutes are considered
a baseline change.
b. It is considered prolonged, if it lasts ≥ 2 minutes
and < 10 minutes.
c. For all gestational ages, an acceleration lasts ≥ 15
seconds and rises ≥ 15 beats above the heart rate
baseline.
d. None of the above
160 La bor

24–6. Decreased fetal heart rate variability most closely 24–10. Which of the following is the most common fetal
reflects which of the following? heart rate deceleration?
a. Fetal hypoxia a. Late deceleration
S
b. Fetal acidemia b. Early deceleration
E
C
c. Fetal hyperglycemia c. Variable deceleration
T
I
O
d. None of the above d. Prolonged deceleration
N
7
24–7. Which of the following would not be expected to 24–11. This deceleration most likely reflects which of the
decrease fetal heart rate variability? following?
a. Meperidine
b. Butorphanol Ons e t Re cove ry
c. Diphenhydramine >30
d. Diabetic ketoacidosis sec

24–8. Evidence would suggest which of the following


concerning magnesium sulfate?
a. It insignificantly decreases fetal heart rate Na dir
variability.
b. It significantly increases the rate of fetal heart rate
accelerations.
Contra ction
c. It insignificantly increases the rate of late fetal
heart rate decelerations.
d. It significantly increases the rate of variable fetal
heart rate decelerations.
Reproduced with permission from Cunningham FG, Leveno KJ, Bloom SL, et al (eds):
24–9. This fetal heart rate pattern may be associated with Intrapartum assessment. In Williams Obstetrics, 24th ed. New York, McGraw-Hill, 2014,
which of the following conditions? Figure 24-14.

240
a. Head compression
210 b. Cord compression
c. Maternal chronic anemia
180
d. Uteroplacental insufficiency
150

120

90

60

30

Reproduced with permission from Cunningham FG, Leveno KJ, Bloom SL, et al (eds):
Intrapartum assessment. In Williams Obstetrics, 24th ed. New York, McGraw-Hill, 2014,
Figure 24-13.

a. Alloimmunization
b. Intracranial hemorrhage
c. Meperidine administration
d. All of the above
Intra pa rtum Assessment 161

24–12. This deceleration most likely reflects which of the 24–13. The acceleration seen prior to the fetal heart
following? deceleration characterized here is most likely due to
which of the following?

C
Ons e t Re cove ry

H
A
≥30

P
sec

T
Fe ta l he a rt

E
R
ra te

2
4
Na dir

P a rtia l Comple te Pa rtia l


Contra ction occlus ion occlus ion
occlus ion

Ute rine
Reproduced with permission from Cunningham FG, Leveno KJ, Bloom SL, et al (eds): contra ction
Intrapartum assessment. In Williams Obstetrics, 24th ed. New York, McGraw-Hill, 2014,
Figure 24-16. Umbilica l ve in
Umbilica l a rte ry
a. Head compression
Fe ta l
b. Cord compression s ys tolic BP
c. Maternal chronic anemia
Umbilica l
d. Uteroplacental insufficiency cord

Reproduced with permission from Cunningham FG, Leveno KJ, Bloom SL, et al (eds):
Intrapartum assessment. In Williams Obstetrics, 24th ed. New York, McGraw-Hill, 2014,
Figure 24-22.

a. Venous compression induces a baroreceptor-


mediated acceleration.
b. Venous compression induces a chemoreceptor-
mediated acceleration.
c. Arterial compression induces a baroreceptor-
mediated acceleration.
d. Partial placenta separation induces a
chemoreceptor-mediated acceleration.
162 La bor

24–14. Which of the following is true of this fetal heart rate 24–17. Centralized monitoring has which of the following
pattern? affects on obstetrical care?
240 240
a. Increases cesarean delivery rates
S
b. Decreases perinatal morbidity rates
E
C
210 210
c. Decreases detection of critical fetal heart rate
T
I
signals as the number of video display screens
O
180 180
N
150 150
increases
7
d. All of the above
120 120

90 90 24–18. Which of the following intrapartum stimulation tests


are useful to exclude fetal acidemia?
60 60
a. Vibroacoustic stimulation
30 30
b. Digital stroking of the fetal scalp
100 100 c. Allis clamp grasping of the fetal scalp
80 80 d. All of the above
60 60
24–19. Which of the following is true of the diagnosis of
40 40
fetal distress and asphyxia?
20 20
a. Labor is ultimately an asphyxiating event.
0 0
b. High degrees of interobserver agreement are
Reproduced with permission from Cunningham FG, Leveno KJ, Bloom SL, et al (eds): found with fetal heart rate pattern interpretation.
Intrapartum assessment. In Williams Obstetrics, 24th ed. New York, McGraw-Hill, 2014,
Figure 24-24. c. Attention should focus only on intrapartum
events as these are the major contributors to poor
a. This variable deceleration will resolve fetal outcome.
spontaneously. d. All of the above
b. This late deceleration should prompt emergent
delivery. 24–20. In 2008, the National Institute of Child Health
and Human Development (NICHD) convened a
c. This prolonged deceleration should prompt
conference and constructed a three-tiered system for
immediate intervention.
fetal heart rate pattern classification. Which of the
d. This fetal bradycardia will be followed by following accurately characterizes the different tiers?
compensatory fetal tachycardia.
a. Category I: absence of early decelerations and
presence of normal baseline variability
24–15. Which of the following approximates the percentage
of deliveries that have decelerations in the second b. Category II: presence of recurrent late
stage of labor? decelerations and absent baseline variability
a. 33% c. Category III: presence of recurrent variable
decelerations and normal baseline variability
b. 50%
d. None of the above
c. 66%
d. 90% 24–21. Since the introduction of the new National
Institute of Child Health and Human Development
24–16. For low-risk pregnancies, continuous fetal (NICHD) classification of fetal heart rate patterns,
monitoring at admission has which of the following which of the following is true regarding its effect on
affects on obstetrical outcome? perinatal and maternal morbidity?
a. Increases cesarean delivery rates a. Cesarean delivery rates have declined.
b. Decreases perinatal mortality rates b. Neonatal morbidity rates have declined.
c. Decreases perinatal morbidity rates c. Identification of fetal acidosis is easier.
d. All of the above d. There is not a consensus on interpretation and
management recommendations for fetal heart rate
patterns.
Intra pa rtum Assessment 163

24–22. Growing evidence suggests which of the following 24–26. Regarding amnioinfusion, which of the following
regarding meconium aspiration syndrome (MAS) is statements is true?
true? a. It improves neonatal outcomes when used

C
a. Antepartum chronic hypoxia rarely leads to MAS. prophylactically for variable decelerations.

H
A
b. Most cases are secondary to acute intrapartum b. It reduces cesarean delivery rates when used

P
prophylactically for variable decelerations.

T
events.

E
c. According to the American College of

R
c. More than ½ of infants with MAS have cord pH

2
values > 7.20. Obstetricians and Gynecologists, it may be used

4
to treat persistent variable decelerations.
d. All of the above
d. All of the above
24–23. Current guidelines from the American Academy
of Pediatricians and the American College of 24–27. Regarding prophylactic amnioinfusion for thick
Obstetricians and Gynecologists recommend meconium, which of the following statements is
which of following regarding intrapartum neonatal true?
suctioning in the presence of meconium? a. It improves neonatal outcomes.
a. Routine intrapartum suctioning decreases the b. It decreases the incidence of meconium aspiration
incidence of meconium aspiration syndrome. syndrome.
b. For vigorous neonates, intubation and suctioning c. It can be completed in only a few cases that have
below the cords should not be performed. thick meconium-stained fluid.
c. The neonate should immediately be handed to d. It is not recommended by the American
the pediatrician for suctioning below the vocal College of Obstetricians and Gynecologists for
cords. meconium-stained fluid.
d. None of the above
24–28. The American College of Obstetricians and
24–24. In the presence of a critical fetal heart rate change, Gynecologists recommends obtaining umbilical cord
what should the obstetrician do to resuscitate the blood gases in which of the following circumstances?
fetus? a. Routinely
a. Halt an oxytocin infusion b. 5-minute Apgar score ≤ 7
b. Examine the cervix to exclude umbilical cord c. Fetal-growth restriction
prolapse d. None of the above
c. Move the patient to a left lateral decubitus
position and correct epidural-related hypotension 24–29. Regarding the mode of fetal heart rate monitoring
d. All of the above during labor, the American Academy of Pediatricians
and the American College of Obstetricians and
24–25. Which of the following is true concerning tocolysis Gynecologists recommend which of the following?
for treatment of nonreassuring fetal heart rate a. In high-risk pregnancies, intermittent
patterns during labor? auscultation is acceptable.
a. Subcutaneous terbutaline lowers cesarean delivery b. In low-risk pregnancies, continuous external
rates. monitoring is preferred.
b. Small doses of intravenous nitroglycerine are not c. In high-risk pregnancies, intermittent
beneficial. auscultation should be preformed every
c. The American College of Obstetricians and 15 minutes during the second stage of labor.
Gynecologists does not recommend tocolysis d. None of the above
for nonreassuring fetal heart rate patterns due to
insufficient evidence regarding its efficacy. 24–30. Uterine contractions are typically associated with
d. None of the above pain after they reach what intrauterine pressure
threshold?
a. 5 mm Hg
b. 15 mm Hg
c. 25 mm Hg
d. 35 mm Hg
164 La bor

24–31. Your patient, a 34-year-old nullipara, is undergoing 24–33. For the patient in Question 24–31, the fetal heart
oxytocin induction of labor, and her cervix is dilated rate pattern responds to conservative measures, and
6 to 7 cm. Her fetus has a cephalic presentation. She a scalp electrode is applied for direct monitoring.
has been having six contractions per 10 minutes for During the next 2 hours, she progresses to complete
S
E
the past 45 minutes. What term correctly describes cervical dilatation. Fetal station is + 1 to + 2, and
C
T
this contraction pattern? fetal head position is left occiput transverse. The
I
O
a. Normal fetal heart rate has lost all variability, and recurrent
N
late decelerations are present. The patient has
b. Hypersystole
7
now developed chorioamnionitis, and you begin
c. Tachysystole antibiotic therapy. Your appropriate next response is
d. Hyperstimulation which of the following?
a. Prepare for cesarean delivery
24–32. During monitoring of the patient in Question
b. Perform midforceps rotation and delivery
24–31, a prolonged fetal heart rate deceleration
is noted during external monitoring. What is an c. Begin maternal pushing, and if prompt descent
acceptable response? and internal rotation are noted, then complete a
low forceps delivery from + 2 station and occiput
a. Prepare for cesarean delivery
anterior position
b. Administer a 0.25-mg dose of terbutaline
subcutaneously d. Clinical judgment should guide your actions, and
all of the responses may be acceptable
c. Halt oxytocin, move patient to a left lateral
decubitus position, and provide oxygen by mask
d. Clinical judgment should guide your actions, and
all of the responses may be acceptable
Intra pa rtum Assessment 165

CHAPTER 24 ANSw ER KEy

Q uestion Letter Pa ge

C
number a nswer cited Hea der cited

H
A
P
24–1 b p. 473 Introduction

T
E
24–2 c p. 473 Electronic Feta l Monitoring

R
2
24–3 a p. 475 Feta l Hea rt Ra te Pa tterns

4
24–4 c p. 474 Externa l (Indirect) Electronic Monitoring
24–5 b p. 477 Ta ble 2 4 -1
24–6 b p. 479 Decrea sed Va ria bility
24–7 c p. 479 Decrea sed Va ria bility
24–8 a p. 479 Decrea sed Va ria bility
24–9 d p. 482 Sinusoida l Hea rt Ra te Pa ttern
24–10 c p. 484 Va ria ble Decelera tion
24–11 a p. 483 Ea rly Decelera tion
24–12 d p. 483 La te Decelera tion
24–13 a p. 484 Va ria ble Decelera tion
24–14 c p. 487 Prolonged Decelera tion
24–15 d p. 487 Feta l Hea rt Ra te Pa tterns During Second-Sta ge La bor
24–16 a p. 487 Admission Feta l Monitoring in Low-Risk Pregna ncies
24–17 c p. 488 Centra lized Monitoring
24–18 d p. 489 Vibroa coustic Stimula tion
24–19 a p. 491 Pa thophysiology a nd Dia gnosis
24–20 a p. 492 N a tiona l Institutes of Hea lth W orkshops on Three-Tier Cla ssifica tion System
24–21 d p. 492 N a tiona l Institutes of Hea lth W orkshops on Three-Tier Cla ssifica tion System
24–22 c p. 493 Meconium in the Amnionic Fluid
24–23 b p. 493 Meconium in the Amnionic Fluid
24–24 d p. 494 Ma na gement O ptions
24–25 c p. 494 Tocolysis
24–26 d p. 495 Prophyla ctic Amnioinfusion for Va ria ble Decelera tions
24–27 d p. 495 Amnioinfusion for Meconium-Sta ined Amnionic Fluid
24–28 c p. 496 Huma n Evidence
24–29 a p. 497 Current Recommenda tions
24–30 b p. 498 Pa tterns of Uterine Activity
24–31 c p. 499 N ew Terminology for Uterine Contra ctions
24–32 d p. 487 Clinica l Correla tion
24–33 d p. 483 Clinica l Correla tion
166

CHAPTER 25

Obstetrical Analgesia and Anesthesia

25–1. Approximately what percentage o maternal deaths 25–5. What percentage o newborns will need naloxone
are attributable to anesthetic complications? treatment in the delivery room i their mother has
a. 1.2% received meperidine in labor?
b. 2.4% a. 1%
c. 3.6% b. 3%
d. 4.8% c. 5%
d. 10%
25–2. Which o the ollowing statements is true regarding
the American College o Obstetricians and 25–6. Which patient should not receive naloxone while in
Gynecologists opinion on which patients should labor?
receive anesthesia in labor? a. A patient with severe preeclampsia
a. All patients with heart disease b. A patient with respiratory depression
b. All patients with severe preeclampsia c. A newborn o a narcotic-addicted mother
c. All patients with gestational diabetes d. A patient who has just received intravenous
d. Any woman who requests it and has no morphine
contraindication to its administration
25–7. What is the direct cause o most maternal deaths
25–3. Which parenteral anesthetic agent has the shortest involving regional anesthesia?
neonatal hal -li e? a. Drug reaction
a. Morphine b. Cardiac arrhythmia
b. Nalbuphine c. High spinal blockade
c. Meperidine d. Central nervous system in ection
d. Butorphanol

25–4. What is the hal -li e o meperidine in the newborn?


a. 4 hr
b. 9 hr
c. 13 hr
d. 21 hr
O bstetrica l Ana lgesia a nd Anesthesia 167

25–8. In the igure below, blockade at which sensory level 25–10. In the image shown below, which ligament is the
would provide the best analgesia during early labor? needle passing through to reach the pudendal nerve?

C
H
A
P
T
E
R
A

2
5
B

Modif ed with permission rom Cunningham FG, Leveno KJ, Bloom SL, et al (eds):
Obstetrical analgesia and anesthesia. In Williams Obstetrics, 24th ed. New York,
McGraw-Hill, 2014, Figure 25-2.

Modif ed with permission rom Eltzschig HK, Lieberman ES, Camann WR: Medical
progress: regional anesthesia and analgesia or labor and delivery. N Engl J Med 348: a. Pudendal ligament
319-332, 2003, Figure 1. b. Sacroiliac ligament
c. Sacrospinous ligament
a. A
d. Sacrotuberous ligament
b. B
c. C 25–11. Which o the ollowing statements is true regarding
d. D butorphanol in labor?
a. Neonatal depression is greater than with
25–9. Which nerve is primarily involved with the pain meperidine.
associated with perineal stretching? b. It can be administered contiguously with
a. Ischial nerve meperidine.
b. Pudendal nerve c. It can be associated with a transient sinusoidal
c. Hypogastric nerve etal heart rate.
d. Frankenhäuser ganglion d. All o the above

25–12. A patient in early labor is sitting up or her epidural.


An anesthetic test dose is given. The patient’s heart
rate and blood pressure rise immediately a ter
administration o the test dose. What has most likely
caused her change in vital signs?
a. The patient just had a contraction.
b. The test dose was given intravenously.
c. The text dose created high spinal blockade.
d. None o the above
168 La bor

25–13. A diabetic, preeclamptic patient requires cesarean 25–16. All EXCEPT which o the ollowing are true or
delivery or breech presentation and is sitting up the quality o regional anesthesia that reaches the
or a spinal anesthetic block. A ter administration dermatome level marked by the X in this igure?
o her spinal block, she has a seizure. Which o the
S
E
ollowing diagnoses should be considered in the
C
V1
T
di erential?
I
O
a. Eclamptic seizure V2
N
V3
b. High spinal blockade
7
C3
c. Pro ound hypoglycemia C4
C5
d. All o the above T1
T2
25–14. Re erring to the patient in Question 25–13, the etal
heart rate tracing is notable or bradycardia while the T4
patient is seizing. Which drug would be most help ul C6
T6
in allowing intubation o the patient?
X
a. Diazepam T8
b. Succinylcholine T10
c. Magnesium sul ate T11
d. All o the above T12
L1
S2 C7
25–15. Which anesthetic is associated with neurotoxicity S3
L2
C8
and cardiotoxicity at virtually identical serum drug
levels?
L3
a. Lidocaine
b. Tetracaine
L4
c. Bupivacaine
d. Ropivacaine

L5

S1

Modif ed with permission rom Cunningham FG, Leveno KJ, Bloom SL, et al (eds):
Obstetrical analgesia and anesthesia. In Williams Obstetrics, 24th ed. New York,
McGraw-Hill, 2014, Figure 25-4A.

a. It is adequate or orceps delivery.


b. It is adequate or a cesarean delivery.
c. It is adequate or spontaneous vaginal delivery.
d. All o the above

25–17. Using the same image in Question 25–16, regional


anesthesia that reaches which dermatome level is
required or cesarean delivery?
a. T 4
b. T 6
c. T 8
d. T 10
O bstetrica l Ana lgesia a nd Anesthesia 169

25–18. Which complication occurs with approximately 15% 25–21. Which o the ollowing interventions has been
o paracervical blocks? shown to reduce the incidence o postdural puncture
a. In ection headache?

C
b. Fetal bradycardia a. Vigorous prehydration

H
A
c. Hematoma ormation b. Prophylactic blood patch

P
T
d. Intravascular injection c. Use o a smaller-gauge needle

E
R
d. Keeping the patient supine during labor

2
25–19. What is the main reason or the addition o glucose

5
to the anesthetic agents chosen or a spinal blockade? 25–22. Absolute contraindications to regional anesthesia
a. To make the solution hyperbaric include all EXCEPT which o the ollowing?
b. To make the solution hypertonic a. Scoliosis
c. To provide glucose to the patient, who should be b. Maternal coagulopathy
NPO c. Skin in ection over the site o needle placement
d. To minimize hypotension associated with spinal d. Use o low-molecular-weight heparin in the prior
blockade 6 hours

25–20. When used prophylactically in the obstetrical 25–23. Which structure in the image here is identi ied by
anesthesia setting, which vasopressor has been the letter X?
associated with etal acidemia? a. Dura mater
a. Ephedrine b. Epidural space
b. Ergonovine c. Ligamentum lavum
c. Phenylephrine d. Internal venous plexus
d. Methylergonovine

L3

L4

Epidural
ne e dle

Modif ed with permission rom Cunningham FG, Leveno KJ, Bloom SL, et al (eds): Obstetrical anesthesia. In Williams Obstetrics, 23rd ed. New
York, McGraw-Hill, 2010, Figure 19-4.
170 La bor

25–24. The spread o anesthesia a ter epidural placement 25–30. An opiate was used or epidural analgesia in
can be in luenced by all EXCEPT which o the a patient’s cesarean delivery, and now she is
ollowing? complaining o itching and being unable to
a. Maternal position empty her bladder. Which drug will eliminate her
S
E
symptoms without a ecting the analgesic action o
C
b. Dose o anesthetic
T
the opiate?
I
c. Type o catheter used
O
a. Naloxone
N
d. Location o catheter tip
b. Cetirizine
7
25–25. Which is the most common complication c. Bupivacaine
encountered during epidural anesthesia? d. Diphenhydramine
a. Fever
25–31. A patient requiring emergent cesarean delivery has
b. Hypotension
a patchy epidural block and needs local in iltration
c. Total spinal blockade o anesthesia to augment the blockade. In the image
d. Ine ective analgesia here, which nerve is identi ied by the letter X?

25–26. All EXCEPT which o the ollowing are associated


with breakthrough pain a ter epidural anesthesia is
initially established?
a. Nulliparity
b. Heavier etal weight
c. Lower maternal body mass index (BMI)
d. Catheter placed at earlier cervical dilation

25–27. Compared with intravenous meperidine, epidural


anesthesia is associated with higher rates o all
EXCEPT which o the ollowing?
a. Cesarean delivery
b. Oxytocin stimulation
c. Operative vaginal delivery
d. Prolonged irst-stage labor
X
25–28. According to the American College o Obstetricians
and Gynecologists, what is the threshold below
which thrombocytopenia may prevent a patient rom
receiving epidural anesthesia?
a. 50,000
b. 75,000
Modif ed with permission rom Cunningham FG, Leveno KJ, Bloom SL, et al (eds):
c. 100,000 Obstetrical analgesia and anesthesia. In Williams Obstetrics, 24th ed. New York,
d. 150,000 McGraw-Hill, 2014, Figure 25-6.

25–29. A patient with a known thrombophilia has just a. Ischial nerve


had a vaginal delivery under epidural anesthesia. b. Intercostal nerve
She had discontinued her low-dose low-molecular-
c. Hypogastric nerve
weight anticoagulant prior to induction o labor.
When would it be sa e to restart her anticoagulant d. Ilioinguinal nerve
postpartum?
a. Prior to removal o her epidural catheter
b. As soon as her epidural catheter is removed
c. At least two hours a ter epidural catheter removal
d. When her partial thromboplastin time (PTT) is
normal
O bstetrica l Ana lgesia a nd Anesthesia 171

25–32. O the ollowing steps taken prior to the induction 25–35. Which pulmonary lobe is most o ten involved in
o general anesthesia, which has been the key actor aspiration as a complication o general anesthesia?
in decreasing maternal mortality rates rom general

C
anesthesia?

H
a. Antacids

A
A

P
b. Preoxygenation

T
E
R
c. Uterine displacement

2
d. Aggressive intravenous hydration

5
25–33. What is the rate o ailed intubation or general
anesthesia in pregnancy?
B
a. 1 in 100
b. 1 in 250
c. 1 in 400 C D
d. 1 in 550

25–34. O agents used or induction o general anesthesia, a. Right upper lobe


which is associated with delirium and hallucinations? b. Right middle lobe
a. Ketamine c. Right lower lobe
b. Propo ol d. Le t lower lobe
c. Thiopental
d. Succinylcholine 25–36. I a patient has emesis o gastric contents
during induction o general anesthesia, which
o the ollowing steps is indicated to limit the
complications o aspiration?
a. Saline lavage
b. Initiation o prophylactic antibiotics
c. Administration o corticosteroid therapy
d. Suctioning o inhaled luid rom pharynx and
trachea
172 La bor

CHAPTER 25 ANSw ER KEy

Q uestion Letter Pa ge
S
number a nswer cited Hea der cited
E
C
T
25–1 a p. 504 Introduction
I
O
25–2 d p. 504 O bstetrica l Anesthesia Services
N
25–3 b p. 507
7
Ta ble 2 5 -3
25–4 c p. 506 Meperidine a nd Prometha zine
25–5 b p. 506 Meperidine a nd Prometha zine
25–6 c p. 507 Effica cy a nd Sa fety of Pa rentera l Agents
25–7 c p. 504 Introduction
25–8 a p. 505 Principles of Pa in Relief
25–9 b p. 505 Principles of Pa in Relief
25–10 c p. 508 Pudenda l Block, Figure 2 5 -2
25–11 c p. 506 Butorpha nol (Sta dol)
25–12 b p. 507 Anesthetic Agents
25–13 d p. 507 Centra l N ervous System Toxicity
25–14 b p. 507 Centra l N ervous System Toxicity
25–15 c p. 508 Ca rdiova scula r Toxicity
25–16 b p. 510 Va gina l Delivery
25–17 a p. 511 Cesa rea n Delivery
25–18 b p. 509 Pa ra cervica l Block
25–19 a p. 510 Va gina l Delivery
25–20 a p. 511 Hypotension
25–21 c p. 512 Postdura l Puncture Hea da che
25–22 a p. 512 Contra indica tions to Spina l Ana lgesia , Ta ble 2 5 -6
25–23 d p. 510 Figure 2 5 -3
25–24 c p. 513 Continuous Lumba r Epidura l Block
25–25 b p. 511 Hypotension, Ta ble 2 5 -5
25–26 c p. 514 Ineffective Ana lgesia
25–27 a p. 515 Effect on La bor, Ta ble 2 5 -8
25–28 a p. 516 Thrombocytopenia
25–29 c p. 516 Anticoa gula tion
25–30 a p. 517 Epidura l O pia te Ana lgesia
25–31 d p. 517 Loca l Infiltra tion for Cesa rea n Delivery, Figure 2 5 -6
25–32 a p. 518 Pa tient Prepa ra tion
25–33 b p. 517 Genera l Anesthesia
25–34 a p. 518 Induction of Anesthesia
25–35 c p. 519 Pa thophysiology
25–36 d p. 520 Trea tment
173

CHAPTER 26

Induction and Augmentation of Labor

26–1. Compared with the induction of labor, the 26–4. Women whose labors are managed with amniotomy
augmentation of labor differs in what regard? are at increased risk for which complication?
a. The fetal membranes are intact. a. Uterine atony
b. Oxytocin is titrated to effect. b. Chorioamnionitis
c. Contractions are pharmacologically stimulated. c. Cervical lacerations
d. Previously commenced labor fails to effect d. All of the above
cervical change.
26–5. A 30-year-old G2P1 at 37 weeks’ gestation with one
26–2. All EXCEPT which of the following are prior cesarean delivery presents with contractions
contraindications to labor induction? and premature rupture of the fetal membranes.
a. Twin gestation Her cervix is 3 cm dilated. She requests a trial of
labor and is deemed an appropriate candidate. An
b. Breech presentation oxytocin infusion is initiated, and 2 hours later, you
c. Fetal-growth restriction are called to the room to evaluate the fetal heart rate
d. Prior vertical hysterotomy cesarean delivery tracing, which is shown below. According to a large
study conducted by the Maternal-Fetal Medicine
26–3. The risk for cesarean delivery is increased in women Units Network, the use of oxytocin increases the risk
undergoing induction of labor in which of the for uterine rupture by what magnitude in women
following situations? with a prior cesarean delivery?
a. Low Bishop score a. Threefold
b. Engaged fetal head b. Sixfold
c. Multiparous parturient c. Tenfold
d. All of the above d. No change from background risk in women
undergoing trial of labor after cesarean
174 La bor

26–6. The patient presented in Question 26–5 is taken 26–11. A 22-year-old primigravida is diagnosed with severe
emergently for cesarean delivery due to uterine preeclampsia at 39 weeks’ gestation. A magnesium
rupture. If misoprostol had been considered as sulfate infusion is initiated for seizure prophylaxis,
an induction agent rather than oxytocin, what is and plans are made for induction of labor. Her
S
E
the safest prostaglandin route and dose to use in a cervix is 3 cm dilated, 50-percent effaced, slightly
C
T
patient with a uterine cesarean scar? soft, and located anteriorly. The fetal head is at –1
I
O
a. Oral administration only station. What is her Bishop score?
N
b. Low-dose preparation only a. 6
7
c. Vaginal administration only b. 7
d. They should be avoided completely. c. 8
d. 9
26–7. Labor induction or augmentation increases the
likelihood of which of the following peripartum 26–12. Of the five elements that comprise the Bishop
complications? scoring system, only three are significantly associated
a. Hysterectomy with predicting successful vaginal delivery. This
simplified Bishop scoring system includes all
b. Uterine atony
EXCEPT which of the following?
c. Postpartum hemorrhage
a. Fetal station
d. All of the above
b. Cervical dilation
26–8. Which of the following women would be most likely c. Cervical effacement
to have a successful induction of labor? d. Cervical consistency
a. G2P1 with a body mass index of 34 and a
neonatal birthweight of 3250 g 26–13. Intracervical administration of dinoprostone
(Prepidil) for the purpose of cervical ripening may
b. G1P0 with a body mass index of 25 and a
be repeated every 6 hours with a maximum of how
neonatal birthweight of 3800 g
many doses?
c. G2P1 with a body mass index of 27 and
a neonatal birthweight of 3150 g a. 2
d. G1P0 with a body mass index of 31 and a b. 3
neonatal birthweight of 2900 g c. 4
d. 5
26–9. Which of the following administration routes is
acceptable for preinduction cervical ripening with 26–14. When administering dinoprostone using the device
prostaglandin E2 (dinoprostone)? shown here, which of the following should be avoided?
a. Intravenous
b. Intravaginal
c. Intramuscular
d. All of the above

26–10. Use of cervical ripening agents is associated with


which of the following outcomes?
a. Labor initiation
b. Decreased cesarean delivery rate
c. Decreased maternal morbidity rate
d. All of the above

a. Subsequent use of oxytocin


b. Use of lubricants during insertion
c. Removal of the device with labor onset
d. Recumbent positioning for the first 2 hours after
insertion
Induction a nd Augmenta tion of La bor 175

26–15. Based on the available literature, the use of 26–17. A 25-year-old G2P1 at 41 weeks’ gestation presents
dinoprostone appears to have what effect on the for preinduction cervical ripening, and a 10-mg
cesarean delivery rate? dinoprostone insert (Cervidil) is placed in the

C
a. Unchanged posterior vaginal fornix. Thirty minutes later, she

H
is noted to have 6 contractions every 10 minutes.

A
b. Decreased

P
What is the most appropriate next step in

T
c. Increased for fetal distress management?

E
R
d. Increased for labor dystocia a. Remove the insert

2
6
b. Irrigate the vagina
26–16. A 16-year-old primigravida is admitted to the
hospital for preterm, premature rupture of the c. Administer supplemental oxygen
fetal membranes at 32 weeks’ gestation. Two days d. Increase intravenous fluid administration rate
later, she complains of contractions and vaginal
bleeding, and the following fetal heart rate tracing 26–18. Misoprostol (prostaglandin E1) is approved by
is noted to be associated with frequent uterine the U.S. Food and Drug Administration for what
contractions. According to the definitions established indication?
by the American College of Obstetricians and a. Labor induction
Gynecologists, what is the appropriate term for this
b. Cervical ripening
condition?
c. Cholelithiasis pain
240 240
d. Peptic ulcer prevention
210 210
26–19. When administered vaginally for labor induction,
180 180
what is the recommended dose of misoprostol
150 150 (prostaglandin E1)?
120 120
a. 25 µg
b. 25 mg
90 90
c. 100 µg
60 60
d. 100 mg
30 30

26–20. Which of the following observations prompted


100 100 investigators to search for clinical agents that
80 80 stimulate nitric oxide (NO) production?
60 60 a. NO is a mediator of cervical ripening.
40 40 b. NO metabolite levels are increased in early labor.
20 20 c. NO production prior to labor is low in postterm
0 0 pregnancies.
d. All of the above
Reproduced with permission from Cunningham FG, Leveno KJ, Bloom SL, et al (eds):
Intrapartum assessment. In Williams Obstetrics, 24th ed. New York, McGraw-Hill, 2014,
Figure 24-24. 26–21. For cervical ripening, the addition of nitric oxide
donors to prostaglandins has been demonstrated to
a. Uterine hypertonus have which of the following outcomes compared
b. Uterine tachysystole with prostaglandins alone?
c. Uterine hyperstimulation a. Shortened time to vaginal delivery
d. Uterine hypercontractility b. Enhanced cervical ripening in term pregnancies
c. Enhanced cervical ripening in preterm
pregnancies
d. None of the above
176 La bor

26–22. When using a transcervical catheter to mechanically 26–25. All EXCEPT which of the following statements
promote cervical ripening, concurrent extraamnionic regarding oxytocin are accurate?
saline infusion through the catheter reduces what a. It was the first polypeptide hormone synthesized.
complication compared with catheter placement
S
b. It may be used for labor induction or
E
without infusion?
C
augmentation.
T
a. Tachysystole
I
c. It can be administered by intravenous or
O
b. Uterine rupture
N
intravaginal routes.
7
c. Chorioamnionitis d. It is one of the most frequently used medications
d. Placental abruption in the United States.

26–23. Compared with prostaglandins for cervical ripening, 26–26. In general, oxytocin infusions should be
transcervical catheters have what benefit? discontinued if the number of contractions per
a. Lower cesarean delivery rate 10 minutes consistently exceeds what value?
b. Lower rates of supplemental oxytocin use a. 3
c. Fewer cases of cardiotocographic changes b. 5
d. All of the above c. 7
d. 10
26–24. For cervical ripening, use of the mechanical dilating
device shown here has which of the following 26–27. What is the mean half-life of oxytocin?
benefits compared with prostaglandins? a. 1 minute
b. 5 minutes
c. 10 minutes
d. 20 minutes

26–28. Potential benefits of a high-dose oxytocin regimen


(4.5 to 6 mU/mL) compared with a low-dose
regimen (0.5–1.5 mU/mL) include which of the
following?
a. Fewer failed inductions
b. Decreased admission-to-delivery intervals
c. Lower rates of intrapartum chorioamnionitis
d. All of the above

26–29. At what oxytocin infusion dose does free-water


clearance begin to decrease markedly?
a. 10 mIU/mL
b. 20 mIU/mL
c. 36 mIU/mL
d. 48 mIU/mL
Reproduced with permission from Cunningham FG, Leveno KJ, Bloom SL, et al (eds):
Abortion. In Williams Obstetrics, 24th ed. New York, McGraw-Hill, 2014, Figure 18-8B.
26–30. On average, epidural analgesia prolongs the active
phase of labor how many minutes?
a. Low cost
a. 60 minutes
b. Patient comfort
b. 90 minutes
c. Lower chorioamnionitis rate
c. 120 minutes
d. Shorter induction-to-delivery intervals
d. 180 minutes
Induction a nd Augmenta tion of La bor 177

26–31. Which of the following can follow amniotomy? 26–32. Membrane stripping has been associated with which
a. Cord prolapse of the following untoward outcomes?
b. Placental abruption a. Chorioamnionitis

C
b. Precipitous labor

H
c. Variable fetal heart rate decelerations

A
c. Patient discomfort

P
d. All of the above

T
d. Premature rupture of the fetal membranes

E
R
2
6
178 La bor

CHAPTER 26 ANSw ER KEy

Q uestion Letter Pa ge
S
number a nswer cited Hea der cited
E
C
T
26–1 d p. 523 Introduction
I
O
26–2 c p. 523 La bor Induction—Contra indica tions
N
26–3 a p. 524 Cesa rea n Delivery Ra te
7
26–4 b p. 524 Chorioa mnionitis
26–5 a p. 524 Rupture of a Prior Uterine Incision
26–6 d p. 524 Rupture of a Prior Uterine Incision
26–7 d p. 524 Uterine Atony
26–8 c p. 524 Fa ctors Affecting Successful Induction
26–9 b p. 525 Ta ble 2 6 -1
26–10 a p. 525 Preinduction Cervica l Ripening
26–11 c p. 525 Cervica l Fa vora bility
26–12 d p. 525 Cervica l Fa vora bility
26–13 b p. 526 Prosta gla ndin E2
26–14 b p. 526 Prosta gla ndin E2
26–15 a p. 526 Prosta gla ndin E2
26–16 b p. 527 Side Effects
26–17 a p. 527 Side Effects
26–18 d p. 527 Prosta gla ndin E1
26–19 a p. 527 Prosta gla ndin E1
26–20 d p. 527 N itric O xide Donors
26–21 d p. 527 N itric O xide Donors
26–22 c p. 528 Tra nscervica l Ca theter
26–23 c p. 528 Tra nscervica l Ca theter
26–24 a p. 528 Hygroscopic Cervica l Dila tors
26–25 c p. 529 O xytocin
26–26 b p. 529 Intra venous O xytocin Administra tion
26–27 b p. 529 Intra venous O xytocin Administra tion
26–28 d p. 529 O xytocin Regimens
26–29 b p. 530 Risks versus Benefits
26–30 a p. 531 Active Pha se Arrest
26–31 d p. 531 Amniotomy for Induction a nd Augmenta tion; Elective Amniotomy
26–32 c p. 532 Membra ne Stripping for La bor Induction
Se c t i o n 8

Del iv er y
180

CHAPTer 27

v ag na D

27–1. Compared with cesarean delivery, spontaneous 27–5. At the end of second-stage labor, the most likely
vaginal delivery has lower associated rates of which position of the fetal occiput at the time of perineal
of the following? distention is which of the following?
a. Hemorrhage a. Occiput anterior
b. Maternal infection b. Occiput posterior
c. Anesthesia-related complications c. Occiput transverse, anterior asynclitic
d. All of the above d. Occiput transverse, posterior asynclitic

27–2. In 2006, a National Institutes of Health (NIH) 27–6. The following drawing represents which event in
State-of-the-Science Conference summarized the delivery?
associations between stress urinary incontinence and
delivery route. Which of the following reflects their
findings?
a. There is no pelvic floor protection from cesarean
delivery.
b. The pelvic floor receives substantive durable
protection from cesarean delivery.
c. The duration of pelvic floor protection from
cesarean delivery is clearly defined.
d. The evidence implicating vaginal delivery as the
main putative agent in stress urinary incontinence
and other pelvic floor disorders is weak and fails
to favor either delivery route.

27–3. Antibiotic prophylaxis against infective endocarditis


is recommended if the mother has which of the
following?
a. Mitral valve prolapse
b. Cyanotic heart disease Reproduced with permission from Cunningham FG, Leveno KJ, Bloom SL, et al (eds):
Vaginal delivery. In Williams Obstetrics, 24th ed. New York, McGraw-Hill, 2014, Figure
c. Prosthetic heart valve 27-1.
d. B and C
a. Crowning
27–4. Regarding patient positioning during second-stage b. Extension
labor, which of the following is true?
c. Expulsion
a. Dorsal lithotomy position is the most widely d. Perineal massage
used.
b. The legs should not be strapped, and this allows 27–7. If expulsive efforts of the parturient are inadequate
quick flexion of the thighs should shoulder for delivery when the fetal head is on the perineum,
dystocia develop. which of the following may be done to aid delivery?
c. Within the leg holder, the popliteal region should a. Low forceps
rest comfortably in the proximal portion and the
b. Ritgen maneuver
heel in the distal portion.
c. Rectal misoprostol
d. All of the above
d. A and B
Va gina l Delivery 181

27–8. Which of the following is true of routine episiotomy 27–12. In the early 20th century, when cesarean delivery
during vaginal delivery? was still associated with excessive maternal mortality
a. Leads to anterior tears involving the urethra and rates, a persistent occiput posterior presentation was

C
labia an important problem during second-stage labor.

H
Which of the following statements is true regarding

A
b. Is preferred instead of individualized use of

P
this figure of a study cohort?
episiotomy

T
e
c. Increases the risk of third- and fourth-degree

r
2
lacerations Te rm pre gna ncie s

7
n = 406
d. B and C

27–9. During the cardinal movements of labor, when the


bisacromial diameter rotates at the introitus after
OA in e a rly la bor OP in e a rly la bor
extension, which of the following is true? n = 347 (85%) n = 61 (15%)
a. This rotation is termed external rotation.
b. The shoulders rotate to a transverse position.
c. Most often, the shoulders require extraction after OP a t de live ry a OP a t de live ry
external rotation. n = 13 (4%) n = 8 (13%)
d. B and C

27–10. Regarding nasopharyngeal suctioning following the


birth of a neonate, the American Heart Association Tota l OP a t de live ry
recommends which of the following? n = 21 (5%)

a. Nasal bulb suctioning should be performed if a 62% of OP pos itions a t de live ry we re OA a t be ginning of la bor
thick meconium is present.
b. Nasal bulb suctioning is performed only if the
initial Apgar score is < 4. Reproduced with permission from Cunningham FG, Leveno KJ, Bloom SL, et al (eds):
Vaginal delivery. In Williams Obstetrics, 24th ed. New York, McGraw-Hill, 2014, Figure
c. The need for nasal bulb suctioning should be 27-6.
determined by the resuscitating team.
d. A and C a. Most occiput anterior presentations at delivery
were initially occiput posterior in early labor.
27–11. Which of the following is true regarding delayed b. Most persistent occiput posterior presentations
umbilical cord clamping after birth of the neonate? were initially occiput anterior in early labor.
a. It is recommended for preterm fetuses. c. Approximately 50% of persistent occiput
b. It delays resuscitation of the depressed infant. posterior presentations were originally occiput
c. There is currently insufficient evidence to support posterior in early labor.
its use in term neonates in the United States. d. Approximately 75% of persistent occiput
d. All of the above posterior presentations were originally occiput
posterior in early labor.
182 Delivery

27–13. During evaluation of a persistent occiput posterior 27–16. Which of the following is the most common
presentation, the fetal scalp is noted at the introitus, neonatal injury with shoulder dystocia?
and the fetal head is palpated above the pubic a. Fractured clavicle
symphysis. Which of the following is appropriate
S
b. Brachial plexus injury
e
action?
C
c. Hypoxic ischemic encephalopathy
T
a. Vacuum-assisted delivery if appropriate anesthesia
i
O
is in place d. B and C
N
b. Manual rotation of the fetal head to occiput
8
27–17. Regarding shoulder dystocia, which of the following
anterior position
is true?
c. Rotational forceps to turn the head to an occiput
anterior position if the provider has appropriate a. It is now relatively predictable.
skills b. Elective inductions in women with suspected
macrosomia may help to reduce maternal and
d. None of the above
neonatal morbidity.
27–14. In the absence of a pelvic architecture abnormality or c. Elective cesarean delivery may be considered for
asynclitism, which of the following is true regarding the diabetic mother whose fetus has an estimated
a transverse fetal head position? weight ≥ 4500 g.
a. Is usually transitory d. B and C
b. Will usually rotate to an occiput anterior position
27–18. Rouse and Owen (1999) predicted that how many
c. Will usually rotate to an occiput posterior prophylactic cesarean deliveries for macrosomia
position would need to be performed to prevent one case of
d. A and B permanent brachial plexus injury?
a. 100
27–15. Which of the following is true regarding the
incidence of shoulder dystocia? b. 500
a. Approximates 1% c. 1000
b. Varies depending on the definition used d. 2000
c. Has increased likelihood because of increasing
fetal birthweights
d. All of the above
Va gina l Delivery 183

27–19. Which maneuver(s) is illustrated in the following


graphic?
a. Rubin maneuver

C
b. McRoberts maneuver

H
A
c. Suprapubic pressure

P
T
d. B and C

e
r
2
7
Reproduced with permission from Cunningham FG, Leveno KJ, Bloom SL, et al (eds): Vaginal delivery. In Williams Obstetrics, 24th ed.
New York, McGraw-Hill, 2014, Figure 27-7.
184 Delivery

27–20. Rubin recommended two maneuvers, one of which 27–22. Which of the following is true regarding shoulder
is depicted here. His maneuvers involve which of the dystocia drills?
following steps? a. A McRoberts maneuver involves at least two
assistants.
S
e
C
b. Suprapubic pressure is the initial approach
T
recommended by the American College of
i
O
Obstetricians and Gynecologists.
N
c. Traction and fundal pressure should be tried
8
first followed by a call for anesthesia, additional
provider help, and pediatric resuscitation.
d. A and B

27–23. Regarding women with prior pelvic reconstructive


surgery and delivery route, which of the following is
true?
a. Vaginal delivery is prohibited in this population.
b. Cesarean delivery is always protective against
symptom recurrence.
c. Most women with prior antiincontinence surgery
can be delivered vaginally without symptom
recurrence.
Reproduced with permission from Cunningham FG, Leveno KJ, Bloom SL, et al (eds):
d. A and B
Vaginal delivery. In Williams Obstetrics, 24th ed. New York, McGraw-Hill, 2014, Figure
27-10B. 27–24. Regarding a hydrocephalic fetus with macrocephaly,
which of the following is true concerning delivery
a. The posterior shoulder is progressively rotated route?
180 degrees in a corkscrew fashion. a. The fetus may not deliver vaginally if the
b. The fetal shoulders are rocked side-to-side by biparietal diameter is < 10 cm.
applying force to the maternal abdomen. b. At the time of cesarean delivery, a long classical
c. The most easily accessible fetal shoulder is pushed incision should be performed.
toward the anterior surface of the fetal chest. c. Cephalocentesis performed suprapubically on the
d. B and C breech fetus may allow a vaginal delivery.
d. B and C
27–21. Which of the following is the initial step in
performing a Zavanelli maneuver? 27–25. Goals of a successful third stage of labor include
a. Perform laparotomy which of the following?
b. Flex the head and push it back into the vagina a. Prevention of uterine inversion
c. Administer subcutaneous terbutaline to relax the b. Prevention of shoulder dystocia
uterus c. Completion of episiotomy repair
d. Restore the fetal head to an occiput anterior or d. All of the above
occiput posterior position
Va gina l Delivery 185

27–26. What important points of placenta delivery are 27–29. Intravenous bolus doses of oxytocin may be
shown in this drawing? particularly dangerous for which of the following
parturients?

C
a. Morbidly obese women

H
A
b. Women with cardiovascular disease

P
T
c. Women suffering significant postpartum

e
r
hemorrhage

2
d. B and C

7
27–30. High-dose oxytocin for extended periods with large
volumes of electrolyte-free, dextrose-containing
solutions can have which of the following effects?
a. Can have a profound diuretic action
b. Can lead to seizures in mothers and newborns
c. Can concentrate electrolytes such as sodium and
lead to water intoxication
d. A and C

27–31. The ergot alkaloid methergine may be given by


which of the following routes?
a. Orally
Reproduced with permission from Cunningham FG, Leveno KJ, Bloom SL, et al (eds): b. Intravenously
Vaginal delivery. In Williams Obstetrics, 24th ed. New York, McGraw-Hill, 2014, Figure
27-12. c. Intramuscularly
d. All of the above
a. The fundus is being elevated.
b. The placenta remains in the uterus. 27–32. Which of the following is true regarding
c. The umbilical cord is being pulled taut. misoprostol?
d. The hand is pushing the fundus toward the a. It is an E2 prostaglandin.
vagina. b. It is more effective than oxytocin in preventing
postpartum hemorrhage.
27–27. Which of the following is most effective to decrease c. It can be given as a single, oral 600-µg dose to
postpartum hemorrhage in third-stage labor? prevent postpartum hemorrhage.
a. Uterotonics d. It is best administered intramuscularly directly
b. Fundal massage into the uterus at the time of hemorrhage.
c. Early cord clamping
d. Manual placenta removal

27–28. Which of the following is the mean half-life of


oxytocin?
a. 1 minute
b. 5 minutes
c. 10 minutes
d. 15 minutes
186 Delivery

27–33. This image represents which of the following 27–35. The restrictive use of episiotomy is associated with
perineal lacerations? which of the following?
a. More healing complications
S
b. Less anterior perineal trauma
e
C
c. Less posterior perineal trauma
T
i
O
d. A and C
N
8
27–36. The following anesthesia methods may be adequate
for perineal repair?
a. Local infiltration
b. Regional anesthesia
c. Pudendal nerve block
d. All of the above

27–37. Regarding repair of a fourth-degree episiotomy or


laceration, which of the following is true?
a. The overlapping technique is acceptable.
b. End-to-end anal sphincter repair is the superior
method.
c. Chromic suture should always be used due to less
tissue inflammation.
Reproduced with permission from Cunningham FG, Leveno KJ, Bloom SL, et al (eds): d. Prophylactic antibiotic administration has strong
Vaginal delivery. In Williams Obstetrics, 24th ed. New York, McGraw-Hill, 2014, Figure
27-15C. evidence to support its use with fourth-degree
laceration repair.
a. First-degree laceration
b. Second-degree laceration r f nc s
c. Third-degree laceration
Rouse DJ, Owen J: Prophylactic cesarean delivery for fetal macrosomia
d. Fourth-degree laceration diagnosed by means of ultrasonography—a Faustian bargain? Am J
Obstet Gynecol 181:332, 1999
27–34. Which of the following is true regarding midline
episiotomies?
a. They increase the incidence of anal sphincter
tears.
b. They should never be used in lieu of spontaneous
laceration.
c. They should be routinely cut during the delivery
of nulliparous patients.
d. A and C
Va gina l Delivery 187

CHAPTer 27 ANSw er Key

Q uestion Letter Pa ge

C
number a nswer cited Hea der cited

H
A
P
27–1 d p. 536 Route of Delivery

T
e
27–2 d p. 536 Route of Delivery

r
2
27–3 d p. 536 Prepa ra tion for Delivery

7
27–4 d p. 536 Prepa ra tion for Delivery
27–5 a p. 537 O cciput Anterior Position
27–6 a p. 537 Delivery of the Hea d
27–7 b p. 537 Delivery of the Hea d
27–8 c p. 537 Delivery of the Hea d
27–9 a p. 538 Delivery of the Shoulders
27–10 c p. 538 Delivery of the Shoulders
27–11 d p. 539 Umbilica l Cord Cla mping
27–12 b p. 539 Persistent O cciput Posterior Position
27–13 d p. 540 Delivery of Persistent O cciput Posterior Position
27–14 d p. 540 O cciput Tra nsverse Position
27–15 d p. 541 Shoulder Dystocia
27–16 b p. 541 Shoulder Dystocia
27–17 c p. 541 Prediction a nd Prevention
27–18 c p. 541 Birthweight
27–19 d p. 542 Ma na gement
27–20 d p. 542 Ma na gement
27–21 d p. 542 Ma na gement
27–22 a p. 542 Ma na gement
27–23 c p. 545 Prior Pelvic Reconstructive Surgery
27–24 c p. 546 Anoma lous Fetuses
27–25 a p. 546 Delivery of the Pla centa
27–26 a p. 546 Delivery of the Pla centa
27–27 a p. 547 Ma na gement of the Third Sta ge
27–28 b p. 547 High-Dose O xytocin
27–29 d p. 547 High-Dose O xytocin
27–30 b p. 547 High-Dose O xytocin
27–31 d p. 548 Ergonovine a nd Methylergonovine
27–32 c p. 548 Misoprostol
27–33 c p. 548 Birth Ca na l La cera tions
27–34 a p. 550 Episiotomy Indica tions a nd Consequences
27–35 c p. 550 Episiotomy Indica tions a nd Consequences
27–36 d p. 551 Repa ir of Episiotomy or Perinea l La cera tion
27–37 a p. 552 Fourth-Degree La cera tion Repa ir
188

CHAPTER 28

Breech Deliver

28–1. What percentage of term singleton pregnancies 28–6. Which of the following is a known risk factor for
present breech? breech presentation?
a. 1–2% a. Oligohydramnios
b. 3–4% b. Maternal diabetes
c. 5–6% c. Prior forceps delivery
d. 7–8% d. Anterior placental implantation

28–2. Regarding the prevalence of breech presentation, 28–7. After reviewing all available studies, which of the
which of the following statements is true? following general statements can be made regarding
a. It is stable throughout pregnancy. vaginal delivery of the term breech fetus compared
with cesarean delivery?
b. It approximates 80% at 24 weeks’ gestation.
a. Neonatal mortality rates are lower with cesarean
c. Across pregnancy, it increases with gestational
delivery.
age.
b. Neonatal morbidity rates are lower with cesarean
d. Across pregnancy, it decreases with gestational
delivery.
age.
c. After vaginal breech birth, children at age 2 years
28–3. The Term Breech Collaborative Group studied have lower intelligence scores.
vaginal delivery of the breech fetus. Which of the d. None of the above
following is a criticism of this study?
a. Serious morbidity was defined too strictly. 28–8. Which of the following statements is true regarding
the preterm breech fetus?
b. Only nulliparas were included in the trial.
a. It is always best to deliver a preterm breech by
c. Most of the providers were unskilled at breech
cesarean.
delivery.
b. Neonatal survival rates are equal with vaginal or
d. More than 10% of participants had radiological
cesarean delivery.
pelvimetry.
c. There are no randomized studies regarding
28–4. All EXCEPT which of the following are true optimal delivery route for the preterm breech
regarding the “stargazer” breech fetus? fetus.
a. The fetal head is hyperextended. d. None of the above
b. Forceps are indicated for delivery.
28–9. All EXCEPT which of the following statements are
c. Cesarean delivery is the safest delivery route. true regarding maternal morbidity and mortality in
d. The cervical spinal cord can be injured during breech delivery?
vaginal delivery. a. Hysterotomy extensions can occur with forceps
use.
28–5. What is the risk that breech presentation will
b. Maternal genital tract lacerations can lead to
reoccur at term in a second pregnancy?
infection.
a. 0.5%
c. Maternal death is less likely if the breech is
b. 2% delivered by cesarean.
c. 10% d. Anesthesia needed for relaxation to deliver the
d. 12% breech can lead to postpartum hemorrhage.
Breech Delivery 189

28–10. Which of the following is the least common bone 28–16. A 24-year-old G4P2 presents at term for a routine
fractured in neonates who are delivered vaginally prenatal visit. She has had two prior vaginal
from a breech presentation? deliveries of 6-pound neonates and one prior

C
a. Femur miscarriage. On examination, you suspect a breech-

H
presenting fetus. Which of the following does not

A
b. Radius

P
favor vaginal breech delivery?

T
c. Humerus

E
a. The fetal head is hyperflexed.

R
d. Clavicle
b. Fetal weight approximates 7 lb.

2
8
28–11. Which of the following outcomes that may be c. The patient requests cesarean delivery.
seen with breech presentation is not related to d. The patient has had a prior pregnancy loss.
delivery mode?
a. Erb palsy 28–17. The patient in Question 28-16 is now in advanced
labor and presents to Labor and Delivery. She wishes
b. Hip dysplasia
to attempt vaginal breech delivery. Sonographic
c. Spinal cord injury evaluation shows that her fetus has both hips flexed
d. Sternocleidomastoid muscle hematoma and both knees extended. Which best describes fetal
position?
28–12. Based on imaging studies, which of the following a. Frank breech
biometric thresholds should be used to assess fetal
suitability for vaginal breech delivery? b. Total breech
a. BPD > 80 mm c. Complete breech
b. EFW < 2500 g d. Incomplete breech
c. EFW > 3500 g 28–18. Following emergence of the fetal legs during
d. None of the above vaginal or cesarean delivery of a breech fetus, this
photograph demonstrates which next step?
28–13. What is the best indicator of pelvic adequacy for
vaginal breech delivery?
a. Fetal lie
b. Pelvic radiograph
c. Clinical pelvimetry
d. Normal progression of labor

28–14. Which of the following statements is false regarding


the cardinal movements of breech delivery?
a. The fetal head is born by flexion.
b. The back of the fetus is directed posteriorly.
c. The anterior hip usually descends more rapidly
than the posterior hip.
d. Engagement and descent usually occur with the
bitrochanteric diameter in an oblique plane.
a. Traction on the fetal waist
28–15. Which of the following best describes a breech fetus
b. Continued traction on the fetal legs
that delivers spontaneously up to the umbilicus, but
whose remaining body is delivered with operator c. Placement of thumbs on the fetal sacrum
traction? d. Placement of thumbs on the anterior superior
a. Breech decomposition iliac crests
b. Total breech extraction
c. Partial breech extraction
d. Spontaneous breech delivery
190 Delivery

28–19. To resolve the complication shown in this image, 28–20. What is the utility of the maneuver shown in this
which of the following should be attempted? image?
S
E
C
T
I
O
N
8
Reproduced with permission from Cunningham FG, Leveno KJ, Bloom SL, et al (eds):
Breech delivery. In Williams Obstetrics, 24th ed. New York, McGraw-Hill, 2014, Figure
28-10.

a. The fetus should be rotated through a half circle.


b. The fetus should be pulled downward to release
the arm. Reproduced with permission from Cunningham FG, Leveno KJ, Bloom SL, et al (eds):
Breech delivery. In Williams Obstetrics, 24th ed. New York, McGraw-Hill, 2014, Figure
c. The fetus should be rotated to bring its back 28-11.
directly posterior.
d. The humerus or clavicle should be fractured to a. Breech decomposition
reduce the bisacromial diameter.
b. Safest delivery method for the head of a preterm
breech
c. Fetal head delivery when the back is oriented
posteriorly
d. Release of the aftercoming head in an
incompletely dilated cervix
Breech Delivery 191

28–21. Which of the following is true regarding the forceps 28–22. A patient presents in preterm labor at 30 weeks’
used in this image? gestation. Her cervix is completely dilated, and
the fetus is breech. You are unable to deliver the

C
fetal head. What procedure, used to resolve this

H
complication, is demonstrated in this image?

A
P
T
E
R
2
8
Reproduced with permission from Cunningham FG, Leveno KJ, Bloom SL, et al (eds):
Breech delivery. In Williams Obstetrics, 24th ed. New York, McGraw-Hill, 2014, Figure
28-13C. Reproduced with permission from Cunningham FG, Leveno KJ, Bloom SL, et al (eds):
Breech delivery. In Williams Obstetrics, 24th ed. New York, McGraw-Hill, 2014, Figure
28-14.
a. They have a prominent pelvic curve.
b. They have a downward arc in the shank. a. Symphysiotomy
c. They must be rotated through a 45-degree angle. b. Zavanelli maneuver
d. They must be disarticulated prior to fetal head c. Dührssen incisions
delivery.
d. Mauriceau maneuver

28–23. If the procedure performed in Question 28–22 is


not successful, which of the following may aid fetal
delivery?
a. Piper forceps
b. Fundal pressure
c. Zavanelli maneuver
d. Intravenous nitroglycerin
192 Delivery

28–24. What is the process called in which a frank breech 28–29. Before proceeding with the requested version
presentation is converted to a footling breech attempt, you counsel the patient in Question 28–27
presentation within the upper birth canal? regarding potential risks. Which of the following are
a. Retraction complications of external cephalic version?
S
E
a. Uterine rupture
C
b. Relaxation
T
b. Placental abruption
I
c. Displacement
O
c. Emergency cesarean delivery
N
d. Decomposition
8
d. All of the above
28–25. What is the eponym given to the maneuver
described in Question 28–24? 28–30. All EXCEPT which of the following are absolute
a. Piper maneuver contraindications for external cephalic version?
b. Pinard maneuver a. Placenta previa
c. Mauriceau maneuver b. Prior myomectomy
d. Zavanelli maneuver c. Multifetal gestation
d. Nonreassuring fetal status
28–26. What is likely to be the most adequate method of
anesthesia for a vaginal breech delivery? 28–31. Which of the following interventions has been
a. General anesthesia shown most consistently to increase the success rate
of external cephalic version attempts?
b. Pudendal anesthesia
a. Nifedipine
c. Epidural anesthesia
b. Terbutaline
d. Intravenous sedation
c. Nitroglycerin
28–27. A 26-year-old G2P1 presents for a routine visit at d. Epidural analgesia
32 weeks’ gestation. She is worried because her fetus
was breech during her most recent sonographic 28–32. Internal podalic version is usually reserved for which
examination. Which of the following are correct of the following clinical situations?
statements during your counseling regarding external a. Frank breech deliveries
cephalic version?
b. Complete breech deliveries
a. The success rate is 80%.
c. Delivery of an aftercoming twin
b. It can be performed when she presents in labor. d. Preterm breech deliveries, regardless of
c. It should be performed after 36 weeks’ gestation. presentation
d. Amnionic fluid volume is unrelated to the success
rate.

28–28. The patient in Question 28–27 chooses an external


cephalic version attempt at 37 weeks’ gestation.
Sonographically, the fetus has a transverse lie, the
amnionic fluid index is 18 cm, and the estimated
fetal weight is 2800 g. The placenta is anterior.
Which of the following does not aid successful
version completion?
a. Multiparity
b. Anterior placenta
c. Abundant amnionic fluid
d. Fetal size of 2500–3000 g
Breech Delivery 193

CHAPTER 28 ANSw ER KEy

Q uestion Letter Pa ge

C
number a nswer cited Hea der cited

H
A
P
28–1 b p. 558 Introduction

T
E
28–2 d p. 559 Fig. 2 8 -1

R
2
28–3 b p. 558 Introduction

8
28–4 b p. 559 Cla ssifica tion of Breech Presenta tions
28–5 c p. 559 Risk Fa ctors
28–6 a p. 559 Risk Fa ctors
28–7 d p. 560 Term Breech Fetus
28–8 c p. 561 Preterm Breech Fetus
28–9 c p. 561 Ma terna l Morbidity a nd Morta lity
28–10 b p. 561 Perina ta l Morbidity a nd Morta lity
28–11 b p. 561 Perina ta l Morbidity a nd Morta lity
28–12 b p. 561 Ima ging Techniques: Sonogra phy
28–13 d p. 562 Ma na gement of La bor a nd Delivery
28–14 b p. 563 Ca rdina l Movements with Breech Delivery
28–15 c p. 563 Pa rtia l Breech Extra ction
28–16 c p. 562 Ta ble 2 8 -1
28–17 a p. 559 Fig. 2 8 -2
28–18 c p. 563 Pa rtia l Breech Extra ction
28–19 a p. 564 N ucha l Arm
28–20 c p. 564 Modified Pra gue Ma neuver
28–21 b p. 567 Forceps to Aftercoming Hea d
28–22 c p. 567 Entra pment of the Aftercoming Hea d
28–23 c p. 567 Entra pment of the Aftercoming Hea d
28–24 d p. 568 Fra nk Breech
28–25 b p. 568 Fra nk Breech
28–26 c p. 570 Ana lgesia a nd Anesthesia
28–27 c p. 570 Indica tions
28–28 b p. 570 Indica tions
28–29 d p. 570 Complica tions
28–30 b p. 570 Indica tions
28–31 b p. 570 Tocolysis
28–32 c p. 571 Interna l Poda lic Version
194

CHAPTER 29

Operative Vaginal Deliver

29–1. Accepted maternal indications or operative 29–6. You are called to see your multigravid patient or
vaginal delivery include all EXCEPT which o the prolonged etal bradycardia. She is now 10 cm
ollowing? dilated, and membranes are ruptured. The head is
a. Mitral stenosis at + 2 station and shows poor descent with pushing.
The etal head is positioned le t occiput posterior
b. Spinal cord injury
and is resistant to an attempt o manual rotation to
c. Pelvic loor protection an occiput anterior position. Forceps delivery o this
d. Second-stage labor lasting > 2 hr in a multipara patient would be classi ied as which o the ollowing?
with epidural analgesia a. Low
b. High
29–2. Accepted etal indications or orceps delivery
include which o the ollowing? c. Outlet
a. Nonreassuring etal heart rate pattern d. Low outlet
b. Prevention o intracranial hemorrhage rom
29–7. Which o the ollowing describes orceps that are
maternal pushing in the ragile preterm etal head
applied to the etal head with the scalp visible at the
c. Prevention o intracranial hemorrhage rom introitus without manual separation o the labia?
maternal pushing in the etus with known
a. Low
coagulopathy
b. Mid
d. All o the above
c. High
29–3. Which o the ollowing is true o high orceps d. Outlet
delivery?
a. Indicated or etal distress 29–8. Maternal morbidity with orceps delivery is most
closely predicted by which o the ollowing?
b. No role in modern obstetrics
a. Fetal station
c. Forceps applied when the etal head is engaged
b. Maternal parity
d. Indicated or those with prolonged second-stage
labor c. Degree o etal distress
d. Degree o etal head molding
29–4. All EXCEPT which o the ollowing criteria must be
met prior to operative vaginal delivery? 29–9. Third-degree laceration is more common
a. Membranes ruptured with operative vaginal delivery compared with
spontaneous vaginal delivery. Practices that may
b. Cervix completely dilated
limit rates o this morbidity include which o the
c. Regional anesthesia placed ollowing?
d. Fetal head position determined a. Early orceps disarticulation
b. Selection o vacuum rather than orceps
29–5. In all EXCEPT which o the ollowing settings
would orceps delivery be pre erred to vacuum c. Selection o midline rather than mediolateral
extraction? episiotomy
a. Maternal Mar an disease d. Creation o episiotomy with each operative
vaginal delivery
b. Delivery o a 33-week gestation
c. Mentum anterior ace presentation
d. Rotation o the etal head rom occiput transverse
to occiput anterior position
O pera tive Va gina l Delivery 195

29–10. Compared with spontaneous vaginal delivery, 29–12. The perinatal complication shown here is seen more
operative vaginal delivery is con irmed to have higher requently with which o the ollowing delivery
long-term risks or which o the ollowing? routes?

C
a. Anal incontinence

H
A
b. Urinary incontinence

P
T
c. Pelvic organ prolapse

E
R
d. None o the above

2
9
29–11. Examples o subgaleal and cephalohematoma are
shown here. Compared with orceps delivery, all
EXCEPT which o the ollowing have higher
associated rates with vacuum extraction?

S ca lp Ga le a l a pone uros is
S ubga le a l
S ubga le a l s pa ce
he morrha ge
Pe rios te um

Ga le a l a pone uros is
Ce pha lohe ma toma S ubga le a l s pa ce Reproduced with permission rom Cunningham FG, Leveno KJ, Bloom SL, et al (eds):
Pe rios te um Diseases and injuries o the newborn. In Williams Obstetrics, 23rd ed. New York,
Pa rie ta l bone McGraw-Hill, 2010, Figure 29-13A.

a. Forceps delivery
b. Vacuum extraction
Modif ed with permission rom Cunningham FG, Leveno KJ, Bloom SL, et al (eds): c. Cesarean delivery
Diseases and injuries o the term newborn. In Williams Obstetrics, 24th ed. New York, d. Spontaneous vaginal delivery
McGraw-Hill, 2014, Figure 33-1.

29–13. The rate o which neurodevelopmental disorder is


a. Cephalohematoma
increased by operative vaginal delivery?
b. Shoulder dystocia
a. Epilepsy
c. Subgaleal hemorrhage
b. Cerebral palsy
d. Brachial plexus injury
c. Cognitive delay
d. None o the above

29–14. All EXCEPT which o the ollowing are known


actors associated with a ailed trial o orceps and
need or cesarean delivery?
a. Birthweight > 4000 g
b. Poor maternal pushing e orts
c. Persistent occiput posterior position
d. Absence o regional or general anesthesia
196 Delivery

29–15. Which o the ollowing indings should in luence 29–18. Your primigravid patient had a orceps-assisted
abandonment o operative vaginal delivery and delivery o the newborn shown here a ter 3 hours
election o cesarean delivery? o pushing with epidural analgesia. Prior to
a. Multiple vacuum cup “pop o s” delivery, the etal head position was noted to be le t
S
E
occiput anterior and the sagittal suture was aligned
C
b. Lack o etal head descent with traction
T
45 degrees rom the vertical axis. Which type o
I
c. Inability to articulate the English lock o orceps
O
orceps would have been ideally suited or this
N
d. All o the above delivery?
8
29–16. The opening in this orceps blade mainly serves
which o the ollowing unctions?

a. Protects the etal ears a. Piper


b. Allows blades to grip the etal head irmly b. Simpson
c. O ers a smaller metal sur ace area against the c. Kielland
etal skull d. Tucker-McLane
d. Provides diminished traction orces against the
maternal vaginal sidewall 29–19. Kielland orceps are ideally suited or which o the
ollowing obstetrical situations?
29–17. This pair o orceps is ideally suited or which o the a. Delivery o a etus with a round head
ollowing obstetrical situations? b. Delivery o a etus with a molded head
c. Delivery o a etus with mentum posterior head
position
d. Rotation o the etal head rom occiput transverse
to occiput anterior position

29–20. With delivery o a etus whose head is in an occiput


anterior position, correct blade application and
positioning is re lected by all EXCEPT which o the
ollowing?
a. The English lock can be articulated.
b. One blade is positioned over the etal brow and
a. Delivery o a etus with a round head the other overlies the occiput.
b. Delivery o a etus with a molded head c. The long axis o the blades lies along the
c. Delivery o a etus with mentum posterior occipitomental diameter o the etal head.
presentation d. The orceps blade edge on both the right and le t
d. Rotation o the etal head rom occiput transverse is one ingerbreadth away rom the adjacent etal
to occiput anterior position head lambdoidal suture.
O pera tive Va gina l Delivery 197

29–21. During orceps placement, positioning o this 29–22. During operative delivery o a etus rom a
operator’s right hand deeper into the vagina serves + 2 station and right occiput anterior position,
what role? movements o the orceps ollowing lock articulation

C
should most closely ollow which sequence to e ect

H
delivery?

A
P
a. Outward traction, clockwise rotation, downward

T
E
traction

R
b. Counterclockwise rotation, upward traction,

2
9
outward traction
c. Clockwise rotation, downward and outward
traction, upward traction
d. Downward and outward traction, counter
clockwise rotation, upward traction

Reproduced with permission rom Cunningham FG, Leveno KJ, Bloom SL, et al (eds):
Forceps and vacuum extraction. In Williams Obstetrics, 22nd ed. New York, McGraw-Hill,
2005, Figure 23-7.

a. Protects the etal ear


b. Identi ies the ischial spines
c. Provides etal scalp stimulation during the
procedure
d. Guides the orceps into position and protects the
vaginal sidewall
198 Delivery

29–23. Which o the ollowing pelvic types is generally


associated with persistent occiput posterior position?
a. Android
S
b. Gynecoid
E
C
c. Anthropoid
T
I
d. Platypelloid
O
N
8
Modif ed with permission rom Cunningham FG, Leveno KJ, Bloom SL, et al (eds): Maternal anatomy. In Williams Obstetrics, 24th ed. New York, McGraw-Hill, 2014,
Figure 2-20.
O pera tive Va gina l Delivery 199

29–24. Your primigravid patient at term has a completely 29–28. Centering the vacuum cup over the lexion point
dilated and e aced cervix, and the etal head lies at provides which o the ollowing advantages?
+ 2 station. The etal head position is right occiput a. Extends the etal head

C
posterior. The etus begins to have severe variable
b. Minimizes traction orces

H
decelerations with maternal pushing e orts. Her

A
c. Delivers the smallest diameter through the pelvic

P
labor has not been augmented, her labor epidural is

T
providing adequate analgesia, and her vital signs are outlet

E
R
normal. Despite attempts at maternal repositioning d. All o the above

2
and oxygen supplementation, these variables persist

9
and are deepening. All EXCEPT which o the 29–29. Which o the ollowing letters re lects the correct
ollowing are suitable options? lexion point used or vacuum extraction? The
a. Per orm vacuum extraction rom occiput color-coordinated larger circles re lect the actual cup
posterior position placement or their respective lexion points.
b. Manually rotate the head to occiput anterior
position
c. Per orm vacuum rotation to occiput anterior
position and then complete vacuum extraction
d. Per orm orceps rotation to occiput anterior
position with Kielland orceps and then complete
orceps delivery
A
29–25. Compared with orceps delivery rom an occiput B
anterior position, which o the ollowing is true o C
delivery rom an occiput posterior position?
a. Equal rates o episiotomy
b. Lower rates o etal Erb palsy
c. Higher rates etal acial palsy
d. Equal rates o vaginal laceration

29–26. Which o the ollowing describes the wandering


technique o Kielland orceps blade placement or
the le t occiput transverse position?
a. The anterior blade is swept up and around the
Modif ed with permission rom Cunningham FG, Leveno KJ, Bloom SL, et al (eds):
etal brow. Operative vaginal delivery. In Williams Obstetrics, 24th ed. New York, McGraw-Hill,
b. The anterior blade is swept up and around the 2014, Figure 29-16.
etal occiput.
c. The posterior blade is inserted under the a. A
symphysis and is swept down and around the b. B
etal brow. c. C
d. The posterior blade is inserted under the d. None o the above
symphysis and is swept down and around the
etal occiput. 29–30. Which o the ollowing is the pre erred total negative
pressure generated prior to initiation o traction
29–27. Compared with so t cups or vacuum extraction, during vacuum extraction?
hard cups di er in which o the ollowing regards?
a. 0.2 kg/cm2
a. Lower scalp laceration rate
b. 0.8 kg/cm2
b. Higher subgaleal hemorrhage rates
c. 1.2 kg/cm2
c. Generation o greater traction orce
d. 1.6 kg/cm2
d. All o the above
200 Delivery

29–31. With vacuum extraction, cup rotation, which 29–32. Ideally, traction during vacuum extraction should be
generates torque, is avoided during traction to help applied in which o the ollowing manners?
avert all EXCEPT which o the ollowing? a. Continuously
a. Cephalohematoma
S
b. Intermittently and with contractions
E
C
b. Cup displacement c. Intermittently and between contractions
T
I
c. Scalp laceration
O
d. Intermittently with cycles o 20 seconds o
N
d. Retinal hemorrhage traction ollowed by 1 minute o rest
8
O pera tive Va gina l Delivery 201

CHAPTER 29 ANSw ER KEy

Q uestion Letter Pa ge

C
number a nswer cited Hea der cited

H
A
P
29–1 c p. 574 Indica tions

T
E
29–2 a p. 574 Indica tions

R
2
29–3 b p. 574 Cla ssifica tion a nd Prerequisites

9
29–4 c p. 574 Cla ssifica tion a nd Prerequisites; Ta ble 2 9 -1
29–5 a p. 574 Cla ssifica tion a nd Prerequisites; Ta ble 2 9 -1
29–6 a p. 575 Ta ble 2 9 -1
29–7 d p. 575 Ta ble 2 9 -1
29–8 a p. 575 Morbidity
29–9 a p. 575 La cera tions
29–10 d p. 576 Pelvic Floor Disorders
29–11 d p. 576 Acute Perina ta l Injuries
29–12 a p. 576 Acute Perina ta l Injuries
29–13 d p. 577 Long-Term Infa nt Morbidity
29–14 b p. 577 Tria l of O pera tive Va gina l Delivery
29–15 d p. 577 Tria l of O pera tive Va gina l Delivery; Technique
29–16 b p. 578 Forceps Design
29–17 a p. 578 Forceps Design
29–18 b p. 578 Forceps Design
29–19 d p. 582 Rota tion from O cciput Tra nsverse Positions
29–20 b p. 578 Forceps Bla de Applica tion a nd Delivery
29–21 d p. 578 Forceps Bla de Applica tion a nd Delivery
29–22 c p. 578 Forceps Bla de Applica tion a nd Delivery
29–23 c p. 580 Delivery of O cciput Posterior Positions
29–24 c p. 580 Delivery of O cciput Posterior Positions
29–25 c p. 580 Delivery of O cciput Posterior Positions
29–26 a p. 582 Rota tion from O cciput Tra nsverse Positions
29–27 c p. 583 Va cuum Extra ctor Design
29–28 c p. 583 Technique
29–29 a p. 583 Technique
29–30 b p. 583 Technique
29–31 d p. 583 Technique
29–32 b p. 583 Technique
202

CHAPTER 30

Cesarean Deliver and Peripartum H sterectom

30–1. By de inition, the term cesarean delivery includes 30–5. Although the indications or cesarean delivery are
abdominal delivery o a etus in all EXCEPT which mani old, 85 percent are per ormed or our reasons.
o the ollowing situations? These principle indications include all EXCEPT
a. Delivery o a stillborn in ant which o the ollowing?
b. Delivery o a previable in ant a. Labor dystocia
c. Delivery o an abdominal pregnancy b. Placenta previa
d. Delivery in a mother who has just died c. Previous cesarean delivery
d. Abnormal etal presentation
30–2. The cesarean delivery rate has steadily increased
during the past 30 years with the exception o what 30–6. Compared with planned primary vaginal delivery,
epoch. During this epoch, the vaginal birth a ter the maternal risks associated with planned primary
cesarean (VBAC) rate was increasing commensurate cesarean include which o the ollowing?
with a decreasing cesarean delivery rate? a. Higher morbidity and mortality rates
a. 1970–1978 b. Equivalent morbidity and mortality rates
b. 1980–1988 c. Higher morbidity but decreased mortality rates
c. 1989–1996 d. Higher morbidity but equivalent mortality rates
d. 2000–2008
30–7. Compared with planned primary vaginal birth,
30–3. Reasons or increasing use o cesarean delivery potential maternal bene its o elective primary
include all EXCEPT which o the ollowing? cesarean delivery include which o the ollowing?
a. Rising average maternal age a. Decreased risk or hemorrhage
b. Greater percentage o births to multiparas b. Decreased rehospitalization rate
c. Declining use o operative vaginal delivery c. Decreased risk or thromboembolism
d. Increased use o electronic etal monitoring d. Decreased rate o hysterectomy

30–4. Elective cesarean deliveries are increasingly being


per ormed or what indication?
a. Maternal request
b. Medically indicated preterm birth
c. Concerns regarding pelvic loor injury
d. All o the above
Cesa rea n Delivery a nd Peripa rtum Hysterectomy 203

30–8. A 21-year-old primigravida at 41 weeks’ gestation 30–10. Preoperative preparation or cesarean delivery should
is undergoing labor induction or oligohydramnios. include all EXCEPT which o the ollowing?
She progresses to the second stage o labor, but the a. Administer an antacid

C
etal head does not descend below 0 station despite
b. Place an indwelling bladder catheter

H
3 hours o pushing e orts. Cesarean delivery is

A
c. Place the supine woman in le t lateral tilt

P
undertaken but extraction is di icult and requires

T
upward pressure rom the vagina. A radiograph d. Shave pubic hair i it obstructs the operative ield

E
R
o the newborn head is shown here and reveals a

3
depressed skull racture (white arrow). Approximately 30–11. Recommendations or antibiotic prophylaxis at

0
what percentage o cesarean deliveries are cesarean delivery or women with a signi icant
complicated by some type o etal injury? penicillin allergy include a single dose o which o
the ollowing agents?
a. Clindamycin
b. Levo loxacin
c. Clindamycin plus gentamicin
d. Clindamycin plus metronidazole

30–12. To reduce postoperative in ectious morbidity, the


American College o Obstetricians and Gynecologists
recommends antibiotic prophylaxis be given within
how many minutes prior to delivery?
a. 30
b. 60
c. 90
d. 120

Reproduced with permission rom Cunningham FG, Leveno KJ, Bloom SL, et al (eds): 30–13. When creating a P annenstiel incision, which vessels
Diseases and injuries o the term newborn. In Williams Obstetrics, 24th ed. New York, should be anticipated hal way between the skin and
McGraw-Hill, 2014, Figure 33-1.
ascia, several centimeters rom the midline?
a. External pudendal
a. 0.5%
b. In erior epigastric
b. 1%
c. Super icial epigastric
c. 3%
d. Super icial circum lex iliac
d. 5%
30–14. Compared with a midline incision, a P annenstiel
30–9. Although controversial, cesarean delivery on
incision o ers which o the ollowing bene its?
maternal request should only be considered as an
option when which o the ollowing criteria have a. Less postoperative pain
been met? b. Improved cosmetic result
a. The mother plans to have three subsequent c. Decreased rates o ascial wound dehiscence
pregnancies. d. All o the above
b. There is a history o cerebral palsy in a previous
child.
c. The pregnancy has reached at least 39 weeks’
gestation.
d. The patient is concerned about inadequate pain
control in labor.
204 Delivery

30–15. When per orming dissection through a P annenstiel 30–16. Transverse uterine incisions are generally pre erred
incision, the two ascial layers are incised individually to vertical incisions or all EXCEPT which o the
as is illustrated in this image. The irst layer ollowing reasons?
encountered, which is incised already in this image, a. Ease o closure
S
E
is the aponeurosis o what muscle?
C
b. Decreased rates o postpartum metritis
T
I
c. Less likely to rupture in subsequent pregnancy
O
N
d. Lower risk o incisional adhesions to bowel or
8
omentum

30–17. Failure to recognize dextrorotation o the uterus


prior to hysterotomy increases the risk o damage to
what structure?
a. Le t ureter
b. Right ureter
c. Le t uterine artery
d. Right uterine artery

30–18. Caudad separation o the bladder rom the lower


uterine segment, as shown in the igure below,
usually does not exceed 5 cm in depth. In what
clinical situation, however, may extended dissection
be recommended?
Reproduced with permission rom Word L, Hof man BL: Surgeries or benign gynecologic a. Placenta previa
conditions. In Hof man BL, Schorge JO, Schaf er JI, et al (eds): Williams Gynecology, b. Planned cesarean hysterectomy
2nd ed. New York, McGraw-Hill, 2012, Figure 41-2.1.
c. Second-stage cesarean delivery
a. Transversalis d. All o the above
b. External oblique
c. Internal oblique
d. Transversus abdominis

Ve s icoute rine
s e ros a

Bla dde r
Lowe r ute rine
s e gme nt

Reproduced with permission rom Cunningham FG, Leveno KJ, Bloom SL, et al (eds): Cesarean delivery and peri-
partum hysterectomy. In Williams Obstetrics, 24th ed. New York, McGraw-Hill, 2014, Figure 30-3.
Cesa rea n Delivery a nd Peripa rtum Hysterectomy 205

30–19. During cesarean delivery, a hysterotomy incision is 30–20. Shown in the image below are two methods o
made in the lower uterine segment, as shown here. extending the hysterotomy once the endometrial
In which o the ollowing settings is it imperative to cavity has been entered. Compared with blunt

C
incise relatively higher on the uterus to avoid uterine extension, the use o bandage scissors or sharp

H
vessel laceration or unintended entry into the vagina? extension has been associated with an increase in

A
P
which o the ollowing?

T
E
R
3
0
Myome trium

Amnionic
sac

Reproduced with permission rom Cunningham FG, Leveno KJ, Bloom SL, et al (eds):
Cesarean delivery and peripartum hysterectomy. In Williams Obstetrics, 24th ed. New
York, McGraw-Hill, 2014, Figure 30-4.

a. An anemic mother
b. A completely dilated cervix
c. A breech-presenting etus
d. Cesarean per ormed prior to labor onset
Amnionic
sac

Reproduced with permission rom Cunningham FG, Leveno KJ, Bloom SL, et al (eds):
Cesarean delivery and peripartum hysterectomy. In Williams Obstetrics, 24th ed.
New York, McGraw-Hill, 2014, Figure 30-5.

a. Estimated blood loss


b. Blood trans usion rate
c. Postoperative in ection rate
d. None o the above
206 Delivery

30–21. A 22-year-old primigravida undergoes a cesarean 30–23. Which o the ollowing is a disadvantage o uterine
delivery or breech presentation. As the hysterotomy exteriorization to repair the hysterotomy during
is being closed, her blood pressure is noted to cesarean delivery?
be 82/40. The estimated blood loss is normal. a. Increased blood loss
S
E
Concentrated administration o which o the
C
b. Increased operative injury rate
T
ollowing could explain this inding?
I
c. Increased nausea and vomiting rate
O
a. Oxytocin
N
d. Increased postoperative in ection rate
b. Methergine
8
c. Misoprostol 30–24. Which o the ollowing conditions likely contributes
d. Tranexamic acid to postoperative adhesion ormation?
a. In ection
30–22. Compared with manual extraction, spontaneous
b. Local tissue ischemia
delivery o the placenta with undal massage, as
shown in the igure, has been shown to reduce the c. Failure to achieve hemostasis
risk o which complication? d. All o the above
a. Retained placenta
b. Postpartum in ection
c. Deep-vein thrombosis
d. Amnionic luid embolism

Lowe r ute rine


s e gme nt

Ute rine incis ion

P la ce nta

Reproduced with permission rom Cunningham FG, Leveno KJ, Bloom SL, et al (eds): Cesarean delivery and peri-
partum hysterectomy. In Williams Obstetrics, 24th ed. New York, McGraw-Hill, 2014, Figure 30-9.
Cesa rea n Delivery a nd Peripa rtum Hysterectomy 207

30–25. What is a potential advantage o closure o the 30–29. When per orming the step shown in this image as
parietal peritoneum prior to closure o the ascia? a part o a peripartum hysterectomy, particular care
a. Less adhesion ormation must be taken to avoid injury to what structure?

C
b. Shorter operative times

H
A
c. Avoidance o distended bowel

P
T
d. Decreased postoperative pain

E
R
3
30–26. Subcutaneous tissue greater than what depth should

0
be closed with suture to avoid postoperative wound
disruption?
a. 2 cm
b. 4 cm
c. 6 cm
d. 10 cm

30–27. All EXCEPT which o the ollowing would be


considered potential indications or a classical
(vertical) hysterotomy?
a. Cervical cancer
b. Densely adhered bladder
c. Back-up transverse etal lie
d. Signi icant maternal obesity

30–28. A 33-year-old G2P1 with one prior cesarean


delivery presents at 35 weeks’ gestation with
active vaginal bleeding. She is taken emergently
Reproduced with permission rom Cunningham FG, Leveno KJ, Bloom SL, et al (eds):
or repeat cesarean delivery and is ound to have a Cesarean delivery and peripartum hysterectomy. In Williams Obstetrics, 24th ed. New
placenta previa with accreta that requires cesarean York, McGraw-Hill, 2014, Figure 30-18A.
hysterectomy. Compared with patients who have
this procedure per ormed electively, this woman a. Ureter
is at increased risk or which o the ollowing b. Bladder
complications?
c. Urethra
a. Bowel injury
d. Sigmoid colon
b. Urinary tract injury
c. Venous thromboembolism
d. All o the above
208 Delivery

30–30. When cystotomy complicates cesarean delivery, the 30–31. Women who have normal blood volume expansion
bladder should be closed with a two- or three-layer and a hematocrit o at least 30 volume percent
running closure. Which layer is being closed in the will usually tolerate blood loss up to what volume
image shown? without hemodynamic compromise?
S
E
a. 2000 mL
C
T
b. 3000 mL
I
O
N
c. 4000 mL
8
d. 5000 mL

30–32. Although luid sequestration in the “third space” is


not typically seen in women who undergo cesarean
delivery, this extracellular luid sequestration can be
Ute rine problematic in women who have what pathological
incis ion process?
Bla dde r a. Sepsis
b. Preeclampsia
Cys totomy c. Excessive hemorrhage
d. All o the above

Reproduced with permission rom Cunningham FG, Leveno KJ, Bloom SL, et al (eds):
Cesarean delivery and peripartum hysterectomy. In Williams Obstetrics, 24th ed. New
York, McGraw-Hill, 2014, Figure 30-23A.

a. Mucosa
b. Serosa
c. Muscularis
d. Visceral peritoneum
Cesa rea n Delivery a nd Peripa rtum Hysterectomy 209

CHAPTER 30 ANSw ER KEy

Q uestion Letter Pa ge

C
number a nswer cited Hea der cited

H
A
P
30–1 c p. 587 Introduction

T
E
30–2 c p. 587 Cesa rea n Delivery in the United Sta tes

R
3
30–3 b p. 587 Cesa rea n Delivery in the United Sta tes

0
30–4 d p. 587 Cesa rea n Delivery in the United Sta tes
30–5 b p. 587 Cesa rea n Delivery Indica tions a nd Risks
30–6 a p. 588 Ma terna l Morta lity a nd Morbidity
30–7 a p. 588 Ma terna l Morta lity a nd Morbidity
30–8 b p. 589 N eona ta l Morbidity
30–9 c p. 589 Pa tient Choice in Cesa rea n Delivery
30–10 d p. 590 Periopera tive Ca re
30–11 c p. 590 Infection Prevention
30–12 b p. 590 Infection Prevention
30–13 c p. 591 Abdomina l Incision
30–14 d p. 591 Abdomina l Incision
30–15 b p. 591 Tra nsverse Incisions
30–16 b p. 592 Hysterotomy
30–17 c p. 592 Low Tra nsverse Cesa rea n Section
30–18 b p. 592 Low Tra nsverse Cesa rea n Section
30–19 b p. 592 Low Tra nsverse Cesa rea n Section
30–20 a p. 593 Uterine Incision
30–21 a p. 594 Delivery of the Fetus
30–22 b p. 596 Pla centa l Delivery
30–23 c p. 596 Uterine Repa ir
30–24 d p. 597 Adhesions
30–25 c p. 597 Abdomina l Closure
30–26 a p. 597 Abdomina l Closure
30–27 c p. 598 Cla ssica l Cesa rea n Incision—Indica tions
30–28 b p. 599 Peripa rtum Hysterectomy—Indica tions
30–29 a p. 600 Peripa rtum Hysterectomy—Technique
30–30 a p. 604 Cystotomy
30–31 a p. 604 Peripa rtum Ma na gement—Intra venous Fluids
30–32 d p. 604 Ana lgesia , Vita l Signs, Intra venous Fluids
210

CHAPTER 31

Prior Cesarean Deliver

31–1. What is the least common neonatal morbidity seen 31–5. The patient in Question 31–4 had this type of
with elective repeat cesarean delivery? uterine incision at her first delivery. Which of the
a. Sepsis following statements is most accurate in planning
the timing of her second pregnancy?
b. Respiratory distress syndrome
c. Transient tachypnea of the newborn
d. None of the above

31–2. What is most the most common maternal


complication seen with repeat cesarean delivery?
a. Transfusion
b. Hysterectomy
c. Wound infection
d. Deep-vein thrombosis

31–3. Which of the following contributes to a rising rate of


primary cesarean delivery?
a. Maternal request
b. Reduced use of oxytocin
c. Increased rate of breech presentation
a. The risk of uterine rupture is unrelated to the
d. All of the above interdelivery interval.
31–4. A patient underwent a primary cesarean in her first b. Lowest uterine rupture rates are seen in women
pregnancy for a frank breech presentation at term. In who wait at least 24 months to become pregnant.
general, what is the rate of successful vaginal delivery c. Interdelivery periods < 18 months are associated
in a subsequent pregnancy in patients with this with a tripling of the uterine rupture risk.
original indication? d. None of the above
a. 30%
b. 50% 31–6. The patient in Question 31–5 returns for an
initial prenatal care visit 12 months after her last
c. 75% delivery and is interested in vaginal birth with this
d. 90% pregnancy. When is the most appropriate time to
begin discussion on this topic?
a. At the first prenatal visit
b. When the hospital consent is signed
c. When she is admitted to Labor and Delivery
d. None of the above
Prior Cesa rea n Delivery 211

31–7. For the patient in Question 31–5, what is her 31–10. What was the rate of cesarean delivery in the United
absolute risk for uterine rupture if she chooses a trial States in 2011?
of labor with a future pregnancy? a. 20%

C
a. 1/1000 b. 26%

H
A
b. 7/1000 c. 33%

P
T
c. 1/100 d. 38%

E
R
d. 5/100

3
31–11. How does the risk of hysterectomy, uterine rupture,

1
31–8. The patient in Question 31–5 also wants more or operative injury change if a patient’s attempt at
information about neonatal risks. Which of the vaginal delivery is unsuccessful and she undergoes a
following statements regarding neonatal outcomes is repeat cesarean delivery?
most accurate? a. Twofold higher
a. The perinatal death rate associated with a trial of b. Fivefold higher
labor is 1.3%.
c. Tenfold higher
b. The perinatal mortality rate is 11 times greater if
she chooses a trial of labor. d. No risk difference
c. Trial of labor is associated with a reduced risk for 31–12. Which of the following neonatal complications is
hypoxic ischemic encephalopathy. more frequently seen in newborns following elective
d. There is no difference in perinatal mortality rates repeat cesarean delivery?
if she chooses planned repeat cesarean section or a. 5-minute Apgar score < 7
trial of labor.
b. Hypoxic ischemic encephalopathy
31–9. Sonographic examination of this pregnancy is done c. Transient tachypnea of the newborn
in the office with the following finding. Now, how d. Neonatal intensive care unit admission
should the patient be counseled regarding her risk
of complications associated with vaginal birth after 31–13. The rate of which of the following is significantly
cesarean (VBAC)? increased in newborns of women attempting trial
of labor compared with those delivered by repeat
cesarean?
a. Term neonatal death
b. Intrapartum stillbirth
c. Term hypoxic ischemic encephalopathy
d. All of the above

31–14. The American College of Obstetricians and


Gynecologists considers which of the following
factors the most important in selecting a suitable
candidate for trial of labor?
a. Prior uterine incision type
b. Infection at the time of the original surgery
c. Gestational age at the time of the original surgery
Reproduced with permission from Cunningham FG, Leveno KJ, Bloom SL, et al (eds): d. Degree of uterine distention during the current
Multifetal gestation. In Williams Obstetrics, 23rd ed. New York, McGraw-Hill, 2010,
Figure 39-7B. pregnancy

a. The risk for uterine rupture is not increased with


this finding.
b. A repeat cesarean section should be planned
because of low rates of successful VBAC.
c. Permitting an attempt at vaginal birth will
depend on the gestational age at delivery.
d. None of the above
212 Delivery

31–15. How does the type of uterine closure seen in 31–17. Two years previously, your patient had the type of
this image affect the risk of complications in a cesarean delivery depicted in the image below for
subsequent pregnancy? a preterm twin gestation. She is currently pregnant
with a singleton gestation. What do the authors feel
S
E
is the optimal management plan for delivery in this
C
T
scenario?
I
O
N
8
Used with permission from Dr. Donald Anderson.

a. The risk of uterine rupture is reduced.


b. The risk of uterine rupture is increased. a. Schedule a repeat cesarean delivery at 39 weeks’
c. The risk of uterine dehiscence is reduced. gestation
d. Data are unclear regarding the role that closure b. Perform a repeat cesarean delivery when she
type plays in subsequent uterine rupture. presents in early labor
c. Schedule a cesarean delivery as soon as pulmonary
31–16. A nonpregnant patient presents with vaginal maturity is documented in the fetus
bleeding 7 months after having a primary cesarean d. Allow a trial of labor only if there is immediate
delivery for arrest of descent during the second stage availability of obstetrical and anesthesia staff in
of labor. Transvaginal sonography reveals the finding the hospital where she plans to deliver
below in this sagittal image of the uterus. Which of
the following statements is accurate? 31–18. A 28-year-old G3P2 underwent a primary low
transverse cesarean delivery for a prolapsed cord
at term in her last pregnancy. She is currently at
38 weeks’ gestation, and her obstetrician estimates
the fetal size to be 6½ lb. Her cervix is 2-cm dilated.
What is the most favorable prognostic indicator that
this patient will have a vaginal delivery?
a. Cervical examination
b. Obstetrical history
c. Estimated fetal weight
d. Indication for her prior cesarean delivery

a. Future pregnancies should be discouraged.


b. She should undergo an exploratory laparotomy.
c. Her risk for uterine rupture in a future pregnancy
is increased.
d. All of the above
Prior Cesa rea n Delivery 213

31–19. What does the graph below suggest about the effect 31–23. During routine exploration of the lower uterine
of maternal obesity on a successful vaginal birth after segment following a vaginal delivery in a patient
cesarean (VBAC) attempt? with a previous cesarean delivery, the obstetrician

C
discovers a defect in the myometrium. The patient’s

H
vital signs are stable, and she has a normal amount

A
P
of vaginal bleeding. What is the most appropriate

T
next step in the management of this patient?

E
R
a. Immediate exploratory laparotomy

3
1
b. Diagnostic laparoscopy 6 weeks postpartum
c. Abdominal computed tomography (CT) scan to
assess the defect size
d. None of the above

31–24. What is the greatest risk factor for uterine dehiscence


or incomplete uterine rupture?
a. Grand multiparity
b. Multifetal gestation
c. Prior cesarean delivery
d. Use of uterotonic agents
a. Obesity has no effect on VBAC success rates.
b. The overall success rate for all women is 85%. 31–25. During uterine rupture, which of the following
c. The success rate in women with a BMI of 35 is improves the chances of a neonate surviving and
70%. retaining normal neurological function?
d. The overall success rate for obese parturients is a. Cephalic presentation
60%. b. Intact fetal membranes prior to rupture
31–20. Use of which of the following methods of labor c. Location of placenta away from the rupture site
induction is contraindicated in a patient with a prior d. Reassuring fetal heart rate tracing prior to rupture
cesarean delivery?
a. Oxytocin 31–26. According to investigators from Utah, delivery
within how many minutes following uterine rupture
b. Amniotomy is associated with normal neonatal neurological
c. Misoprostol outcomes?
d. None of the above a. 18 minutes
b. 30 minutes
31–21. What is the most common sign of uterine rupture?
c. 40 minutes
a. Abdominal pain
d. None of the above
b. Vaginal bleeding
c. Loss of fetal station 31–27. When uterine rupture occurs, what is the risk for
d. Fetal heart rate decelerations neonatal death?
a. 0.5%
31–22. Which of the following is a valid reason to avoid b. 1%
epidural analgesia during labor in patients with prior
cesarean delivery? c. 5%
a. It may mask the signs of uterine scar dehiscence. d. 25%
b. Higher doses of medication are required in
women undergoing trial of labor.
c. It is associated with decreased success rates for
vaginal birth after cesarean.
d. None of the above
214 Delivery

31–28. While undergoing labor induction at 41 weeks’ 31–30. Which of the following statements regarding the
gestation, a primigravida has spontaneous uterine figure below is accurate?
rupture that is repaired. How should she be
counseled about the recurrence risk in future 8
S
Wound/ute rine infe ction
E
pregnancies?
C
P la ce nta previa
T
a. 2–3% 7
I
Tra ns fus ion
O
b. 25–30% Hys te re ctomy
N
8
c. 50% 6 P la ce nta a ccre ta

)
d. No increase in risk

t
n
e
c
r
5

e
p
31–29. Which of the following obstetrical complications

(
n
o
is NOT increased with the diagnosis shown in the

i
t
a
4
following images?

c
i
l
p
m
o
c
3

r
o
f
k
s
i
R
2

0
Firs t S e cond Third Fourth ≥ Fifth
(6201) (15,808) (6324) (1452) (347)
Numbe r of re pe a t ce s a re a n de live ry
(Numbe r of wome n)

Reproduced with permission from Cunningham FG, Leveno KJ, Bloom SL, et al (eds):
Prior cesarean delivery. In Williams Obstetrics, 24th ed. New York, McGraw-Hill, 2014,
A
Figure 31-5.

a. The risk for wound infection is halved with the


second repeat cesarean delivery.
b. Placenta previa complicates approximately 6% of
first repeat cesarean deliveries.
c. The risk of placenta accreta continues to increase
as the number of repeat cesarean deliveries
increases.
d. All of the above

a. Hemorrhage
b. Hysterectomy
c. Maternal mortality
d. None of the above
Prior Cesa rea n Delivery 215

31–31. In this figure, the trend from 1989 to 1995 may be 31–32. Which of the following statements regarding
explained by which of the following? maternal deaths from uterine rupture is most
a. Increased demand for cesarean delivery by accurate?

C
patients a. Worldwide mortality rate from uterine rupture is

H
A
b. Increased numbers of pregnancies secondary to < 1%.

P
fertility treatments b. Individual maternal mortality risk can be

T
E
c. Increased awareness of risks of vaginal birth after predicted based on comorbidities.

R
3
cesarean delivery c. Rates for maternal mortality vary widely

1
d. A recommendation from American College depending on availability of medical services.
of Obstetricians and Gynecologists that most d. None of the above
women with a previous low transverse cesarean
section should attempt vaginal birth in a
subsequent pregnancy

35
30 VBAC a
s
h
t
r
i
25
b
Tota l ce s a re a n de live ryb
e
v
20
i
l
0
0
15
1
P rima ry ce s a re a n de live ryc
r
e
10
p
e
t
5
a
R
0
1989 1991 1993 1995 1997 1999 2001 2003 2004
Ye a r

Reproduced with permission from Cunningham FG, Leveno KJ, Bloom SL, et al (eds): Prior cesarean
delivery. In Williams Obstetrics, 23rd ed. New York, McGraw-Hill, 2010, Figure 26-1.
216 Delivery

CHAPTER 31 ANSw ER KEy

Q uestion Letter Pa ge
S
number a nswer cited Hea der cited
E
C
T
31–1 a p. 615 Figure 3 1 -3
I
O
31–2 c p. 618 Figure 3 1 -5
N
31–3 a p. 609 1 0 0 Yea rs of Controversy
8
31–4 d p. 614 Indica tion for Prior Cesa rea n Delivery
31–5 c p. 614 Interdelivery Interva ls
31–6 a p. 610 Fa ctors Tha t Influence a Tria l of La bor
31–7 b p. 610 Ma terna l Risks
31–8 b p. 610 Ma terna l Risks
31–9 a p. 614 Multifeta l G esta tions
31–10 c p. 609 1 0 0 Yea rs of Controversy
31–11 b p. 610 Ma terna l Risks
31–12 c p. 612 Feta l a nd N eona ta l Risks
31–13 c p. 611 Ta ble 3 1 -2
31–14 a p. 612 Prior Incision Type
31–15 d p. 612 Prior Incision Type
31–16 c p. 614 Ima ging of Prior Incision
31–17 c p. 614 Prior Uterine Rupture
31–18 b p. 614 Prior Va gina l Delivery
31–19 c p. 615 Ma terna l O besity
31–20 c p. 615 Cervica l Ripening a nd La bor Stimula tion
31–21 d p. 616 Epidura l Ana lgesia
31–22 d p. 616 Epidura l Ana lgesia
31–23 d p. 616 Uterine Sca r Explora tion
31–24 c p. 617 Cla ssifica tion
31–25 c p. 617 Decision-to-Delivery Time
31–26 a p. 617 Decision-to-Delivery Time
31–27 c p. 617 Decision-to-Delivery Time
31–28 b p. 618 Hysterectomy versus Repa ir
31–29 d p. 618 Complica tions with Multiple Repea t Cesa rea n Deliveries
31–30 d p. 618 Figure 3 1 -5
31–31 d p. 609 1 0 0 Yea rs of Controversy
31–32 c p. 617 Decision-to-Delivery Time
Se c t i o n 9

Th e New bo r N
218

Ch APTer 32

T N n

32–1. Rem val f fluid fr m the newb rn respirat ry tra t 32–6. What differentiates se ndary apnea fr m primary
urs thr ugh whi h f the f ll wing me hanisms? apnea in the newb rn?
a. Physi al mpressi n f the th rax a. It is res lved with simple stimulati n.
. Abs rpti n f fluid int the pulm nary . It is a mpanied by a fall in heart rate.
ir ulati n c. Clini al signs are markedly different f r the tw
c. Abs rpti n f fluid int the pulm nary lymphati nditi ns.
system d. Respirat ry eff rts will n t sp ntane usly resume
d. All f the ab ve with ut interventi n.

32–2. Whi h f the f ll wing events aids in l sure f the 32–7. A multipar us patient with rapid lab r has m derate
du tus arteri sus in the newb rn? staining f her amni ni fluid with me nium.
a. Fall in pulm nary arterial pressure She re eived a single 50-mg d se f meperidine
intraven usly f r pain relief. F ll wing variable
. h ra i mpressi n during delivery
de elerati ns t 90 beats per minute (bpm) with
c. P stnatal a umulati n f arb n di xide pushing, she delivers the ne nate. What is the first
d. In reased mpressi n f pulm nary vas ulature step in newb rn resus itati n?
a. Administer nal x ne (Nar an)
32–3. Delayed rem val r abs rpti n f amni ni fluid
. Bulb su ti n the r pharynx
fr m the pulm nary system results in whi h f the
f ll wing nditi ns? c. Warm, dry, and stimulate the newb rn
a. Respirat ry distress syndr me d. Pr phyla ti ally intubate and su ti n
. Persistent pulm nary hypertensi n
32–8. he newb rn in Questi n 32–7 has a heart rate
c. ransient ta hypnea f the newb rn f 80 bpm and gasping respirati ns. Whi h f the
d. Premature l sure f the du tus arteri sus f ll wing sh uld be the next step f resus itati n?
a. Intubati n and r m air administrati n
32–4. What type f ell pr du es surfa tant in the fetal
. P sitive pressure ventilati n with r m air
lung?
c. P sitive pressure ventilati n with 50% xygen
a. ype I pneum ytes
d. Nasal ntinu us airway pressure with 28% xygen
. ype II pneum ytes
c. Sm th mus le ells 32–9. Whi h f the f ll wing s enari s is an indi ati n f r
d. Squam us epithelial ells end tra heal intubati n?
a. Gestati nal age < 28 weeks
32–5. A 25-year- ld primigravida explains t her
. Heart rate < 100 bpm f r > 60 se nds
bstetri ian that she is nsidering a h me delivery
with a l al midwife. Whi h f the f ll wing is an c. Heart rate < 100 bpm f r > 30 se nds
a urate statement regarding h me births? d. Meperidine administrati n 1 h ur pri r t
a. Ne natal death rates are d ubled f r infants b rn delivery
at h me.
32–10. Whi h f the f ll wing dem nstrates rre t
. One in 10 ne nates b rn at h me needs extensive pla ement f the end tra heal tube in the ne nate?
resus itati n.
a. Symmetri hest wall m ti n
c. When patients are pr perly sele ted, ne natal
mpli ati n rates are n t in reased. . Equal breath s unds bilaterally
d. N ne f the ab ve c. Absen e f gurgling s unds in epigastrium
d. All f the ab ve
The N ewborn 219

32–11. Where is the rre t l ati n f fingers during hest 32–15. Whi h f the f ll wing is NOT an appr priate use
mpressi ns in the ne nate? f the 1- and 5-minute Apgar s res?
a. Pr gn sti indi at r f ne natal survival

C
. Pr gn sti indi at r f l ng-term neur l gi

h
A
ut me

P
T
c. Measure f effe tiveness f ne natal resus itati n

e
r
eff rt

3
d. All f the ab ve are appr priate uses

2
32–16. Whi h f the f ll wing nditi ns may influen e the
Apgar s re?
a. Immaturity
. Maternal infe ti n
c. Medi ati n administrati n
d. All f the ab ve

32–17. A 25-year- ld primigravida with gestati nal


hypertensi n has new- nset vaginal bleeding that is
Used with permissi n fr m Stephanie Raynish. a mpanied by fetal de elerati ns. She underg es
an emergen y esarean delivery, and the pla ental
a. A findings bel w are n ted. A rding t the Ameri an
C llege f Obstetri ians and Gyne l gists and
. B
the Ameri an A ademy f Pediatri s, whi h f the
c. C f ll wing w uld supp rt the diagn sis f hyp xia-
d. N ne f the ab ve indu ed metab li a idemia?

32–12. What is the rre t rati f hest mpressi ns t


breaths per minute in the term ne nate?
a. 30 mpressi ns: 90 breaths
. 45 mpressi ns: 15 breaths
c. 90 mpressi ns: 30 breaths
d. 60 mpressi ns: 30 breaths

32–13. Despite ventilat ry supp rt and hest mpressi ns,


the ne natal heart rate is 45 bpm. What is the
appr priate management?
a. In rease respirat ry rate
. Administer intramus ular nal x ne
c. Administer intraven us fluid b lus
d. Administer end tra heal epinephrine

32–14. Whi h f the f ll wing is NOT an element f the


Apgar s re?
Repr du ed with permissi n fr m Cunningham FG, Leven KJ, Bl m SL, et al (eds):
a. Cry Obstetri al hem rrhage. In Williams Obstetri s, 23rd ed. New Y rk, M Graw-Hill, 2010,
Figure 35-5.
.C l r
c. Mus le t ne a. 5-minute Apgar s re f 2
d. Respirat ry eff rt . Umbili al artery pH f 7.05
c. ransient ta hypnea f the newb rn
d. Ne natal seizure in the first 24 h urs f life
220 The N ewborn

32–18. Whi h f the f ll wing a ti ns is riti al in 32–22. Whi h f the f ll wing regimens is NOT
btaining an appr priate umbili al bl d sample f r re mmended pr phylaxis f r the nditi n sh wn
a id-base analysis? here?
a. he spe imen is lle ted in a heparinized
S
e
syringe.
C
T
. he spe imen must be immediately pla ed n i e
I
o
until testing is perf rmed.
N
c. he spe imen is best btained fr m sampling f
9
the arteries n the fetal surfa e f the pla enta.
d. he rd segment used f r the sample is is lated
immediately f ll wing delivery f the pla enta.

32–19. H w d es the n rmal result f an umbili al ven us


sample f rd bl d differ fr m that f an umbili al
arterial sample?
a. he pH is higher.
Repr du ed with permissi n fr m Levsky ME, DeFl ri P: Ophthalm l gi nditi ns.
. he Pco 2 is higher. In Kn p KJ, Sta k LB, St rr w AB, et al (eds): T e Atlas f Emergen y Medi ine, 3rd
c. he base ex ess is higher. ed. New Y rk, M Graw-Hill, 2010, Figure 2-1.

d. N ne f the ab ve.
a. 1% tetra y line phthalmi intment
32–20. Metab li a idemia at birth is m st l sely linked . 1% silver nitrate phthalmi s luti n
t l ng-term neur l gi al ut me in whi h f the c. 0.5% erythr my in phthalmi intment
f ll wing ir umstan e?
d. 300-mg eftriax ne intramus ular inje ti n
a. When the birthweight is < 1000 g
. When infant has a 5-minute Apgar < 7 32–23. Ne natal njun tivitis an f ll w vaginal delivery
c. When the gestati nal age at delivery is in a m ther with an a tive hlamydial infe ti n.
< 36 weeks Whi h f the f ll wing is a true statement regarding
hlamydial njun tivitis?
d. When the m ther is a kn wn gr up B Streptococcus
arrier a. 1% f newb rns b rn t w men with a tive hla-
mydial infe ti ns will devel p njun tivitis.
32–21. A rding t the Ameri an C llege f Obstetri ians . Pr phylaxis f r g n rrheal njun tivitis ade-
and Gyne l gists, whi h f f ll wing is NOT an quately nveys pr te ti n against hlamydial
indi ati n f r rd bl d gas determinati n? infe ti n.
a. win gestati n c. Pr phyla ti phthalmi treatment f r hlamydial
. Maternal thyr id disease njun tivitis d es n t reliably redu e the in i-
den e f the disease.
c. Vaginal bleeding bef re delivery
d. reatment f newb rn hlamydial infe ti n
d. Abn rmal fetal heart rate tra ing during lab r nsists f a single d se f eftriax ne 100 mg/kg
given intraven usly r intramus ularly.

32–24. During her prenatal urse, a patient is dis vered t


be p sitive f r b th the hepatitis B surfa e antigen
and the e antigen. Whi h f the f ll wing is the
appr priate evaluati n and management f r her
newb rn?
a. Administer hepatitis B immune gl bulin immedi-
ately after delivery
. est f r ne natal hepatitis B antigen status t
determine appr priate therapy
c. Administer hepatitis B immune gl bulin and
immunize against hepatitis B during h spital stay
d. N ne f the ab ve
The N ewborn 221

32–25. What is the reas n that vitamin K is administered t 32–28. A patient wh is ex lusively breast feeding her infant
newb rns within 1 h ur f birth? is n erned be ause her hild has l st 6 z in the
a. enhan e newb rn b ne devel pment first 3 days f life. What advi e sh uld y u give this

C
patient?
. prevent vitamin K–dependent hem rrhagi

h
a. Swit h feedings entirely t a high- al ri infant

A
disease f the newb rn

P
f rmula

T
c. redu e the in iden e f ne r tizing enter li-

e
tis in premature infants . Expe t the infant t regain its weight ver the

r
next week

3
d. augment the l wer vitamin K levels f und in

2
breast milk mpared with th se in mmer ial c. Supplement the breast feedings with f rmula
infant f rmulas until the infant’s weight stabilizes
d. N ne f the ab ve
32–26. Whi h f the f ll wing is the m st a urate
statement regarding newb rn s reening? 32–29. Me nium st ling in the delivery r m suggests
a. he United States federal g vernment mandates whi h f the f ll wing?
newb rn s reening. a. An imperf rate anus
. S me newb rn s reening tests d n t rely n . Intrapartum fetal distress
hemat l gi testing.
c. Gastr intestinal tra t paten y
c. All newb rn s reening tests are perf rmed within
d. C n urrent intrauterine infe ti n
the first 24 h urs after delivery.
d. N ne f the ab ve 32–30. A redu ed risk f whi h f the f ll wing is NOT a
benefit f ne natal male ir um isi n?
32–27. A patient wh se lab r was mpli ated by
a. Phim sis
h ri amni nitis and gestati nal hypertensi n
returns t y ur ffi e n p stpartum day 5 t have . Penile an er
her bl d pressure he ked. She brings her newb rn c. End metrial an er in female partner
and is n erned ab ut the appearan e f the d. Human immun defi ien y virus infe ti n
umbili al rd stump, whi h is sh wn bel w. What
is the appr priate resp nse t her n ern? 32–31. Whi h f the f ll wing is NOT a re mmended
meth d f anesthesia f r ne natal ir um isi n?
a. L al infiltrati n with lid aine
. pi al lid aine-pril aine ream
c. D rsal penile bl k with lid aine
d. Ring bl k with lid aine and epinephrine

32–32. Whi h f the f ll wing are ass iated with redu ed


maternal p stpartum lengths f stay?
a. De reased ne natal m rtality rates
. De reased ne natal infe ti n rates
c. In reased ne natal readmissi n rates
d. In reased rates f ex lusive breast feeding

Used with permissi n fr m Kelly Buru a.

a. his is a n rmal finding, and n treatment is


required.
. here is n ern f r rd ne r sis and a need f r
l al rese ti n.
c. his likely represents an infe ti n f the rd,
and antibi ti s are indi ated.
d. he patient sh uld hasten the pr ess by vering
the remaining umbili al rd with petr leum jelly
and gauze.
222 The N ewborn

Ch APTer 32 ANSw er Key

Q uestion Letter Pa ge
S
number a nswer cited Hea der cited
e
C
T
32–1 d p. 624 Initia tion of Air Brea thing
I
o
32–2 a p. 624 Initia tion of Air Brea thing
N
32–3 c p. 624 Initia tion of Air Brea thing
9
32–4 p. 624 Initia tion of Air Brea thing
32–5 a p. 625 N ewborn Resuscita tion
32–6 d p. 625 N ewborn Resuscita tion
32–7 c p. 626 Figure 3 2 -2
32–8 p. 625 Assessment a t 3 0 Seconds of Life
32–9 p. 625 Assessment a t 6 0 Seconds of Life
32–10 d p. 626 Tra chea l Intuba tion
32–11 c p. 626 Chest Compressions
32–12 c p. 626 Chest Compressions
32–13 d p. 626 Epinephrine a nd Volume Expa nsion
32–14 a p. 627 Ta ble 3 2 -1
32–15 p. 627 Apga r Score
32–16 d p. 627 Apga r Score
32–17 d p. 630 Meta bolic Acidemia
32–18 a p. 628 Umbilica l Cord Blood Acid-Ba se Studies
32–19 a p. 630 Ta ble 3 2 -2
32–20 a p. 630 Meta bolic Acidemia
32–21 c p. 630 Recommenda tions for Cord Blood G a s Determina tions
32–22 d p. 631 G onococca l Infection
32–23 c p. 631 Chla mydia l Infection
32–24 c p. 631 Hepa titis B Immuniza tion
32–25 p. 631 Vita min K
32–26 p. 631 N ewborn Screening
32–27 a p. 632 Ca re of Skin a nd Umbilica l Cord
32–28 p. 633 Feeding a nd W eight Loss
32–29 c p. 633 Stools a nd Urine
32–30 c p. 633 N ewborn Ma le Circumcision
32–31 d p. 633 Anesthesia for Circumcision
32–32 c p. 634 Hospita l Discha rge
223

CHAPTER 33

Diseases and Injuries of the Term Ne born

33–1. Of the following, which is the most common cause 33–6. Which of the following is NOT strongly predictive
of respiratory distress syndrome in a term infant? of cerebral palsy?
a. Severe asphyxia a. Perinatal infection
b. Meconium aspiration b. Birthweight < 2000 g
c. Elective cesarean delivery c. Obstetrical complication
d. Perinatal infection with sepsis syndrome d. Birth earlier than 32 weeks’ gestation

33–2. Which of the following statements regarding 33–7. Which of the following statements regarding
intrapartum amnioinfusion is true? intrapartum fetal heart rate monitoring is true?
a. It improves the perinatal death rate. a. There are no specific fetal heart rate patterns
b. It decreases cesarean delivery rates. predictive of cerebral palsy.
c. It prevents meconium aspiration syndrome. b. Evidence does not support its ability to predict or
reduce cerebral palsy risk.
d. It has been used successfully in women with
variable fetal heart rate decelerations. c. An abnormal fetal heart rate pattern in fetuses
that ultimately develop cerebral palsy may reflect
33–3. Hyperalertness, irritability, and hyper/hypotonia a preexisting neurological abnormality.
define which level of hypoxic-ischemic d. All are true.
encephalopathy?
a. Mild 33–8. Which of the following statements regarding Apgar
scores and cerebral palsy is true?
b. Severe
a. Survivors with Apgar scores of 0 at 10 minutes
c. Moderate
have surprisingly good outcomes.
d. None of the above
b. 1- and 5-minute Apgar scores are generally good
predictors of long-term neurological impairment.
33–4. Which form(s) of cerebral palsy result(s) from acute
peripartum ischemia? c. With a 20-minute Apgar score of ≤ 2, there
is a 60-percent mortality rate and a 57-percent
a. Ataxia
cerebral palsy rate.
b. Spastic diplegia
d. All are true.
c. Spastic quadriplegia
d. All of the above 33–9. The threshold for clinically significant cord blood
acidemia is which of the following?
33–5. Which of the following is NOT a risk factor for a. < 6.9
neonatal acidosis?
b. < 7.0
a. Thick meconium
c. < 7.1
b. General anesthesia
d. < 7.2
c. Prior cesarean delivery
d. Maternal age < 35 years
224 The N ewborn

33–10. Encephalopathy develops in 40 percent of newborns 33–15. After 35 weeks’ gestation, what is the expected mean
with a base deficit of which of the following? cord hemoglobin concentration?
a. 8 to 10 mmol/L a. Less than 10 g/dL
S
b. 10 to 12 mmol/L b. Less than 14 g/dL
E
C
c. 12 to 16 mmol/L c. Approximately 17 g/dL
T
I
O
d. > 16 mmol/L d. Approximately 27 g/dL
N
9
33–11. Regarding neuroimaging in the neonatal period, 33–16. Which of the following is NOT a cause of neonatal
which of the following statements is true? polycythemia?
a. Imaging precisely determines the timing of a a. Chronic hypoxia
brain injury. b. Fetal liver trauma
b. Sonographic studies are generally normal on the c. Fetal-growth restriction
day of birth.
d. Twin-twin transfusion syndrome
c. Magnetic resonance imaging is most helpful at
2 weeks of life. 33–17. Which of the following statements regarding serum
d. Computed-tomography scans will detect bilirubin levels in newborns is true?
abnormalities on the first day of life in term a. Even in the mature newborn, serum bilirubin
infants. levels usually increase for 3 to 4 days.

33–12. Which of the following is LEAST likely to cause b. In preterm infants, the bilirubin level does not
increase to the same extent as in term infants.
isolated intellectual disability?
a. Gene mutation c. In 25 percent of term newborns, bilirubin levels
cause clinically visible skin color changes.
b. Perinatal hypoxia
d. Ten to 20 percent of neonates delivered at
c. Chromosomal abnormality ≥ 35 weeks’ gestation have a maximum serum
d. Congenital malformation bilirubin level > 20 mg/dL.

33–13. According to the Centers for Disease Control and 33–18. Which of the following is NOT a characteristic of
Prevention (2012), what is the frequency of autism acute bilirubin encephalopathy?
in the United States? a. Lethargy
a. 0.1% b. Hypotonia
b. 0.5% c. Spasticity
c. 1% d. Poor feeding
d. 5%
33–19. The mainstay of prevention and treatment of
33–14. According to the Centers for Disease Control neonatal hyperbilirubinemia is which of the
and Prevention (2012), what is the frequency of following?
attention deficit hyperactivity disorders in the a. Ursodiol
United States?
b. Hydration
a. 1%
c. Phototherapy
b. 5%
d. Exchange transfusions
c. 7%
d. 10%
Disea ses a nd Injuries of the Term N ewborn 225

33–20. Which of the following is NOT a vitamin K– 33–25. Which of the following statements regarding the
dependent clotting factor? condition depicted in the figure below is true?
a. Factor III

C
b. Factor V

H
S ca lp

A
c. Factor VII Pe rios te um

P
T
d. Factor IX

E
R
S kull bone

3
33–21. What is recommended by the American Academy of

3
Pediatrics and the American College of Obstetricians
and Gynecologists for routine prophylaxis against
hemorrhagic disease of the newborn?
a. Vitamin K1 1 mg intramuscularly
b. Phytonadione 5 mg intravenously
c. Phytonadione 25 mg by mouth once daily for
two doses
d. 2 mg elemental iron/kg/day by mouth divided Reproduced with permission from Cunningham FG, Leveno KJ, Bloom SL, et al (eds):
into 1 to 3 doses Diseases and injuries of the fetus and newborn. In Williams Obstetrics, 23rd ed. New
York, McGraw-Hill, 2010, Figure 29-12.
33–22. Which of the following statements regarding
hemorrhagic disease of the newborn is true? a. It enlarges rapidly.
a. It becomes apparent 4 to 6 months after birth. b. It resolves in hours.
b. Mothers who took anticonvulsant drugs are at c. It is identified immediately at delivery.
lower risk. d. It results from laceration of the emissary or
c. It results from abnormally high levels of proteins diploic veins.
C and S.
d. The disorder is characterized by spontaneous 33–26. Which of the following statements regarding the
internal or external bleeding beginning any time condition depicted in the figure below is true?
after birth.
S ca lp
33–23. Which of the following is associated with severe fetal
thrombocytopenia? Pe rios te um
a. Preeclampsia
S kull bone
b. Systemic lupus erythematosus
c. Immunological thrombocytopenia
d. Neonatal autoimmune thrombocytopenia

33–24. Which of the following types of neonatal intracranial


hemorrhage is common and almost always benign?
a. Subdural
b. Intracerebellar
c. Intraventricular
Reproduced with permission from Cunningham FG, Leveno KJ, Bloom SL, et al (eds):
d. Primary subarachnoid Diseases and injuries of the fetus and newborn. In Williams Obstetrics, 23rd ed. New
York, McGraw-Hill, 2010, Figure 29-12.

a. It is rare.
b. It reaches maximum size at birth.
c. It may persist for weeks to months.
d. It becomes apparent hours after delivery.
226 The N ewborn

33–27. A 33-year-old G1P0 at 42 weeks’ gestation is 33–28. Which of the following statements regarding birth
delivered by cesarean for arrest of descent. The injury of the brachial plexus is NOT true?
patient pushed for 3 hours without progressing past a. Nerve roots at C3–7 and T 1 are injured.
+ 1 station. At the time of cesarean, difficulty in
S
b. The incidence is 1.5/1000 vaginal deliveries.
E
delivering the head is encountered. Several different
C
c. Chances of recovery are good if there is no
T
maneuvers are required to disimpact the head. After
I
avulsion.
O
delivery, a radiograph is performed of the fetal head.
N
It is provided below. Which of the following is NOT d. Risk factors include breech delivery and shoulder
9
the likely etiology of the depressed skull fracture? dystocia.

33–29. A 25-year-old G4P3 presents in active labor at


10 cm dilation. The fetal presentation is breech.
She proceeds to deliver vaginally. After delivery, the
right arm of the newborn is noted to be straight and
internally rotated with the elbow extended and the
wrists and fingers flexed. The affected nerve root is
likely:
a. C2–3
b. C3–4
c. C5–6
d. C8-T 1

33–30. A 20-year-old G2P1 undergoes a forceps-assisted


vaginal delivery for a nonreassuring fetal heart
tracing. After delivery, the newborn is noted to have
an asymmetric grimace. A photograph is provided
below. Which nerve is most likely injured?

Reproduced with permission from Cunningham FG, Leveno KJ, Bloom SL, et al (eds):
Diseases and injuries of the fetus and newborn. In Williams Obstetrics, 23rd ed. New
York, McGraw-Hill, 2010, Figure 29-14.

a. Forceful retraction of the bladder blade


b. Skull compression against the sacral promontory
c. Upward transvaginal hand pressure by an assistant
Reproduced with permission from Cunningham FG, Leveno KJ, Bloom SL, et al (eds):
d. Pressure from the surgeon’s hand as the head is Diseases and injuries of the term newborn. In Williams Obstetrics, 24th ed. New York,
lifted upward McGraw-Hill, 2014, Figure 33-3.

a. Cranial nerve V
b. Cranial nerve VII
c. Cranial nerve IX
d. Cranial nerve XI
Disea ses a nd Injuries of the Term N ewborn 227

33–31. Which of the following birth fractures is the most 33–32. The image below represents the most common
common? type of humeral fracture seen, frequently after an
a. Femoral fracture uneventful birth.What is this type of fracture called?

C
b. Humeral fracture

H
A
c. Clavicular fracture

P
T
d. Mandibular fracture

E
R
3
3
Reproduced with permission from Menkes JS: Initial evaluation and management of
orthopedic injuries. In Tintinalli JE, Stapczynski JS, Cline DM, et al (eds): Tintinalli’s
Emergency Medicine: A Comprehensive Study Guide, 7th ed. New York, McGraw-Hill,
2011, Figure 264-2G.

a. Torus
b. Spiral
c. Segmental
d. Greenstick

References:
Centers for Disease Control and Prevention: Key findings: trends
in the prevalence of developmental disabilities in U.S. children
1997–2008. 2012. Available at: http://www.cdc.gov/ncbddd/features/
birthdefects-dd-keyfindings. html. Accessed June 6, 2013
228 The N ewborn

CHAPTER 33 ANSw ER KEy

Q uestion Letter Pa ge
S
number a nswer cited Hea der cited
E
C
T
33–1 d p. 637 Respira tory Distress Syndrome
I
O
33–2 d p. 638 Prevention
N
33–3 a p. 639 N eona ta l Encepha lopa thy
9
33–4 c p. 639 N eona ta l Encepha lopa thy
33–5 d p. 640 Sentinel Event
33–6 c p. 640 Incidence a nd Epidemiologica l Correla tes
33–7 d p. 641 Intra pa rtum Feta l Hea rt Ra te Monitoring
33–8 c p. 642 Apga r Scores
33–9 b p. 642 Umbilica l Cord Blood G a s Studies
33–10 d p. 642 Umbilica l Cord Blood G a s Studies
33–11 b p. 642 N euroima ging in N eona ta l Period
33–12 b p. 643 Intellectua l Disa bility a nd Seizure Disorders
33–13 b p. 643 Autism Spectrum Disorders
33–14 c p. 643 Autism Spectrum Disorders
33–15 c p. 643 Anemia
33–16 b p. 643 Polycythemia a nd Hyperviscosity
33–17 a p. 644 Hyperbilirubinemia
33–18 c p. 644 Acute Bilirubin Encepha lopa thy a nd Kernicterus
33–19 c p. 644 Prevention a nd Trea tment
33–20 a p. 644 Hemorrha gic Disea se of the N ewborn
33–21 a p. 644 Hemorrha gic Disea se of the N ewborn
33–22 d p. 644 Hemorrha gic Disea se of the N ewborn
33–23 d p. 644 Thrombocytopenia
33–24 d p. 646 Ta ble 3 3 -5
33–25 d p. 646 Extra cra nia l Hema toma s
33–26 b p. 646 Extra cra nia l Hema toma s
33–27 a p. 647 Skull Fra ctures
33–28 a p. 648 Bra chia l Plexopa thy
33–29 c p. 648 Bra chia l Plexopa thy
33–30 b p. 648 Fa cia l Pa ra lysis
33–31 c p. 648 Fa cia l Pa ra lysis
33–32 d p. 648 Fra ctures
229

CHAPTER 34

The Preterm Ne born

34–1. Neonates born prematurely compared with term 34–4. A 30-week newborn displays grunting, nasal
newborns have higher associated rates of which of flaring, chest retractions, and diminished oxygen
the following? saturation. Chest radiography shows a moderate left
a. Congenital malformations pneumothorax, seen here. In general, all EXCEPT
which of the following should be included in the
b. Necrotizing enterocolitis
initial differential diagnosis of respiratory distress in
c. Bronchopulmonary dysplasia a preterm neonate?
d. All of the above

34–2. Following birth, neonatal alveoli must rapidly clear


amnionic fluid and remain expanded to permit gas
exchange. All EXCEPT which of the following assist
this transition?
a. Lymphatic absorption of amnionic fluid
b. Pulmonary vasculature absorption of amnionic
fluid
c. Chest compression by the vaginal walls during
delivery
d. Surfactant-related increases in alveolar surface
tension

34–3. Clinical signs of respiratory distress syndrome


include all EXCEPT which of the following?
a. Grunting a. Sepsis
b. Hypertension b. Diaphragmatic hernia
c. Respiratory acidosis c. Bronchopulmonary dysplasia
d. Chest wall retraction during inspiration d. Persistent fetal circulation
230 The N ewborn

34–5. With respiratory distress syndrome, as shown here, 34–9. o lower rates of respiratory distress syndrome, the
a chest radiograph will most likely display which of American College of Obstetricians and Gynecologists
the following? (ACOG) recommends antenatal corticosteroid
therapy for women at risk of preterm delivery at
S
E
what gestational ages?
C
T
a. 20–30 weeks
I
O
b. 22–32 weeks
N
9
c. 24–34 weeks
d. 26–36 weeks

34–10. After 34 weeks’ gestation, approximately what


percentage of newborns develops respiratory distress
syndrome?
a. 0.5%
b. 4%
c. 9%
d. 16%

a. Perihilar infiltrate 34–11. Complications of hyperoxia used to treat respiratory


distress syndrome typically include all EXCEPT
b. Diffuse reticulogranular infiltrate
which of the following?
c. Increased central bronchovascular markings
a. Pulmonary hypertension
d. Honeycomb pattern with or without air-fluid
b. Necrotizing enterocolitis
levels
c. Retinopathy of prematurity
34–6. Once diagnosed, respiratory distress syndrome is d. Bronchopulmonary dysplasia
preferably treated with which of the following?
a. Intravenous corticosteroids 34–12. A 35-year-old G3P2 with two prior cesarean
deliveries presented for her initial prenatal visit at
b. High inspired-oxygen concentration
28 weeks’ gestation. Her fundal height measured
c. High-frequency oscillatory ventilation 30 cm, and a sonographic evaluation performed
d. All the above at that time yielded measurements that were
concurrent with her last menstrual period. She
34–7. All EXCEPT which of the following are available desires repeat cesarean delivery and has now reached
surfactant products for use in humans? 40 weeks’ gestation without spontaneous labor onset.
a. Exosurf (synthetic) Suitable measures for fetal lung maturity include all
EXCEPT which of the following?
b. Infasurf (calf-derived)
a. Lamellar body count
c. Survanta (bovine-derived)
b. Surfactant – albumin ratio
d. Surfamet (murine-derived)
c. Lecithin – sphingomyelin ratio
34–8. Your patient delivers her son prematurely at d. Mature type II pneumocyte – type I pneumocyte
27 weeks’ gestation. He develops respiratory distress ratio
syndrome, and his neonatologist plans treatment
with surfactant. You counsel her that among its 34–13. Clinical settings that may alter lecithin –
benefits, prophylactic surfactant lowers subsequent sphingomyelin assessment of fetal lung maturity
rates of all EXCEPT which of the following? include all EXCEPT which of the following?
a. Pneumonitis a. Blood-stained amnionic fluid
b. Pneumothorax b. Meconium-stained amnionic fluid
c. Neonatal death c. Maternal corticosteroid administration
d. Bronchopulmonary dysplasia d. Elevated fetal fibronectin concentrations in
amnionic fluid
The Preterm N ewborn 231

34–14. Values greater than what lecithin – sphingomyelin 34–18. With retinopathy of prematurity, which of the
ratio threshold indicate fetal lung maturity? following is the ultimate step leading to blindness?
a. 2 a. Retinal detachment

C
b. 4 b. Vitreous humor hemorrhage

H
A
c. 6 c. Retinal neovascularization

P
T
d. 10 d. Retinal vessel vasoconstriction

E
R
3
34–15. his diagram shows the normal rise in the 34–19. Which of the following modalities is initially

4
concentration of lecithin relative to that of preferred to identify brain abnormalities in the
sphingomyelin. At what gestational age does newborn?
this begin? a. Sonography
b. Computed tomography
12
c. Magnetic resonance imaging
s
)
d
L
i
m
p
d. Positron emission tomography
i
10
l
o
0
h
0
p
1
/
s
34–20. In addition to subdural hemorrhage, the other main
g
8
o
m
h
p
(
Le cithin neonatal intracranial hemorrhage categories include
d
f
o
i
6 all EXCEPT which of the following?
u
l
n
f
o
c
i
a. Cephalohematoma
i
t
n
a
4 S phingomye lin
r
o
t
i
b. Subarachnoid hemorrhage
n
n
e
m
c
2
a
n
c. Intracerebellar hemorrhage
o
n
C
i
0 d. Peri- or intraventricular hemorrhage
12 16 20 24 28 32 36 40 Pos t-
We e ks ’ ge s ta tion te rm 34–21. his coronal sonographic image is from a neonate
born at 26 weeks’ gestation. In preterm newborns,
Reproduced with permission from Cunningham FG, Leveno KJ, Bloom SL, et al (eds): which cerebral lesion, shown here (asterisk), is
T e preterm newborn. In Williams Obstetrics, 24th ed. New York, McGraw-Hill, 2014,
Figure 34-1. strongly associated with adverse neurodevelopmental
outcomes?
a. 24 weeks
b. 28 weeks
c. 30 weeks
d. 34 weeks

34–16. Lamellar bodies are counted in amnionic fluid


samples to assess fetal lung maturity. hese
phospholipid storage organelles are derived from
which of the following?
a. Amniocytes
b. ype II pneumocytes
c. Syncytiotrophoblast
d. Pulmonary endothelium

34–17. Which of the following are common clinical findings


in a preterm neonate with necrotizing enterocolitis? a. Subdural hemorrhage
a. Bloody stools b. Subgaleal hemorrhage
b. Scaphoid abdomen c. Intraventricular hemorrhage
c. Hyperperistalic colon d. None of the above
d. None of the above
232 The N ewborn

34–22. Because of the ventricular dilatation shown in the 34–28. Which of the following statements about
image from Question 34–21, which grade best periventricular leukomalacia is true?
describes this hemorrhage? a. It is associated with cerebral palsy.
a. I
S
b. Cyst size does not correlate with risk of cerebral
E
C
b. II palsy.
T
I
c. III c. Cysts seen on neuroimaging studies require
O
N
d. IV approximately 2 days to develop.
9
d. All of the above
34–23. Which of the following is the most grave sequela in
those who survive intraventricular hemorrhage? 34–29. Which of the following types of cerebral palsy is the
a. Proliferative gliosis least common?
b. Ependymal cell atrophy a. Diplegia
c. Periventricular leukomalacia b. Hemiplegia
d. Choroidal neovascularization c. Spastic quadriplegia
d. Choreoathetoid types
34–24. Approximately what percentage of all neonates born
before 34 weeks’ gestation will have evidence of 34–30. Preterm neonates are most susceptible to brain
intraventricular hemorrhage? ischemia and periventricular leukomalacia. his
a. 0.5 stems from which of the following vascular
limitations of prematurity?
b. 10
a. A fine cortical capillary network that limits blood
c. 30
flow
d. 50
b. Poor anastomoses between cortical and
ventricular vascular systems
34–25. Several organizations recommend antenatal
corticosteroid therapy for women at risk for preterm c. Vessels with underdeveloped tunica media and
delivery to prevent which of the following? poor autoregulation
a. Neonatal death d. Vessels with a low density of receptors specific for
adrenergic vasodilators
b. Intraventricular hemorrhage
c. Respiratory distress syndrome 34–31. Which of the following are underlying risks for
d. All of the above periventricular leukomalacia?
a. Ischemia
34–26. In addition to corticosteroids, which of the following
b. Perinatal infection
has the greatest evidence supporting its efficacy
antenatally to reduce the sequelae of periventricular c. Intraventricular hemorrhage
hemorrhage? d. All of the above
a. Vitamin K
b. Vitamin E
c. Indomethacin
d. Magnesium sulfate

34–27. Which of the following obstetrical practices has


been shown to be superior to lower the rate of
intraventricular hemorrhage?
a. Forceps delivery
b. Cesarean delivery
c. Spontaneous vaginal delivery
d. None is superior
The Preterm N ewborn 233

CHAPTER 34 ANSw ER KEy

Q uestion Letter Pa ge

C
number a nswer cited Hea der cited

H
A
P
34–1 d p. 653 Introduction

T
E
34–2 d p. 653 Respira tory Distress Syndrome

R
3
34–3 b p. 653 Clinica l Course

4
34–4 c p. 653 Clinica l Course
34–5 b p. 653 Clinica l Course
34–6 c p. 654 Trea tment
34–7 d p. 654 Surfa cta nt Prophyla xis
34–8 a p. 654 Surfa cta nt Prophyla xis
34–9 c p. 654 Antena ta l Corticosteroids
34–10 b p. 654 Antena ta l Corticosteroids
34–11 b p. 654 Complica tions
34–12 d p. 655 Amniocentesis for Feta l Lung Ma turity
34–13 d p. 655 Lecithin-Sphingomyelin Ra tio
34–14 a p. 655 Lecithin-Sphingomyelin Ra tio
34–15 d p. 655 Lecithin-Sphingomyelin Ra tio
34–16 b p. 655 O ther Tests
34–17 a p. 655 N ecrotizing Enterocolitis
34–18 a p. 656 Retinopa thy of Prema turity
34–19 a p. 656 Bra in Disorders
34–20 a p. 656 Intra cra nia l Hemorrha ge
34–21 c p. 656 Periventricula r-Intra ventricula r Hemorrha ge
34–22 c p. 657 Incidence a nd Severity
34–23 c p. 656 Pa thology
34–24 d p. 657 Incidence a nd Severity
34–25 d p. 657 Prevention with Antena ta l Corticosteroids
34–26 d p. 657 O ther Preventa tive Methods
34–27 d p. 657 O ther Preventa tive Methods
34–28 a p. 656 Periventricula r-Intra ventricula r Hemorrha ge; Periventricula r Leukoma la cia
34–29 d p. 657 Cerebra l Pa lsy
34–30 b p. 658 Ischemia
34–31 d p. 657 Cerebra l Pa lsy
234

CHAPTER 35

Stillbirth

35–1. In the United States, which o the ollowing 35–5. A 41-year-old G6P5 presents or etal sonographic
gestational ages has the lowest associated etal evaluation at 19 weeks’ gestation. The ollowing
mortality rate per 1000 births? image is obtained, which demonstrates an absent
a. 20 weeks calvarium (an arrow indicates the chin and asterisks
mark the eyes). Her medical history is signi icant or
b. 30 weeks
poorly controlled hypertension and diabetes. Her
c. 39 weeks obstetrical history is signi icant or having a previous
d. 42 weeks child with Down syndrome and a major cardiac
de ect. Which o the ollowing conditions likely
35–2. In the United States, the de inition o a etal death contributed to this particular etal anomaly?
includes all EXCEPT which o the ollowing
characteristics?
a. Death occurs prior to etal expulsion.
b. No signs o li e are apparent at birth.
c. Induced terminations o pregnancy are excluded.
d. It applies only to gestational ages greater than
14 weeks.

35–3. Reporting requirements or etal deaths are


determined by which level o government?
a. Local
b. State
c. County
d. Federal

35–4. The etal mortality rate has declined since 1990 or a. Diabetes mellitus
which o the ollowing gestational age ranges?
b. Advanced maternal age
a. < 20 weeks
c. History o an anomalous in ant in a prior
b. 20–27 weeks pregnancy
c. > 28 weeks d. History o an aneuploid in ant in a prior
d. None o the above pregnancy
Stillbirth 235

35–6. The patient rom Question 35–5 declines 35–10. Determining the cause o a stillbirth via systematic
intervention. She represents at 35 weeks’ gestation evaluation can be important or which o the
or a prenatal care visit and is ound to have a ollowing reasons?

C
stillbirth. Induction is undertaken, and she delivers a. Helps parents cope with the loss

H
the in ant pictured below. Based on the Stillbirth

A
b. May prompt a speci ic therapy

P
Collaborative Research Writing Group’s categories,

T
how would this stillbirth be classi ied with regard to c. Allows or more accurate counseling regarding

E
R
the underlying cause? recurrence risk

3
d. All o the above

5
35–11. A 40-year-old G5P4 presents at 28 weeks’ gestation
with complaints o contractions and vaginal bleeding.
Her blood pressure is 166/98, and heavy proteinuria
is ound on urinalysis. Her cervix is 8 cm dilated,
and there is active vaginal bleeding. No etal heart
tones can be ound, and a stillbirth is con irmed by
sonographic examination. She is stabilized, has a
vaginal delivery, and the placenta is noted to have
the ollowing appearance. What is the most common
associated risk actor or this condition?

Used with permission rom Dr. Tif any Woodus.

a. Unknown
b. Possible
c. Probable
d. None o the above

35–7. A ter a systematic evaluation, approximately what


percentage o stillbirths could be assigned a cause in
the Stillbirth Collaborative Research Network study
(2011)?
a. 25% a. High parity
b. 50% b. Hypertension
c. 75% c. Family history
d. 90% d. Advanced maternal age

35–8. Re erencing the study rom Question 35–7, what 35–12. For the patient presented in Question 35–11, what
was the categorical leading cause o etal death? additional testing is recommended?
a. In ection a. Autopsy
b. Fetal mal ormations b. Karyotyping
c. Placental abnormalities c. Examination o the etus
d. Obstetrical complications d. All o the above

35–9. All EXCEPT which o the ollowing are recognized 35–13. For the patient presented in Question 35–11, what
risk actors or antepartum stillbirth? additional maternal blood test might be use ul in
this particular clinical situation?
a. Smoking
a. Kleihauer-Betke staining
b. Multiparity
b. Serum glucose measurement
c. A rican American race
c. Lupus anticoagulant testing
d. Advanced maternal age
d. Anticardiolipin antibody testing
236 The N ewborn

35–14. The American College o Obstetricians and 35–17. Almost hal o all etal deaths are associated with
Gynecologists recommends karyotyping o stillborns what pregnancy complication?
with which o the ollowing characteristics? a. Preeclampsia
a. Those with morphologic anomalies
S
b. Oligohydramnios
E
C
b. Those born to women o advanced maternal age c. Fetal mal ormations
T
I
c. Those whose serum screening tests indicated an
O
d. Fetal-growth restriction
N
increased risk or aneuploidy
9
d. All stillborns 35–18. At approximately what gestational age should
antenatal testing begin in women with a history o a
35–15. Acceptable specimens or cytogenetic analysis stillbirth in a prior pregnancy?
rom a stillborn include all EXCEPT which o the a. 26 weeks
ollowing?
b. 28 weeks
a. A placental block
c. 30 weeks
b. A etal skin specimen
d. 32 weeks
c. A etal patella specimen
d. An umbilical cord segment 35–19. For women with an unexplained stillbirth in a
previous pregnancy, recommendations or delivery
35–16. I parents decline an autopsy, in addition to a planning would include an induction o labor at
etogram shown here, other tests that can aid what gestational age in a well-dated pregnancy?
determination o the cause o death include all a. 37 weeks
EXCEPT which o the ollowing?
b. 38 weeks
c. 39 weeks
d. 40 weeks

References:
Stillbirth Collaborative Research Network Writing Group: Causes o
death among stillbirths. JAMA 306(22):2459, 2011

a. Photography
b. Viral culture
c. Chromosomal studies
d. Magnetic resonance imaging
Stillbirth 237

CHAPTER 35 ANSw ER KEy

Q uestion Letter Pa ge

C
number a nswer cited Hea der cited

H
A
P
35–1 b p. 661 Introduction

T
E
35–2 d p. 661 Definition of Feta l Morta lity

R
3
35–3 b p. 661 Definition of Feta l Morta lity

5
35–4 c p. 661 Definition of Feta l Morta lity; Figure 3 5 -3
35–5 a p. 662 Ca uses of Feta l Dea th
35–6 c p. 662 Ca uses of Feta l Dea th
35–7 c p. 662 Ca uses of Feta l Dea th
35–8 d p. 662 Ca uses of Feta l Dea th; Ta ble 3 5 -1
35–9 b p. 662 Risk Fa ctors for Feta l Dea th
35–10 d p. 664 Eva lua tion of the Stillborn
35–11 b p. 663 Ta ble 3 5 -1
35–12 d p. 664 Clinica l Exa mina tion
35–13 a p. 664 La bora tory Eva lua tion
35–14 d p. 664 La bora tory Eva lua tion
35–15 b p. 664 La bora tory Eva lua tion
35–16 b p. 665 Autopsy
35–17 d p. 665 Ma na gement of W omen with a Prior Stillbirth
35–18 d p. 665 Ma na gement of W omen with a Prior Stillbirth
35–19 c p. 665 Ma na gement of W omen with a Prior Stillbirth
This page intentionally left blank
Se c t i o n 1 0

Th e Pu er Per iu m
240

Ch APTer 36

T P p

36–1. By de inition, the puerperium lasts what time 36–6. Your multigravid patient delivered 12 hours ago in a
interval ollowing delivery? delivery with an estimated blood loss o 500 mL and
a. 3 weeks without perineal laceration. Her epidural catheter
has been removed, and she now complains o strong
b. 6 weeks
uterine contractions that accompany initial attempts
c. 9 weeks at nursing. Her temperature is 37.0°C; pulse, 84 bpm;
d. 12 weeks and blood pressure, 98/66 mm Hg. She has voided
twice or a total o 800 mL. Her undus is irm and
36–2. According to data rom the Pregnancy Risk minimal bright red blood is noted on her perineal
Assessment Surveillance System, which o the pad. his clinical picture should prompt which o
ollowing concerns is most requently expressed by the ollowing management plans?
women in the irst 2 to 9 months postpartum? a. ransvaginal sonography
a. Altered libido b. Analgesia administration and reassurance
b. Altered body image c. Hematocrit assessment and initiation o
c. Breast- eeding issues uterotonics
d. Ability to return to work d. Hematocrit assessment and initiation o
prophylactic broad-spectrum antibiotics
36–3. ypically, one inger can easily be inserted through
the internal cervical os or up to what time interval 36–7. Lochia, in its various orms, typically resolves a ter
postpartum? how many weeks postpartum?
a. 3 days a. 1 week
b. 10 days b. 5 weeks
c. 21 days c. 9 weeks
d. 42 days d. 13 weeks

36–4. What interval ollowing delivery is required or the 36–8. Which organism has been implicated in late
typical uterus to complete involution? postpartum hemorrhage?
a. 1 week a. Escherichia coli
b. 4 weeks b. Neisseria gonorrhoeae
c. 10 weeks c. Mycoplasma genitalium
d. 16 weeks d. Chlamydia trachomatis

36–5. Sonographically, approximately what percentage o


women will have demonstrable uterine tissue or luid
in their endometrial cavity 2 weeks postpartum?
a. 20%
b. 40%
c. 60%
d. 80%
The Puerperium 241

36–9. Your patient is now 14 days postpartum ollowing 36–11. Your patient had a vaginal delivery yesterday
an uncomplicated vaginal delivery. She notes complicated by a irst-degree perineal laceration
heavy vaginal bleeding and passage o clots. Her a ter a 12-hour labor. She is asymptomatic, a ebrile,

C
temperature is 37.1°C; pulse, 88 bpm; and blood and has normal vital signs. Her uterus is irm and

h
pressure, 112/72 mm Hg. Physical indings reveal nontender. Her remaining physical examination is

A
P
approximately 60 mL o clot in the vaginal vault, unremarkable. She has voided several times, and total

T
a closed cervical os, and a 12-week-size boggy urine output or the past 12 hours is 1400 mL. Her

e
r
nontender uterus. Her hematocrit has declined complete blood count rom this morning reveals the

3
2 volume percent rom that measured ollowing ollowing: hemoglobin, 9.8 g/dL; hematocrit, 31

6
delivery and is 30 volume percent. A sagittal volume percent; white blood cell count, 16,000/mm3;
sonographic image o her uterus is shown here, and platelet count 118,000/mm3. Which o the ollowing
calipers measure the endometrial cavity. he most is the next best management step?
appropriate management or this patient includes a. Chest radiograph
which o the ollowing?
b. Urinalysis and urine culture
c. Continue routine postpartum care
d. Blood culture and intravenous broad-spectrum
antibiotics

36–12. You counsel your puerperal patient that most


women irst approach their prepregnancy weight by
which time interval ollowing delivery?
a. 2 months
b. 6 months
c. 12 months
d. 24 months

36–13. Compared with colostrum, mature milk typically


has greater amounts o which o the ollowing per
volume weight?
a. Dilatation and curettage a. Fat
b. Methergine administration b. Protein
c. Uterine artery embolization c. Minerals
d. Intravenous broad-spectrum antibiotics d. All o the above

36–10. Secondary postpartum hemorrhage is de ined as 36–14. Concentrations o which two vitamins are reduced
bleeding rom 24 hours to 12 weeks postpartum. or absent rom mature breast milk and require
It may be caused by all EXCEPT which o the supplementation?
ollowing? a. Vitamins C and D
a. Endomyometritis b. Vitamins D and K
b. Placental abruption c. Vitamins B6 and K
c. Abnormal involution d. None o the above
d. Retained placental tissue
36–15. Bene its o breast eeding include lower rates o all
EXCEPT which o the ollowing?
a. Puerperal mastitis
b. Maternal breast cancer
c. In ant respiratory in ections
d. Maternal postpartum weight retention
242 The Puerperium

36–16. All EXCEPT which o the ollowing are included in 36–21. Your patient who is breast eeding presents
the World Health Organization steps to success ul 8 weeks postpartum with complaints o breast
breast eeding? pain and associated mass that has enlarged during
S
a. Discourage breast eeding on demand by in ants the last 2 weeks. A signi icantly tender, luctuant
e
C
nonerythematous mass is present. She is a ebrile and
b. Initiate breast eeding within an hour o birth
T
has normal vital signs. Management o this patient
i
O
c. In orm all pregnant women about breast- eeding should primarily include which o the ollowing?
N
advantages
a. Diagnostic excision
1
0
d. Under no circumstances provide ree-o -charge
b. herapeutic needle aspiration
breast milk substitutes
c. Broad-spectrum antibiotic therapy
36–17. Contraindications to breast eeding include maternal d. Incision and drainage and antibiotics with
in ection caused by which o the ollowing viruses? primarily gram-positive coverage
a. Hepatitis C virus
36–22. Your patient presents 5 days postpartum with
b. Genital herpes simplex virus
an axillary mass. She noted it during pregnancy,
c. Human immunode iciency virus although it was much smaller. It became signi icantly
d. All o the above larger and tender yesterday. Her ace is shielded, and
a sagittal photograph o her breast, axilla, and arm is
36–18. o minimize neonatal drug exposure rom maternal shown below. She denies ever or other complaints.
medications, which drug quality is pre erred during Management o this patient should primarily include
selection? which o the ollowing?
a. Long hal -li e
b. Poor lipid solubility
c. Superior oral absorption
d. None o the above

36–19. All EXCEPT which o the ollowing therapies are


absolutely contraindicated during breast eeding?
a. Methotrexate
b. hyroid radioablation
c. Radionuclide imaging with technetium
d. Selective serotonin-reuptake inhibitors

36–20. In a woman who chooses not to breast eed,


treatment o bilateral breast engorgement that is
associated with ever in the irst ew postpartum days
is managed best with which o the ollowing? Used with permission rom Dr. William Grif th.

a. Breast pumping
b. Antibiotics with gram-positive coverage a. Needle aspiration

c. Antipyretic analgesia and breast binding b. Observation and reassurance

d. All o the above c. Axillary lymph node excision


d. Antibiotic therapy with primarily gram-positive
coverage
The Puerperium 243

36–23. All EXCEPT which o the ollowing are typical 36–25. Your patient is a multipara who developed a swollen
components o in-hospital puerperal care? anterior cervical lip during her protracted irst- and
a. Encourage bed rest second-stage labor. Immediately postpartum, the

C
anterior lip was seen prolapsed past the introitus as
b. Monitor voiding requency and volume

h
shown here. How is this condition best managed?

A
c. Assess uterus regularly to con irm contracted tone

P
T
d. Advance diet as tolerated or all puerperal patients

e
r
3
36–24. Your patient had a 3-hour second-stage labor and

6
vaginal delivery without laceration. As her epidural
analgesia subsides, she complains o perineal pain.
Her temperature is 37.2°C; pulse, 84 bpm; and blood
pressure, 120/68 mm Hg. Her irst void yielded
300 mL o urine. Management o this patient should
primarily include which o the ollowing?

a. Vaginal packing
b. Strict bed rest and Foley catheter placement
c. Digital replacement o the prolapse and
observation or urinary retention
d. All o the above

36–26. Which o the ollowing is not associated with


postpartum urinary retention?
a. Prolonged labor
Used with permission rom Dr. Marlene Corton.
b. Chorioamnionitis
a. Perineal cool pack c. Oxytocin induction
b. Surgical evacuation d. Perineal laceration
c. Diagnostic needle aspiration
36–27. A depressed mood within the irst week postpartum
d. Broad-spectrum intravenous antibiotic therapy
in a patient with no prior psychiatric history should
prompt which o the ollowing management plans?
a. Observation
b. Psychiatric admission
c. Psychiatric consultation
d. Initiation o antidepressant medications
244 The Puerperium

36–28. Following spontaneous vaginal delivery, your patient 36–30. For women who are not breast eeding, menses
complains o right leg weakness. Examination reveals will typically return during which time rame
an inability to lex the right hip and extend the leg. postpartum?
S
A common risk actor or this complication includes a. 2–4 weeks
e
C
which o the ollowing?
b. 6–8 weeks
T
a. Eclampsia
i
O
c. 12–16 weeks
N
b. Epidural anesthesia
d. 16–20 weeks
1
c. Prolonged second-stage labor
0
d. Fourth-degree perineal laceration 36–31. Correct statements regarding contraception in breast
eeding women in general include which o the
36–29. Your multigravid patient delivered 5 hours ago in ollowing?
a precipitous delivery without perineal laceration. a. Depot medroxyprogesterone lowers the quality o
Her epidural catheter has been removed, and she breast milk.
now complains o pain with lexion or extension o b. Estrogen-progestin birth control pills do a ect
either leg and with walking or weight bearing. She the quality o breast milk.
has normal lower extremity sensation and muscle
strength. She is a ebrile, vital signs are normal, c. Progestin-only birth control pills do not a ect the
and laboratory testing is unremarkable. Her pelvic quantity o breast milk.
radiograph is shown here. Management includes d. Estrogen-progestin birth control pills do not
which o the ollowing? a ect the quantity o breast milk.

36–32. Because o deep-vein thrombosis risks, combination


hormonal contraception should not be initiated
sooner than how many weeks postpartum?
a. 2 weeks
b. 4 weeks
c. 6 weeks
d. 8 weeks

Reproduced with permission rom Cunningham FG, Leveno KJ, Bloom SL, et al (eds):
T e puerperium. In Williams Obstetrics, 23rd ed. New York, McGraw-Hill, 2010,
Figure 30-4.

a. Supine bed rest


b. ightly itted pelvic binder
c. Surgery or all cases with separation > 3 cm
d. All o the above
The Puerperium 245

Ch APTer 36 ANSw er Key

Q uestion Letter Pa ge

C
number a nswer cited Hea der cited

h
A
P
36–1 b p. 668 Introduction

T
e
36–2 c p. 669 Ta ble 3 6 -1

r
3
36–3 a p. 668 Uterus

6
36–4 b p. 668 Uterus
36–5 d p. 669 Sonogra phic Findings
36–6 b p. 670 Afterpa ins
36–7 b p. 670 Lochia
36–8 d p. 670 Subinvolution
36–9 b p. 670 La te Postpa rtum Hemorrha ge
36–10 b p. 670 La te Postpa rtum Hemorrha ge
36–11 c p. 671 Hema tologica l a nd Coa gula tion Cha nges
36–12 b p. 671 Pregna ncy-Induced Hypervolemia
36–13 a p. 672 Brea st Ana tomy a nd Products
36–14 b p. 672 Brea st Ana tomy a nd Products
36–15 a p. 673 Immunologica l Consequences of Brea st Feeding; N ursing
36–16 a p. 674 Ta ble 3 6 -3
36–17 c p. 674 Contra indica tions to Brea st Feeding
36–18 b p. 674 Drugs Secreted in Milk
36–19 d p. 674 Drugs Secreted in Milk
36–20 c p. 675 Brea st Engorgement
36–21 b p. 675 O ther Issues with La cta tion
36–22 b p. 675 O ther Issues with La cta tion
36–23 a p. 675 Hospita l Ca re
36–24 a p. 676 Perinea l Ca re
36–25 c p. 676 Perinea l Ca re
36–26 b p. 676 Bla dder Function
36–27 a p. 676 Pa in, Mood, a nd Cognition
36–28 c p. 676 O bstetrica l N europa thies
36–29 b p. 676 Musculoskeleta l Injuries
36–30 b p. 678 Contra ception
36–31 c p. 678 Contra ception
36–32 b p. 678 Contra ception
246

CHAPTER 37

Puerperal Complications

37–1. Infection causes what percentage of pregnancy- 37–6. You are called to the bedside of a 16-year-old
related deaths? primipara who underwent a cesarean delivery
a. 1% 28 hours ago for nonreassuring fetal status. She is
complaining of abdominal cramps and chills. She
b. 6%
has a fever of 38.7°C and is not tolerating oral
c. 11% intake. During examination, her lungs are clear, and
d. 22% breasts are soft. She does, however, have uterine and
parametrial tenderness. Her hematocrit is stable,
37–2. Which of the following is the most significant risk white blood cell count is 15,000 cells/µL, and
factor for the development of puerperal uterine urinalysis is unremarkable. Which of the following is
infection? the likely diagnosis?
a. Use of epidural a. Atelectasis
b. Length of labor b. Postpartum metritis
c. Route of delivery c. Small bowel obstruction
d. Number of vaginal examinations d. Septic pelvic thrombophlebitis

37–3. As a new, young obstetrician-gynecologist, you strive 37–7. The management of the patient in Question 37–6
to avoid postcesarean infection in your patients. should include all EXCEPT which of the following?
Which of the following interventions would be most a. Antipyretics
helpful in this pursuit?
b. Blood cultures
a. Changing scalpels after skin incision
c. Intravenous fluids
b. Changing gloves after delivery of the fetal head
d. Intravenous broad-spectrum antibiotics
c. Single dose of antibiotics prior to skin incision
d. Copious irrigation of the abdomen before 37–8. After 72 hours of intravenous antibiotics, the
incision closure patient in Question 37–6 has yet to defervesce. She
continues to have fevers as high as 38.5°C. Which
37–4. Which of the following is NOT a risk factor for of the following is the LEAST likely explanation for
puerperal pelvic infection? her condition?
a. Obesity a. Drug fever
b. General anesthesia b. Infected hematoma
c. Advanced maternal age c. Parametrial phlegmon
d. Meconium-stained amnionic fluid d. Septic pelvic thrombophlebitis

37–5. Organisms from which group have been implicated


in late-onset, indolent metritis?
a. Chlamydia
b. Pseudomonas
c. Clostridium
d. Staphylococcus
Puerpera l Complica tions 247

37–9. The patient in Question 37–6 has been receiving 37–13. A 29-year-old multipara presents to the emergency
intravenous gentamicin and clindamycin. Which of room 7 days after a cesarean delivery. She complains
the following statements regarding this regimen is of drainage from her incision, fever, and nausea and

C
true? vomiting. A photograph of her abdomen is provided.

H
a. This regimen has a 50% to 60% response rate. The patient’s skin is erythematous, and malodorous

A
P
pus is oozing from the right angle of the incision.
b. If the gentamicin is administered once daily,

T
Which of the following is the most appropriate

E
serum levels will probably be inadequate.

R
management of this patient?

3
c. It may be ineffective in this patient if the

7
offending organism group is Enterococcus.
d. The gentamicin should be changed to
metronidazole so Clostridium difficile is covered.

37–10. Which of the following statements regarding


perioperative prophylaxis at the time of cesarean
delivery is NOT true?
a. Prophylactic antimicrobials reduce the pelvic
infection rate by 70% to 80%.
b. There is no benefit to prophylactic antimicrobials
in cases of nonelective cesarean delivery.
c. Single-dose prophylaxis with ampicillin or a
first-generation cephalosporin is as effective as a
multiple-dose regimen.
d. Women known to be colonized with methicillin-
resistant Staphylococcus aureus (MRSA) should be
a. Culture the wound and discharge her home with
given vancomycin in addition to a cephalosporin.
oral antimicrobial therapy
37–11. Which of the following has been shown to prevent b. Culture the wound, start intravenous
puerperal pelvic infections following cesarean antimicrobial therapy, open the skin incision
delivery? fully, and pack with moist gauze
a. Closure of the peritoneum c. Culture the wound, start intravenous
antimicrobial therapy, open the skin incision
b. Changing gloves after delivery of the placenta
fully, and place a wound vacuum device
c. Prenatal treatment of asymptomatic vaginal
d. Culture the wound, start intravenous
infections
antimicrobial therapy, perform surgical
d. Allowing the placenta to separate and deliver exploration, and provide aggressive supportive
spontaneously care

37–12. Which of the following is a risk factor for wound


infection?
a. Anemia
b. Obesity
c. Diabetes mellitus
d. All of the above
248 The Puerperium

37–14. For the patient in Question 37–13, you decide to 37–16. An 18-year-old primigravida at 41 weeks’ gestation
administer intravenous broad-spectrum antimicrobial presented to Labor and Delivery for decreased fetal
therapy and perform wound exploration and movement. She was found to have oligohydramnios,
S
debridement in the operating room. Purulent material and an induction of labor was initiated. Twenty-
E
C
is noted to be issuing from below the fascia. For this four hours later, the patient was 4 cm dilated with
T
reason, the fascia is opened. Pus is also draining from no cervical change for 4 hours, despite adequate
I
O
the uterine incision, and a photograph of her uterus contractions. In addition, the patient had been
N
is provided here. Which of the following is the most diagnosed with chorioamnionitis, and intravenous
1
0
appropriate management of this patient? antimicrobial therapy was begun. A cesarean delivery
was performed for failure to progress. Postpartum,
the patient continued to have fever for 5 days despite
treatment with intravenous ampicillin, gentamicin,
and clindamycin. Which of the following is a
reasonable next step in the management of this
patient?
a. Surgical exploration to exclude abscess
b. Stop antibiotics and discharge the patient home
c. Continue current management for 5 additional
days and then reassess
d. Further evaluate the patient using computed
tomography (CT) or magnetic resonance (MR)
imaging

37–17. You order a pelvic CT scan for the patient in


Question 37–16. An image from that study is
Reproduced with permission from Cunningham FG, Leveno KJ, Bloom SL, et al (eds): provided. What is the most likely diagnosis?
Puerperal complications. In Williams Obstetrics, 24th ed. New York, McGraw-Hill,
2014, Fig. 37-4.

a. Abdominal wash out and closure of the patient’s


fascia, subcutaneous layer, and skin
b. Hysterectomy, abdominal wash out, and closure
of the patient’s fascia without closure of the skin
and subcutaneous layer
c. Debridement of the uterine incision, closure of
the uterine incision, abdominal wash out, and
closure of both the patient’s fascia and superficial
layers
d. Reapproximation of the uterine incision with
delayed absorbable suture, abdominal wash out,
and closure of the patient’s fascia without closure
of the skin and subcutaneous layer
Reproduced with permission from Cunningham FG, Leveno KJ, Bloom SL, et al (eds):
Puerperal complications. In Williams Obstetrics, 24th ed. New York, McGraw-Hill,
37–15. Which of the following statements regarding 2014, Fig. 37-7.
parametrial phlegmon is true?
a. Phlegmons are usually bilateral. a. Pelvic abscess
b. Phlegmons typically do not involve the broad b. Psoas muscle abscess
ligament. c. Ovarian vein thrombosis
c. Associated fever typically resolves in 5 to 7 days d. Retained products of conception
with antibiotic treatment but may persist longer.
d. A phlegmon should be considered when
postpartum fever persists longer than 24 hours
despite intravenous antimicrobial therapy.
Puerpera l Complica tions 249

37–18. The patient in Question 37–17 appears quite 37–22. Which of the following is NOT a risk factor for
well despite her continued fevers. Based on her episiotomy dehiscence?
CT findings, which of the following is the most a. Smoking

C
appropriate management of this patient?
b. Coagulation disorder

H
a. Continue antibiotics until she is afebrile

A
c. Human papillomavirus infection

P
b. Stop antibiotics and discharge the patient home

T
d. Intrapartum antimicrobial therapy

E
R
c. Stop antibiotics and start lifelong anticoagulation

3
d. Continue antibiotics and start a 12-month course 37–23. The patient in Question 37–21 undergoes surgical

7
of anticoagulation debridement of her separated and infected fourth-
degree laceration. A photograph taken at the
37–19. Which of the following statements regarding septic end of the surgery is provided. To permit early
pelvic thrombophlebitis is true? reapproximation, which of the following is essential
a. Patients complain of severe pain. postoperatively?
b. The overall incidence is 1/10,000 deliveries.
c. Intravenous heparin causes fever to dissipate.
d. The ovarian veins are involved because they drain
the upper uterus and therefore, the placental
implantation site.

37–20. What percentage of women with septic pelvic


thrombophlebitis have clot extending into the vena Ante rio r vag inal wall
cava? Vag inal o pe ning

a. 1%
b. 5% Dis rupte d pe rine um

c. 10%
d. 25% Re c tal muc o s a

37–21. A 22-year-old primipara presents to the emergency


room 4 days after a forceps-assisted vaginal delivery
complaining of intense perineal pain, fever, and a
foul vaginal odor. During examination, infection
and dehiscence of a fourth-degree laceration is noted.
Reproduced with permission from Cunningham FG, Leveno KJ, Bloom SL, et al (eds):
Which of the following is the most appropriate Puerperal infection. In Williams Obstetrics, 23rd ed. New York, McGraw-Hill, 2010,
management of this patient? Figure 31-7.
a. Oral broad-spectrum antimicrobial therapy and
outpatient reevaluation in 3 days a. Sitz baths completed several times daily
b. Intravenous antimicrobial therapy and hospital b. Provision of adequate analgesia during each
admission for observation and sitz baths debriding wound scrub
c. Intravenous antimicrobial therapy and hospital c. A debriding wound scrub completed twice daily
admission for surgical debridement and local with povidone-iodine solution
wound care d. All of the above
d. Oral broad-spectrum antimicrobial therapy,
placement of a Foley catheter with leg bag, daily
sitz baths at home, and outpatient reevaluation in
2 days
250 The Puerperium

37–24. When is an infected episiotomy ready for early 37–29. What percentage of women with mastitis develop an
repair? abscess?
a. 24 hours after debridement a. 1%
S
b. Once any improvement is seen in the tissue b. 5%
E
C
T
c. Once the patient becomes afebrile for 24 hours c. 10%
I
O
d. When the wound is free of infection and covered d. 20%
N
by pink granulation tissue
1
0
37–30. A 20-year-old primipara presents to your office
37–25. A 16-year-old primipara is brought to the emergency 4 weeks after a term spontaneous vaginal delivery.
room three days after having a vaginal delivery. She is breast feeding and reports severe right breast
The girl is confused and lethargic. Her mother pain. She had a fever at home of 101°F. The
reports that she has had a fever at home associated patient appears ill. You perform an examination
with nausea, vomiting, diarrhea, and headache. and note a tender right breast fullness, which is
During examination, you note very mild uterine warm and erythematous. A photograph is provided
tenderness and a diffuse macular erythematous rash. below. The patient reports feeding her baby every
The patient is hypotensive, tachycardic, and febrile. 3 hours, although the newborn latches poorly on
Laboratory studies reveal leukocytosis, transaminitis, the right breast. She has no complaints regarding
prolongation of her partial thromboplastin time her left breast, which appears normal. Which of the
(PTT), and an elevated creatinine level. Which of following is the most likely diagnosis?
the following is the most likely diagnosis?
a. Cyclospora
b. Meningitis
c. Viral hepatitis
d. Toxic shock syndrome

37–26. When selecting antimicrobial therapy for the patient


in Question 37–25, it is important that you select an
agent that covers which of the following?
a. Staphylococcus aureus
b. Group A β -hemolytic Streptococcus
c. Both of the above
d. Neither of the above

37–27. When is suppurative mastitis most likely to develop?


a. One week after delivery Used with permission from Dr. Emily Adhikari.

b. Two weeks after delivery


a. Mastitis
c. Three weeks after delivery
b. Engorgement
d. Eight weeks after delivery
c. Blocked milk duct
37–28. Which of the following is NOT true regarding the d. Inflammatory breast cancer
symptoms of suppurative mastitis?
a. Patients report severe pain.
b. It is almost always bilateral.
c. Marked engorgement usually precedes
inflammation.
d. Chills or rigor are soon followed by fever and
tachycardia.
Puerpera l Complica tions 251

37–31. Which of the following is the most appropriate 37–32. Sonographic examination of the right breast is
management of the patient in Question 37–30? performed. An image is provided. Which of the
a. Milk culture and mammogram following is the most appropriate management of

C
this patient?
b. Analgesia and frequent breast feeding

H
A
c. Milk culture, breast sonography, initiation of

P
T
antimicrobial therapy, analgesia, and breast

E
R
feeding/pumping

3
d. Initiation of antimicrobial therapy, milk culture,

7
and discontinuation of breast feeding or pumping
on the right side

a. Breast biopsy
b. Surgical drainage
c. Increased breast feeding/pumping and
discontinuation of antibiotics
d. Continuation of antibiotics for 48 hours with
surgical drainage if the patient does not defervesce
252 The Puerperium

CHAPTER 37 ANSw ER KEy

Q uestion Letter Pa ge
S
number a nswer cited Hea der cited
E
C
T
37–1 c p. 682 Puerpera l Infections
I
O
37–2 c p. 683 Predisposing Fa ctors
N
1
37–3 c p. 683 Cesa rea n Delivery
0
37–4 c p. 683 O ther Risk Fa ctors
37–5 a p. 683 Common Pa thogens
37–6 b p. 684 Pa thogenesis a nd Clinica l Course
37–7 b p. 684 Trea tment
37–8 a p. 684 Trea tment
37–9 c p. 684 Choice of Antimicrobia ls
37–10 b p. 685 Periopera tive Prophyla xis
37–11 d p. 685 O ther Methods of Prophyla xis
37–12 d p. 686 Abdomina l Incisiona l Infections
37–13 d p. 686 N ecrotizing Fa sciitis
37–14 b p. 686 N ecrotizing Fa sciitis
37–15 c p. 687 Pa ra metria l Phlegmon
37–16 d p. 688 Ima ging Studies
37–17 c p. 689 Figure 3 7 -7
37–18 a p. 688 Septic Pelvic Thrombophlebitis
37–19 d p. 688 Septic Pelvic Thrombophlebitis
37–20 d p. 688 Septic Pelvic Thrombophlebitis
37–21 c p. 689 Perinea l Infections
37–22 d p. 689 Perinea l Infections
37–23 d p. 690 Ta ble 3 7 -3
37–24 d p. 690 Ea rly Repa ir of Infected Episiotomy
37–25 d p. 690 Toxic Shock Syndrome
37–26 c p. 690 Toxic Shock Syndrome
37–27 c p. 691 Brea st Infections
37–28 b p. 691 Brea st Infections
37–29 c p. 691 Brea st Infections
37–30 a p. 691 Figure 3 7 -8 a
37–31 c p. 691 Brea st Infections—Ma na gement
37–32 b p. 692 Brea st Abscess
253

CHAPTER 38

Contraception

38–1. Which o the ollowing is considered long-acting 38–6. The Skyla IUD can be di erentiated rom the other
reversible contraception (LARC)? FDA-approved levonorgestrel-releasing IUD by
a. Copper intrauterine device which o the ollowing characteristics?
b. Depot medroxyprogesterone acetate a. White, T-shaped rame
c. Combination hormonal contraceptive patch b. Silver ring atop the stem
d. All o the above c. Copper bracelets on the lateral T arms
d. Single braided brown string attached to the
38–2. What is the goal o the United States Medical T- rame base
Eligibility Criteria (US MEC) published by the
Centers or Disease Control and Prevention? 38–7. Contraceptive e icacy with this method is believed
a. Guide contraceptive selection or women with to result rom which o the ollowing?
comorbidities
b. Provide legal criteria or contraception provision
to minors
c. Provide inancial criteria or Medicaid
contraception eligibility
d. None o the above

38–3. With typical use, which o the ollowing


contraceptive methods has the highest ailure rate
within the irst year o use?
a. Withdrawal
b. Spermicides
c. Male condom
d. Progestin-only pills

38–4. O U.S. Food and Drug Administration (FDA)-


approved intrauterine devices (IUDs), hormone-
eluting devices release which o the ollowing?
a. Norgestimate
b. Etonogestrel
c. Norethindrone a. Spermicidal action
d. Levonorgestrel b. Ovulation inhibition
38–5. O U.S. FDA-approved intrauterine devices, c. Cervical mucus thickening
which o the ollowing has the longest duration o d. All o the above
contraceptive e icacy?
a. Paragard, copper intrauterine device
b. Skyla, levonorgestrel-releasing intrauterine device
c. Mirena, levonorgestrel-releasing intrauterine
device
d. Skyla and Mirena have equivalent durations o
e icacy
254 The Puerperium

38–8. During insertion, which o the ollowing increases 38–10. Your patient complains o lower abdominal ache in
this IUD-related complication? the months ollowing placement o her IUD. During
examination, no IUD strings are seen. Which o the
S
ollowing is the most appropriate next management step?
E
C
a. Obtain KUB radiograph
T
I
b. Reexamine her ollowing her next menses
O
N
c. Per orm β -human chorionic gonadotropin
1
(β -hCG) assay
0
d. Attempt intrauterine IUD retrieval by means o
an IUD hook or orceps

38–11. The patient in Question 38–10 has a negative


β -hCG test result. Which o the ollowing is the best
next management step?
a. Diagnostic laparoscopy
b. Diagnostic hysteroscopy
c. Transvaginal sonography
Used with permission rom Dr. Kimberly Kho. Reproduced with permission rom Werner d. Magnetic resonance imaging
CL, Moschos E, Grif th WF, et al (eds): Contraception. In Williams Gynecology, 2nd
Edition Study Guide. New York, McGraw-Hill, 2012, Q5-9.
38–12. Her sonographic images are shown here, and an
arrow indicates a sagittal view o the IUD. Which o
a. Breast eeding the ollowing is the best next clinical step?
b. Insertion during menses
c. Midplane uterine position
d. None o the above

38–9. Compared with the copper IUD, this intrauterine


device o ers which o the ollowing long-term
advantages?

a. Lower rates o menorrhagia B


b. Longer duration o e ective use
a. Hysterectomy
c. Lower rates o ovarian cyst ormation
b. Diagnostic laparoscopy
d. None o the above
c. Dilation and curettage
d. Operative hysteroscopy
Contra ception 255

38–13. Which o the ollowing statements regarding IUD 38–15. An asymptomatic woman with 8 weeks o
use and in ection are correct? amenorrhea is ound to have a positive pregnancy
a. Prophylaxis with doxycycline 100 mg orally in a test result, strings o her Mirena IUD visible at the

C
single dose is required prior to IUD insertion. cervical os, and sonographic indings shown here.

H
No adnexal pathology was seen sonographically. She

A
b. Endocarditis prophylaxis is required or those

P
desires to continue the pregnancy i possible. What

T
with mitral valve prolapse prior to IUD insertion.
is the most appropriate management or her?

E
c. Negative Neisseria gonorrhoeae and Chlamydia

R
3
trachomatis test results are required prior to IUD

8
insertion.
d. None o the above

38–14. This photograph shows placental membranes


ollowing a term delivery. Antenatally, this
pregnancy was at increased risk or which o the
ollowing?

a. Fetal mal ormations


b. Intrauterine in ection
a. IUD removal

c. Fetal-growth restriction
b. IUD removal and dilatation and curettage

d. All o the above


c. IUD removal and 10-day course o doxycycline
d. Initiate olate and schedule irst prenatal care visit
or 2 weeks
256 The Puerperium

38–16. I an intrauterine device is le t in situ during 38–20. Which o the ollowing is a suspected method o
pregnancy, what is the risk o this pregnancy e icacy or progestin-containing contraceptive
complication? implants?
S
a. Endometrial atrophy
E
C
b. Ovulation inhibition
T
I
O
c. Cervical mucus thickening
N
d. All o the above
1
0
38–21. According to the US MEC, in addition to pregnancy
and current breast cancer, which o the ollowing
are absolute contraindications to progestin-releasing
implants?
a. Prior cervical cancer and prior deep-vein
thrombosis
b. Prior ectopic pregnancy and prior deep-vein
thrombosis
Used with permission rom Dr. Clarice Grimes. c. Prior cervical cancer, prior ectopic pregnancy, and
diagnosed depression
a. 10% d. None o above
b. 25%
38–22. Rates o which o the ollowing are increased with
c. 50%
use o extended cycle hormonal contraception
d. 75% compared with traditional cyclic hormonal
contraception use?
38–17. Which o is true regarding the timing o IUD
a. Headaches
insertion?
b. Escape ovulation
a. May be inserted immediately ollowing vaginal
delivery c. Endometrial cancer
b. May be inserted during the menstrual cycle d. Unpredictable bleeding
ollicular phase once pregnancy is excluded
38–23. Your patient calls you on Monday stating that she
c. May be inserted immediately ollowing dilation
missed taking her birth control pills during the
and curettage or irst-trimester abortion
weekend. All EXCEPT which o the ollowing are
d. All o the above true counseling statements?
a. Take three pills today and then inish the current
38–18. Which hormone is released rom the contraceptive pill pack
implant currently marketed in the United States?
b. Take one pill today, inish the pill pack, but add
a. Norgestimate a barrier method until her next menses
b. Etonogestrel c. Stop pills, use a barrier method until the next
c. Norethindrone menses, then restart a new pack on irst day o
d. Levonorgestrel menses
d. Discard current pill pack, start new pill pack
38–19. A new patient requests removal o her Implanon today, but add an additional barrier method or
implant. You are unable to locate the device by 7 days
palpation. What is the best next management step?
a. Sonography 38–24. Drugs whose e icacy may be diminished by
b. Fluoroscopy combination oral contraceptive pill use include
which o the ollowing?
c. Radiography
a. Lisinopril
d. Computed-tomography scanning
b. Macrodantin
c. Acetaminophen
d. Metoclopramide
Contra ception 257

38–25. Which o the ollowing is not a physiologic e ect 38–30. With transdermal contraceptive patch use, the patch
exerted by the estrogen component o combination should be replaced how requently?
hormonal contraceptives?

C
a. Elevated serum ibrinogen levels

H
A
b. Lowered serum ree testosterone levels

P
T
c. Elevated serum ollicle-stimulating hormone

E
R
levels

3
d. Lowered serum low-density-lipoprotein (LDL)

8
cholesterol levels

38–26. I prescribing combination oral contraceptive pills to


a woman with hypertension, which clinical criteria
should be met?
a. Nonsmoker
b. Younger than 35 years
c. Hypertension well controlled
d. All o the above a. Daily
b. Weekly
38–27. Which o the ollowing is not an absolute c. Twice weekly
contraindication to initiation o combination oral d. Every 3 weeks
contraceptive pills?
a. Thrombotic disorders 38–31. With transdermal contraceptive patch use,
b. Two weeks postpartum contraceptive ailure may occur in which o the
c. Migraines with ocal neurologic de icits ollowing situations?
d. Uncomplicated systemic lupus erythematosus a. Patient weight > 90 kg
with negative antiphospholipid antibody testing b. Concurrent penicillin antibiotic use
c. Patch worn while in hot tub or sauna
38–28. The relative risk o which o the ollowing cancers d. All o the above
has been associated with current combination oral
contraceptive pill use? 38–32. With this method o contraception, a new ring is
a. Ovarian used how requently?
b. Melanoma
c. Cervical
d. Endometrial

38–29. A proven bene it o combination oral contraceptive


pills includes which o the ollowing?
a. Increased bone density
b. Improved cognitive memory
c. Lowered total cholesterol levels
d. Decreased rates o thromboembolism

Reproduced with permission rom Cunningham FG, Leveno KJ, Bloom SL, et al (eds): Con-
traception. In Williams Obstetrics, 24th ed. New York, McGraw-Hill, 2014, Figure 38-5.

a. Daily
b. Weekly
c. Monthly
d. Every 3 months
258 The Puerperium

38–33. Compared with combination oral contraceptive 38–38. The incidence o which o the ollowing is increased
pills, the vaginal contraceptive ring is associated with with diaphragm use?
higher rates o which o the ollowing? a. Cystocele
S
a. Toxic shock syndrome b. Cervicitis
E
C
b. Contraceptive ailure
T
c. Urinary tract in ection
I
O
c. Cervical intraepithelial neoplasia d. Pelvic in lammatory disease
N
d. None o the above
1
0
38–39. Which o the ollowing is NOT a variation o
38–34. Risks and bene its o depot medroxyprogesterone periodic abstinence as a amily planning method?
acetate (DMPA) use or longer than 2 years should a. Withdrawal method
be evaluated care ully, especially in which o the
b. Cervical mucus method
ollowing patients?
c. Calendar rhythm method
a. Hypertensive women
d. Temperature rhythm method
b. Perimenopausal women
c. Women with a amily history o breast cancer 38–40. Compared with latex diaphragms, this product o ers
d. Women with a history o simple endometrial which advantages?
hyperplasia

38–35. Potential side e ects o DMPA commonly may


include which o the ollowing?
a. Amenorrhea
b. Irregular uterine bleeding
c. Delayed return o ertility ollowing DMPA
cessation
d. All o the above

38–36. Your patient has diabetes and hypertension but


pre ers to use “pills” or contraception. She declines
an intrauterine device and barrier methods. She is
considering a progestin-only contraceptive and avors
progestin-only pills. You counsel her regarding the
advantages and which o the ollowing disadvantages
o progestin-only pills compared with combination
oral contraceptives?
a. Higher ailure rate
b. Higher rate o irregular bleeding
a. Greater e icacy
c. Higher relative rate o ectopic pregnancy with
method ailure b. Prescription not required
d. All o the above c. Not associated with subsequent vaginitis
d. Not associated with toxic shock syndrome
38–37. Latex condom e icacy is enhanced by which o the
ollowing? 38–41. Current methods o emergency contraception
a. Reservoir tip include all EXCEPT which o the ollowing?
b. Oil-based spermicide a. Ulipristal acetate
c. Concurrent emale condom use b. Levonorgestrel-releasing IUD
d. All o the above c. Levonorgestrel-containing pills
d. Combination oral contraceptive pills
Contra ception 259

CHAPTER 38 ANSw ER KEy

Q uestion Letter Pa ge

C
number a nswer cited Hea der cited

H
A
P
38–1 a p. 695 Introduction

T
E
38–2 a p. 695 Introduction

R
3
38–3 b p. 696 Ta ble 3 8 -2

8
38–4 d p. 696 Long-a cting Reversible Contra ception: Intra uterine Devices
38–5 a p. 696 Long-a cting Reversible Contra ception: Intra uterine Devices
38–6 b p. 696 Long-a cting Reversible Contra ception: Intra uterine Devices
38–7 a p. 697 Contra ceptive Action
38–8 a p. 697 Perfora tion
38–9 a p. 699 Menstrua l Cha nges
38–10 c p. 697 Lost Device
38–11 c p. 697 Lost Device
38–12 b p. 697 Lost Device
38–13 d p. 699 Infection
38–14 b p. 700 Pregna ncy with a n IUD
38–15 a p. 700 Pregna ncy with a n IUD
38–16 c p. 700 Pregna ncy with a n IUD
38–17 d p. 700 Timing
38–18 b p. 703 Long-a cting Reversible Contra ception: Progestin Impla nts
38–19 a p. 703 Long-a cting Reversible Contra ception: Progestin Impla nts
38–20 d p. 704 Actions a nd Side Effects
38–21 d p. 704 Contra indica tions to Progestin-O nly Contra ceptives
38–22 d p. 705 Administra tion
38–23 a p. 705 Administra tion
38–24 c p. 708 Ta ble 3 8 -5
38–25 c p. 708 Meta bolic Cha nges
38–26 d p. 709 Ca rdiova scula r effects
38–27 d p. 698 Ta ble 3 8 -3
38–28 c p. 709 N eopla sia
38–29 a p. 710 Ta ble 3 8 -6
38–30 b p. 710 Tra nsderma l Pa tch
38–31 a p. 710 Tra nsderma l Pa tch
38–32 c p. 710 Tra nsva gina l Ring
38–33 d p. 710 Tra nsva gina l Ring
38–34 b p. 711 Actions a nd Side Effects
38–35 d p. 711 Actions a nd Side Effects
38–36 d p. 711 Progestin-O nly Pills
38–37 a p. 712 Ma le Condom
38–38 c p. 712 Dia phra gm Plus Spermicide
38–39 a p. 713 Fertility-Awa reness Ba sed Methods
38–40 b p. 714 Contra ceptive Sponge
38–41 b p. 714 Hormona l Emergency Contra ception
260

CHAPTER 39

Sterilization

39–1. Your patient is a 26-year-old G3P2 who desires 39–4. Although highly e ective and shown here, which
permanent sterilization ollowing her upcoming method is an uncommonly used technique or
delivery. Her past medical and surgical histories puerperal sterilization?
are unremarkable. During your counseling session
you quote ailures rates or puerperal sterilization;
you discuss alternative methods o contraception,
including the advantages o vasectomy; and
you discuss possible operative and anesthesia
complications. Which other preoperative counseling
points regarding emale tubal sterilization are true?
a. Ease o sterilization reversal
b. Higher risk o later menstrual irregularities
c. Choice o either an abdominal or a hysteroscopic
approach or puerperal sterilization
d. None o the above

39–2. Which o the ollowing aspects o normal


postpartum maternal anatomy are not advantageous
or puerperal sterilization?
a. Noninvoluted uterus
Reproduced with permission rom Hof man BL: Surgeries or benign gynecologic condi-
b. Lax anterior abdominal wall tions. In Schorge JO, Schaf er JI, Halvorson LM, et al (eds) Williams Gynecology. New
c. Engorged mesosalpinx vessels York, McGraw-Hill, 2008, Figure 41-24.5.

d. All o the above


a. Uchida
39–3. Outcomes associated with per orming puerperal b. Pomeroy
tubal sterilization the morning a ter vaginal delivery c. Parkland
include which o the ollowing? d. Modi ied Pomeroy
a. Increases the risk o anesthesia-related
complications 39–5. Sterilization in the puerperium is typically per ormed
b. Provides longer opportunity to assess neonatal using which o the ollowing anesthetic methods?
well-being a. Spinal anesthesia
c. Increases the risk o postpartum hemorrhage b. General anesthesia
complicating the postoperative course c. Incision in iltration
d. All o the above d. Transversus abdominis plane block
Steriliza tion 261

39–6. During puerperal sterilization and ollowing 39–10. With this method o interval sterilization, which o
peritoneal cavity entry, the small bowel and the ollowing energy sources is pre erred?
omentum continue to dri t into your operative ield.

C
All EXCEPT which o the ollowing techniques can

H
improve visualization?

A
P
a. Enlarge the incision

T
E
b. Use a wider retractor

R
3
c. Place patient in reverse Trendelenburg position

9
d. Pack the omentum cephalad with an opened
4 × 4 gauze sponge whose extraabdominal end
has been tagged with a hemostat

39–7. Prior to ligation, the allopian tube is most reliably


identi ied by which o its ollowing attributes?
a. Fimbria
b. Pronounced vascularity
c. Round tubular structure
d. Location posterior to the uteroovarian ligament Reproduced with permission rom Hof man BL: Surgeries or benign gynecologic
conditions. In Schorge JO, Schaf er JI, Halvorson LM, et al (eds) Williams Gynecology.
New York, McGraw-Hill, 2008, Figure 41-29.1.
39–8. With the Parkland method o emale tubal
sterilization, it is recommended that approximately
what length o tubal segment be excised to allow a. Harmonic ultrasound
adequate separation o tubal stumps? b. Bipolar electrosurgical coagulation
c. Unipolar electrosurgical coagulation
d. Nd:YAG (neodymium-doped yttrium aluminum
garnet) laser

39–11. Which o the ollowing is true o nonpuerperal


emale tubal sterilization per ormed in the United
States?
a. Commonly per ormed via minilaparotomy
b. Commonly per ormed using general anesthesia
c. Typically requires overnight hospitalization
d. None o the above

39–12. O available methods, which o the ollowing is


commonly used or nonpuerperal tubal sterilization
in the United States?
a. Laparoscopic clip application
a. 0.5 cm b. Laparoscopic supracervical hysterectomy
b. 1 cm c. Distal salpingectomy via posterior colpotomy
c. 2 cm d. Hysteroscopic intratubal quinine instillation
d. 4 cm

39–9. The success o the Pomeroy procedure relies upon


the use o what type o suture ligature?
a. Plain gut
b. Chromic gut
c. Synthetic polyglactin braided
d. Synthetic polydioxanone mono ilament
262 The Puerperium

39–13. This image shows the cumulative probability o 39–15. This surgical specimen is the result o which
pregnancy per 1000 procedures by ive methods intraoperative event during puerperal sterilization?
o tubal sterilization using data rom the U.S.
S
Collaborative Review o Sterilization (CREST)
E
C
study. When counseling your patient regarding
T
puerperal bilateral midsegment salpingectomy, which
I
O
o the ollowing long-term ailure rates per procedure
N
is most accurate?
1
0
30 Inte rva l pa rtia l s a lpinge ctomy
e
S ilicone ba nd or s pring clip
r
u
+ + Bipola r coa gula tion
l
+
i
25
a
f
P ue rpe ra l pa rtia l s a lpinge ctomy
s
f
e
o
r
X X Unipola r coa gula tion
u
20
y
d
t
i
e
l
i
c
b
+
o
a
15
r
b
p
o
0
r
p
0
0
10
e
1
v
r
i
t
+
e
a
p
l
5
u
m
+ X X
u
C
0
1 Ye a r 3 Ye a rs 5 Ye a rs 12 Ye a rs Reproduced with permission rom Cunningham FG, Leveno KJ, Bloom SL (eds): Anat-
Inte rval afte r s te rilizatio n omy. In Williams Obstetrics, 23rd ed. New York, 2014, Figure 2-15B.

Reproduced with permission rom Cunningham FG, Leveno KJ, Bloom SL (eds) Williams
Obstetrics, 23rd ed. New York, 2010, Figure 5-15.
a. Incidental appendectomy
a. 1 in 150 b. Correct allopian tube ligation
b. 1 in 650 c. Incorrect round ligament ligation
c. 1 in 1100 d. Incorrect uteroovarian ligament ligation
d. 1 in 2000 39–16. With Filshie clip application or nonpuerperal
emale tubal sterilization, which o the ollowing
39–14. Puerperal sterilization ailures result most commonly is a possible reason or method ailure with this
rom which o the ollowing? technique?
a. Ligation o incorrect structure a. Tubal stump istula
b. Postoperative acute salpingitis b. Electrosurgical circuit ailure
c. Use o absorbable suture or ligation c. Intercourse too soon a ter the procedure
d. Intercourse too soon ollowing ligation d. Surgery scheduled in the ollicular phase

39–17. I pregnancy occurs ollowing bilateral bipolar


tubal electrosurgical coagulation, what is the rate o
ectopic tubal implantation?
a. 10%
b. 30%
c. 50%
d. 75%
Steriliza tion 263

39–18. Your patient is a 37-year-old G4P4 who underwent 39–21. Your 37-year-old patient underwent bilateral tubal
puerperal sterilization 14 months ago and now bipolar coagulation or sterilization 7 years ago.
presents or her annual examination. She has a Review o her operative report reveals that you

C
history o polycystic ovarian syndrome with irregular coagulated three contiguous spots along a 3-cm

H
menses and notes that her last menses was 7 weeks length that incorporated most o the tube’s isthmic

A
P
ago. Appropriate management should irst be which portion. Now in a new relationship, she presents

T
o the ollowing? desiring counseling regarding reestablishment

E
R
a. Transvaginal sonographic examination o ertility. Which o the ollowing is a correct

3
counseling statement regarding tubal reanastomosis?

9
b. Serum human chorionic gonadotropin level
assessment a. Highest pregnancy rates ollow ampullary-to-
isthmic reanastomosis.
c. Initiation o oral contraceptives or menstrual
regulation b. Almost 10 percent o women who conceive
ollowing reanastomosis will have an ectopic
d. Medroxyprogesterone 10 mg daily or 10 days to
pregnancy.
initiate a withdrawal menses
c. Reanastomosis a ter tubal electrosurgical
39–19. Which o the ollowing conditions is increased in coagulation has higher pregnancy rates than
women ollowing this procedure compared with reanastomosis ollowing Filshie clip application.
those not undergoing this surgery? d. All o the above

39–22. This device achieves sterilization by which o the


ollowing methods?

a. Lies within the cervical canal to secrete


spermicide
b. Wraps around the allopian tube to occlude the
Used with permission rom Dr. Michelle Medel. lumen
c. Is placed within the uterine cavity to agglutinate
a. Risk o ovarian cancer the endometrium
b. Incidence o menorrhagia d. Is placed within the allopian tube ostia to
c. Risk o pelvic in lammatory disease promote occlusive tissue ingrowth
d. None o the above
39–23. According to Food and Drug Administration
recommendations, which o the ollowing imaging
39–20. Studies support that tubal ligation is a risk or which
procedures is per ormed a ter the Essure method o
o the ollowing psychological or sexual side e ects?
tubal sterilization to document procedure success?
a. Regret
a. Hysterosalpingography 6 weeks a ter surgery
b. Decreased libido b. Saline in usion sonography 3 months a ter
c. Decreased sexual satis action surgery
d. All o the above c. Three-dimensional transvaginal sonography
8 weeks a ter surgery
d. None o the above
264 The Puerperium

39–24. All EXCEPT which o the ollowing are true 39–29. Most vasectomy ailures occur during the irst year
regarding the Essure method o tubal sterilization? ollowing the procedure. The cumulative ailure rate
a. Requires tissue ingrowth or method success per 1000 procedures at years 2, 3 and 5 is stable and
S
approximates which o the ollowing?
b. Requires hysterosalpingography 3 months
E
C
postprocedure a. 10
T
I
b. 50
O
c. Requires proximal tubal thermal injury to incite
N
tissue ingrowth c. 80
1
d. Requires alternative method o contraception
0
d. 125
until tubal occlusion is documented
39–30. Vasectomy ailures may result rom which o the
39–25. Pregnancy ollowing the Essure method o ollowing?
transcervical tubal sterilization may result rom a. Recanalization
which o the ollowing?
b. Incomplete surgical occlusion
a. Insert expulsion
c. Intercourse too soon a ter the procedure
b. Misinterpretation o hysterosalpingogram
d. All o the above
c. Noncompliance with required
hysterosalpingogram 39–31. A long-term complication ollowing vasectomy
d. All the above includes which o the ollowing?
a. Regret
39–26. During vasectomy, which o the ollowing structures
is ligated? b. Atherogenesis
a. Epididymis c. Testicular cancer
b. Spermatic cord d. All o the above
c. Ductus de erens 39–32. Pregnancy rates a ter vasectomy reversal increase
d. E erent ductile with all EXCEPT which o the ollowing?
a. Microsurgical technique
39–27. Compared with vasectomy, which o the ollowing is
higher with emale tubal sterilization? b. Younger emale partner age
a. Cost c. Longer time duration rom vasectomy to reversal
b. Failure rate d. Normal sperm quality noted during reversal
procedure
c. Surgical complication rate
d. All o the above

39–28. To avoid conception ollowing vasectomy, an


alternative orm o contraception should be used
until semen analysis documents aspermia. Complete
sperm expulsion rom the reproductive tract takes
approximately how long?
a. 1 week
b. 4 weeks
c. 8 weeks
d. 12 weeks
Steriliza tion 265

CHAPTER 39 ANSw ER KEy

Q uestion Letter Pa ge

C
number a nswer cited Hea der cited

H
A
P
39–1 d p. 720 Fema le Steriliza tion

T
E
39–2 c p. 720 Puerpera l Tuba l Steriliza tion

R
3
39–3 b p. 720 Puerpera l Tuba l Steriliza tion

9
39–4 a p. 720 Puerpera l Tuba l Steriliza tion
39–5 a p. 720 Surgica l Technique
39–6 c p. 720 Surgica l Technique
39–7 a p. 720 Surgica l Technique
39–8 c p. 721 Figure 3 9 -1
39–9 a p. 721 Figure 3 9 -2
39–10 b p. 721 N onpuerpera l (Interva l) Surgica l Tuba l Steriliza tion
39–11 b p. 721 N onpuerpera l (Interva l) Surgica l Tuba l Steriliza tion
39–12 a p. 721 N onpuerpera l (Interva l) Surgica l Tuba l Steriliza tion
39–13 a p. 721 Contra ceptive Fa ilure
39–14 a p. 721 Contra ceptive Fa ilure
39–15 b p. 721 Contra ceptive Fa ilure
39–16 a p. 721 Contra ceptive Fa ilure
39–17 c p. 721 Contra ceptive Fa ilure
39–18 b p. 721 Contra ceptive Fa ilure
39–19 d p. 722 O ther Effects
39–20 a p. 722 O ther Effects
39–21 b p. 722 Reversa l of Tuba l Steriliza tion
39–22 d p. 722 Intra tuba l Devices
39–23 d p. 722 Intra tuba l Devices
39–24 c p. 722 Intra tuba l Devices
39–25 d p. 722 Intra tuba l Devices
39–26 c p. 723 Ma le Steriliza tion
39–27 d p. 723 Ma le Steriliza tion
39–28 d p. 723 Ma le Steriliza tion
39–29 a p. 723 Ma le Steriliza tion
39–30 d p. 723 Ma le Steriliza tion
39–31 a p. 724 Long-Term Effects
39–32 c p. 724 Restora tion of Fertility
This page intentionally left blank
Se c t i o n 1 1

Obst et r ic a l c Ompl ic a t iOn s


268

c Ha pt er 40

H v D od

40–1. What proportion o maternal deaths can be 40–5. For the patient in Question 40–4, a sonographic
attributed to hypertensive disorders in pregnancy? examination is per ormed to estimate gestational age.
. 1 in 2 One image is shown below. Which o the ollowing
may explain the development o preeclampsia in this
. 1 in 6
patient?
. 1 in 10
d. 1 in 20

40–2. A patient has a blood pressure o 110/72 mm Hg


on her irst prenatal visit at 8 weeks’ gestation. She
develops hypertension in the third trimester, and
at delivery, her blood pressure is 148/94 mm Hg.
Urine protein by dipstick is trace, her creatinine level
is 0.76 mg/dL, and her hypertension has resolved
by the time o her hospital discharge. What is her
correct diagnosis?
. Preeclampsia
. Chronic hypertension
. Gestational hypertension
d. Superimposed preeclampsia

40–3. A patient with antepartum baseline blood pressure


. Increased volume o chorionic villi
measurements o 90/65 mm Hg has blood pressures
o 130–140/80–86 mm Hg at delivery. She has an . Extensive remodeling o the spiral arterioles
increased risk o which o the ollowing obstetric . Increased invasion o extravillous trophoblastic
complications? tissue
. Eclampsia d. None o the above
. Placental abruption
40–6. What is the underlying etiology or proteinuria that
. Nonreassuring etal heart rate tracing
is seen with preeclampsia?
d. None o the above
. Increased capillary permeability
40–4. A multiparous patient who has received no prenatal . Increased renal artery resistance
care presents to Labor and Delivery with a complaint . Increased glomerular iltration rate
o vaginal bleeding. Her undal height is 24 cm. d. Increased systemic vascular resistance
Which o the ollowing laboratory tests supports the
diagnosis o preeclampsia? 40–7. All EXCEPT which o the ollowing increase a
. Creatinine 1.14 mg/dL woman’s predisposition to develop preeclampsia
. Platelet count 103,000/µL syndrome?
. Alkaline phosphatase 138 IU/L . Obesity
d. Total protein o 258 mg in a 24-hour urine . Smoking
collection . Nulliparity
d. Multiple gestation
Hypertensive Disorders 269

40–8. What is a possible explanation or the increased 40–11. Which o the ollowing physiological responses is
incidence o preeclampsia seen in patients whose typically seen in preeclamptic patients?
pregnancies are complicated by the aneuploidy . Increased production o nitric acid

c
re lected by the karyotype results shown here?
. Decreased reactivity to norepinephrine

H
. Increased antiangiogenic actor levels

a
. Increased sensitivity to angiotensin II

p
. Increased requency o placental mosaicism

t
d. All o the above

e
r
. Higher requency o spiral arteriole atherosis

4
d. Increased levels o oxidative products in the

0
placenta

40–9. Shown here, macrophages, which contribute to 40–12. The typical blood volume o a gravida at term is
vessel atherosis, are illed with what substance? 4500 mL. In patients with preeclampsia, which o
the ollowing would be the expected blood volume?
. 2500 mL
. 3200 mL
. 4500 mL
d. 5000 mL

40–13. In patients with preeclampsia, limited blood volume


expansion during pregnancy a ects maternal cardiac
unction by which mechanism?
. Decreases preload
. Increases a terload
. Increases stroke volume
d. Decreases cardiac output
Modif ed with permission rom Cunningham FG, Leveno KJ, Bloom SL, et al (eds):
Hypertensive disorders. In Williams Obstetrics, 24th ed. New York, McGraw-Hill, 2014,
Figure 40-3A. 40–14. A neonate is born vaginally a ter 6 hours o labor
and a ew minutes o pushing. He has petechiae
. Lipids
covering his scalp and chest, and his pediatric nurse
. Interleukins notices oozing at the site o a heel stick. His initial
. Nitric oxide platelet count is 32,000/µL. Which o the ollowing
d. Tumor necrosis actor-α maternal hypertensive conditions predisposes to this
neonatal condition?
40–10. Which o the ollowing nutritional supplements has . Eclampsia
been shown to reduce the incidence o preeclampsia?
. Preeclampsia
. Calcium . Gestational hypertension
. Vitamin E d. None o the above
. Ascorbic acid
d. None o the above
270 O bstetrica l Complica tions

40–15. Which o the ollowing is responsible or the 40–17. Which o the ollowing leads to increased uric acid
clinical sign o preeclampsia seen in the ollowing levels in patients with preeclampsia?
photograph? . Increased tubular reabsorption
s
. Increased placental production
e
c
t
. Decreased glomerular iltration rate
i
O
d. All o the above
n
1
1
40–18. Your obstetrical patient presents with a blood
pressure o 160/104 mm Hg, 3+ proteinuria,
and right upper quadrant discom ort at 36 weeks’
gestation. Following induction o labor, she delivers
vaginally. She has uterine atony, and her estimated
blood loss is 1500 mL. Her serum creatinine rises
rom 0.98 mg/dL predelivery to 1.42 mg/dL. What
is the most likely explanation or this inding?
. Postpartum hemorrhage
. Severe preeclampsia alone
. Subcapsular liver hematoma
d. Dehydration secondary to prolonged induction

40–19. The ollowing computed-tomography image is rom


a study obtained on a postpartum hypertensive
patient with con usion. Cerebral edema was
diagnosed. For what associated morbidity is the
patient at risk?
Reproduced with permission rom Cunningham FG, Leveno KJ, Bloom SL, et al (eds):
Hypertensive disorders. In Williams Obstetrics, 24th ed. New York, McGraw-Hill, 2014,
Figure 40-19A.

. Endothelial injury
. Increased sodium retention
. Reduced plasma oncotic pressure
d. All o the above

40–16. A primigravida delivered 4 hours ago. Her blood


pressure was 152/90 mm Hg be ore delivery,
and 1+ proteinuria was ound by dipstick. Her
delivery was unevent ul, and her estimated blood
loss was 500 mL. You get a call rom her nurse
because her urine output or the past 4 hours is
only 118 mL. Her BP is 148/88, pulse is 84, she
has 12 respirations per minute, and no evidence o
ongoing bleeding is noted. Which o the ollowing
. Hemiplegia
treatment options is most appropriate or this
patient? . Cystic leukomalacia
. Continue observation . Retinal artery occlusion
. Give 10 mg intravenous urosemide d. Transtentorial herniation
. Trans use 2 units o packed red blood cells
d. Give 500 mL bolus o intravenous normal saline
Hypertensive Disorders 271

40–20. All EXCEPT which o the ollowing are indicated 40–26. What is the concern surrounding use o
treatments in the management o the patient corticosteroids to enhance etal lung maturation in
discussed in Question 40–19? women with severe hypertension prior to 34 weeks’

c
. Mannitol gestation?

H
. It exacerbates maternal hypertension.

a
. Dexamethasone

p
. Time needed or corticosteroid administration

t
. Intravenous immune globulin

e
may delay delivery.

r
d. Antihypertensive medication

4
. It may trigger eclampsia in women with a genetic

0
40–21. In a low-risk population, treatment with which o predisposition to seizures.
the ollowing medications resulted in a reduced d. It is associated with an increased rate o neonatal
incidence o preeclampsia? intraventricular hemorrhage.
. Aspirin
40–27. What salutary e ect does dexamethasone possibly
. Pravastatin have when used in the treatment o HELLP
. Hydrochlorothiazide (hemolysis, elevated liver enzymes, low platelet
d. None o the above count) syndrome?
. Decreased maternal mortality rate
40–22. Your obstetrical patient is admitted to the hospital . Decreased rate o acute renal ailure
or evaluation o new-onset hypertension at
30 weeks’ gestation. All EXCEPT which o the . Faster aspartate trans erase recovery time
ollowing should be part o your evaluation? d. Increased platelet count in severe
. Maternal weight thrombocytopenia
. Cell- ree etal DNA testing 40–28. Which o the ollowing explains the indings seen in
. Fetal sonographic evaluation the chest radiograph below, obtained rom a patient
d. Maternal urine protein:creatinine ratio with eclampsia?

40–23. The patient in Question 40–22 has blood pressures


o 140–150/85–100 mm Hg during the next 5 days.
Which o the ollowing should prompt consideration
or premature delivery?
. Headache
. Worsening pedal edema
. 3+ proteinuria on dipstick
d. Fetal biophysical pro ile score o 8

40–24. Three days a ter admission, the patient in Question


40–23 develops severe preeclampsia and delivery
is indicated. Sonographic evaluation reveals a
cephalic presentation and estimated etal weight o
1405 g. I labor induction is attempted, what is the
approximate rate o success ul vaginal delivery?
. 10% Reproduced with permission rom Emerman CL, Anderson E, Cline DM: Community-
acquired pneumonia, aspiration pneumonia, and nonin ectious pulmonary inf ltrates. In
. 30% Tintinalli JE, Stapczynski JS, Cline DM, et al (eds): Tintinalli’s Emergency Medicine:
. 50% A Comprehensive Study Guide, 7th ed. New York, McGraw-Hill, 2011, Figure 68-4.

d. 80%
. Pneumothorax
40–25. In studies evaluating the antenatal use o labetalol or . Pulmonary edema
treatment o early mild preeclampsia, which o the . Pulmonary embolus
ollowing is reduced?
d. Aspiration pneumonia
. Blood pressure
. Fetal-growth restriction
. Length o inpatient hospitalization
d. All o the above
272 O bstetrica l Complica tions

40–29. An eclamptic patient who has received a 4-g loading 40–33. A pregnant patient has a seizure at home and is
dose o magnesium sul ate has another seizure. evaluated by an emergency room physician. He
Which o the ollowing medications can be given as consults with a neurologist who, a ter excluding
s
adjuvant anticonvulsant therapy? other etiologies or the seizure, makes the diagnosis
e
c
. Midazolam o eclampsia. The neurologist recommends
t
phenytoin or eclampsia prophylaxis to the on-call
i
. Thiopental
O
obstetrician. What is the best response to this
n
. Additional magnesium sul ate recommendation?
1
1
d. All o the above . Agree and load the patient with phenytoin
. Give intravenous loading dose o magnesium
40–30. What is the target magnesium level when used or sul ate and oral phenytoin
eclampsia prophylaxis?
. Explain there is a reduction o recurrent seizure
. 2.0–3.5 mg/dL activity with magnesium sul ate and start
. 4.8–8.4 mg/dL magnesium sul ate
. 8.4–10.4 mg/dL d. None o the above
d. None o the above
40–34. Antenatal use o nitroglycerin to control severe
40–31. What clinical sign or test can be used to detect maternal hypertension can lead to which o the
hypermagnesemia prior to development o ollowing complications?
respiratory depression? . Fetal acidosis
. Heart rate . Fetal oliguria
. Patellar re lex . Fetal cyanide toxicity
. Presence o clonus d. Reduced etal heart rate variability seen during
d. Visual ield testing monitoring

40–32. Which o the ollowing strategies or administering 40–35. Use o hydroxyethyl starch to expand intravascular
magnesium sul ate or eclampsia prophylaxis volume improves which o the ollowing pregnancy
should be used in the setting o an elevated serum outcomes in women with preeclampsia?
creatinine? . Eclampsia
. Give no loading dose and start in usion at 2 g/hr . Fetal death rate
. Give 3-g loading dose ollowed by 2 g/hr in usion . Gestational age at delivery
. Give 3-g loading dose, check magnesium level, d. None o the above
and then titrate in usion rate
d. None o the above 40–36. Preeclampsia is a marker or all EXCEPT which o
the ollowing morbidities later in li e?
. Metabolic syndrome
. Chronic renal disease
. Ischemic heart disease
d. Nonalcoholic steatohepatitis
Hypertensive Disorders 273

c Ha pt er 40 a n sw er Key

Q uestion Letter Pa ge

c
number a nswer cited Hea der cited

H
a
p
40–1 p. 728 Introduction

t
e
40–2 p. 728 Terminology a nd Dia gnosis

r
4
40–3 d p. 728 Dia gnosis of Hypertensive Disorders

0
40–4 p. 729 Ta ble 4 0 -1
40–5 p. 731 Etiopa thogenesis; Figure 4 0 -2
40–6 p. 731 Preecla mpsia Syndrome
40–7 p. 731 Preecla mpsia Syndrome
40–8 p. 733 Immunologica l Fa ctors
40–9 p. 733 Figure 4 0 -3
40–10 d p. 734 N utritiona l Fa ctors
40–11 p. 735 Increa sed Pressor Responses
40–12 p. 737 Blood Volume; Figure 4 0 -7
40–13 p. 736 Ca rdiova scula r System
40–14 d p. 738 N eona ta l Thrombocytopenia
40–15 d p. 739 Endocrine Cha nges; Fluid a nd Electrolyte Cha nges
40–16 p. 739 Kidney
40–17 d p. 739 Kidney
40–18 p. 740 Acute Kidney Injury
40–19 d p. 744 N euroima ging Studies
40–20 p. 745 Cerebra l Edema
40–21 d p. 748 Antihypertensive Drugs; Low-Dose Aspirin
40–22 p. 749 Eva lua tion
40–23 p. 750 Considera tion for Delivery
40–24 p. 750 Elective Cesa rea n Delivery
40–25 p. 752 Antihypertensive Thera py for Mild to Modera te Hypertension
40–26 p. 754 G lucocorticoids for Lung Ma tura tion
40–27 d p. 754 Corticosteroids to Ameliora te HELLP Syndrome
40–28 d p. 756 Immedia te Ma na gement of Seizure
40–29 d p. 758 Ma gnesium Sulfa te to Control Convulsions
40–30 p. 758 Ta ble 4 0 -1 1
40–31 p. 759 Pha rma cology a nd Toxicology
40–32 d p. 759 Pha rma cology a nd Toxicology
40–33 p. 760 Ma terna l Sa fety a nd Effica cy of Ma gnesium Sulfa te
40–34 p. 763 O ther Antihypertensive Agents
40–35 d p. 764 Ta ble 4 0 -1 3
40–36 d p. 769 Long-term Consequences
274

CHAPTER 41

Obstetrical Hemorrhage

41–1. Which of the following is true of obstetrical 41–3. For a woman measuring 5 ft 0 in. and 120 lb, what
hemorrhage? is her expected pregravid blood volume?
a. It is the leading cause of maternal death a. 3000 mL
worldwide. b. 3250 mL
b. It is no longer a leading cause of maternal death c. 3500 mL
in the United States.
d. 3800 mL
c. Only 5% of maternal deaths in the United States
are due to obstetrical hemorrhage. 41–4. Assuming a 50% increase in blood volume during
d. All of the above normal pregnancy, what would the blood volume
of the woman from Question 41–2 approximate at
41–2. Given the diagram shown here, what can be said 38 weeks’ gestation?
regarding blood loss at delivery? a. 4000 mL
a. It is common to lose more than 500 mL with b. 4500 mL
vaginal delivery.
c. 5250 mL
b. It is common to lose more than 1000 mL with
d. 5700 mL
cesarean delivery.
c. It is common to lose more than half of maternal
blood volume with cesarean hysterectomy.
d. All of the above

70
< 500 mL
60
500–1000 mL
50 1000–1500 mL
s
e
> 2500 mL
s
a
40
c
f
o
t
n
30
e
c
r
e
P
20

10

0
Va gina l Re pe a t ce s a re a n Re pe a t ce s a re a n
de live ry de live ry with hys te re ctomy

Reproduced with permission from Cunningham FG, Leveno KJ, Bloom SL, et al (eds): Maternal physiology. In Williams
Obstetrics, 24th ed. New York, McGraw-Hill, 2014, Figure 41-1.
O bstetrica l Hemorrha ge 275

41–5. Midtrimester bleeding is associated with which of 41–10. For Ms. Jones, the patient from Question 41–8, one
the following? dose of a uterotonic agent is given and the fundus
a. Placenta previa is massaged. Despite this, she continues to bleed.

C
Which of the following is suitable treatment in this
b. Placental abruption

H
situation?

A
c. Higher rates of adverse pregnancy outcomes

P
a. Administer Methergine, 0.2 mg intramuscularly

T
d. All of the above

E
b. Perform laparotomy to prepare for postpartum

R
4
hysterectomy
41–6. What might delay a practitioner from recognizing

1
dangerous postpartum hemorrhage? c. Mobilize a team that includes obstetricians,
nurses, and anesthesiologists.
a. Initial catecholamine release after hemorrhage
d. All of the above
b. A normal blood pressure in a severely
preeclamptic woman
41–11. For Ms. Jones, the patient from Question 41–8,
c. Persistent light bleeding during fourth-degree you have provided general anesthesia, evaluated the
vaginal laceration repair genital tract again, and administered three doses
d. All of the above of Hemabate. Despite this, she continues to bleed,
and you have initiated whole blood transfusion.
41–7. In treating uterine atony after delivery, which of the Which of the following is suitable treatment in this
following is true? situation?
a. Fundal massage should be performed. a. Consider laparotomy and uterine compression
b. A 20-unit oxytocin bolus should be administered suture placement
intravenously. b. Continue to administer Hemabate
c. Oxytocin diluted in a crystalloid solution intramuscularly every 20 minutes
at a concentration of 200 U/min should be c. Insert a Bakri postpartum balloon or large Foley
administered intravenously. catheter balloon into the uterine cavity and inflate
d. All of the above the balloon
d. All of the above
41–8. Ms. Jones, a 32-year-old G3P2 gravida with
chronic hypertension, had a normal labor that 41–12. For the patient in Question 41–8, despite Bakri
arrested during the second stage at + 1 station. She balloon placement, heavy uterine bleeding continues.
complained of mild dyspnea and fatigue. The fetus You perform a laparotomy and find an atonic uterus.
had a left occiput anterior presentation and was Which of the following is suitable treatment in this
delivered by forceps. Completion of the third stage situation?
followed quickly, and the fundus was noted to be a. Internal iliac artery ligation
firm. Brisk vaginal bleeding was then noted. What is
b. Uterine compression suture placement
the most likely cause of bleeding?
c. Bilateral uterine artery and uteroovarian pedicle
a. Uterine atony
ligation
b. Uterine rupture
d. All of the above
c. Retained placenta
d. Genital tract laceration 41–13. For the patient in Question 41–8, despite these
measures, Ms. Jones continues to bleed now from
41–9. The genital tract of the patient from Question 41–8 all surgical edges, and the uterus remains atonic.
was carefully examined, and no lacerations were Which of the following is suitable treatment in this
noted. Prophylactic oxytocin at 200 mU/min was situation?
being administered. The examiner then noted that a. Initiate hysterectomy
although the fundus felt firm, the lower uterine
b. Administer 20-unit oxytocin intravenous bolus
segment was boggy. Which of the following is
suitable treatment in this situation? c. Evaluate for dilutional coagulopathy and continue
uterine compression
a. Hemabate, 250 µg intramuscularly
d. All of the above
b. Methergine, 0.2 mg intramuscularly
c. A 20-unit oxytocin intravenous bolus
d. All are suitable
276 O bstetrica l Complica tions

41–14. For the patient in Question 41–8, after correction 41–15. Which of the following is suitable treatment for this
of coagulopathy, bleeding stops and the uterus situation?
firms. Postoperatively, Ms. Jones is admitted to the
S
intensive care unit, where she is slowly weaned from
E
C
the ventilator. She is discharged on postoperative day
T
4. What condition, represented by the histological
I
O
image of lung shown here, is consistent with the
N
clinical events for Ms. Jones?
1
1
Reproduced with permission from Cunningham FG, Leveno KJ, Bloom SL, et al (eds):
Dermatological disorders. In Williams Obstetrics, 23rd ed. New York, McGraw-Hill,
2010, Figure 35-23A.

a. Immediate recognition and calls for assistance


improve outcome.
b. If recognized quickly, fundal massage and
a. Pulmonary embolism uterotonic agents are initiated.
b. Amnionic fluid embolism c. The patient is evaluated for regional anesthesia,
c. Primary pulmonary hypertension large-bore intravenous access is established, and
rapid crystalloid infusion is begun while you wait
d. None of the above for blood to arrive.
d. All of the above

41–16. Which of the following statements concerning


uterine inversion are true?
a. Placenta accreta is a risk factor.
b. Fundal placental implantation is a risk factor.
c. It has an incidence that approximates 1 in 2000
deliveries.
d. All of the above
O bstetrica l Hemorrha ge 277

41–17. Your patient delivered precipitously and without 41–20. Which of the following regarding placental
perineal laceration. During the first hour abruption are true?
postpartum, the vulvar mass shown here continues a. Classically, it is associated with pain and vaginal

C
to expand, and the patient complains of significant bleeding.

H
pain. Evaluation reveals a BP of 90/40 mm Hg,

A
b. It is the most frequent cause of clinically

P
pulse of 120 bpm, and no fever. Which of the significant consumptive coagulopathy.

T
following are reasonable management approaches to

E
c. Abruption causing fetal death is usually associated

R
the condition depicted here?

4
with blood loss that is equivalent to half of

1
maternal blood volume.
d. All the above

41–21. With placental abruption, which condition(s) would


preclude vaginal delivery?
a. Intrauterine fetal demise and prior classical
cesarean hysterotomy
b. Term fetus at + 2 station, brisk vaginal bleeding,
and mild coagulopathy
c. Intrauterine fetal demise and a herpes simplex
virus ulcer on the maternal perineum
d. All of the above

41–22. Which of the following statements is true concerning


placenta previa?
a. Ice packs and observation are planned. a. A low-lying placenta at 27 weeks’ gestation is
unlikely to persist until term.
b. This large supralevator hematoma may require
angiographic embolization. b. Persistent placenta previa is more common in
women with prior cesarean deliveries.
c. This likely connects with the ischioanal fossa, and
laparotomy is planned to exclude or evacuate a c. Approximately 40% of placenta previas that cover
large retroperitoneal hematoma. the internal cervical os at 20 weeks’ gestation will
persist until term.
d. The point of maximal expansion is incised, clots
are evacuated, bleeding points are ligated, and the d. All of the above
evacuated space is obliterated by sutures.
41–23. Placentas that lie within close proximity of the
41–18. All EXCEPT which of the following are associated internal cervical os but do not reach it are termed
with primary uterine rupture? low lying. What is the boundary threshold that
defines a low-lying placenta?
a. Hydramnios
a. 1.0 cm
b. Forceps delivery
b. 2.0 cm
c. Breech extraction
c. 3.0 cm
d. Prior cesarean delivery
d. 4.0 cm
41–19. Which of the following has the highest associated
relative risk for placental abruption? 41–24. The incidence of placenta previa increases with
which of the following factors?
a. Thrombophilia
a. Increasing parity
b. Prior abruption
b. Increasing maternal age
c. Chorioamnionitis
c. Increasing number of cesarean deliveries
d. Preterm ruptured membranes
d. All of the above
278 O bstetrica l Complica tions

41–25. Histologically, which of the following best describes 41–27. Using Doppler flow color mapping, as shown here,
the villi found at placenta increta sites? which two factors are highly predictive of placenta
a. Invade into the myometrium accreta? B = bladder.
S
b. Attached to the myometrium
E
C
T
c. Attached to the endometrium
I
O
d. Penetrate through the myometrium
N
1
1
41–26. This picture represents which of the following?

Used with permission from Dr. Christopher Ripperda.

a. Large intraplacental lakes; fundal placenta


b. Less than 1 mm between uterine serosa-bladder
interface and retroplacental vessels; lateral
placenta
a. Placenta accreta c. Less than 1 mm between uterine serosa-bladder
b. Placenta increta interface and retroplacental vessels; large placental
lakes
c. Placenta percreta
d. None of the above
d. Placental abruption
41–28. Management of placenta accreta typically requires
which of the following procedures?
a. Classical cesarean; hysterectomy
b. Low transverse cesarean; hysterectomy
c. Classical cesarean; myometrial resection
d. Low vertical cesarean; myometrial resection
O bstetrica l Hemorrha ge 279

41–29. Which obstetrical conditions can lead to significant 41–33. With ongoing obstetrical hemorrhage, rapid blood
consumptive coagulation? transfusion is typically initiated at which hematocrit
a. Placental abruption threshold?

C
b. Amnionic fluid embolism a. 20%

H
A
c. Gram-negative bacterial sepsis b. 25%

P
T
d. All of the above c. 30%

E
R
d. 35%

4
41–30. Above which threshold are serum fibrinogen levels

1
considered adequate to promote coagulation? 41–34. The ideal treatment of hypovolemia from
a. 50 mg % catastrophic hemorrhage is which of the following?
b. 150 mg % a. Whole blood
c. 250 mg % b. Packed red blood cells
d. 400 mg % c. Whole blood and platelets
d. Packed red blood cells and plasma
41–31. One of the most important vital signs with
obstetrical hemorrhage is which of the following? 41–35. Morbidity from volume replacement with only
a. Urine output packed red blood cells and crystalloid infusion would
include which of the following?
b. Blood pressure
a. Thrombocytopenia
c. Oxygen saturation
b. Hypofibrinogenemia
d. Point-of-care hematocrit
c. Dilutional coagulopathy
41–32. Acute resuscitation of hypovolemia is preferably d. All of the above
done with which of the following?
a. Colloids and ephedrine 41–36. Each unit of packed red blood cells raises the
hematocrit by what amount?
b. Crystalloids and ephedrine
a. 2–3%
c. Crystalloids and packed red blood cells
b. 3–4%
d. Colloid solutions and packed red blood cells
c. 4–5%
d. 5–6%
280 O bstetrica l Complica tions

CHAPTER 41 ANSw ER KEy

Q uestion Letter Pa ge
S
number a nswer cited Hea der cited
E
C
T
41–1 a p. 780 Introduction
I
O
41–2 d p. 781 Definition a nd Incidence
N
1
41–3 a p. 781 Ta ble 4 1 -1
1
41–4 b p. 781 Ta ble 4 1 -1
41–5 d p. 782 Antepa rtum Hemorrha ge
41–6 d p. 783 Blood Loss Estima tion
41–7 a p. 784 Uterine Atony a fter Pla centa l Delivery
41–8 d p. 784 Ca uses of O bstetrica l Hemorrha ge
41–9 a p. 785 Uterotonic Agents
41–10 c p. 786 Bleeding Unresponsive to Uterotonic Agents
41–11 d p. 786 Bleeding Unresponsive to Uterotonic Agents
41–12 d p. 787 Surgica l Procedures
41–13 c p. 787 Surgica l Procedures
41–14 b p. 782 Ta ble 4 1 -2 ; Amnionic-Fluid Embolism
41–15 a p. 787 Recognition a nd Ma na gement
41–16 d p. 787 Uterine Inversion
41–17 d p. 790 Ma na gement
41–18 d p. 790 Rupture of the Uterus
41–19 b p. 795 Predisposing Fa ctors
41–20 d p. 796 Clinica l Findings a nd Dia gnosis
41–21 a p. 798 Va gina l Delivery
41–22 d p. 799 Pla centa l Migra tion
41–23 b p. 800 Cla ssifica tion
41–24 d p. 801 Incidence a nd Associa ted Fa ctors
41–25 a p. 804 Cla ssifica tion
41–26 b p. 804 Cla ssifica tion
41–27 c p. 806 Clinica l Presenta tion a nd Dia gnosis
41–28 a p. 807 Cesa rea n Delivery a nd Hysterectomy
41–29 d p. 809 Pa thologica l Activa tion of Coa gula tion
41–30 b p. 810 Fibrinogen a nd Degra da tion Products
41–31 a p. 814 Ma na gement of Hemorrha ge
41–32 c p. 815 Fluid Resuscita tion
41–33 b p. 815 Blood Repla cement
41–34 a p. 815 Blood Component Products
41–35 d p. 816 Dilutiona l Coa gulopa thy
41–36 b p. 816 Pa cked Red Blood Cells
281

CHAPTER 42

Preterm Labor

42–1. he term small-for-gestational age is generally used to 42–5. A ter achieving a birthweight o at least 1000 grams,
designate newborns whose birthweight is less than neonatal survival rates reach 95 percent at
what percentile? approximately what gestational age with regard to
a. 3% newborn sex?
b. 5% a. 28 weeks or both males and emales
c. 10% b. 30 weeks or both males and emales
d. 15% c. 28 weeks or emales and 30 weeks or males
d. 30 weeks or emales and 28 weeks or males
42–2. he neonatal mortality rate is expected to be
lowest or newborns born at which o the ollowing 42–6. Cesarean delivery or neonates born at the threshold
gestational ages? o viability has been demonstrated to protect against
a. 36 weeks 6 days which o the ollowing adverse newborn outcomes?
b. 37 weeks 4 days a. Seizures
c. 39 weeks 6 days b. Intraventricular hemorrhage
d. 41 weeks 2 days c. Respiratory distress syndrome
d. None o the above
42–3. Late-preterm births, de ined as those between 34 and
36 weeks’ gestation, compose what percentage o all 42–7. Compared with neonates born at term, the risks
preterm births? to those born between 34 and 36 weeks’ gestation
a. 35% include which o the ollowing?
b. 50% a. Increased serious morbidity and mortality rates
c. 70% b. Equivalent serious morbidity and mortality rates
d. 85% c. Increased serious morbidity but decreased
mortality rates
42–4. Which o the ollowing etiologies is largely d. Increased serious morbidity but equivalent
responsible or the increase in preterm birth rates in mortality rates
the United States during the past 20 years?
a. riplet pregnancies 42–8. Maternal stress may potentiate preterm labor by
which o the ollowing mechanisms involving
b. Spontaneous preterm labor corticotropin-releasing hormone (CRH)?
c. Preterm rupture o etal membranes a. Increased production o maternal-derived CRH
d. Indicated (iatrogenic) preterm birth b. Decreased production o maternal-derived CRH
c. Increased production o placental-derived CRH
d. Decreased production o placental-derived CRH
282 O bstetrica l Complica tions

42–9. A 26-year-old G2P1 presents at 29 weeks’ gestation 42–11. A 24-year-old G2P1 at 6 weeks’ gestation presents
complaining o leaking clear luid rom her vagina. or prenatal care and complains o bleeding, pain ul
A speculum examination reveals scant pooling o gums. Her obstetric history is signi icant or two
S
luid in the posterior vagina, and the microscopic prior preterm births. An oral examination reveals the
E
C
analysis o the luid reveals the ollowing pattern. indings noted in the image below. You counsel her
T
You diagnose premature rupture o the etal that periodontal disease treatment in pregnancy has
I
O
membranes (PROM). O the known risk actors or been proven to have which o the ollowing avorable
N
this condition, which is most commonly identi ied outcomes?
1
1
in such patients?

Reproduced with permission rom Gonsalves WC, Usatine RP: Gingivitis and Periodontal
Reproduced with permission rom Birnbaumer DM: Microscopic f ndings. In Knoop KJ, Disease. In Usatine RP, Smith MA, Chumley H, et al (eds): T e Color Atlas o Family
Stack LB, Storrow AB, et al (eds): T e Atlas o Emergency Medicine, 3rd ed. New York, Medicine. New York, McGraw-Hill, 2009, Figure 38-1.
McGraw-Hill, 2010, Figure 25.24.

a. Improved periodontal health


a. Smoking b. Decreased rates o preterm birth
b. Low socioeconomic status c. Decreased rates o low birthweight
c. Prior pregnancy complicated by PROM d. All o the above
d. None o the above
42–12. Intervals shorter than how many months between
42–10. All EXCEPT which o the ollowing li estyle actors pregnancies have been associated with an increased
has been identi ied as an antecedent or preterm risk or preterm birth?
labor? a. 18
a. Frequent coitus b. 24
b. Illicit drug use c. 36
c. Young maternal age d. 48
d. Inadequate maternal weight gain
42–13. A 33-year-old G2P2 is contemplating pregnancy but
is hesitant since her two prior deliveries occurred at
28 and 29 weeks’ gestation, respectively. You in orm
her that her risk or a recurrent preterm birth less
than 34 weeks’ gestation approximates what value?
a. 15%
b. 25%
c. 40%
d. 70%
Preterm La bor 283

42–14. A 22-year-old G2P1 at 14 weeks’ gestation 42–17. Which o the ollowing is true regarding sonographic
complains o a malodorous vaginal discharge. A evaluation o the cervix as a part o the assessment
saline preparation o the discharge is prepared, or preterm birth risk?

C
and indings are illustrated in this image. You

H
recommend antimicrobial treatment or this

A
P
condition or what principal reason?

T
E
R
4
2
a. ransabdominal approach is pre erred to avoid
cervical manipulation.
b. In research populations, women with
a. Resolution o symptoms progressively shorter cervices had increased
b. Prevention o preterm birth preterm labor rates.
c. Avoidance o spontaneous abortion c. Women with prior preterm birth and with
d. reatment o intraamnionic in ection cervical lengths equal to 35 mm will bene it rom
cerclage placement.
42–15. Characteristics o Braxton Hicks contractions can d. All o the above
include all EXCEPT which o the ollowing?
a. Pain ul 42–18. Potential indications to per orm the procedure
demonstrated in this image include which o the
b. Nonrhythmical
ollowing?
c. Irregular pattern
d. Associated with cervical change

42–16. Per ormance o routine cervical examinations at each


prenatal care visit has been demonstrated to e ect
what outcome?
a. Decreased preterm birth rate
b. Increased interventions or preterm labor
c. Increased rate o premature rupture o etal
membranes
d. None o the above

Reproduced with permission rom Cunningham FG, Leveno KJ, Bloom SL, et al (eds):
Abortion. In Williams Obstetrics, 24th ed. New York, McGraw-Hill, 2014, Figure 18-5C.

a. Recurrent midtrimester losses


b. Short cervix identi ied sonographically
c. hreatened preterm labor with cervical dilatation
d. All o the above
284 O bstetrica l Complica tions

42–19. 17-Hydroxyprogesterone caproate has been 42–24. A 25-year-old primigravida at 34 weeks and 5 days’
demonstrated in a randomized, controlled trial to gestation by certain dating criteria is ound to have
decrease the preterm birth rate in women with which preterm rupture o the etal membranes. What is the
S
o the ollowing characteristics? most appropriate management strategy?
E
C
a. Nulliparous a. Expedited delivery
T
I
b. Carrying twins b. Expectant management
O
N
c. Prior preterm birth c. Administer a course o corticosteroids ollowed by
1
delivery
1
d. None o the above
d. Expectant management unless etal lung maturity
42–20. Based on the known natural history o preterm is con irmed
premature ruptured membranes, approximately what
percentage o women will be delivered within 42–25. Corticosteroids administered to women at risk or
48 hours o membrane rupture when this complication preterm birth have been demonstrated to decrease
occurs between 24 and 34 weeks’ gestation? rates o respiratory distress i the birth is delayed or
a. 20% at least what amount o time a ter the initiation o
therapy?
b. 40%
a. 12 hours
c. 70%
b. 24 hours
d. 90%
c. 36 hours
42–21. A 20-year-old primigravida at 18 weeks’ gestation d. 48 hours
presents a ter she noticed a gush o luid rom
her vagina. You con irm the diagnosis o preterm 42–26. When antimicrobials have been administered to
rupture o the etal membranes. Sonographic orestall preterm birth in women with preterm labor,
evaluation con irms anhydramnios. In the unlikely rates o which o the ollowing untoward perinatal
event that she remains undelivered at a viable outcomes have been consistently reduced?
gestational age, perinatal survival would be unlikely a. Neonatal death
because o underdevelopment o what organ system? b. Cerebral palsy
a. Brain c. Chronic lung disease
b. Lungs d. None o the above
c. Heart
d. Kidneys 42–27. Although bed rest is commonly prescribed or
women deemed to be at increased risk or preterm
42–22. What is the only reliable indicator o clinical birth, limited data exist to support a bene it o this
chorioamnionitis in women with preterm rupture o recommendation. Which o the ollowing negative
the etal membranes? outcomes have been reported in pregnant women
placed on bed rest compared with those without this
a. Fever
restriction?
b. Leukocytosis
a. Greater bone loss
c. Fetal tachycardia
b. Impaired etal growth
d. Positive cervical or vaginal cultures
c. Greater maternal weight gain
42–23. Several antibiotic regimens have been used to d. Higher rates o preeclampsia
prolong the latency period in women with preterm
rupture o the etal membranes who are attempting
expectant management. Which antibiotic should be
avoided in this setting because it has been associated
with an increased risk o necrotizing enterocolitis in
the newborn?
a. Ampicillin
b. Amoxicillin
c. Erythromycin
d. Amoxicillin-clavulanate
Preterm La bor 285

42–28. A 21-year-old primigravida presents at 28 42–30. he combination o ni edipine with what other
weeks’ gestation in active preterm labor, and tocolytic agent can potentially cause dangerous
intravenous terbutaline is administered or neuromuscular blockade?

C
tocolysis. Approximately 2 hours a ter therapy a. Atosiban

H
initiation, she begins to cough, and her peripheral

A
b. erbutaline

P
oxygen saturation is noted to be 80 percent. he

T
ollowing chest radiograph is obtained. In which c. Indomethacin

E
R
o the ollowing clinical settings is the risk or this d. Magnesium sul ate

4
complication increased?

2
42–31. A 28-year-old primigravida at 27 weeks’ gestation
presents with regular pain ul uterine contractions,
and her cervix is 8 cm dilated. he etus has a
vertex presentation. he etal heart rate tracing
is reassuring. Which o the ollowing procedures
will help decrease the risk or intraventricular
hemorrhage in her neonate?
a. Episiotomy
b. Cesarean delivery
c. Forceps-assisted vaginal delivery
d. None o the above

42–32. Although the e icacy is somewhat controversial,


intrapartum administration o magnesium sul ate to
women who deliver preterm has been demonstrated
to reduce rates o which o the ollowing neonatal
outcomes?
a. win pregnancy
a. Cerebral palsy
b. Maternal sepsis
b. Necrotizing enterocolitis
c. Concurrent administration o corticosteroids
c. Neonatal seizure activity
d. All o the above
d. Bronchopulmonary dysplasia
42–29. What reversible complication can be seen when
indomethacin is used or tocolysis longer than 24 to
48 hours?
a. Oligohydramnios
b. Placental abruption
c. Neonatal necrotizing enterocolitis
d. Neonatal intraventricular hemorrhage
286 O bstetrica l Complica tions

CHAPTER 42 ANSw ER KEy

Q uestion Letter Pa ge
S
number a nswer cited Hea der cited
E
C
T
42–1 c p. 829 Introduction
I
O
42–2 c p. 829 Definition of Preterm
N
1
42–3 c p. 829 Definition of Preterm
1
42–4 d p. 829 Definition of Preterm
42–5 c p. 832 Morbidity in Preterm Infa nts
42–6 d p. 833 Threshold of Via bility
42–7 a p. 835 La te Preterm Birth
42–8 c p. 837 Ma terna l-Feta l Stress
42–9 d p. 839 Preterm Prema ture Rupture of Membra nes
42–10 a p. 841 Lifestyle Fa ctors
42–11 a p. 841 Periodonta l Disea se
42–12 a p. 841 Interva l between Pregna ncies
42–13 c p. 841 Prior Preterm Birth; Ta ble 4 2 -5
42–14 a p. 842 Ba cteria l Va ginosis
42–15 d p. 842 Symptoms
42–16 d p. 843 Cervica l Cha nge
42–17 b p. 843 Length
42–18 d p. 844 Cervica l Cercla ge
42–19 c p. 844 Prior Preterm Birth a nd Progestin Compounds
42–20 d p. 847 N a tura l History
42–21 b p. 848 Risks of Expecta nt Ma na gement
42–22 a p. 848 Clinica l Chorioa mnionitis
42–23 d p. 848 Antimicrobia l Thera py
42–24 a p. 849 Ma na gement Recommenda tions; Ta ble 4 2 -9
42–25 b p. 850 Corticosteroids for Feta l Lung Ma tura tion
42–26 d p. 851 Antimicrobia ls
42–27 a p. 851 Bed Rest
42–28 d p. 852 Ritodrine
42–29 a p. 852 Prosta gla ndin Inhibitors
42–30 d p. 853 Ca lcium Cha nnel Blockers
42–31 d p. 854 Delivery; Prevention of N eona ta l Intra cra nia l Hemorrha ge
42–32 a p. 854 Ma gnesium for Feta l N europrotection
287

CHAPTER 43

Postterm Pregnanc

43–1. What is the threshold of completed weeks after 43–4. This graphic suggests which of the following
which a pregnancy is considered prolonged? regarding perinatal mortality rates (PMR)?
a. 40 weeks
70
b. 41 weeks 60 60

s
c. 42 weeks

h
50

t
r
i
b
d. 43 weeks 40 33
1943–1952

0
27

0
30

0
1
15 13
43–2. Which of the following is true regarding calculated 20 10 11

r
e
p
gestational age? 10

y
t
i
a. Underestimated if based on last menstrual period 7.2
l
a
t
r
(LMP) alone
o
m
1977–1978
b. Overestimated if based on first-trimester 4.0
l
a
3.1 3.0
t
a
sonographic examination 2.3 2.4
n
i
r
c. Overestimated if based on second-trimester
e
P
sonographic examination and LMP compared
with LMP alone 37 38 39 40 41 42 43 44
d. None of the above We e ks ’ ge s ta tion a t de live ry

Reproduced with permission from Cunningham FG, Leveno KJ, Bloom SL, et al (eds):
43–3. Rare fetal-placental factors associated with postterm Postterm pregnancy. In Williams Obstetrics, 24th ed. New York, McGraw-Hill, 2014,
pregnancy include which of the following? Figure 43-2.
a. Wilms tumor
b. Anencephaly a. PMR was lowest in 1943–1952.
c. Adrenal hyperplasia b. PMR was higher in 1977–1978.
d. Autosomal-recessive placental sulfatase c. PMR increases after 41 weeks.
overproduction d. PMR increases after 40 weeks.
288 O bstetrica l Complica tions

43–5. Major causes of death in postterm pregnancy include 43–7. Which of the following is true concerning the
which of the following? syndrome afflicting this infant?
a. Placenta accreta
S
b. Placental abruption
E
C
T
c. Unexplained stillbirth in diabetic patients
I
O
d. Cephalopelvic disproportion in prolonged labor
N
1
1
43–6. This graphic illustrates which of the following
regarding perinatal mortality rates (PMR)?

Pe rina ta l morta lity ra te


Pe rina ta l ris k index
10
s
h
8
t
r
i
b
0
6
0
0
1
r
4
e
p
s
h
t
2
a
e
D
0
37 38 39 40 41 42 43
We e k of birth

Reproduced with permission from Cunningham FG, Leveno KJ, Bloom SL, et al (eds):
Postterm pregnancy. In Williams Obstetrics, 24th ed. New York, McGraw-Hill, 2014,
Figure 43-3.

Reproduced with permission from Cunningham FG, Leveno KJ, Bloom SL, et al (eds):
a. Highest PMR occurs at 43 weeks. Postterm pregnancy. In Williams Obstetrics, 24th ed. New York, McGraw-Hill, 2014,
Figure 43-4.
b. Highest PMR occurs at 38 weeks.
c. Perinatal risk index, which accounts for risk of all a. Features include simian crease and low-set ears.
ongoing pregnancies, is highest at 36 weeks
b. Its incidence in pregnancies between 41 and
d. None of the above 43 weeks is 20%.
c. Neurological deficits are found in 23% of affected
newborns.
d. Associated oligohydramnios substantially increases
its likelihood at 42 weeks.

43–8. In postterm gestations, which of the following


suggests compromise of fetal oxygenation?
a. Decreased hematocrit
b. Proapoptotic gene upregulation
c. Elevated erythropoietin level
d. None of the above
Postterm Pregna ncy 289

43–9. This figure demonstrates which of the following? 43–12. This graphic illustrates which of the following?

180 180 1,400

C
150 150 1,300

H
1,200

A
120 120

P
1,100

)
90 90

T
L
m
Uppe r ra nge

E
60 60 1,000

(
R
e
30 30 900

m
4
Me a n

3
u
800

l
o
100 100

v
700

d
80 80

i
u
60 60
600

l
f
c
40 40
500

i
n
20 20

o
i
400

n
0 0

m
300

A
Reproduced with permission from Cunningham FG, Leveno KJ, Bloom SL, et al (eds):
Postterm pregnancy. In Williams Obstetrics, 24th ed. New York, McGraw-Hill, 2014, 200 Lowe r ra nge
Figure 43-6. 100
0
a. Late deceleration 38 39 40 41 42 43
b. Variable deceleration We e ks ’ ge s ta tion
c. Prolonged deceleration Reproduced with permission from Cunningham FG, Leveno KJ, Bloom SL, et al (eds):
Postterm pregnancy. In Williams Obstetrics, 23rd ed. New York, McGraw-Hill, 2010,
d. None of the above
Figure 37-8.

43–10. In postterm pregnancies, which of the following is


a. Amnionic fluid volume decreases from term until
true of most cases of fetal distress?
43 weeks.
a. Associated with cord occlusion
b. The largest amount of amnionic fluid is present
b. Associated with prolonged labor before term.
c. Not correlated with viscous meconium c. The smallest amount of fluid in the upper range
d. Caused by uteroplacental insufficiency is seen at 41 weeks.
d. The greatest amount of fluid in the lower range is
43–11. Which of the following increases the risk of approximately 700 mL at 38 weeks.
meconium aspiration syndrome?
a. Fetal acidemia
b. Oligohydramnios
c. Postterm pregnancy
d. A and B
290 O bstetrica l Complica tions

43–13. This graphic illustrates which of the following? 43–16. Concerning an unfavorable cervix, research supports
which of the following statements?
40
a. A cervical length ≤ 3 cm was predictive of
S
successful induction.
E
C
30 b. Of women at 42 weeks, 92% have an unfavorable
T
I
cervix, when defined as a Bishop score < 7.
O
N
s
c. The risk of cesarean delivery is increased twofold
m
20
1
a
in those with a closed cervix at 42 weeks
r
1
G
undergoing labor induction.
10
d. All of the above

0 43–17. Concerning membrane stripping, research supports


20 22 24 26 28 30 32 34 36 38 40 42 which of the following statements?
We e ks ’ ge s ta tion a. Decreases need for induction
Reproduced with permission from Cunningham FG, Leveno KJ, Bloom SL, et al (eds):
b. Increases maternal infection rates
Postterm pregnancy. In Williams Obstetrics, 24th ed. New York, McGraw-Hill, 2014, c. Complications include bleeding and pain
Figure 43-5.
d. All of the above
a. Fetuses lose weight after 40 weeks.
43–18. This graphic illustrates which of the following?
b. Fetal growth continues until at least 42 weeks.
c. The peak of fetal growth occurs in the late
midtrimester. 100%
)
t
100
n
d. Maternal and fetal morbidity associated with
e
c
r
macrosomia would be mitigated with timely
e
p
80
(
induction.
s
e
m
o
43–14. In the presence of macrosomia, which of the 60
c
t
46%
u
following is true?
o
l
40
a
a. Early induction decreases maternal and fetal
m
27%
r
o
morbidity rates. 19%
n
20
b
A
b. Cesarean delivery should be performed for
estimated fetal weight > 4000 g.
c. Cesarean delivery is recommended for estimated 6.7 cm < 5 cm < 2 cm < 1 cm
fetal weights > 4500 g if there is prolonged (5th pe rce ntile )
second-stage labor.
Amnionic fluid inde x La rge s t ve rtica l pocke t
d. None of the above
Reproduced with permission from Cunningham FG, Leveno KJ, Bloom SL, et al (eds):
43–15. What percentage of postterm stillbirths are Postterm pregnancy. In Williams Obstetrics, 23rd ed. New York, McGraw-Hill, 2010,
Figure 37-9.
growth-restricted?
a. 10%
a. An AFI < 1 cm had 100% abnormal outcomes.
b. 25%
b. Abnormal fetal outcomes occur when the AFI is
c. 33% at the 5th centile.
d. 50% c. When the AFI is < 2 cm, abnormal outcomes
were increased relative to those at 5th centile.
d. If the largest vertical pocket (LVP) was < 5 cm,
the chance of an abnormal outcome was 27%.
Postterm Pregna ncy 291

43–19. Concerning the station of the vertex in nulliparous 43–22. Concerning this algorithm that summarizes the
pregnancies at the beginning of induction for American College of Obstetricians and Gynecologists
postterm pregnancy, research supports which of the (ACOG) recommendations, which of the following

C
following statements? is true?

H
a. The cesarean delivery rate is directly related to

A
P
station.

T
Comple te d 41 We e ks

E
b. The cesarean delivery rate is 6% if the vertex is at No othe r complica tions

R
–1 station.

4
3
c. The cesarean delivery rate is 43% if the vertex is
at –3 station. S ome choos e to initia te
fe ta l s urve illa nce
d. All of the above

43–20. Concerning induction versus fetal testing in Comple te d 42 We e ks


prolonged pregnancies, research supports which of
the following statements?
a. Testing is associated with decreased cesarean No complica tions Complica tions evide nce for:
delivery rates. (1) Fe ta l compromis e
(2) Oligohydra mnios
b. Most studies are performed during the 43rd week
of gestation. Fe ta l s urve illa nce a La bor induction b
Amnionic fluid (P re fe ra ble with
c. No differences in perinatal death and cesarean volume a s s e s s me nta La bor induction b
favora ble ce rvix)
delivery rates are found between the two
approaches.
Reproduced with permission from Cunningham FG, Leveno KJ, Bloom SL, et al (eds):
d. None of the above Postterm pregnancy. In Williams Obstetrics, 24th ed. New York, McGraw-Hill, 2014,
Figure 43-10.
43–21. In Alexander’s study (2001) comparing induction
versus awaiting spontaneous labor at 42 weeks’ a. Induction should be performed at 41 completed
gestation, investigators found which of the weeks.
following? b. If no complications exist, labor may be induced at
a. Decreased cesarean delivery rate in the induction 42 completed weeks.
group c. Fetal surveillance should be initiated for a
b. Increased cesarean delivery rate for fetal distress in pregnancy with oligohydramnios at 42 completed
the spontaneous labor group weeks.
c. Increased cesarean delivery rate in the induction d. All of the above
group related to the use of meperidine analgesia
during labor 43–23. Which of the following is true regarding
d. Increased cesarean delivery rate in the induction amnioinfusion?
group related to nulliparity, epidural anesthesia, a. Requires amniotomy
and an unfavorable cervix
b. Reduces late decelerations
c. Reduces the rate of meconium aspiration
syndrome
d. Does not affect the cesarean delivery rate in
postterm fetuses
292 O bstetrica l Complica tions

43–24. Concerning the nulliparous woman with a postterm 43–29. With reassuring status by antepartum surveillance
gestation in early labor, which of the following is true? verified, what might your next step be for the patient
a. If there are repetitive variable decelerations, in Question 43-28?
S
amnioinfusion is appropriate. a. Elective cesarean delivery
E
C
b. If cephalopelvic disproportion is suspected, b. Weekly antepartum surveillance
T
I
immediate cesarean delivery should be performed.
O
c. Amniocentesis to verify pulmonary maturity
N
c. If there is thin meconium, strong consideration followed by induction
1
should be given to effect prompt cesarean
1
d. B or C
delivery.
d. All of the above 43–30. Your patient is a 16-year-old G1P0 at 41 weeks’
gestation with complaints of decreased fetal
43–25. The American Academy of Pediatrics supports which movement. Her care started during her first
of the following practices? trimester. She has a stated last menstrual period that
a. Routine suctioning of newborns with meconium agrees with 16-week sonographic measurements.
Examination reveals a 1- to 2-cm dilated cervix,
b. Intubation of the depressed newborn with
cephalic presentation, estimated fetal weight of 9 lb,
meconium
and a ballottable head. What is a reasonable next
c. Meconium suctioning after the head is born to step?
minimize meconium aspiration syndrome
a. Nonstress test
d. None of the above
b. Labor induction
43–26. In the postterm fetus, which of the following is true c. Umbilical artery Doppler evaluation
regarding amniotomy? d. A or B
a. May identify thick meconium
b. May exacerbate cord compression 43–31. After a reassuring nonstress test (NST) result, the
patient in Question 43-30 returns to your office
c. Is essential to effect continuous fetal monitoring 1 week later. Examination now reveals a 2-cm dilated
d. A and B cervix, cephalic presentation, and estimated fetal
weight of 9½ lb. Fetal movement is good, and the
43–27. Your patient is a 32-year-old G3P2 at 406/7 weeks’ vertex is ballotable. What is a reasonable next step?
gestation. Her first prenatal visit occurred after a. NST
30 weeks’ gestation. Examination today reveals a 1-cm
dilated cervix, cephalic presentation, no ballotment of b. Labor induction
the head, and good fetal movement. What is the best c. Amnionic fluid surveillance
next step in the management of this patient? d. All of the above
a. Nonstress test
b. Labor induction 43–32. Your patient is a 22-year-old G1P0 at 42 weeks’
gestation by excellent dating criteria. She arrives for
c. Oxytocin challenge test induction and cervical examination reveals 1 to 2 cm
d. Sonographic assessment of the amnionic fluid dilatation, cephalic presentation, and a head that
index (AFI) is not ballottable. The estimated fetal weight is 8½
lb. Her fetal heart rate pattern is category II. You
43–28. One week later, the patient in Question 43–27 perform an amniotomy, place a direct fetal scalp
returns to your office. Now her examination reveals electrode monitor, and encounter viscous meconium.
a 1- to 2-cm dilated cervix, cephalic presentation, What is an appropriate next step?
and an easily ballottable head. However, she notes a. Cesarean delivery
decreased fetal movement. Your management plan
should include which of the following? b. Low-dose oxytocin protocol
a. Nonstress test c. Place intrauterine pressure catheter
b. Labor induction d. All may be considered.
c. Oxytocin challenge test
d. A or B Reference:
Alexander JM, McIntire DD, Leveno KJ: Prolonged pregnancy:
induction of labor and cesarean births. Obstet Gynecol 97:911, 2001
Postterm Pregna ncy 293

CHAPTER 43 ANSw ER KEy

Q uestion Letter Pa ge

C
number a nswer cited Hea der cited

H
A
P
43–1 c p. 862 Introduction

T
E
43–2 d p. 862 Estima ted G esta tiona l Age Using Menstrua l Da tes

R
4
43–3 b p. 863 Incidence

3
43–4 c p. 863 Perina ta l Morta lity
43–5 c p. 863 Perina ta l Morta lity
43–6 d p. 863 Perina ta l Morta lity
43–7 d p. 864 Postma turity Syndrome
43–8 c p. 864 Pla centa l Dysfunction
43–9 c p. 865 Feta l Distress a nd O ligohydra mnios
43–10 a p. 865 Feta l Distress a nd O ligohydra mnios
43–11 d p. 865 Feta l Distress a nd O ligohydra mnios
43–12 a p. 865 Feta l Distress a nd O ligohydra mnios
43–13 b p. 867 Ma crosomia
43–14 c p. 867 Ma crosomia
43–15 c p. 866 Feta l-G rowth Restriction
43–16 d p. 867 Unfa vora ble Cervix
43–17 c p. 868 Cervica l Ripening
43–18 b p. 866 O ligohydra mnios
43–19 d p. 868 Sta tion of Vertex
43–20 c p. 868 Induction versus Feta l Testing
43–21 d p. 868 Induction versus Feta l Testing
43–22 b p. 864 Pla centa l Dysfunction
43–23 a p. 869 Intra pa rtum Ma na gement
43–24 a p. 869 Intra pa rtum Ma na gement
43–25 b p. 869 Intra pa rtum Ma na gement
43–26 d p. 869 Intra pa rtum Ma na gement
43–27 d p. 866 O ligohydra mnios
43–28 a p. 869 Ma na gement Recommenda tions
43–29 d p. 869 Ma na gement Recommenda tions
43–30 d p. 869 Ma na gement Recommenda tions
43–31 d p. 869 Ma na gement Recommenda tions
43–32 d p. 869 Intra pa rtum Ma na gement
294

CHAPTER 44

Fetal-Gro th Disorders

44–1. What can be said regarding birthweight in the 44–4. Amino acids undergo which type of transport from
United States? maternal to fetal circulation?
a. All low-birthweight neonates are born preterm. a. Active transport
b. Of low-birthweight neonates, 3% are born at b. Passive diffusion
term. c. Facilitated diffusion
c. The percentage of low-birthweight neonates has d. None of the above
been decreasing since the mid-1980s.
d. None of the above 44–5. This graph depicts the relationship between
birthweight percentile and perinatal mortality and
44–2. Given the graphic below, what can be said regarding morbidity rates. Below which threshold value of
the velocity of fetal growth at different gestational birthweight percentile do perinatal mortality rates
ages? increase most rapidly?

80 100 Morbidity 175

90 Morta lity
)
t
60
n
150
e
P
80
c
e
)
r
g
e
r
(
i
p
n
y
(
40

a
70
a
y
t
d
100

a
t
i
r
l
d
e
m
i
p
60
b
o
r
h
20
o
r
t
t
w
m
a
o
l
50 75

i
l
r
t
G
a
y
t
r
a
a
0
n
40

t
i
e
r
e
(
50
p
p
e
30
e
r
v
i
1
t
0
20 25 30 35 40 45
a
l
20

0
u
0
25
m
Ge s ta tiona l a ge a t la s t ultra s ound (we e ks )
)
u
C
10
Reproduced with permission from Cunningham FG, Leveno KJ, Bloom SL, et al (eds):
Fetal growth disorders. In Williams Obstetrics, 24th ed. New York, McGraw-Hill, 2014,
Figure 44-1.
> 10 10 9 8 7 6 5 4 3 2 1 <1
Birthwe ight pe rce ntile
a. At 24 weeks’ gestation, growth averages 5 g/day.
b. At 34 weeks’ gestation, growth averages Reproduced with permission from Cunningham FG, Leveno KJ, Bloom SL, et al (eds):
Fetal growth disorders. In Williams Obstetrics, 24th ed. New York, McGraw-Hill, 2014,
25–30 g/day. Figure 44-3.
c. There is considerable variation in the velocity of
fetal growth. a. 3
d. All of the above b. 5
c. 7
44–3. Elevated C-peptide levels are associated with which
of the following? d. 10
a. Hyperinsulinemia
b. Hypercholesterolemia
c. Fetal-growth restriction
d. All of the above
Feta l-Growth Disorders 295

44–6. Symmetrical growth restriction is characterized by a 44–11. Which of the following is true regarding maternal
reduction in which of the following? nutrition during pregnancy?
a. Head size a. Providing micronutrient supplementation to

C
b. Body size undernourished women consistently lowers rates

H
of small-for-gestational-age newborns.

A
c. Both body and head size

P
b. For all maternal weight categories, excessive

T
d. Both body and femur length

E
maternal weight gain during pregnancy is

R
associated with large-for-gestational-age newborns.

4
44–7. Which of the following correctly represents current

4
thinking on asymmetrical versus symmetrical growth c. For all maternal weight categories, maternal
restriction? weight gain in the second and third trimesters
that is less than recommended is associated with
a. Neonatal morbidity rates are higher with fetal-growth restriction.
asymmetrical growth restriction.
d. All of the above
b. Uteroplacental insufficiency leads to asymmetrical
growth restriction in most cases of preeclampsia.
44–12. Women screened during pregnancy for psychosocial
c. Assigning specific morbidity to specific fetal- risk factors compared with pregnant women who
growth restriction patterns is a straightforward do not undergo such screening have which of the
process. following?
d. None of the above a. Lower preterm birth rates
b. More appropriate interventions
44–8. Growing evidence suggests that fetal-growth
restriction affects organ development, especially c. Lower rates of low-birthweight newborns
which of the following? d. All of the above
a. Brain
44–13. Which of the following vascular diseases in women
b. Heart
during pregnancy leads to the highest perinatal
c. Kidney morbidity rates?
d. Thyroid a. Class F diabetics
b. Chronic hypertension
44–9. Compared with appropriately grown fetuses of
equivalent gestational age, growth-restricted fetuses c. Ischemic heart disease
have which of the following perinatal advantages? d. Valvular heart disease
a. Lower stillbirth rate
44–14. Which of the following is true concerning diabetes
b. Lower perinatal mortality rate
in pregnancy?
c. Lower rate of respiratory distress syndrome
a. Compared with type 1 diabetics, type 2 diabetics
d. None of the above have a higher risk of delivering a large-for-
gestational-age (LGA) newborn.
44–10. Which of the following is true regarding women
with pregravid weights less than 100 lb compared b. Type 1 diabetics have a proportionately higher
risk of delivering a small-for-gestational-age
with normal-weight women?
(SGA) than an LGA newborn.
a. They have a twofold risk of having growth-
restricted fetuses. c. Type 1 diabetics without vascular involvement
have a proportionately higher risk of delivering an
b. They have a slightly increased risk of having a LGA newborn than an SGA one.
fetus with aneuploidy.
d. None of the above
c. The risk of fetal-growth restriction may be
modulated by appropriate maternal gestational
44–15. Which of the following are true concerning chronic
weight gain.
hypoxia?
d. All of the above
a. Women with cyanotic heart disease have a higher
rate of growth-restricted fetuses.
b. Neonates born at lower altitudes have a lower risk
of being small for gestational age.
c. Neonates born at higher altitudes have a lower
risk of being large for gestational age.
d. All of the above
296 O bstetrica l Complica tions

44–16. Regarding maternal anemia, which of the following 44–20. Which of the following drugs are associated with
is true? fetal-growth restriction?
a. It confers a high associated risk of fetal-growth a. Metoclopramide
S
restriction. b. Diphenhydramine
E
C
b. Fetal-growth restriction rates are higher in women c. Cyclophosphamide
T
I
with sickle-cell trait.
O
d. Low-dose aspirin
N
c. Fetal-growth restriction is related to restricted
1
maternal blood volume expansion. 44–21. This growth-restricted newborn was born at
1
d. All of the above 36 weeks’ gestation. All EXCEPT which of the
following are infectious causes of fetal-growth
44–17. Which of the following is the primary autoantibody restriction?
that predicts obstetrical antiphospholipid antibody
syndrome?
a. Lupus anticoagulant
b. Anticardiolipin antibodies
c. Anti-beta-glycoprotein-I antibodies
d. Anti-double-stranded DNA antibodies

44–18. Fetal-growth restriction is associated with all


EXCEPT which of the following?
a. Prior infertility
b. Placental chorangioma
c. Inherited thrombophilia
d. Velamentous cord insertion

44–19. The graph below suggests which of the following


regarding fetal-growth restriction and multifetal
pregnancy?

4000 S ingle tons


3500
3000
Twins
)
g
(
2500
t
h
Triple ts
g
2000
i
e
w
Reproduced with permission from Cunningham FG, Leveno KJ, Bloom SL, et al (eds):
1500 Qua druple ts
h
Fetal growth disorders. In Williams Obstetrics, 24th ed. New York, McGraw-Hill, 2014,
t
r
i
Figure 44-7.
B
1000
500 a. Congenital syphilis
0 b. Maternal tuberculosis
20 25 30 35 40 45
Ge s ta tiona l a ge a t de live ry (we e ks ) c. First-trimester primary cytomegalovirus infection
d. Antepartum maternal seroconversion for
Reproduced with permission from Cunningham FG, Leveno KJ, Bloom SL, et al (eds): toxoplasmosis
Fetal growth disorders. In Williams Obstetrics, 24th ed. New York, McGraw-Hill, 2014,
Figure 44-5.
44–22. In which chromosomal aneuploidy is fetal-growth
a. Most growth-restricted neonates result from restriction virtually always present?
quadruplet pregnancies. a. 45,X
b. Fetal-growth restriction can be detected at 20 b. Trisomy 13
weeks’ gestation in quadruplets. c. Trisomy 18
c. In multifetal gestations, fetal-growth restriction d. Trisomy 21
typically becomes apparent in the early third
trimester.
d. All of the above
Feta l-Growth Disorders 297

44–23. Which of the following is true regarding serial 44–28. For the patient in Question 44–27, when will you
fundal height measurements to detect fetal-growth reevaluate fetal growth?
restriction? a. 1 week

C
a. Sensitivity < 25%, specificity < 50% b. 2 weeks

H
A
b. Sensitivity < 35%, specificity > 90% c. 3 weeks

P
T
c. Sensitivity < 70%, specificity < 50% d. 6 weeks

E
R
d. Sensitivity < 85%, specificity > 90%

4
44–29. Your next obstetrical sonographic evaluation of the

4
44–24. Which of the following sonographic assessments patient in Question 44–27 is performed 4 weeks after
of fetal-growth restriction is the most predictive of the first one and now at an estimated gestational age
obstetrical outcome? of 36 weeks. The fetus now has measurements similar
a. Biometric growth < 3rd percentile to a 30-week fetus. Growth restriction seems more
likely. What is appropriate at this time?
b. Biometric growth < 5th percentile
a. Delivery
c. Biometric growth < 3rd percentile and abnormal
umbilical artery Doppler velocimetry results b. Strict bed rest
d. Biometric growth < 5th percentile and absent c. Umbilical artery Doppler velocimetry
end-diastolic flow noted during umbilical artery d. Sonographic fetal biometry in 1 week
Doppler velocimetry study
44–30. For the patient in Question 44–27, studies indicate
44–25. Which of the following is true regarding a systolic/diastolic (S/D) ratio of 4, and the patient
oligohydramnios? has an amnionic fluid index (AFI) of 9 cm. What is
a. It is associated with fetal-growth restriction. appropriate at this time?
b. It is associated with congenital fetal a. Delivery
malformations. b. Betamethasone administration
c. It is associated with a higher cesarean delivery c. Sonographic fetal biometry in 1 week
rate. d. Serial umbilical artery Doppler studies and AFI
d. All of the above assessment

44–26. Which of the following is true concerning the 44–31. For the patient in Question 44–27, during the
prevention of fetal-growth restriction? next week, umbilical artery Doppler velocimetry
a. In the United States, malaria prophylaxis assists indicates reversed end-diastolic flow (REDF), and
prevention. the amnionic fluid index (AFI) is 4 cm. What is
appropriate at this time?
b. Preconceptional and antepartum smoking
cessation assists prevention. a. Deliver the fetus
c. For the gravida with chronic hypertension, b. Plan delivery at 38 weeks after amniocentesis for
antihypertensive therapy assists prevention. pulmonary maturity
d. Aspirin therapy assists prevention and is c. Continue serial umbilical artery Doppler studies
recommended by the American College of and AFI assessment
Obstetricians and Gynecologists. d. All are reasonable

44–27. Ms. Smith is a 37-year-old multigravida who 44–32. According the American College of Obstetricians
presents to your office at 32 weeks’ gestation and Gynecologists, which of the following is the
as calculated by her last menstrual period. Her threshold above which macrosomia is defined?
hematocrit is 29 volume percent, and she has sickle- a. 4000 g
cell trait. During sonographic evaluation, the fetus
b. 4250 g
has biometric values that correlate with a 28-week
fetus. What is the most likely explanation? c. 4500 g
a. Aneuploidy d. 5000 g
b. Chronic hypoxia
c. Poor pregnancy dating
d. First-trimester cytomegalovirus infection
298 O bstetrica l Complica tions

44–33. For the prediction of macrosomia, how does clinical 44–34. In pregnancies with estimated fetal weights
estimation of fetal weight compare with sonographic > 4000 g after 37 weeks’ gestation, prophylactic
estimation? labor induction has which of the following effects?
S
a. Less accurate a. Increases the cesarean delivery rate
E
C
b. Similar accuracy b. Decreases the shoulder dystocia rate
T
I
O
c. Modestly more accurate c. Decreases the postpartum hemorrhage rates
N
d. Significantly more accurate d. All of the above
1
1
Feta l-Growth Disorders 299

CHAPTER 44 ANSw ER KEy

Q uestion Letter Pa ge

C
number a nswer cited Hea der cited

H
A
P
44–1 b p. 872 Introduction

T
E
44–2 c p. 872 Feta l G rowth

R
4
44–3 a p. 872 Feta l G rowth

4
44–4 a p. 872 Feta l G rowth
44–5 a p. 874 Feta l G rowth Restriction, Definition
44–6 c p. 875 Symmetrica l versus Asymmetrica l G rowth Restriction
44–7 a p. 875 Symmetrica l versus Asymmetrica l G rowth Restriction
44–8 b p. 876 Feta l Undergrowth
44–9 d p. 877 Accelera ted Lung Ma tura tion
44–10 c p. 877 Constitutiona lly Sma ll Mothers
44–11 b p. 877 G esta tiona l W eight G a in a nd N utrition
44–12 d p. 878 Socia l Depriva tion
44–13 c p. 878 Va scula r Disea se
44–14 c p. 878 Pregesta tiona l Dia betes
44–15 d p. 878 Chronic Hypoxia
44–16 c p. 878 Anemia
44–17 a p. 878 Antiphospholipid Antibody Syndrome
44–18 c p. 879 Inherited Thrombophilia s; Infertility; Pla centa l a nd Cord Abnorma lities
44–19 c p. 879 Multiple G esta tions
44–20 c p. 879 Drugs with Tera togenic a nd Feta l Effects
44–21 d p. 879 Ma terna l a nd Feta l Infections
44–22 c p. 880 Chromosoma l Aneuploidies
44–23 b p. 880 Uterine Funda l Height
44–24 c p. 880 Sonogra phic Mea surements of Feta l Size
44–25 d p. 881 Amnionic Fluid Volume Mea surement
44–26 b p. 882 Prevention
44–27 c p. 880 Recognition of Feta l-G rowth Restriction
44–28 c p. 880 Sonogra phic Mea surements of Feta l Size
44–29 c p. 882 Ma na gement
44–30 d p. 883 Figure 4 4 -9
44–31 a p. 883 Figure 4 4 -9
44–32 c p. 885 Empirica l Birthweight
44–33 b p. 885 Dia gnosis
44–34 a p. 886 Prophyla ctic La bor Induction
300

CHAPTER 45

Multifetal Pregnanc

45–1. Compared with singleton pregnancies, multifetal 45–5. When trying to establish chorionicity of the
gestations have a higher risk of all EXCEPT which pregnancy shown in the image here, which of the
of the following complications? following statements is true?
a. Preeclampsia
b. Hysterectomy
c. Maternal death
d. Postterm pregnancy

45–2. Compared with singleton pregnancies, multifetal


gestations have an infant mortality rate that is how
many times greater?
a. Twofold
b. Threefold
c. Fivefold
d. Tenfold

45–3. Which of the following mechanisms may prevent


monozygotic twins from being truly “identical”? Reproduced with permission from Cunningham FG, Leveno KJ, Bloom SL, et al (eds):
Multifetal pregnancy. In Williams Obstetrics, 24th ed. New York, McGraw-Hill, 2014,
a. Postzygotic mutation Figure 45-7B.
b. Unequal division of the protoplasmic material
c. Variable expression of the same genetic disease a. There are two placentas.
d. All of the above b. The twins must be monozygotic.
c. The twins share the same amnion.
45–4. A patient delivers a twin gestation in which one d. The twins must have arisen from two separate
infant has blood type A and one has type O. ova.
The patient and her husband are both type O. A
particular phenomenon is proposed as the etiology of 45–6. Which of the following factors increases the risk for
the discordant blood types. How would you explain monozygotic twinning?
this to the mother?
a. Increased parity
a. The proposed phenomenon does not b. Increased maternal age
spontaneously occur in humans.
c. The father is an identical twin.
b. It involves fertilization of one ovum that splits
into two during the same menstrual cycle. d. None of the above
c. It involves fertilization of two ova within the
same menstrual cycle, but not at the same coitus.
d. It involves fertilization of two ova separated in
time by an interval as long as or longer than a
menstrual cycle.
Multifeta l Pregna ncy 301

45–7. The first-trimester sonographic image here shows 45–10. What can be confirmed about the placenta being
two fetal heads arising from a shared body. How examined in the image here?
many days after fertilization must the division of

C
this zygote have occurred to lead to the abnormality

H
shown?

A
P
T
E
R
4
5
a. Dizygosity
b. Monozygosity
c. One chorion, two amnions
Reproduced with permission from Cunningham FG, Leveno KJ, Bloom SL, et al (eds): d. Two chorions, two amnions
Multifetal pregnancy. In Williams Obstetrics, 24th ed. New York, McGraw-Hill, 2014,
Figure 45-16.
45–11. Which of the following is true regarding the rate of
a. 0–3 days monozygotic twinning?
b. 4–7 days a. It approximates 1 in 250 worldwide.
c. 8–12 days b. It is increased with maternal age and parity.
c. It is lower for Hispanic women than for white
d. More than 13 days
women.
45–8. A patient with twins is referred for prenatal d. It can be modified by FSH (follicle-stimulating
care. At the referring clinic, she had several hormone) treatment.
sonographic examinations that establish these to be
monochorionic twins. Today, you see only one fetus 45–12. Which of the following statements is true regarding
sonographically. Which of the following statements chorionicity in multifetal pregnancy?
is false regarding the risk of a vanishing twin? a. Dichorionic pregnancies are always dizygotic.
a. The risk exceeds 10% in multifetal gestations. b. Monochorionic membranes should have four
b. The risk is higher in monochorionic than in layers.
dichorionic pregnancies. c. Monochorionic pregnancies are always
c. This risk is increased if she used assisted monozygotic.
reproductive technologies to conceive. d. Chorionicity is accurately determined by
d. A vanishing twin does not affect first-trimester measuring the thickness of the dividing
biomarker testing if it occurs after 10 weeks’ membranes during sonographic examination in
gestation. the first trimester.

45–13. Among the following choices, which is the strongest


45–9. What is the approximate risk of triplet or higher-
risk factor for multifetal pregnancy?
order multifetal gestation if ovarian stimulation
and intrauterine insemination is used to achieve a. Advanced maternal age
pregnancy? b. Use of clomiphene citrate
a. 10% c. African American ethnicity
b. 20% d. Maternal history of being a twin herself
c. 30%
d. 40%
302 O bstetrica l Complica tions

45–14. A patient presents for prenatal care at 12 weeks’ 45–17. A fetus that is part of a dichorionic twin pair is
gestation and wants to know about specific risks to estimated to weigh 2000 g at 33 weeks’ gestation.
her pregnancy. She has spontaneously conceived a What can be said about its growth?
S
monochorionic twin gestation. Which statement is
E
5000
C
false regarding these twins?
T
a. They have a higher risk of pregnancy loss than
I
S ingle tons 75 th
O
fraternal twins. 4000 50 th
N
25 th

)
1
g
b. Those born at term have a higher risk of cognitive

(
1
3000 50 th

t
delay than term singletons.

h
g
i
e
c. They have twice the risk of malformations

w
2000

h
compared with singleton pregnancies. Twins

t
r
i
B
d. They have a lower risk of pregnancy loss 1000
than identical twins conceived with assisted
reproductive technologies. 0
25 30 35 40
45–15. The differential diagnosis of clinically suspected Ge s ta tiona l a ge (we e ks )
twins includes all EXCEPT which of the following?
Reproduced with permission from Cunningham FG, Leveno KJ, Bloom SL, et al (eds):
a. Obesity Multifetal pregnancy. In Williams Obstetrics, 24th ed. New York, McGraw-Hill, 2014,
Figure 45-8.
b. Hydramnios
c. Leiomyomas
a. The fetus already shows growth restriction.
d. Blighted ovum
b. The fetus will be growth restricted at term.
45–16. Regarding maternal adaptations to multifetal c. The fetal growth is adequate for gestational age.
pregnancy, which of the following is lower in d. Growth differences will not be apparent until
twin pregnancy compared with that in a singleton delivery.
pregnancy?
a. Blood volume expansion 45–18. Among complications that may be seen in twin
pregnancies, which of the following may be seen in
b. Blood pressure at term dichorionic pregnancies?
c. Blood loss at delivery a. Acardiac twin
d. Systemic vascular resistance b. Fetus-in-fetu
c. Twin-twin transfusion syndrome
d. Complete mole with coexisting normal twin

45–19. What is the major cause of increased neonatal


morbidity rates in twins?
a. Preterm birth
b. Congenital malformations
c. Abnormal growth patterns
d. Twin-twin transfusion syndrome
Multifeta l Pregna ncy 303

45–20. When diagnosed at 20 weeks’ gestation, which of 45–23. A pair of monochorionic twins presents at 20 weeks’
the following statements is true regarding the twin gestation with sonographic findings that suggest
vascular complication seen in the image here? twin-twin transfusion syndrome. There is significant

C
growth discordance, no bladder is visualized in the

H
smaller twin, neither twin has ascites or hydrops, and

A
P
umbilical Doppler studies are normal. What would

T
be the assigned Quintero stage?

E
R
a. Stage I

4
5
b. Stage II
c. Stage III
d. Stage IV

45–24. The recipient cotwin in a monochorionic twin


gestation affected by twin-twin transfusion syndrome
may experience all EXCEPT which of the following
neonatal complications?
a. Thrombosis
Used with permission from Dr. Jodi Dashe. b. Hypovolemia
c. Kernicterus
a. It precludes vaginal delivery.
d. Heart failure
b. It implies the twins are conjoined.
c. It has a 50% associated fetal mortality rate. 45–25. What percentage of Quintero stage I cases remain
d. It can be monitored effectively with daily stable without intervention?
sonography. a. 25%
b. 50%
45–21. Which are the most common vascular anastomoses
seen in monochorionic twin placentas? c. 75%
a. Deep vein-vein d. 90%
b. Deep artery-vein 45–26. Which of the following therapies for severe twin-
c. Superficial artery-vein twin transfusion syndrome has been shown in a
d. Superficial artery-artery randomized trial to improve survival rates of at least
one twin to age 6 months?
45–22. Which of the following statements is true in twin- a. Septostomy
reversed-arterial-perfusion (TRAP) sequence? b. Amnioreduction
a. It is caused by a large arteriovenous placental c. Selective feticide
shunt.
d. Laser ablation of vascular anastomoses
b. The donor is at risk of cardiomegaly and high-
output heart failure. 45–27. What is the calculated fetal growth discordance of a
c. The most effective treatment is injection of KCl twin pair where the estimated fetal weight of twin A
into the recipient twin. is 800 g and that of twin B is 600 g?
d. Placental arterial perfusion pressure in the a. 10%
recipient exceeds that of the donor. b. 15%
c. 25%
d. 33%
304 O bstetrica l Complica tions

45–28. A second sonographic evaluation of the twin 45–33. Which of the following findings can predict a lower
pair described in Question 45–27 shows 27% risk of preterm birth in twins?
discordance. One fetus is male and one is female. a. Closed cervix on digital examination
S
Which mechanism is not the likely cause of their
b. Negative fetal fibronectin assessment
E
C
discordance?
c. Normal cervical length measured by transvaginal
T
a. Unequal placental sharing
I
O
sonography
N
b. Different growth potential d. All of the above
1
c. Histological placental abnormality
1
d. Suboptimal implantation of one placental site 45–34. Which is the most common presentation of twins in
labor?
45–29. With growing discordance, rates of which of the a. Vertex/vertex
following neonatal complications are increased? b. Vertex/breech
a. Neonatal sepsis c. Breech/vertex
b. Necrotizing enterocolitis
d. Vertex/transverse
c. Intraventricular hemorrhage
d. All of the above 45–35. For twins in labor, risk factors for an unstable fetal
lie include all EXCEPT which of the following?
45–30. Which of the following is the most important a. Small fetuses
predictor of neurological outcome of the survivor
b. Polyhydramnios
after death of a cotwin?
c. Increased maternal parity
a. Chorionicity
d. Vertex/vertex presentation
b. Gestational age at time of demise
c. Malformations present in the deceased twin 45–36. Which of the following scenarios presents the best
d. Length of time between demise and delivery of opportunity for a vaginal trial of labor?
survivor a. Nonvertex/vertex presentation
b. Vertex/nonvertex presentation
45–31. Which of the following methods of antepartum fetal
surveillance has been shown to improve outcomes in c. Nonvertex second twin whose estimated fetal
twin pregnancies? weight is < 1500 g
a. Nonstress test d. Vertex second twin whose estimated fetal weight
is > 20% larger than the presenting vertex twin
b. Biophysical profile
c. Doppler velocimetry of the umbilical artery
d. None of the above

45–32. Which of the following interventions has been


shown to decrease the rate of preterm birth in twins?
a. Cerclage
b. Betamimetics
c. 17-Hydroxyprogesterone caproate
d. None of the above
Multifeta l Pregna ncy 305

CHAPTER 45 ANSw ER KEy

Q uestion Letter Pa ge

C
number a nswer cited Hea der cited

H
A
P
45–1 d p. 891 Introduction

T
E
45–2 c p. 891 Introduction

R
4
45–3 d p. 892 Dizygotic versus Monozygotic Twinning

5
45–4 c p. 892 Superfeta tion a nd Superfecunda tion
45–5 b p. 896 Sonogra phic Determina tion of Chorionicity-
45–6 d p. 892 Frequency of Twinning
45–7 d p. 893 Figure 4 5 -1
45–8 d p. 892 The “Va nishing” Twin
45–9 a p. 895 Infertility Thera py
45–10 d p. 896 Pla centa l Exa mina tion
45–11 a p. 892 Frequency of Twinning
45–12 d p. 896 Sonogra phic Determina tion of Chorionicity; G enesis of Monozygotic Twins
45–13 b p. 894 Fa ctors Tha t Influence Twinning
45–14 b p. 899 Pregna ncy Complica tions; Long-Term Infa nt Development
45–15 d p. 896 Clinica l Eva lua tion
45–16 d p. 898 Ma terna l Ada pta tions to Multifeta l Pregna ncy
45–17 c p. 899 Low Birthweight; Figure 4 5 -8
45–18 d p. 902 Aberra nt Twinning Mecha nisms
45–19 a p. 900 Preterm Birth
45–20 a p. 901 Monoa mniotic Twins; Figure 4 5 -1 3
45–21 d p. 904 Monochorionic Twins a nd Va scula r Ana stomoses
45–22 b p. 908 Aca rdia c Twin
45–23 b p. 907 Dia gnosis
45–24 b p. 904 Twin-Twin Tra nsfusion Syndrome
45–25 c p. 907 Ma na gement a nd Prognosis
45–26 d p. 907 Ma na gement a nd Prognosis
45–27 c p. 909 Dia gnosis
45–28 a p. 909 Etiopa thogenesis
45–29 d p. 909 Dia gnosis
45–30 a p. 910 Dea th of O ne Fetus
45–31 d p. 912 Tests of Feta l W ell-Being
45–32 d p. 913 Prevention of Preterm Birth
45–33 d p. 913 Prediction of Preterm Birth
45–34 a p. 915 Feta l Presenta tion
45–35 d p. 915 Feta l Presenta tion
45–36 b p. 916 Cepha lic-N oncepha lic Presenta tion
Se c t i o n 1 2

Med ic a l a n d Su r g ic a l
c o Mpl ic a t io n S
308

c Ha pt er 46

g c s s M ev

46–1. A 20-year-old primigravida at 13 weeks’ gestation 46–4. Which o the ollowing statements regarding
presents with nausea, vomiting, and lower abdominal anesthetic agents and teratogenicity is most accurate?
pain or the past 24 hours. On examination, she . There is no evidence that anesthetic agents are
has mild guarding in the right lower quadrant o harm ul.
her abdomen. Which o the ollowing statements b. The evidence is contradictory regarding minimal
regarding diagnostic evaluation is most accurate? exposure.
. Magnetic resonance imaging can be used or . Halogenated agents carry an increased risk o
evaluation o the appendix. etal mal ormations during the irst 8 weeks o
b. Ultrasound is the pre erred method o evaluating pregnancy.
the appendix during pregnancy. . Regional anesthesia is the pre erred route, as
. Imaging should not be per ormed until the paralytic agents are linked to increased rates o
patient has been observed or 24 hours. neural-tube de ects.
. Computed tomography o the maternal abdomen
should be avoided because o etal exposure to 46–5. At 30 weeks’ gestation, a 38-year-old multipara
ionizing radiation. presents with recurrent cholelithiasis, and
cholecystectomy is indicated. Which o the ollowing
46–2. The patient in Question 46–1 undergoes an is true?
uncomplicated appendectomy. Which o the . Laparoscopic approach is not indicated because o
ollowing statements is most accurate? her gestational age.
. She is not at increased risk or excessive adverse b. Continuous etal heart rate monitoring should be
pregnancy outcomes. employed during the procedure.
b. Laparotomy is the pre erred approach to . The indication or the surgical approach o the
minimize risk o injury to the gravid uterus. procedure is the same as or nonpregnant emale
. There is an increased risk o pregnancy loss when patients.
surgery is per ormed in the irst trimester. . All o the above
. There is an increase in anesthesia-related
complications or surgery per ormed during early
pregnancy.

46–3. The increased neonatal death rate seen in gravidas


who undergo nonobstetrical surgery is attributable to
higher rates o which o the ollowing?
. Preterm birth
b. Fetal-growth restriction
. Congenital anomalies in patients with irst-
trimester procedures
. Fetal metabolic acidemia associated with
emergent cesarean delivery
G enera l Considera tions a nd Ma terna l Eva lua tion 309

46–6. I insu lation pressures are kept below 20 mm Hg, 46–9. Which o the ollowing actors does not a ect
all EXCEPT which o the ollowing are unchanged the dose o ionizing radiation delivered during
in women undergoing the procedure depicted below? pregnancy?

c
. Type o imaging study

H
a
b. Type and age o equipment

p
t
. Distance rom radiation source to uterus

e
r
. Gestational age at the time o the procedure

4
6
46–10. Which o the ollowing is not a potential e ect o
radiation exposure during pregnancy?
. Microcephaly
b. Spontaneous abortion
. Monozygotic twinning
. Fetal-growth restriction

46–11. Radiation exposure during which period has the


highest risk or lethality?
. Be ore implantation
b. During etal cardiac development
Reproduced with permission rom Hof man BL, Wu D: Introduction to Diagnostic Lapa- . During closure o the proximal neural tube
roscopy (update) in Schorge JO, Schaf er JI, Halvorson LM, et al (eds): Williams Gynecol-
ogy Online. New York, McGraw-Hill, 2009. http://www.accessmedicine.com. . None o the above

. Heart rate 46–12. The greatest risk or severe mental retardation in


b. Cardiac index humans occurs when high-dose radiation exposure
occurs at which o the ollowing gestational ages?
. Mean arterial pressure
. Preimplantation
. Systemic vascular resistance
b. 5–8 weeks
46–7. Which o the ollowing steps in preparation o a . 8–15 weeks
patient or laparoscopic surgery di ers when the . 25–30 weeks
patient is pregnant?
. Use o general anesthesia 46–13. What is the mean decrease in intelligence quotient
b. Presurgical bowel cleansing (IQ) scores seen with in utero exposure to 100 rads?
. Placement o a nasogastric tube . 5 points
. Positioning on the surgical table b. 15 points
. 25 points
46–8. Compared with laparoscopy per ormed in the irst . No change
trimester, procedures per ormed later in gestation
may use which o the ollowing techniques to reduce 46–14. Which o the ollowing is an example o a stochastic
surgical complications? e ect rom etal exposure to ionizing radiation?
. Gasless procedure . Childhood cancer
b. Open entry technique b. Mental retardation
. Le t upper quadrant port placement . Spontaneous abortion
. All o the above . Fetal-growth restriction

46–15. What is the pre erred method o gastrointestinal


tract evaluation during pregnancy?
. Endoscopy
b. Barium enema
. Computed tomography
. Abdominal sonography
310 Medica l a nd Surgica l Complica tions

46–16. Which o the ollowing is not a reason why it 46–20. Which o the ollowing radionuclide studies delivers
is di icult to calculate etal exposure during the highest dose o radiation to the etus?
luoroscopic procedures? . Thyroid scan with 131I at 18 weeks’ gestation
S
. The number o images taken may vary. b. Brain scan with 99mTc DTPA at 15 weeks’
e
c
b. The amnionic luid volume around the etus is gestation
t
i
variable.
o
. Liver scan using 99mTc sul ur colloid at 10 weeks’
n
. Total luoroscopy time or any given procedure is gestation
1
variable. . Abdominal abscess evaluation with 67Ga citrate at
2
. Amount o time the etus is within the radiation 6 weeks’ gestation
ield is di icult to quanti y.
46–21. A woman given 131I or Graves disease treatment
46–17. A 26-year-old woman at 29 weeks’ gestation is an discovers she was pregnant at the time o the
unrestrained patient in a motor vehicle accident. administration. Which o the ollowing is a
She is con used and complaining o a headache. recognized complication in this scenario?
Computed tomographic (CT) scanning o her head . Fetal cretinism
is ordered to exclude acute bleeding. What should b. Childhood leukemia
her amily be told about the risk to her etus rom
the imaging study? . Fetal retinoblastoma
. The risk is negligible. . None o the above
b. Computed tomography does not involve ionizing 46–22. An obstetrician discovers a breast mass in a woman
radiation, thus there is no risk. who is having her irst prenatal visit at 10 weeks’
. There is a small risk o ionizing radiation causing gestation. The biopsy is positive or cancer, and
childhood cancer with this study, but it is the breast surgeon would like to per orm a sentinel
balanced by the necessity o the test. lymphoscintigram during the surgical procedure.
. None o the above Which o the ollowing is an accurate statement?
. Pregnancy does not alter the use o 99mTc-sul ur
46–18. At 14 weeks’ gestation, a patient presents with colloid.
chest pain and shortness o breath. Using standard b. 99mTc-sul ur should not be used at all during
protocols or pulmonary embolus evaluation, which pregnancy.
o the ollowing statements is the most accurate
regarding dosimetry exposure when comparing . Surgery and the sentinel lymphoscintigram
should be delayed until a ter 15 weeks’ gestation.
ventilation-per usion scans and CT scans?
. Because o etal concerns, use o the sentinel
. Exposure is greater with CT scans
lymphoscintigram with 99mTc-sul ur should be
b. Exposure is equivalent or the two types o scans based on the individual’s risk o tumor spread.
. Exposure is greater with ventilation-per usion
scans 46–23. Which o the ollowing is an advantage o magnetic
. None o the above resonance imaging during pregnancy?
. Enhanced so t-tissue contrast
46–19. The computed tomography protocol or which b. Multiplanar acquisition o images
o the ollowing conditions is associated with the
highest dosimetry exposure to the etus? . No ionizing radiation exposure to the etus
. Renal stone . All o the above
b. Appendicitis 46–24. All EXCEPT which o the ollowing are true
. Pulmonary embolus statements regarding contrast agent use with
. Cerebral aneurysm magnetic resonance imaging?
. Developmental delay is seen in in ants exposed in
the third trimester.
b. There are no documented adverse e ects with
irst-trimester exposure in humans.
. Developmental delay is seen in the o spring o
animals with prenatal exposure.
. Use during pregnancy is not recommended unless
there is overwhelming bene it to the patient.
G enera l Considera tions a nd Ma terna l Eva lua tion 311

46–25. Which o the ollowing is not a use ul application o 46–29. A 41-year-old women presents to the emergency
magnetic resonance imaging during pregnancy? room with shoulder pain and nausea. Her
. Renal evaluation or possible urolithiasis electrocardiogram shows ST segment elevation.

c
Cardiac catheterization and angioplasty are
b. Angiography or suspected pulmonary embolus

H
per ormed. A terward, she is ound to be 6 weeks

a
. Evaluation o a bladder lap mass postoperatively

p
pregnant. Which o the ollowing statements

t
. Evaluation o the uterine wall or placental regarding radiation exposure to the embryo is most

e
r
invasion accurate?

4
6
. The dose is much less than that or barium
46–26. In comparing magnetic resonance (MR) imaging enema.
and sonography, which is most accurate or antenatal b. The dose to the embryo is similar to that during
etal weight estimation? cerebral angiography.
. MR imaging is more accurate. . The dose during the procedure is similar to that
b. Sonography is more accurate. during a barium swallow.
. Sonography and MR imaging are equivalent. . None o the above
. No in ormation is available comparing the two
techniques 46–30. Which o the ollowing has solved problems related
to etal movement during the imaging technique
46–27. Laparoscopic surgery or the condition shown in the demonstrated below?
image below has been associated with increased rates
o which o the ollowing complications?

. Increased stillbirth rate


b. Increased preterm delivery
. Increased spontaneous abortion
Reproduced with permission rom Cunningham FG, Leveno KJ, Bloom SL, et al (eds):
. All o the above Fetal imaging. In Williams Obstetrics, 24th ed. New York, McGraw-Hill, 2014, Fig.
10-44A.
46–28. In animal studies, what causes decreased
uteroplacental blood low when intraabdominal . Use o ast-acquisition imaging
insu lation pressures exceeded 15 mm Hg? b. Administration o a maternal sedative
. Decreased heart rate . Reduction in slice number acquired per study
b. Increased cardiac output . Timing o study during period o decreased etal
. Increased per usion pressure movement
. Increased placental vessel resistance
312 Medica l a nd Surgica l Complica tions

46–31. Which o the ollowing statements is most accurate 46–32. In ormation regarding deterministic e ects o
regarding the imaging modality used below? ionizing radiation during pregnancy has been
obtained rom which o the ollowing sources?
S
. Animal models
e
c
b. Atomic bomb survivors
t
i
o
. Childhood cancer registries
n
. All o the above
1
2
. It is not associated with known adverse etal
e ects.
b. It is associated with very low doses o ionizing
radiation.
. Tracking o total exposure rom multiple studies
should be per ormed.
. Because o known risks to the etus, its use should
be limited during pregnancy.
G enera l Considera tions a nd Ma terna l Eva lua tion 313

c Ha pt er 46 a n Sw er Key

Q uestion Letter Pa ge

c
number a nswer cited Hea der cited

H
a
p
46–1 p. 936 Ma terna l Indica tions

t
e
46–2 p. 927 Surgica l Procedures During Pregna ncy

r
4
46–3 p. 927 Perina ta l O utcomes

6
46–4 p. 927 Perina ta l O utcomes
46–5 p. 928 Ta ble 4 6 -2
46–6 b p. 928 Hemodyna mic Effects
46–7 p. 928 Technique
46–8 p. 928 Technique
46–9 p. 930 X-Ra y Dosimetry
46–10 p. 930 Deterministic Effects of Ionizing Ra dia tion
46–11 p. 931 Anima l Studies
46–12 p. 931 Huma n Da ta
46–13 p. 931 Huma n Da ta
46–14 p. 932 Stocha stic Effects of Ionizing Ra dia tion
46–15 p. 932 Fluoroscopy a nd Angiogra phy
46–16 b p. 932 Fluoroscopy a nd Angiogra phy
46–17 p. 933 Computed Tomogra phy
46–18 b p. 933 Computed Tomogra phy
46–19 b p. 933 Ta ble 4 6 -7
46–20 p. 935 Ta ble 4 6 -8
46–21 p. 934 N uclea r Medicine Studies
46–22 p. 934 N uclea r Medicine Studies
46–23 p. 934 Ma gnetic Resona nce Ima ging
46–24 p. 935 Contra st Agents
46–25 b p. 936 Ma terna l Indica tions
46–26 p. 936 Feta l Indica tions
46–27 b p. 928 Perina ta l O utcomes
46–28 p. 928 Perina ta l O utcomes
46–29 p. 933 Ta ble 4 6 -6
46–30 p. 936 Feta l Indica tions
46–31 p. 937 Ta ble 4 6 -9
46–32 p. 930 Deterministic Effects of Ionizing Ra dia tion
314

CHAPTER 47

Critical Care and Trauma

47–1. Which of the following statements is true of 47–6. Which of the following is least likely to contribute
obstetrical patients who require intensive care? to the development of noncardiogenic pulmonary
a. The highest use is before delivery. edema in the obstetrical patient?
b. The associated mortality rate can reach 10%. a. Sepsis syndrome
c. Pulmonary embolus is the most common b. Magnesium sulfate administration
indication for intensive care. c. Resuscitation during acute hemorrhage
d. All of the above d. Preeclampsia-related endothelial activation

47–2. Which of the following is the most common 47–7. A pregnant patient with chronic hypertension
complication in pregnancy that leads to an intensive develops superimposed preeclampsia. She is
care unit admission? diagnosed with pulmonary edema on the first
a. Sepsis postpartum day. Which of the following statements
regarding this complication is true?
b. Hemorrhage
a. Her edema may be cardiogenic.
c. Pulmonary embolus
b. Her edema may be related to increased capillary
d. Hypertensive disease
permeability.
47–3. All EXCEPT which of the following monitoring c. Brain natriuretic peptide levels are less
modalities should be routinely used when informative in pregnant patients compared with
transporting a critically ill obstetrical patient to levels in nonpregnant patients.
another hospital for care? d. All of the above
a. Electrocardiography
47–8. Which tocolytic drug has the strongest association
b. Regular vital signs
with the development of pulmonary edema in
c. Tocodynamic monitoring obstetrical patients?
d. Continuous pulse oximetry a. Atosiban
b. Terbutaline
47–4. Which of the following physiological changes is
typical of pregnancy? c. Indomethacin
a. Colloid oncotic pressure increases. d. Magnesium sulfate
b. Systemic vascular resistance decreases.
c. Pulmonary vascular resistance increases.
d. Left ventricular stroke work index decreases.

47–5. Which of the following statements is true of


pulmonary artery catheter monitoring in the acutely
ill gravida?
a. It has been shown to improve survival.
b. It is essential for the care of patients with severe
preeclampsia.
c. Its use in critically ill obstetrical patients is of
limited value.
d. It aids in the management of patients with low
injury-severity scores.
Critica l Ca re a nd Tra uma 315

47–9. To confirm the diagnosis suggested by the image 47–12. What would shift the oxyhemoglobin dissociation
here, which of the following should not be present? curve to the right, favoring oxygen delivery to the
fetus?

C
H
Capillary Alve o lar

A
100

P
T
E
R
)
t
FETAL

n
4
80

e
7
c
MATERNAL

r
e
p
(
n
60

o
i
t
a
r
u
t
a
40

s
n
e
g
y
x
O
20

0
Reproduced with permission from Cunningham FG, Leveno KJ, Bloom SL, et al (eds): 0 10 20 30 40 50 60 70 80
Critical care and trauma. In Williams Obstetrics, 24th ed. New York, McGraw-Hill, P O2 (mm Hg )
2014, Figure 47-1.
Reproduced with permission from Cunningham FG, Leveno KJ, Bloom SL, et al (eds):
a. Oxygen desaturation Critical care and trauma. In Williams Obstetrics, 24th ed. New York, McGraw-Hill,
2014, Figure 47-2.
b. PaO 2:Fio2 ratio < 200
c. Evidence of heart failure a. Fever
d. Severe permeability pulmonary edema b. Hypocapnia
c. Metabolic alkalosis
47–10. What is the best way to increase oxygen delivery
to peripheral tissues in an obstetrical patient with d. Decreased 2,3-diphosphoglycerate levels
hemorrhage that is complicated by acute respiratory
distress syndrome? 47–13. What is the approximate mortality risk for an
obstetrical patient who requires mechanical
a. Blood transfusion ventilation for any amount of time?
b. Intravenous antibiotics a. 0.5%
c. High-frequency oscillation ventilation b. 5%
d. Oxygen therapy to increase arterial Po2 to c. 20%
200 mm Hg
d. 40%
47–11. What is the final healing phase of acute respiratory
distress syndrome termed? 47–14. Of infectious causes of acute respiratory distress
syndrome, which possible etiology is less likely in an
a. Fibrotic obstetrical patient?
b. Secretory a. Pneumonia
c. Exudative b. Pyelonephritis
d. Fibroproliferative c. Chorioamnionitis
d. Puerperal pelvic infection
316 Medica l a nd Surgica l Complica tions

47–15. A patient presents to the emergency department 7 days 47–16. Which of the following pathogens is known for its
following a cesarean delivery with abdominal exotoxin that can cause extensive tissue necrosis and
tenderness, purulence from her wound, and shortness gangrene?
S
of breath. Which of the following interventions a. Escherichia coli
E
C
would most likely lead to the improvement seen in
b. Proteus mirabilis
T
her chest radiographs taken 2 day apart?
I
O
c. Staphylococcus aureus
N
d. Klebsiella pneumoniae
1
2
47–17. Which of the following mediators of sepsis syndrome
causes myocardial depression?
a. Bradykinin
b. CD4 T cells
c. Interleukin-6
d. Tumor necrosis factor-α

47–18. Which of the following clinical signs defines


progression from the warm phase of septic shock to
the cold phase of septic shock?
a. Oliguria
b. Tachypnea
c. Leukocytosis
d. Pulmonary hypertension

A 47–19. Which intervention is least likely to be beneficial in


the treatment of early sepsis?
a. Correction of anemia
b. Albumin administration
c. Broad-spectrum antimicrobials
d. Aggressive hydration with crystalloid solutions

47–20. A young primigravida presents with fever,


tachycardia, hypotension, and uterine tenderness
after an attempt at pregnancy termination using
abortifacient drugs. Which management decision is
most likely to lead to improvement?
a. Hysterectomy
b. Diagnostic laparoscopy
c. Curettage of uterine contents
d. Uterotonic agent administration

47–21. A 26-year-old G2P1 who is admitted with


pyelonephritis has persistent fever after 48 hours
of antimicrobial therapy. All EXCEPT which
B
of the following are reasonable next steps in the
management of this patient?
Reproduced with permission from Cunningham FG, Leveno KJ, Bloom SL, et al (eds): a. Blood cultures
Renal and urinary tract disorders. In Williams Obstetrics, 24th ed. New York, McGraw-
Hill, 2014, Figure 53-3A&C. b. Renal sonography
c. Computed tomography of abdomen
a. Surfactant therapy
d. Change of antibiotics based on original urine
b. Methylprednisolone therapy culture sensitivity
c. Nitric oxide administration
d. Positive end-expiratory pressure of 5-15 mm Hg
Critica l Ca re a nd Tra uma 317

47–22. A G1P1 patient presents 1 week postpartum with 47–25. A patient presents late in pregnancy for prenatal
fever, abdominal pain, and hypotension. At the care and continues to miss appointments. She is
time of exploratory laparotomy, the uterus is found typically friendly and pleasant when she comes to

C
to have the soft, bullous outpouchings shown here. her appointments, but today she is quiet during her

H
Which treatment will most likely lead to survival of visit that is attended by her husband. You suspect

A
P
this patient? intimate partner violence. If true, which of the

T
following statements is accurate?

E
R
a. She is at increased risk of depression.

4
7
b. She is at increased risk for placental abruption.
c. She is at increased risk for a low-birthweight
infant.
d. All of the above

47–26. Which of the following is not a risk factor for


domestic violence in pregnancy?
a. Poverty
b. Illicit drug use
c. Maternal age > 35
d. Low socioeconomic status
Reproduced with permission from Cunningham FG, Leveno KJ, Bloom SL, et al (eds):
Critical care and trauma. In Williams Obstetrics, 24th ed. New York, McGraw-Hill, 47–27. A patient is involved in a motor vehicle accident and
2014, Figure 47-7.
at delivery has the complication seen in the image
below. Given this finding, which of the following
a. Hysterectomy statements is least likely to be accurate?
b. Debridement of necrotic tissue
c. Hysterotomy for evacuation of retained placenta
d. Empirical treatment for group A β -hemolytic
streptococcus

47–23. What regimen is considered appropriate prophylaxis


against Chlamydia trachomatis for a sexual assault
victim?
a. Cefixime, 400 mg orally as a single dose
b. Azithromycin, 1 g orally as a single dose
c. Erythromycin, 500 mg orally twice daily for
7 days
d. Metronidazole gel 0.75%, 5 g intravaginally daily Reproduced with permission from Cunningham FG, Leveno KJ, Bloom SL, et al (eds):
for 5 days Critical care and trauma. In Williams Obstetrics, 24th ed. New York, McGraw-Hill,
2014, Figure 47-11B.

47–24. What is the most important risk factor for intimate


partner homicide? a. The speed of the car was higher than 30 mph.
a. Unwanted pregnancy b. Prior to delivery, the fetal heart rate tracing was
nonreassuring.
b. Use of illicit drugs
c. Prior domestic violence c. The intrauterine pressure generated by this
trauma was more than 500 mm Hg.
d. Low socioeconomic status
d. The patient had contractions every 30 minutes in
the first 4 hours after the trauma.
318 Medica l a nd Surgica l Complica tions

47–28. Which of the following is a critical part of adequate 47–31. What is the most common etiology of all traumatic
resuscitation of the pregnant patient with cardiac fetal deaths?
arrest? a. Suicide
S
a. Cricoid pressure b. Stab wound
E
C
b. Steep Trendelenburg positioning
T
c. Gunshot wound
I
O
c. Left lateral decubitus positioning d. Motor vehicle collision
N
d. All of the above
1
2
47–32. A pregnant patient requires laparotomy after a
47–29. What is the likelihood of intact neurological survival gunshot wound to the abdomen. Which of the
for a neonate delivered via perimortem cesarean following would make you more likely to deliver the
section 30 minutes after maternal cardiac arrest? fetus at the time of surgery?
a. 1% a. Gestational age of 35 weeks
b. 8% b. Reassuring fetal heart rate tracing
c. 15% c. Uterine contractions every 20 minutes
d. 25% d. Limited evaluation of the injury because of the
gravid uterus
47–30. A pregnant patient at 34 weeks’ gestation presents to
the trauma bay in the emergency room after a motor 47–33. All EXCEPT which of the following statements
vehicle accident. She was the restrained passenger; are accurate regarding thermal injuries during
the car was traveling approximately 40 mph; and the pregnancy?
air bag did not deploy. What complication is least a. Severe burns usually trigger onset of spontaneous
likely to be encountered? labor.
a. Fetal skull fracture b. Pregnancy does not alter maternal outcome after
b. Maternal bowel injury thermal injury.
c. Maternal pelvic fracture c. Fetal mortality rates are typically higher than
d. Maternal long bone fracture maternal mortality rates.
d. A remote history of abdominal burns and
contractures conveys a higher risk of preterm birth.
Critica l Ca re a nd Tra uma 319

CHAPTER 47 ANSw ER KEy

Q uestion Letter Pa ge

C
number a nswer cited Hea der cited

H
A
P
47–1 b p. 940 O bstetrica l Intensive Ca re; Ta ble 4 7 -2

T
E
47–2 d p. 941 Ta ble 4 7 -2

R
4
47–3 c p. 940 O bstetric Critica l Ca re

7
47–4 b p. 942 Hemodyna mic Cha nges in Pregna ncy
47–5 c p. 941 Pulmona ry Artery Ca theter
47–6 b p. 942 N onca rdiogenic Increa sed Permea bility Edema
47–7 d p. 943 Ca rdiogenic Hydrosta tic Edema
47–8 b p. 942 N onca rdiogenic Increa sed Permea bility Edema
47–9 c p. 943 Definitions
47–10 a p. 944 Ma na gement
47–11 a p. 943 Etiopa thogenesis
47–12 a p. 945 Feta l O xygena tion; Figure 4 7 -2
47–13 c p. 945 Mecha nica l Ventila tion
47–14 a p. 943 Etiopa thogenesis
47–15 d p. 945 Positive End-Expira tory Pressure
47–16 c p. 946 Etiopa thogenesis
47–17 c p. 946 Etiopa thogenesis
47–18 a p. 947 Clinica l Ma nifesta tions
47–19 b p. 948 Ma na gement
47–20 c p. 948 Surgica l Trea tment
47–21 a p. 948 Surgica l Trea tment
47–22 a p. 948 Surgica l Trea tment
47–23 b p. 952 Ta ble 4 7 -7
47–24 c p. 951 Physica l Abuse—Intima te Pa rtner Violence
47–25 d p. 951 Physica l Abuse—Intima te Pa rtner Violence
47–26 c p. 951 Physica l Abuse—Intima te Pa rtner Violence
47–27 d p. 953 Pla centa l Injuries—Abruption or Tea r; Figure 4 7 -9
47–28 c p. 956 Ca rdiopulmona ry Resuscita tion
47–29 d p. 956 Cesa rea n Delivery
47–30 b p. 951 Automobile Accidents; O ther Blunt Tra uma
47–31 d p. 952 Feta l Injury a nd Dea th
47–32 d p. 955 Cesa rea n Delivery
47–33 d p. 955 Therma l Injury
320

CHAPTER 48

Obesit

48–1. Which of the following best describes obesity in the 48–3. A 22-year-old G1P0 presents for prenatal care at 8
United States? weeks’ gestation. She has a BMI of 26. The National
a. Endemic Institutes of Health classifies this patient as which of
the following?
b. Epidemic
a. Normal
c. Pandemic
b. Overweight
d. Syndemic
c. Obese–class 1
48–2. Body mass index (BMI) is defined as which of the d. Obese–class 2
following?
a. Height in inches divided by weight in pounds 48–4. A 19-year-old G1P0 presents for her first prenatal
squared. care appointment. She weighs 270 pounds and is
67 inches tall. Using the figure provided, what is the
b. Weight in pounds divided by height in inches
patient’s BMI?
squared.
a. < 18.5
c. Weight in kilograms divided by height in meters
squared. b. 19–25
d. Height in meters divided by weight in kilograms c. 25–30
squared. d. > 30.0
We ight (pounds )
90 110 130 150 170 190 210 230 250 270 290 310 330 350
2 6'6

Unde rwe ight Norma l ra nge Ove rwe ight Obe s e


BMI < 18.5 BMI 18.5–25 BMI 25–30 BMI > 30
1.9 6'3
)
s
e
1.8 5'11
h
)
c
s
n
r
i
e
d
t
e
n
m
a
(
t
e
t
h
e
g
f
1.7
(
5'7
i
e
t
H
h
g
i
e
H
1.6 5'3

1.5 4'11
40 50 60 70 80 90 100 110 120 130 140 150 160
We ight (kg)
Reproduced with permission from Cunningham FG, Leveno KJ, Bloom SL, et al (eds): Obesity. In Williams Obstetrics, 24th ed. New York,
McGraw-Hill, 2014, Fig. 48-1.
O besity 321

48–5. Based on this figure, which of the following 48–8. A 35-year-old woman presents to your office
statements is true? for preconceptional counseling. She has chronic
hypertension that is controlled with medication. Her

C
60 White, Non-His pa nic blood pressure today is 140/90 mm Hg, and her

H
59
waist circumference is 36 inches. A fasting glucose

A
Bla ck, Non-His pa nic

P
50 level is 115 mg/dL, high-density lipoprotein (HDL)

T
His pa nic concentration is 60 mg/dL, and triglyceride level is

E
R
145 mg/dL. How many criteria does she meet for
)
40

4
t
41
n
metabolic syndrome?

8
e
c
r
a. 1
e
p
30 32
(
b. 2
y
t
i
s
e
20 c. 3
b
O
16 17 d. 4
10
10 48–9. Which of the following is not part of the metabolic
syndrome?
0
2–5 6–11 12–19 Wome n a. Diabetes
Girls (a ge in yrs ) (≥ 20 yrs ) b. Dyslipidemia
Reproduced with permission from Cunningham FG, Leveno KJ, Bloom SL, et al (eds):
Obesity. In Williams Obstetrics, 24th ed. New York, McGraw-Hill, 2014, Fig. 48-2.
c. Hypertension
d. Polycystic ovaries
a. The prevalence of obesity increases with age.
48–10. Nonalcoholic fatty liver disease (NAFLD)
b. Race is not a factor in the prevalence of female
contributes what percentage of chronic liver disease
obesity.
cases in Western countries?
c. Obesity is more common in Hispanic women
a. 1%
than in black, non-Hispanic women.
b. 10%
d. Obesity is more common in white, non-Hispanic
women than in Hispanic women. c. 25%
d. 50%
48–6. Which of the following statements regarding
adiponectin is not true? 48–11. Rates of which of the following are increased in
a. It enhances insulin sensitivity. obese women?
b. It blocks the hepatic release of glucose. a. Infertility
c. It contributes to the development of b. Recurrent miscarriage
hypertension. c. First-trimester pregnancy loss
d. It has cardioprotective effects on circulating d. All of the above
plasma lipids.
48–12. A 25-year-old G2P1 at 12 weeks’ gestation
48–7. Which of the following does not contribute to presents to your office for prenatal care. She has a
insulin resistance? BMI of 27. Based on the Institute of Medicine’s
a. Leptin recommendations, how much weight should she
gain this pregnancy?
b. Resistin
a. None
c. Adiponectin
b. 11–20 pounds
d. Tumor necrosis factor-alpha (TNF-α )
c. 15–25 pounds
d. 30–40 pounds

48–13. What percentage of women having their first baby


gain more weight than recommended?
a. 1%
b. 10%
c. 25%
d. 75%
322 Medica l a nd Surgica l Complica tions

48–14. Which of the following adverse pregnancy effects is 48–20. Serum levels of which of the following are increased
not of increased prevalence in overweight women in obese pregnant women?
compared with women of normal BMI? a. Insulin
S
a. Preeclampsia b. Interleukin-6
E
C
b. Urinary infection c. C-reactive protein
T
I
O
c. Gestational diabetes d. All of the above
N
d. Emergency cesarean delivery
1
48–21. Which of the following is the highest-ranking
2
48–15. Incidence of which of the following medical modifiable risk factor for stillbirth?
problems is not increased with increasing BMI? a. Obesity
a. Macrosomia b. Smoking
b. Preeclampsia c. Diabetes
c. Gestational diabetes d. Maternal age
d. All have increased incidence
48–22. The rate of which of the following is not increased
48–16. You have two similar pregnant women in your in fetuses of obese pregnant women?
practice. Both are 25 years old, primiparous, a. Stillbirths
and free of diabetes and hypertension. One has a b. Heart defects
BMI of 25, and the other has a BMI of 35. What
is the difference in their chances of developing c. Renal agenesis
preeclampsia? d. Neural-tube defects
a. Equal risk
48–23. Which of the following has the strongest influence
b. A 3% chance versus a 12% chance on the prevalence of the condition depicted in this
c. A 10% chance versus a 25% chance photograph?
d. A 10% chance versus a 40% chance

48–17. In the study by Weiss and colleagues from 2004,


what was the cesarean delivery rate for morbidly
obese women?
a. 10%
b. 21%
c. 47%
d. 62%

48–18. Compared with normal-weight patients, obese


women have increased rates of all EXCEPT which of
the following?
a. Prepregnancy BMI
a. Hypertension
b. Gestational diabetes
b. Wound infection
c. Gestational weight gain
c. Vaginal delivery
d. Pregestational diabetes
d. Gestational diabetes
48–24. Which of following contributes to childhood
48–19. Which of the following statements regarding obesity obesity?
and pregnancy is not true?
a. Maternal obesity
a. Obese women are more likely to breast feed.
b. Maternal gestational diabetes
b. Postpartum depression rates are increased in
obese women. c. Maternal-child environment subsequent to birth
c. Obese women have greater weight retention d. All of the above
1 year after delivery.
d. Second-trimester dilatation and evacuations take
longer in obese women.
O besity 323

48–25. During the prenatal care of an obese woman, which 48–27. Regarding cesarean delivery in obese women, which
of the following commonly collected measures will of the following should be done to reduce the
likely be LEAST useful in patient management? chances of wound disruption?

C
a. Fundal height a. Vertical skin incision

H
A
b. Urine protein b. Weight loss during the pregnancy

P
T
c. Blood pressure c. Closure of the subcutaneous layer

E
R
d. Cervical dilatation d. All of the above

4
8
48–26. When performing a cesarean delivery on the obese 48–28. Which of the following is not recommended by the
pregnant woman, which of the following is not an American College of Obstetricians and Gynecologists
advantage of the incision depicted in this figure? to lower thromboembolic complications in the obese
parturient?
a. Hydration
b. Early mobilization
c. Compression stockings
d. Full anticoagulation with heparin

48–29. Which of the following complications is more likely


in a pregnant woman who has undergone bariatric
surgery?
a. Diabetes
b. Preeclampsia
c. Large-for-gestational age neonate
d. Small-for-gestational age neonate

48–30. Which of the following complications is more


common after Roux-en-Y gastric bypass than after
gastric banding?
a. Hypertension
b. Cesarean delivery
c. Gestational diabetes
A d. Small-for-gestational age newborn

48–31. A 30-year-old G3P2 at 14 weeks’ gestation presents


for prenatal care. She reports that she had bariatric
surgery 2 years ago, losing 100 pounds. You
correctly counsel her which of the following?
a. She is at increased risk for diabetes.
b. She is at increased risk for hypertension.
c. She does not need to see her bariatric doctor
during the pregnancy.
d. She needs to be assessed for vitamin deficiencies,
and she may need vitamin B12, vitamin D, or
calcium supplementation.
B

Reproduced with permission from Cunningham FG, Leveno KJ, Bloom SL, et al (eds):
48–32. When caring for a pregnant woman who has
Obesity. In Williams Obstetrics, 24th ed. New York, McGraw-Hill, 2014, Fig. 48-7A&B. undergone bariatric surgery, vigilance for which of
the following is essential?
a. Provides the least intervening tissue a. Appendicitis
b. Permits access to the lower uterine segment b. Bowel obstruction
c. Greatly reduces the chances of wound infection c. Acute cholecystitis
d. Prevents moisture from collecting on the incision d. Gastroesophageal reflux disease
324 Medica l a nd Surgica l Complica tions

CHAPTER 48 ANSw ER KEy

Q uestion Letter Pa ge
S
number a nswer cited Hea der cited
E
C
T
48–1 a p. 961 O besity
I
O
48–2 c p. 961 Definitions
N
1
48–3 b p. 961 Definitions
2
48–4 d p. 962 Figure 4 8 -1
48–5 a p. 962 Figure 4 8 -2
48–6 c p. 961 Adipose Tissue a s a n O rga n System
48–7 c p. 962 Adipocytokines in Pregna ncy
48–8 c p. 963 Ta ble 4 8 -1
48–9 d p. 962 Meta bolic Syndrome
48–10 c p. 963 N ona lcoholic Fa tty Liver Disea se
48–11 d p. 964 Pregna ncy a nd O besity
48–12 c p. 965 Ma terna l W eight G a in a nd Energy Requirement
48–13 d p. 965 Ma terna l W eight G a in a nd Energy Requirement
48–14 d p. 965 Ta ble 4 8 -3
48–15 d p. 966 Figure 4 8 -5
48–16 b p. 966 Figure 4 8 -6
48–17 c p. 965 Ma terna l Morbidity
48–18 c p. 965 Ma terna l Morbidity
48–19 a p. 965 Ma terna l Morbidity
48–20 d p. 966 Preecla mpsia
48–21 a p. 967 Perina ta l Morta lity
48–22 c p. 967 Perina ta l Morbidity
48–23 a p. 967 Perina ta l Morbidity
48–24 d p. 967 Feta l Progra mming a nd Childhood Morbidity
48–25 a p. 968 Prena ta l Ca re
48–26 c p. 968 Figure 4 8 -7
48–27 c p. 968 Surgica l a nd Anesthetic Concerns
48–28 d p. 968 Surgica l a nd Anesthetic Concerns
48–29 d p. 969 Ba ria tric Surgery
48–30 d p. 969 Ta ble 4 8 -4
48–31 d p. 969 Recommenda tions
48–32 b p. 969 Recommenda tions
325

CHAPTER 49
00

Cardiovascular Disorders

49–1. What percent o pregnancies are complicated by 49–6. All EXCEPT which o the ollowing are
heart disease? electrocardiogram changes seen in normal
a. 0.1% pregnancy?
b. 1% a. Increased voltage
c. 4% b. 15-degree le t axis deviation
d. 8% c. Premature atrial contractions
d. ST segment changes in in erior leads
49–2. Which o the ollowing parameters increases by
40 percent in pregnancy? 49–7. A pregnant woman with preexisting cardiac disease is
a. Heart rate com ortable at rest but cannot stand up to brush her
teeth without experiencing chest pain. What is her
b. Cardiac output New York Heart Association classi ication?
c. Mean arterial pressure a. I
d. Le t ventricular stroke work index b. II
49–3. Which aspect o cardiac physiology does not change c. III
in pregnancy? d. IV
a. Mean arterial pressure
49–8. Excluding those associated with Mar an syndrome,
b. Pulmonary vascular resistance which congenital heart lesions carry the greatest risk
c. Le t ventricular contractility o heritability?
d. Le t ventricular stroke work index a. Aortic stenosis
b. Pulmonary stenosis
49–4. Which o the ollowing is never a normal inding in
pregnancy? c. Tetralogy o Fallot
a. Dyspnea d. Coarctation o the aorta
b. Systolic murmur 49–9. A pregnant patient at 13 weeks’ gestation has known
c. Diastolic murmur heart disease. All EXCEPT which o the ollowing
d. Exercise intolerance are predictive o poor outcomes in pregnancy?
a. Prior stroke
49–5. When encountered in an obstetrical patient, which b. Cardiac symptoms at rest
o the ollowing are considered expected variations
rom the nonpregnant state? c. Ejection raction o 30%
a. Larger cardiac silhouette seen on chest radiograph d. Mitral valve area o 2.5 cm2
b. Tricuspid regurgitation seen during 49–10. Which o the ollowing statements is true regarding
echocardiography antepartum and intrapartum care o patients with
c. Increased le t atrial end-diastolic dimensions seen cardiovascular disease?
during echocardiography a. Vaginal delivery is pre erred.
d. All o the above b. Spinal blockade is the recommended anesthetic.
c. These patients should avoid pneumococcal
vaccination.
d. Invasive monitoring with pulmonary artery
catheter is required.
326 Medica l a nd Surgica l Complica tions

49–11. A patient with prior valve replacement heart surgery 49–14. I a patient requires cardiac bypass or valve
presents or prenatal care. Her chest radiograph replacement during pregnancy, which o the
lateral view is shown here and arrows point to her ollowing can she expect?
S
St. Jude valve. With mechanical valves, what is the a. Fetal death rate o 5%
E
C
maternal mortality rate in pregnancy?
b. Minimal miscarriage risk
T
I
O
c. Maternal mortality rate o 20%
N
d. Maternal mortality rate similar to nonpregnant
1
2
women

49–15. A pregnant patient with mitral stenosis requires


percutaneous transcatheter balloon dilatation o the
mitral valve. What can you tell the patient as part o
your presurgical counseling?
a. It is pre erred to open surgical repair.
b. Le t atrial pressures will improve as the mitral
valve area increases.
c. Balloon dilatation is success ul in more than 90
percent o procedures.
d. All o the above

a. 0.1–0.4% 49–16. A patient who had a heart transplant as a child is


now considering pregnancy. What complications can
b. 3–4% she expect during pregnancy?
c. 7–9% a. A tissue rejection risk less than 5%
d. 12–15% b. A high likelihood o vaginal delivery
49–12. In a pregnant woman with a mechanical heart valve, c. A risk o hypertension that approximates 50%
which o the ollowing anticoagulation regimens d. A heart that will not undergo the normal
would be considered inadequate or prevention o physiologic changes o pregnancy
thromboembolism involving the prosthesis?
a. War arin 49–17. A patient in labor gives a history o mitral stenosis.
She is asymptomatic and currently taking a
b. Low-dose un ractionated heparin beta blocker, but she is not sure why her doctor
c. Adjusted-dose un ractionated heparin recommended it. What complication is the
d. Adjusted-dose low-molecular-weight heparin prophylactic beta blocker trying to prevent in these
patients?
49–13. Which o the ollowing anticoagulants is not a. Irregular heart rhythm
compatible with breast eeding? b. Mural thrombus ormation
a. War arin c. Tachycardia leading to pulmonary edema
b. Un ractionated heparin d. Le t ventricular hypertrophy and dilatation
c. Low-molecular-weight heparin
d. All are compatible
Ca rdiova scula r Disorders 327

49–18. The patient whose heart is seen in this image has 49–20. For patients with congenital heart disease, what
systemic lupus erythematosus and has had a prior is the most common adverse cardiovascular event
stroke. The le t atrium (LA) and le t ventricle (LV) encountered in pregnancy?

C
are identi ied, and the arrows point to nonin ectious a. Arrhythmia

H
in lammatory vegetations on the mitral lea lets.

A
b. Heart ailure

P
Which o the ollowing is the likely condition

T
associated with her mitral insu iciency? c. Thromboembolic event

E
R
d. Cerebrovascular hemorrhage

4
9
49–21. Which congenital de ect is associated most o ten
with “paradoxical embolism”?
a. Eisenmenger syndrome
b. Atrial septal de ect
c. Patent ductus arteriosus
d. Ventricular septal de ect

49–22. A patient with the congenital lesion shown in this


igure is now pregnant. She has not had surgery to
correct her lesion, and her hematocrit is 68 volumes
percent. Her expected risk or pregnancy loss
approximates which o the ollowing?

Aorta

P ulmona ry
a rte ry
Na rrow pulmona ry
Modif ed with permission rom Roldan CA: Connective tissue disease and the heart. In outflow tra ct
Craw ord MH (ed): Current Diagnosis & Treatment: Cardiology, 3rd ed. New York, “Ove rriding”
McGraw-Hill, 2009, Figure 33-2. a ortic va lve

a. Antiphospholipid antibodies
b. In arction o the papillary muscle
Ve ntricula r
c. Calci ication o the mitral annulus s e pta l de fe ct Right ve ntricula r
hype rtrophy
d. None o the above

49–19. A patient with critical aortic stenosis presents to


the labor and delivery unit, and her cervix is 6-cm
dilated. All EXCEPT which o the ollowing are
suitable management steps? Reproduced with permission rom Harris IS, Foster E: Congenital heart disease in adults.
In Craw ord MH (ed): Current Diagnosis & Treatment: Cardiology, 3rd ed. New York,
a. Limit activity McGraw-Hill, 2009, Figure 28-23.

b. Provide narcotic epidural anesthesia


c. Per orm pulmonary artery catheterization a. 40%
d. Decrease cardiac preload by limiting intravenous b. 60%
luids c. 80%
d. 100%
328 Medica l a nd Surgica l Complica tions

49–23. This echocardiography image shows a short-axis view 49–27. An obese primigravida develops acute dyspnea while
o the heart o a emale with Eisenmenger syndrome. in labor. Her chest radiograph is shown here. What
A markedly dilated pulmonary artery (PA) is seen. intervention will most likely resolve her symptoms?
S
All EXCEPT which o the ollowing are common
E
C
underlying de ects that lead to this condition?
T
I
O
N
1
2
a. Intravenous urosemide
Reproduced with permission rom Harris IS, Foster E: Congenital heart disease in adults.
In Craw ord MH (ed): Current Diagnosis & Treatment: Cardiology, 3rd ed. New York, b. Intravenous hydralazine
McGraw-Hill, 2009, Figure 28-26B.
c. 3 L oxygen by ace mask
d. Therapeutic heparinization
a. Atrial septal de ect
b. Valvular heart disease 49–28. With in ective endocarditis, what is the most
c. Patent ductus arteriosus common causative organism in intravenous drug
d. Ventricular septal de ect users?
a. Enterococcus species
49–24. Which World Health Organization classi ication b. Staphylococcus aureus
best categorizes patients with idiopathic pulmonary
c. Streptococcus viridans
hypertension?
d. Staphylococcus epidermidis
a. I
b. II 49–29. Which o the ollowing is an adequate intravenous
c. III regimen or prophylaxis o bacterial endocarditis or
d. IV the at-risk patient in labor who is penicillin allergic?
a. Ce azolin, 1 g
49–25. A pregnant patient develops heart ailure without an b. Ce triaxone, 1 g
identi iable underlying cause. I she does not recover
c. Clindamycin, 600 mg
baseline cardiac unction by 6 months postpartum,
which o the ollowing best approximates her 5-year d. All are adequate
mortality rate?
49–30. Patients with QT prolongation are at risk o
a. 5%
developing which o the ollowing arrhythmias?
b. 20%
a. Atrial ibrillation
c. 40%
b. Torsades de pointes
d. 80%
c. Wol -Parkinson-White
49–26. What is the most common cause o heart ailure d. Paroxysmal supraventricular tachycardia
during pregnancy and the puerperium?
a. Obesity
b. Viral myocarditis
c. Valvular heart disease
d. Chronic hypertension with severe preeclampsia
Ca rdiova scula r Disorders 329

49–31. Which o the ollowing is a threshold or aortic root 49–32. A 42-year-old gravida at 32 weeks’ gestation
size above which a patient with Mar an syndrome is with diabetes presents with chest pain and has an
at higher risk o dissection during pregnancy? abnormal electrocardiogram. All EXCEPT which o

C
a. 20 mm the ollowing statements are correct regarding the

H
evaluation and management o suspected myocardial

A
b. 30 mm

P
in arction in pregnancy?

T
c. 40 mm

E
a. The patient should be delivered immediately.

R
d. 50 mm
b. The patient should be started on a beta blocker.

4
9
c. The patient should receive daily low-dose aspirin.
d. Troponin levels can be used to aid diagnosis in
pregnancy.
330 Medica l a nd Surgica l Complica tions

CHAPTER 49 ANSw ER KEy

Q uestion Letter Pa ge
S
number a nswer cited Hea der cited
E
C
T
49–1 b p. 973 Introduction
I
O
49–2 b p. 973 Ca rdiova scula r Physiology; Ta ble 4 9 -1
N
1
49–3 c p. 973 Ca rdiova scula r Physiology
2
49–4 c p. 974 Dia gnosis of Hea rt Disea se; Ta ble 4 9 -2
49–5 d p. 975 Dia gnostic Studies
49–6 a p. 976 Electroca rdiogra phy
49–7 c p. 976 Cla ssifica tion of Functiona l Hea rt Disea se
49–8 a p. 976 Congenita l Hea rt Disea se in O ffspring; Ta ble 4 9 -4
49–9 d p. 976 Cla ssifica tion of Functiona l Hea rt Disea se
49–10 a p. 977 Peripa rtum Ma na gement Considera tions
49–11 b p. 979 Va lve Repla cement before Pregna ncy
49–12 b p. 979 Anticoa gula tion
49–13 d p. 979 Recommenda tions for Anticoa gula tion
49–14 d p. 980 Ca rdia c Surgery During Pregna ncy
49–15 d p. 980 Mitra l Va lvotomy During Pregna ncy
49–16 c p. 981 Pregna ncy a fter Hea rt Tra nspla nta tion
49–17 c p. 981 Mitra l Stenosis
49–18 a p. 982 Mitra l Insufficiency
49–19 d p. 983 Aortic Stenosis
49–20 a p. 984 Congenita l Hea rt Disea se
49–21 b p. 984 Atria l Septa l Defects
49–22 d p. 985 Cya notic Hea rt Disea se
49–23 b p. 985 Eisenmenger Syndrome
49–24 a p. 987 Ta ble 4 9 -8
49–25 d p. 988 Ca rdiomyopa thies; Prognosis
49–26 d p. 990 Hea rt Fa ilure
49–27 a p. 990 Hea rt Fa ilure: Ma na gement
49–28 b p. 990 Infective Endoca rditis
49–29 d p. 991 Ta ble 4 9 -1 0
49–30 b p. 992 Prolonged Q T Interva l
49–31 c p. 993 Effect of Pregna ncy on Ma rfa n Syndrome
49–32 a p. 994 Myoca rdia l Infa rction During Pregna ncy
331

CHAPTER 50
00

Chronic H pertension

50–1. A 36-year-old primigravida has a 2-year history of 50–5. What is the target blood pressure you are attempting
chronic hypertension that is well controlled with to achieve with this therapy?
medication. She monitors her blood pressure at a. < 120/90 mm Hg
home, and at her 16-week appointment, she is
b. < 130/80 mm Hg
thrilled to report a drop in her blood pressure. What
is the best response to this information? c. < 140/90 mm Hg
a. She should discontinue her medication at this d. < 150/100 mm Hg
time.
50–6. Which of the following factors is an indication for
b. Chronic hypertension typically improves early in
evaluation of renal function in a patient with chronic
pregnancy.
hypertension who is considering pregnancy?
c. This finding predicts a favorable prognosis for her
a. Age of 45 years
pregnancy.
b. Preexisting diabetes mellitus
d. None of the above
c. Hypertension duration of 3 years
50–2. Which of the following factors may influence blood d. All of the above
pressure?
a. Race 50–7. How should a patient whose blood pressure is
120/80 mm Hg at her annual examination be
b. Weight
counseled?
c. Gender
a. Initiate 30 minutes of daily exercise
d. All of the above
b. Limit alcohol consumption to two drinks per day
50–3. Which of the following blood pressure measurements c. Start pravastatin to reduce her cardiovascular risk
meets the criteria for “prehypertension” as defined d. Plan no recommendation as her blood pressure is
by the Joint National Committee 7? normal
a. Diastolic blood pressure 70–80 mm Hg
b. Diastolic blood pressure 90–95 mm Hg
c. Systolic blood pressure 120–130 mm Hg
d. Systolic blood pressure 140–150 mm Hg

50–4. An African-American patient presents for her


annual gynecologic examination and has a blood
pressure reading of 142/92 mm Hg. After checking
it at home several more times, she calls to report
persistent readings of 140–150/90–100 mm Hg.
What medication is appropriate as initial therapy for
most nonpregnant patients?
a. Enalapril
b. Labetalol
c. Losartan
d. Hydrochlorothiazide
332 Medica l a nd Surgica l Complica tions

50–8. A new patient with chronic hypertension presents 50–11. The chest radiograph below was obtained from
to discuss future pregnancy. She brings additional a patient with chronic hypertension 1 day after
records with her, including a copy of this image, delivery at 38 weeks’ gestation. Which of the
S
which depicts her cerebrovascular accident from 3 following statements is true regarding this condition?
E
C
years ago. What is the most appropriate response to
T
her question regarding pregnancy outcomes?
I
O
N
1
2
a. It occurs in approximately 1% of gravidas with
chronic hypertension.
b. It is most often seen in gravidas with a history of
cardiomyopathy.
c. It is the most frequent delivery complication seen
in women with chronic hypertension.
Reproduced with permission from Ropper AH, Samuels MA: Cerebrovascular diseases.
In Adams & Victor’s Principles of Neurology, 9th ed. New York, McGraw-Hill, 2009,
d. None of the above
Figure 34-21.
50–12. Administration of which of the following
a. Her risk for recurrence approaches 80%. antihypertensive medications may contribute to
development of the condition seen in Question
b. Pregnancy is relatively contraindicated.
50–11?
c. Outcomes are good if blood pressure is controlled
a. Losartan
prior to conception.
b. Nifedipine
d. If her serum creatinine level is < 2.5 mg/dL, the
pregnancy should be uneventful. c. Lisinopril
d. Hydralazine
50–9. The diagnosis of chronic hypertension is supported
when hypertension is present prior to what threshold 50–13. Which of the following postpartum events
gestational age? predisposes to development of the condition in
a. 8 weeks Question 50–11?
b. 14 weeks a. Increased peripheral resistance
c. 20 weeks b. Increased left-ventricular workload
d. 24 weeks c. Increased intravascular fluid volume
d. All of the above
50–10. Which of the following comorbidities is most
frequently seen in pregnant women with chronic 50–14. What treatment is indicated when this develops
hypertension? postpartum?
a. Hypothyroidism a. Digoxin
b. Pregestational diabetes b. Furosemide
c. Systemic lupus erythematosus c. Hydralazine
d. Antiphospholipid antibody syndrome d. Fluid restriction
Chronic Hypertension 333

50–15. The risk of maternal death is increased by what 50–18. What dietary supplement may reduce the incidence
degree in women with chronic hypertension? the event shown in Question 50–16?
a. Twofold a. Vitamin C

C
b. Fivefold b. Vitamin E

H
A
c. Tenfold c. Folic acid

P
T
d. Fiftyfold d. Omega-3 fatty acids

E
R
5
50–16. What is the risk of the following pregnancy 50–19. Serum assays for which of the following can

0
complication in women with chronic hypertension? accurately predict the development of superimposed
preeclampsia in a patient with chronic hypertension?
a. Inhibin
Ute rine wa ll b. Pregnancy-associated plasma protein A
c. Vascular endothelial growth factor receptor 1
d. None of the above
P la ce nta
50–20. Results from randomized controlled trials suggest
what benefit is gained from low-dose aspirin
use during pregnancy in women with chronic
hypertension?
a. No benefit
b. Reduced preterm birth rates
c. Reduced maternal mortality rates
d. Reduced fetal-growth restriction rates

50–21. A patient with no prenatal care presents to Labor


He mmorha ge and Delivery in the third trimester with a complaint
of decreased fetal movement for 1 day. She takes
an unknown medication for chronic hypertension.
What is the risk of stillbirth in a gravida with
chronic hypertension?
a. 1 per 20 births
Reproduced with permission from Cunningham FG, Leveno KJ, Bloom SL, et al (eds): b. 1–2 per 100 births
Obstetrical hemorrhage. In Williams Obstetrics, 23rd ed. New York, McGraw-Hill, 2010,
Figure 35-4.
c. 1–2 per 1000 births
d. 1 per 5000 births
a. 0.1%
b. 0.3–0.5%
c. 1–2%
d. 10%

50–17. For women with chronic hypertension, what


additional factor further increases the risk of the
event shown in Question 50–16?
a. Oligohydramnios
b. Maternal smoking
c. Multifetal gestation
d. Pregestational diabetes
334 Medica l a nd Surgica l Complica tions

50–22. The fetal heart rate tracing for the patient in 50–25. Thiazide medications have which of the following
Question 50-21 is shown below. Sonographic side effects that limit their use in pregnancy?
evaluation is performed, and her amnionic fluid a. Postural hypotension
S
index is 2.3 cm, which is markedly abnormal.
b. Reduced volume expansion
E
C
What class of antihypertensive medications may be
T
responsible for this finding? c. Oligohydramnios in the third trimester
I
O
d. Teratogenicity when taken in the first trimester
N
240
1
2
210 50–26. Which of the following is a true statement regarding
the use of oral hydralazine as an antihypertensive
190
B agent?
150 a. It is an effective monotherapy in pregnancy.
A b. It promotes sodium and water diuresis in
120
pregnant women.
90
c. It should not be combined with calcium-channel
60 blocking agents.
30 d. None of the above

100
50–27. A new obstetrical patient presents for care at 8
weeks’ gestation. Her blood pressure is 146/92.
80
You order an echocardiogram, assess baseline renal
60 function, and send her to an ophthalmologist for
40 evaluation. Her eye examination is normal, and
20 her serum creatinine level is 0.71 mg/dL. Her
echocardiogram reveals moderate left ventricular
0
hypertrophy. What is the most appropriate next step
Reproduced with permission from Cunningham FG, Leveno KJ, Bloom SL, et al (eds): in her management?
Intrapartum assessment. In Williams Obstetrics, 24th ed. New York, McGraw-Hill, 2014,
Figure 24-21. a. Start antihypertensive therapy
b. Proceed with routine prenatal care
a. Loop diuretics c. Repeat the entire evaluation in the second
b. β -Adrenergic receptor blockers trimester
c. Calcium-channel blocking agents d. Plan expectant management with biweekly blood
d. Angiotensin-converting enzyme inhibitors pressure checks

50–23. A 30-year-old G3P2 presents for prenatal care. 50–28. Regarding the patient in Question 50-27, what is
Her prior pregnancies have both been complicated her risk of developing superimposed preeclampsia?
by mild gestational hypertension near term. She is a. 5%
currently 12 weeks pregnant, and her blood pressure b. 10%
is 146/88 mm Hg. Which of the following is the c. 20–30%
most appropriate next step in her management?
d. 50%
a. Continued observation
b. Begin treatment with methyldopa 50–29. If the patient from Question 50-27 actually develops
c. Begin treatment with hydrochlorothiazide superimposed preeclampsia, what is the likelihood
d. None of the above she will deliver before 37 weeks’ gestation?
a. 2%
50–24. Which of the following pregnancy complications b. 10%
is reduced when antihypertensive therapy is started c. 20%
during pregnancy?
d. 50%
a. Preterm delivery
b. Maternal morbidity
c. Development of severe hypertension
d. Neonatal intensive care admissions
Chronic Hypertension 335

50–30. All EXCEPT which of the following support the 50–32. Which of the following medications is preferred for
diagnosis of superimposed preeclampsia? intrapartum prevention of eclampsia?
a. Decreased platelet count a. Phenytoin

C
b. Increased serum creatinine level b. Midazolam

H
A
c. Elevated serum alkaline phosphatase level c. Phenobarbital

P
T
d. Elevated serum aspartate aminotransferase level d. Magnesium sulfate

E
R
5
50–31. According to the American College of Obstetricians

0
and Gynecologists, what antenatal assessment has
been conclusively shown to be of benefit in the
management of pregnancies complicated by chronic
hypertension?
a. Nonstress testing
b. Biophysical profile
c. Umbilical artery Doppler studies
d. Serial sonographic evaluations of fetal growth
336 Medica l a nd Surgica l Complica tions

CHAPTER 50 ANSw ER KEy

Q uestion Letter Pa ge
S
number a nswer cited Hea der cited
E
C
T
50–1 b p. 1000 Introduction
I
O
50–2 d p. 1000 G enera l Considera tions
N
1
50–3 c p. 1001 Ta ble 5 0 -1
2
50–4 d p. 1001 Ta ble 5 0 -1
50–5 c p. 1001 Trea tment a nd Benefits for N onpregna nt Adults
50–6 b p. 1001 Preconceptiona l Counseling
50–7 a p. 1001 Trea tment a nd Benefits for N onpregna nt Adults; Ta ble 5 0 -2
50–8 b p. 1001 Preconceptiona l Counseling
50–9 c p. 1002 Dia gnosis a nd Eva lua tion in Pregna ncy
50–10 b p. 1002 Associa ted Risk Fa ctors
50–11 d p. 1003 Ma terna l Morbidity a nd Morta lity
50–12 b p. 1006 Ca lcium-Cha nnel Blocking Agents
50–13 d p. 1008 Postpa rtum Hypertension
50–14 b p. 1008 Postpa rtum Hypertension
50–15 b p. 1003 Ma terna l Morbidity a nd Morta lity
50–16 c p. 1004 Pla centa l Abruption
50–17 b p. 1004 Pla centa l Abruption
50–18 c p. 1004 Pla centa l Abruption
50–19 d p. 1004 Prevention of Superimposed Preecla mpsia
50–20 a p. 1004 Prevention of Superimposed Preecla mpsia
50–21 b p. 1004 Perina ta l Morbidity a nd Morta lity
50–22 d p. 1006 Angiotensin-Converting Enzyme Inhibitors
50–23 a p. 1005 Blood Pressure Control
50–24 c p. 1005 Blood Pressure Control
50–25 b p. 1005 Diuretics
50–26 d p. 1005 Va sodila tors
50–27 a p. 1007 Recommenda tions for Thera py in Pregna ncy
50–28 c p. 1007 Pregna ncy-Aggra va ted Hypertension or Superimposed Preecla mpsia
50–29 d p. 1007 Pregna ncy-Aggra va ted Hypertension or Superimposed Preecla mpsia
50–30 c p. 1007 Pregna ncy-Aggra va ted Hypertension or Superimposed Preecla mpsia
50–31 d p. 1007 Feta l Assessment
50–32 d p. 1007 Intra pa rtum Considera tion
337

CHAPTER 51
00

Pulmonar Disorders

51–1. What per entage pregnant w men have hr ni 51–7. A pregnant patient rep rts asthma sympt ms
asthma? 3 days per week, and she is awakened by these
a. 1% appr ximately twi e per m nth. Between
exa erbati ns, her FEV1 ( r ed expirat ry v lume in
b. 4%
1 se nd) is ≥ 80 per ent predi ted. Her asthma is
c. 14% best lassi ied by whi h the ll wing des ript rs?
d. 20% a. Intermittent
b. Mild persistent
51–2. What happens t vital apa ity in pregnan y?
c. M derate persistent
a. In reases by 20%
d. Very severe persistent
b. De reases by 25%
c. In reases by 40% 51–8. A pregnant patient rep rts asthma sympt ms that
d. De reases by 45% pr mpt daily use her inhaler, and she is awakened
by these sympt ms appr ximately twi e weekly.
51–3. What e e t d es pr gester ne have n tidal v lume? Between exa erbati ns, her FEV1/FVC ( r ed
a. In reases by 20% expirat ry v lume in 1 se nd/ r ed vital apa ity)
is redu ed by 5 per ent. Her asthma is best lassi ied
b. De reases by 25%
by whi h the ll wing des ript rs?
c. In reases by 40%
a. Intermittent
d. De reases by 45%
b. Mild persistent
51–4. What happens t arb n di xide pr du ti n in c. M derate persistent
pregnan y? d. Very severe persistent
a. In reases by 20%
51–9. Whi h the ll wing best hara terizes persistent
b. De reases by 25%
asthma?
c. In reases by 30%
a. Sympt ms n e daily
d. De reases by 40%
b. FEV1 < 60% predi ted
51–5. What happens t residual v lume in pregnan y? c. S me limitati n n rmal a tivities
a. In reases by 20% d. Single, daily use sh rt-a ting β -ag nists r
sympt ms
b. De reases by 20%
c. In reases by 40% 51–10. A 25-year- ld G1P0 at 20 weeks’ gestati n
d. De reases by 40% presents with an asthma exa erbati n. Her FEV1
is 50 per ent. Her Pco 2 is n rmal, and her Po 2 is
51–6. Whi h the ll wing statements regarding de reased. hese values are very n erning and
physi l gi pulm nary hanges in pregnan y is true? likely re le t whi h the ll wing?
a. Basal xygen nsumpti n de reases. a. She is in respirat ry ailure.
b. Ventilati n is in reased by m re requent b. She has a signi i ant metab li a id sis.
breathing. c. A n rmal Pco 2 indi ates atigue and early C 2
c. Ventilati n is in reased be ause deeper retenti n.
breathing. d. A etus ann t survive when the m ther’s FEV1
d. Ventilati n is in reased be ause in reased hest dr ps t 50%.
wall mplian e.
338 Medica l a nd Surgica l Complica tions

51–11. Up t what per entage w men with mild 51–17. Whi h the ll wing d es n t des ribe
r m derate asthma will have an intrapartum the phylline?
exa erbati n? a. Methylxanthine
S
a. 2% b. Br n h dilat r
E
C
b. 20%
T
c. C rti ster id
I
O
c. 40% d. Antiin lammat ry agent
N
d. 60%
1
2
51–18. Whi h the ll wing is e e tive r a ute asthma?
51–12. he etal resp nse t maternal hyp xemia is whi h a. Zileut n
the ll wing?
b. Cr m lyn
a. De reased ardia utput
c. M ntelukast
b. In reased umbili al bl d l w
d. Epinephrine
c. De reased systemi vas ular resistan e
d. De reased pulm nary vas ular resistan e 51–19. A 19-year- ld G2P1 at 23 weeks’ gestati n presents
with an asthma exa erbati n. Her FEV1 is 35%,
51–13. Whi h the ll wing is a sign a p tentially atal and she is a ebrile and therwise well. Y u treat her
asthma atta k? with an inhaled β -ag nist treatment and inhaled
a. a hy ardia rti ster ids. A ter three d ses the inhaled
β -ag nist, the patient has an xygen saturati n
b. Central yan sis
94% and an FEV1 45%. Whi h the ll wing
c. Lab red breathing is the best next management step?
d. Pr l nged expirati n a. rans er t the intensive are unit r status
asthmati us
51–14. Whi h the ll wing statements regarding
b. Dis harge her h me with a sh rt-a ting β -ag nist
pulm nary un ti n testing is true?
inhaler and s hedule a 2-week ll w-up i e
a. An FEV1 50% rrelates with severe disease. app intment
b. he peak expirat ry l w rate in reases in n rmal c. Admit r additi nal treatment with inhaled
pregnan y. β -ag nists and intraven us rti ster ids and
c. A w man’s baseline pulm nary un ti n tests bserve r respirat ry distress
sh uld be determined when sympt mati . d. Dis harge her h me with β -ag nist and
d. Pulm nary un ti n testing sh uld be r utine in rti ster id nebulizer treatments and with
the management hr ni and a ute asthma. br ad-spe trum antibi ti s r presumed
pneum nia
51–15. he re mmended treatment r mild persistent
asthma sympt ms in ludes a sh rt-a ting β -ag nist 51–20. A pregnant w man is treated r an asthma
and whi h ther agent(s)? exa erbati n at 37 weeks’ gestati n with a regimen
a. N ther agent that in luded systemi rti ster id therapy. She
presents again at 39 weeks with lab r. Whi h the
b. L w-d se inhaled rti ster ids (ICs)
ll wing w uld n t be part y ur management
c. L w-d se ICs, and a l ng-a ting β -ag nist plan?
d. High-d se ICs, and a l ng-a ting β -ag nist a. Epidural anesthesia, when and i desired by the
patient
51–16. he re mmended treatment r severe persistent
b. Determinati n FEV1 up n admissi n t Lab r
asthma sympt ms in ludes a sh rt-a ting β -ag nist
and Delivery
and whi h ther agent(s)?
c. reatment p stpartum hem rrhage with
a. L w-d se inhaled rti ster ids (ICs) arb pr st tr methamine
b. L w-d se ICs, and a l ng-a ting β -ag nist d. Hydr rtis ne 100 mg intraven usly every
c. High-d se ICs, and a l ng-a ting β -ag nist 8 h urs during lab r and r 24 h urs a ter
d. High-d se ICs, a l ng-a ting β -ag nist, and ral delivery
rti ster ids
Pulmona ry Disorders 339

51–21. n average, h w l ng d es the ugh a ute 51–25. A 19-year- ld G1P0 at 22 weeks’ gestati n with n
br n hitis persist? signi i ant past medi al hist ry presents with ugh,
a. 1–2 days ever, and hest pain n inspirati n r 3 days.

C
She has a ever 38.5°C, has 22 respirati ns per
b. 3–4 days

H
minute, and is xygenating well. he patient’s hest

A
c. 5–7 days

P
radi graph is pr vided here. he best treatment r

T
d. 10–20 days this patient is whi h the ll wing?

E
R
5
51–22. Whi h the ll wing is n t a risk a t r r

1
pneum nia?
a. Asthma
b. Pregnan y
c. Binge drinking
d. Chr ni br n hitis

51–23. Whi h the ll wing m st mm nly auses


pneum nia?
a. In luenza A
b. Mycoplasma pneumoniae
c. Legionella pneumophila
d. Chlamydophila pneumoniae
A
51–24. Whi h the ll wing statements regarding
pneum nia is true?
a. Chest radi graphy is essential r diagn sis.
b. Chest radi graphy a urately predi ts the
eti l gy.
c. he resp nsible path gen an be identi ied in
m st ases.
d. Sputum ultures are re mmended in all
suspe ted ases pneum nia.

Repr du ed with permissi n r m Cunningham FG, Leven KJ, Bl m SL, et al (eds):


Pulm nary dis rders. In Williams bstetri s, 24th ed. New Y rk, M Graw-Hill, 2014,
Figure 51-3.

a. Van my in
b. M xi l xa in
c. Azithr my in
d. Lev l xa in and van my in
340 Medica l a nd Surgica l Complica tions

51–26. What per entage w men with pneum nia devel p 51–29. A 30-year- ld G2P1 at 15 weeks’ gestati n presents
pleural e usi n? with ugh, ever, and weight l ss. She has re ently
a. 2% immigrated t the United States r m Mexi .
S
Y u send a QuantiFER N- B G ld, whi h yields
b. 20%
E
C
a p sitive result. he patient’s hest radi graphy
T
c. 40% is pr vided bel w. Sputum is p sitive r a id-
I
O
d. 60% ast ba illi. Whi h the ll wing is the rre t
N
diagn sis r this patient?
1
2
51–27. A 25-year- ld G3P2 at 19 weeks’ gestati n presents
with ever, ugh, myalgias, and nausea r 2 days.
Her tw hildren at h me have als been si k. By
rapid test, the patient is diagn sed with in luenza A.
he best treatment regimen r this patient is whi h
the ll wing?
a. Amantadine r 5 days
b. Amantadine r 10 days
c. seltamivir r 5 days
d. seltamivir and amantadine r 10 days

51–28. A 33-year- ld G3P0 at 18 weeks’ gestati n presents


r prenatal are. She is br ught t the lini by
a deputy as she is urrently in ar erated n drug
harges. he patient ame t this untry 1 year ag .
She rep rts having previ usly re eived the ba ille
Calmette-Guérin (BCG) va ine. By her hist ry,
she has multiple risks r tuber ul sis. Whi h the
ll wing is the best initial test r this patient?
a. Chest radi graphy
b. Sputum lle ti n
Repr du ed with permissi n r m Phan VD, P p ni k JM: uber ul sis. In intinalli JE,
c. uber ulin skin test Stap zynski JS, Cline DM, et al (eds): intinalli’s Emergen y Medi ine: A C mprehensive
Study Guide, 7th ed. New Y rk, M Graw-Hill, 2011, Figure 70-2.
d. Inter er n-gamma release assay (IGRA)
a. Sar id sis
b. A tive tuber ul sis
c. Latent tuber ul sis
d. Extrapulm nary tuber ul sis

51–30. Whi h the ll wing is the best management plan


r the patient in Questi n 51–29?
a. Initiate is niazid
b. Initiate is niazid and ri ampin
c. Initiate is niazid, ri ampin, ethambut l,
pyrazinamide, and pyrid xine
d. Delay treatment until a ter delivery, then initiate
is niazid, ri ampin, and pyrid xine
Pulmona ry Disorders 341

51–31. A 38-year- ld G1P0 at 22 weeks’ gestati n presents 51–32. his image sh ws ive lasses geneti mutati ns
r prenatal are. he patient has a tive sar id sis, a e ting a peptide that ultimately un ti ns in
and a ph t graph her hands is pr vided here. epithelial- ell membrane transp rt ele tr lytes.

C
Regarding sar id sis in pregnan y, y u rre tly hese mutati ns ause whi h the ll wing

H
unsel her whi h the ll wing? diseases?

A
P
T
Cla s s IV:

E
R
de fe ctive
conduction

5
1
Apica l

+
P KA Cla s s III:
de fe ctive
re gula tion

Repr du ed with permissi n r m W l K, J hns n RA (eds): T e skin in immune, aut - Golgi


immune, and rheumati dis rders. In Fitzpatri k’s C l r Atlas and Syn psis Clini al
Dermat l gy, 6th ed. New Y rk, M Graw-Hill, 2009, Figure 14-51.
Cla s s II:
de fe ctive
a. Perinatal ut mes are p r. proce s s ing

b. Disease pr gressi n in pregnan y is mm n.


ER
c. Severe disease warrants serial determinati n
pulm nary un ti n.
d. All mm n treatment m dalities are abs lutely
Cla s s V:
ntraindi ated in pregnan y. Cla s s I:
re duce d
de fe ctive
prote in functioning
s ynthe s is CFTR
prote in

Ba s ola te ra l

Repr du ed with permissi n r m B u her RC: Cysti f br sis. In L ng DL, Fau i AS,
Kasper DL, et al (eds): Harris n’s Prin iples Internal Medi ine, 18th ed. New Y rk,
M Graw-Hill, 2012, Figure 259-1.

a. Sar id sis
b. Cysti ibr sis
c. Maple syrup urine disease
d. Primary iliary dyskinesia

51–33. he rganism m st str ngly ass iated with ysti


ibr sis is whi h the ll wing?
a. Burkholderia cepacia
b. Staphylococcus aureus
c. Pseudomonas aeruginosa
d. Haemophilus influenzae
342 Medica l a nd Surgica l Complica tions

51–34. Whi h the ll wing statements regarding ysti 51–36. Fetal heart tra ing indings in ases maternal
ibr sis is true? arb n m n xide p is ning in lude all EXCEPT
a. Disease severity is quanti ied by pulm nary whi h the ll wing?
S
un ti n studies. a. Elevated baseline
E
C
b. When the FEV1 is at least 40%, w men t lerate b. Absen e a elerati ns
T
I
pregnan y well.
O
c. Marked beat-t -beat variability
N
c. he best predi t r pregnan y ut me in d. De reased beat-t -beat variability
1
w men with ysti ibr sis is nutriti nal status.
2
d. Diabetes in pregnan y is less likely i the a e ted
w man is h m zyg us r the ∆F508 mutati n.

51–35. he m st mm n ause p is ning w rldwide is


whi h the ll wing?
a. Ri in
b. Cyanide
c. Stry hnine
d. Carb n m n xide
Pulmona ry Disorders 343

CHAPTER 51 ANSw ER KEy

Q uestion Letter Pa ge

C
number a nswer cited Hea der cited

H
A
P
51–1 b p. 1011 Pulmona ry Disorders

T
E
51–2 a p. 1011 Pulmona ry Disorders

R
5
51–3 c p. 1011 Pulmona ry Disorders

1
51–4 c p. 1011 Pulmona ry Disorders
51–5 b p. 1011 Pulmona ry Disorders
51–6 c p. 1011 Pulmona ry Disorders
51–7 b p. 1012 Clinica l Course
51–8 c p. 1012 Clinica l Course
51–9 b p. 1012 Clinica l Course
51–10 c p. 1012 Clinica l Course
51–11 b p. 1013 Effects of Pregna ncy on Asthma
51–12 a p. 1014 Feta l Effects
51–13 b p. 1014 Clinica l Eva lua tion
51–14 d p. 1014 Clinica l Eva lua tion
51–15 b p. 1014 Ma na gement of Chronic Asthma
51–16 c p. 1014 Ma na gement of Chronic Asthma
51–17 c p. 1014 Ma na gement of Chronic Asthma
51–18 d p. 1014 Ma na gement of Chronic Asthma
51–19 c p. 1015 Ma na gement of Acute Asthma
51–20 c p. 1015 La bor a nd Delivery
51–21 d p. 1015 Acute Bronchitis
51–22 b p. 1016 Ba cteria l Pneumonia
51–23 a p. 1016 Incidence a nd Ca uses
51–24 a p. 1016 Dia gnosis
51–25 c p. 1016 Ma na gement
51–26 b p. 1016 Ma na gement
51–27 c p. 1018 Ma na gement
51–28 d p. 1020 Dia gnosis
51–29 b p. 1020 Dia gnosis
51–30 c p. 1021 Active Infection
51–31 c p. 1022 Sa rcoidosis a nd Pregna ncy
51–32 b p. 1022 Cystic Fibrosis
51–33 c p. 1022 Pa thophysiology
51–34 a p. 1022 Pregna ncy with Cystic Fibrosis
51–35 d p. 1023 Ca rbon Monoxide Poisoning
51–36 c p. 1024 Trea tment
344

CHAPTER 52

Thromboembolic Disorders

52–1. What is the approximate inci ence o 52–6. he mutations that cause antithrombin e iciency
thromboembolic events per pregnancy? are almost always inherite in what ashion?
a. 1/10 a. X-linke ominant
b. 1/100 b. X-linke recessive
c. 1/1000 c. Autosomal ominant
d. 1/10,000 d. Autosomal recessive

52–2. Which o the ollowing statements is correct 52–7. Un erstan ing pregnancy physiology is essential to
regar ing the timing o pregnancy-relate eep-vein iagnose protein S e iciency. Concentrations o
thrombosis (DV ) an pulmonary embolism (PE)? which o the ollowing ecline substantially uring
a. DV an PE are more common antepartum. normal pregnancy?
b. DV an PE are more common postpartum. a. Free protein S
c. DV is more common antepartum, an PE is b. otal protein S
more common postpartum. c. Functional protein S
d. PE is more common antepartum, an DV is d. All o the above
more common postpartum.
52–8. During his irst ay o li e, a seemingly healthy
52–3. All EXCEPT which o the ollowing physiologic neonate who was elivere at term su enly
alterations pre ispose to venous thrombosis in evelops ever an i use skin lesions as shown in
pregnancy? the igure. his is most likely a mani estation o
a. Lower extremity venous stasis what thrombophilic con ition?
b. Decrease plasminogen activity
c. En othelial cell injury at elivery
d. Increase synthesis o clotting actors

52–4. Which o the ollowing risk actors or eveloping


thromboembolism in pregnancy is most important?
a. Cesarean elivery
b. Multi etal gestation
c. Postpartum hemorrhage
d. Personal history o thrombosis Repro uce with permission rom Wolf K, Gol smith LA, Katz SI, et al: Fitzpatrick’s
Dermatology in General Me icine, 7th e . New York, McGraw-Hill, 2008, Figure
180-1C.
52–5. Approximately what percentage o thromboembolic
events that occur uring pregnancy can be attribute
to an inherite or acquire thrombophilia? a. Homozygous prothrombin mutation
a. 1% b. Homozygous protein S e iciency
b. 15% c. Heterozygous protein C e iciency
c. 50% d. Heterozygous actor V Lei en mutation
d. 75%
Thromboembolic Disorders 345

52–9. Which o the ollowing inherite thrombophilia 52–14. Which o the ollowing thrombophilic con itions
syn romes is most prevalent? have been most consistently associate with a verse
a. Protein S e iciency pregnancy outcomes?

C
b. Antithrombin e iciency a. Factor V Lei en mutation

H
A
c. Factor V Lei en mutation b. Antiphospholipi syn rome

P
T
d. Prothrombin G20210A mutation c. Antithrombin mutation

E
R
d. Prothrombin G20210A mutation

5
52–10. Which o the ollowing pregnancy-relate

2
complications are increase in women who are 52–15. A 32-year-ol G2P1 at 29 weeks’ gestation presents
heterozygous carriers o the actor V Lei en or a routine prenatal care visit with no complaints.
mutation? During a routine examination, you note that her
a. Preeclampsia legs have the ollowing appearance. Which o the
ollowing statements is true regar ing this con ition
b. Placental abruption
in pregnancy?
c. Fetal-growth restriction
d. None o the above

52–11. All EXCEPT which o the ollowing can be causes


o elevate homocysteine levels?
a. Folate e iciency
b. Vitamin D e iciency
c. Vitamin B6 e iciency
d. Methylene-tetrahy ro olate re uctase mutation

52–12. he American College o Obstetricians an


Gynecologists currently recommen s assessment
o women or hyperhomocysteinemia an the
methylene tetrahy ro olate re uctase mutation in
what clinical situations?
a. Folic aci e iciency
b. First venous thromboembolism
c. Recurrent venous thromboembolism
d. None o the above

52–13. All EXCEPT which o the ollowing women


illustrate clinical criteria that woul prompt
antiphospholipi syn rome laboratory screening?
a. A 33-year-ol G2P1 with a 12-week etal emise.
a. Most cases are on the right si e.
b. A 41-year-ol G2P1 with a history o eclampsia
at 36 weeks in her prior pregnancy. b. A positive Homans sign is virtually iagnostic.
c. A 26-year-ol G5P1 at 6 weeks’ gestation c. Most cases are locate in the ilio emoral veins.
with spontaneous abortions in her last three d. Symptoms typically correlate with the egree o
pregnancies. vessel involvement.
d. A 29-year-ol G1P0 at 29 weeks’ gestation
un ergoing in uction or severe etal-growth
restriction.
346 Medica l a nd Surgica l Complica tions

52–16. he patient escribe in Question 52–15 un ergoes 52–19. his image obtaine uring venography shows
compression ultrasonography o the lower a illing e ect (arrows) in the popliteal vein.
extremities with the ollowing in ings. he calipers Although this mo ality remains the gol stan ar
S
elineate a vessel lumen. What next step is most or iagnosing eep-vein thrombosis, it is use
E
C
appropriate in her management? in requently or all EXCEPT o the ollowing
T
reasons?
I
O
N
1
2
a. Venography
b. Embolectomy
c. Anticoagulation
d. Ventilation-per usion scan

52–17. Magnetic resonance imaging is a use ul a junctive


imaging technique or iagnosing eep-vein
thrombosis or all EXCEPT which o the ollowing
reasons?
Repro uce with permission rom Liem K, Moneta GL: Venous an lymphatic isease.
a. Lower cost than compression ultrasonography In Brunicar i FC, An ersen DK, Billiar R, et al (e s): Schwartz’s Principles o Surgery,
9th e . New York, McGraw-Hill, 2010, Figure 24-8.
b. Improve evaluation o the ilio emoral veins
c. Ability to iagnosis nonthrombotic con itions
a. It is invasive.
d. Ability to reconstruct pelvic venous system
b. Intravenous contrast is require .
anatomy
c. It has a low negative-pre ictive value.
52–18. d-Dimer concentrations can be elevate in which o d. It increases the risk or proce ure-associate
the ollowing pregnancy-relate complications? thrombosis.
a. Preeclampsia
52–20. Anticoagulation in pregnancy shoul pre erentially
b. Placenta previa
be accomplishe with which me ication?
c. Gestational iabetes
a. War arin
d. Prurigo o pregnancy
b. Dabigatran
c. Un ractionate heparin
d. Low-molecular-weight heparin
Thromboembolic Disorders 347

52–21. Pulmonary embolism occurs in approximately what 52–26. What is the most common presenting symptom in
percentage o patients with untreate eep-vein patients with a pulmonary embolus?
thrombosis? a. Cough

C
a. 10% b. Dyspnea

H
A
b. 35% c. Syncope

P
T
c. 60% d. Chest pain

E
R
d. 85%

5
52–27. Results rom which o the ollowing tests, when

2
52–22. I un ractionate heparin is selecte as the treatment normal, e ectively exclu e a iagnosis o pulmonary
or thromboembolism, the initial intravenous ose embolism?
is continue or at least how many ays be ore a. Chest ra iograph
converting to subcutaneous osing? b. Electrocar iogram
a. 2 c. Oxygen saturation
b. 5 d. None o the above
c. 10
d. 14 52–28. Circulatory collapse requires obstruction o at least
what percentage o the pulmonary vascular tree?
52–23. In women who are ully anticoagulate with low- a. 25%
molecular-weight heparin, perio ic monitoring o b. 50%
anti- actor Xa levels is particularly important when
which o the ollowing comorbi ities is present? c. 75%
a. Pneumonia d. 90%
b. Renal insu iciency 52–29. Which o the ollowing clinical in ings in the
c. Chronic hypertension setting o pulmonary embolism increases the
d. Gestational iabetes mortality rate?
a. Hypoxia
52–24. Women receiving therapeutic oses o low- b. Hemoptysis
molecular-weight heparin shoul not receive
neuraxial blocka e (e.g., epi ural anesthesia) or how c. Marke elevation o d- imers
long a ter the last ose was a ministere ? d. Right ventricular ys unction
a. 12 hours
52–30. What is the proce ure-relate mortality rate with
b. 24 hours pulmonary angiography?
c. 36 hours a. 0.005%
d. 48 hours b. 0.05%
52–25. he risk or heparin-in uce osteoporosis is c. 0.5%
increase by all EXCEPT which o the ollowing? d. 5%
a. Cigarette smoking
b. A rican American race
c. Anticoagulation longer than 6 months
d. Un ractionate heparin as the anticoagulant
348 Medica l a nd Surgica l Complica tions

52–31. A 35-year-ol G4P3 with three prior cesarean 52–32. Which o the ollowing statements is accurate
eliveries presents at 39 weeks’ gestation regar ing the use o thrombolytics in pregnancy?
complaining o rupture etal membranes an a. hey shoul be avoi e entirely.
S
shortness o breath. During preoperative evaluation,
b. hey shoul be avoi e in the last trimester.
E
C
she is note to have tachycar ia an hypoxia,
T
an compute tomographic (C ) angiography is c. hey shoul be avoi e in the irst trimester.
I
O
emergently per orme . One image rom this stu y is d. hey shoul be a ministere when clinically
N
shown here, an the arrow points to a illing e ect. in icate irrespective o pregnancy timing.
1
2
Which o the ollowing is the most appropriate next
step?

Repro uce with permission rom Cunningham FG, Leveno KJ, Bloom SL, et al (e s):
T romboembolic isor ers. In Williams Obstetrics, 24th e . New York, McGraw-Hill,
2014, Figure 38-5.

a. Pulmonary angiography
b. hrombolytic a ministration
c. Placement o a vena cava ilter
d. Un ractionate heparin in usion
Thromboembolic Disorders 349

CHAPTER 52 ANSw ER KEy

Q uestion Letter Pa ge

C
number a nswer cited Hea der cited

H
A
P
52–1 c p. 1028 Introduction

T
E
52–2 c p. 1028 Introduction

R
5
52–3 b p. 1028 Pa thophysiology

2
52–4 d p. 1028 Pa thophysiology
52–5 c p. 1028 Pa thophysiology
52–6 c p. 1030 Antithrombin Deficiency
52–7 d p. 1031 Protein S Deficiency
52–8 b p. 1031 Protein S Deficiency
52–9 c p. 1031 Activa ted Protein C Resista nce—Fa ctor V Leiden Muta tion
52–10 d p. 1031 Activa ted Protein C Resista nce—Fa ctor V Leiden Muta tion
52–11 b p. 1032 Hyperhomocysteinemia
52–12 d p. 1032 Hyperhomocysteinemia
52–13 b p. 1033 Antiphospholipid Antibodies
52–14 b p. 1033 Thrombophilia s a nd Pregna ncy Complica tions
52–15 c p. 1035 Deep-Vein Thrombosis—Clinica l Presenta tion
52–16 c p. 1036 Deep-Vein Thrombosis—Compression Ultra sonogra phy/ Ma na gement
52–17 a p. 1036 Deep-Vein Thrombosis—Ma gnetic Resona nce Ima ging
52–18 a p. 1036 Deep-Vein Thrombosis—d -Dimer Screening Tests
52–19 c p. 1036 Deep-Vein Thrombosis—Venogra phy
52–20 d p. 1036 Deep-Vein Thrombosis—Ma na gement
52–21 c p. 1036 Deep-Vein Thrombosis—Ma na gement
52–22 b p. 1037 Unfra ctiona ted Hepa rin
52–23 b p. 1038 Low-Molecula r-W eight Hepa rin
52–24 b p. 1039 La bor a nd Delivery
52–25 b p. 1040 Hepa rin-Induced O steoporosis
52–26 b p. 1040 Pulmona ry Embolism—Clinica l Presenta tion
52–27 d p. 1040 Pulmona ry Embolism—Clinica l Presenta tion
52–28 c p. 1041 Ma ssive Pulmona ry Embolism
52–29 d p. 1041 Ma ssive Pulmona ry Embolism
52–30 c p. 1043 Intra va scula r Pulmona ry Angiogra phy
52–31 c p. 1044 Vena Ca va Filter
52–32 d p. 1044 Thrombolysis
350

CHAPTER 53

Renal and Urinar Tract Disorders

53–1. Which of the following statements regarding the 53–5. An asymptomatic 17-year-old G1P0 at 16 weeks’
physiologic changes of pregnancy is true? gestation presents for her first prenatal visit, and
a. Glomerular filtration decreases. a urine culture is sent as part of routine care.
he results show > 100,000 gram-negative rods.
b. Effective renal plasma flow increases.
he diagnosis is which of the following?
c. Serum creatinine concentration increases.
a. Cystitis
d. Glomeruli become larger because of an increased
b. Pyelonephritis
number of cells (hyperplasia).
c. Diverticulitis
53–2. Which of the following is a threshold for proteinuria d. Asymptomatic bacteriuria
in pregnancy, above which levels are considered
abnormal? 53–6. If the patient in Question 53–5 is not treated, what
a. 50 mg/day is the chance that she will develop a symptomatic
infection during pregnancy?
b. 100 mg/day
a. 5%
c. 200 mg/day
b. 10%
d. 300 mg/day
c. 25%
53–3. A serum creatinine level in pregnancy that d. 50%
persistently exceeds what threshold value should
prompt a suspicion for intrinsic renal disease? 53–7. he patient in Question 53–5 refuses any treatment
a. 0.3 mg/dL that she must take over multiple days. Which of the
following regimens would be an acceptable single-
b. 0.5 mg/dL
dose oral treatment?
c. 0.7 mg/dL
a. Ampicillin, 2 g
d. 0.9 mg/dL
b. Ciprofloxacin, 250 mg
53–4. A 27-year-old G1P0 at 14 weeks’ gestation presents c. Nitrofurantoin, 100 mg
for prenatal care. She reports having donated a d. rimethoprim-sulfamethoxazole, 160/800 mg
kidney to a family member 2 years ago. She has no
other significant medical history. In regard to her 53–8. After treatment of the patient in Question 53–5,
having only one kidney, which of the following is what is her risk of recurrence?
true? a. 10%
a. She has a significantly increased risk for b. 30%
developing preeclampsia.
c. 50%
b. She is likely to manifest severe edema in the third
d. 70%
trimester.
c. She will require close monitoring for frequent 53–9. Lower urinary tract symptoms with pyuria but
electrolyte abnormalities. a sterile urine culture are likely due to which
d. If her renal function is normal, she can expect to pathogen?
have a normal pregnancy. a. Escherichia coli
b. Proteus mirabilis
c. Chlamydia trachomatis
d. Klebsiella pneumoniae
Rena l a nd Urina ry Tra ct Disorders 351

53–10. What is the leading cause of septic shock in pregnancy? 53–13. You admit the patient in Question 53–11.
a. Pneumonia Intravenous antimicrobial therapy and hydration
with crystalloid solutions are begun. he next
b. Pyelonephritis

C
day, she is tachypneic and her oxygen saturation

H
c. Breast abscess is low. She is hypotensive, continues to spike

A
P
d. Chorioamnionitis high temperatures, and appears ill. You initiate

T
oxygen via face mask. What is the next step in your

E
R
53–11. A 16-year-old G1P0 at 22 weeks’ gestation presents management?

5
to the emergency room complaining of fever, nausea

3
a. Chest radiograph
with vomiting, chills, dysuria, and flank pain. Her
b. Renal sonography
temperature is 102oF, and right costovertebral
angle tenderness is noted. A urinalysis reveals many c. Modification of antimicrobial therapy
leukocytes, many bacteria, small blood, and large d. Computed tomographic chest angiography
ketones. he patient’s urine culture will most likely
reveal what organism? An image of the organism is 53–14. A chest radiograph for the patient in Question
provided here. 53–11 is provided here. he most likely diagnosis is
which of the following?

Reproduced with permission from Levinson W: Gram-negative rods related to the


enteric tract. In Review of Medical Microbiology and Immunology, 12th ed. New York,
McGraw-Hill, 2012, Figure 18-1.

a. Escherichia coli Reproduced with permission from Cunningham FG, Leveno KJ, Bloom SL, et al (eds):
Maternal physiology. In Williams Obstetrics, 24th ed. New York, McGraw-Hill, 2014,
b. Proteus mirabilis Figure 53-3A.
c. Chlamydia trachomatis
d. Klebsiella pneumoniae a. Pulmonary embolism
b. Mild pulmonary edema
53–12. For the patient in Question 53–11, which of the c. Community-acquired pneumonia
following best describes appropriate management?
d. Acute respiratory distress syndrome
a. Discharge her home with oral antibiotics,
antiemetics, and close surveillance
53–15. After discharge from the hospital for acute
b. 23-hour inpatient observation with intravenous pyelonephritis, patients should receive oral
antimicrobials and fluid restriction to prevent antimicrobial treatment for how long?
pulmonary edema
a. 3–5 days
c. One dose of intravenous antimicrobials in b. 7–14 days
the emergency room, discharge home, oral
antibiotics, and close surveillance c. 21–28 days
d. Hospitalization, intravenous antimicrobial d. Oral antimicrobial therapy is not necessary.
therapy and hydration, close monitoring of vital
signs, and evaluation of electrolytes
352 Medica l a nd Surgica l Complica tions

53–16. Which of the following promotes nephrolithiasis? 53–20. Obstetrical management of a pregnant renal
a. Hydration transplant recipient should include which of the
following?
b. Low-sodium diet
S
a. Glucose tolerance testing
E
c. Low-calcium diet
C
b. Renal function monitoring
T
d. hiazide diuretics
I
O
c. Fetal-growth restriction surveillance
N
53–17. A 24-year-old G2P1 at 15 weeks’ gestation presents d. All of the above
1
2
to the emergency room with severe pain that radiates
from her back to lower abdomen. he patient is 53–21. Which of the following statements is not true
afebrile, and urinalysis only reveals red blood cells. regarding polycystic kidney disease, the autosomal
One of the patient’s renal sonograms is provided dominant disease demonstrated in the image here?
here. What is the best management plan for this
patient?

Reproduced with permission from Smith MA: Polycystic kidney disease. In Usatine RP,
Smith MA, Chumley H, et al (eds): T e Color Atlas of Family Medicine. New York,
McGraw-Hill, 2009, Figure 67-1.

a. Ureteral stenting
a. Most cases are caused by a mutation on
b. Percutaneous nephrolithotomy chromosome 8.
c. Intravenous hydration and analgesics b. en percent of patients die from berry aneurysm
d. Intravenous hydration, analgesics, and rupture.
antimicrobial therapy c. It causes 5–10% of end-stage renal disease in the
United States.
53–18. he rate of which of the following obstetrical
d. One third of patients have coexistent
complications is not increased in renal transplant
asymptomatic hepatic cysts.
recipients treated with cyclosporine and tacrolimus?
a. Preeclampsia 53–22. Which of the following does not cause acute
b. Preterm birth nephritic syndrome?
c. Fetal malformation a. IgA nephropathy
d. Fetal-growth restriction b. Infective endocarditis
c. Systemic lupus erythematosus
53–19. A 26-year-old woman who underwent renal d. Focal segmental glomerulosclerosis
transplant 1 year ago presents for preconceptional
counseling. You recommend she delay pregnancy 53–23. Of the following, which is the most common cause
until which of the following benchmarks is achieved? of nephrotic syndrome?
a. Her creatinine ranges from 2 to 3 mg/dL. a. Amyloidosis
b. She has been in good general health for 1 to b. Minimal change disease
2 years.
c. Membranous glomerulonephritis
c. She documents no evidence of graft rejection for
3 months. d. Focal segmental glomerulosclerosis
d. All of the above
Rena l a nd Urina ry Tra ct Disorders 353

53–24. Select the correct statement regarding nephrotic 53–30. When managing a pregnant woman on dialysis,
syndrome. which of the following is an important modification?
a. Heavy proteinuria is the hallmark. a. Reduce doses of daily oral multivitamins

C
b. here is increased risk of thromboembolism. b. Stop erythropoietin because of harmful fetal effects

H
A
c. In pregnancy, prognosis is dependent on the c. Add additional bicarbonate and less calcium to

P
T
degree of hypertension and renal insufficiency. dialysate

E
R
d. All of the above d. Extend dialysis to 5–6 times weekly to avoid

5
hypotension

3
53–25. Which of the following is not a known complication
of nephrotic syndrome in pregnancy? 53–31. A 24-year-old G1P0 at 37 weeks’ gestation
a. Preeclampsia presents to Labor and Delivery complaining of
headache, lower abdominal pain, and decreased fetal
b. Polycythemia movement. Her initial blood pressure is 140/90,
c. Preterm birth and no fetal heart tones are seen sonographically.
d. Peripheral edema She has 4+ urine protein. he patient is diagnosed
with severe preeclampsia and with fetal demise
53–26. Which of the following contributes the most to end- due to a suspected placental abruption. You send
stage renal disease requiring dialysis and transplant? a panel of laboratory tests, start magnesium sulfate
a. Diabetes infusion, transfuse 2 units of packed red blood
cells, and induce labor. he patient has a prompt
b. Hypertension vaginal delivery with an estimated 1 L blood loss.
c. Glomerulonephritis Postpartum, her urine output is 20 mL/hr, heart rate
d. Polycystic kidney disease is 120 beats per minute, and blood pressure is 90/60.
Which of the following is your plan of management?
53–27. A 22-year-old G1P0 at 14 weeks’ gestation with a. Initiate dopamine drip
a history of type 1 diabetes presents for prenatal b. Administer a loop diuretic
care. Her diabetes is poorly controlled, and her
serum creatinine is 3.5 mg/dL. While counseling c. Provide a one-time intravenous bolus of
her regarding the prognosis of this pregnancy, how crystalloid solution
would you categorize her renal impairment? d. Replace intravascular volume with crystalloid
solution and blood
a. Mild renal insufficiency
b. Moderate renal insufficiency 53–32. he condition depicted in this renal sonogram
c. Severe renal insufficiency can cause all EXCEPT which of the following
d. None of the above complications?

53–28. For the patient in Question 53–27, her pregnancy-


induced hypervolemia will surely be reduced by the
renal insufficiency. his patient will likely expand
her blood volume by what amount?
a. 0%
b. 10%
c. 25%
d. 55%

53–29. Which of the following would not be part of your


pregnancy management plan for the patient in
Question 53–27?
a. Low-protein diet
b. Excellent glucose control
c. Frequent blood pressure monitoring a. Oliguria
d. Serial sonographic examinations to follow fetal
b. Hypotension
growth
c. Renal failure
d. Elevated serum creatinine
354 Medica l a nd Surgica l Complica tions

53–33. Regarding the condition depicted in this


photograph, all EXCEPT which of the following
statements are true during pregnancy?
S
E
C
T
I
O
N
1
2
Photograph contributed by Gracy Jilson, WHNP.

a. It is usually managed surgically.


b. Recurrent urinary tract infections may develop.
c. Urine can collect within the sac and dribble out.
d. It originates from an enlarging paraurethral gland
abscess that ruptures into the urethral lumen and
leaves a persistent diverticular sac and ostium into
the urethra.
Rena l a nd Urina ry Tra ct Disorders 355

CHAPTER 53 ANSw ER KEy

Q uestion Letter Pa ge

C
number a nswer cited Hea der cited

H
A
P
53–1 b p. 1051 Pregna ncy-Induced Urina ry Tra ct Cha nges

T
E
53–2 d p. 1051 Assessment of Rena l Function During Pregna ncy

R
5
53–3 d p. 1051 Assessment of Rena l Function During Pregna ncy

3
53–4 d p. 1052 Pregna ncy a fter Unila tera l N ephrectomy
53–5 d p. 1053 Asymptoma tic Ba cteriuria
53–6 c p. 1053 Significa nce
53–7 a p. 1053 Ta ble 5 3 -1
53–8 b p. 1053 Trea tment
53–9 c p. 1054 Cystitis a nd Urethritis
53–10 b p. 1054 Acute Pyelonephritis
53–11 a p. 1054 Clinica l Findings
53–12 d p. 1055 Ta ble 5 3 -2
53–13 a p. 1055 Ta ble 5 3 -2
53–14 d p. 1055 Figure 5 3 -3
53–15 b p. 1054 Ma na gement
53–16 c p. 1056 N ephrolithia sis
53–17 c p. 1057 Ma na gement
53–18 c p. 1057 Pregna ncy O utcomes
53–19 b p. 1057 Pregna ncy O utcomes
53–20 d p. 1058 Ma na gement
53–21 a p. 1058 Polycystic Kidney Disea se
53–22 d p. 1059 Ta ble 5 3 -3
53–23 d p. 1060 Ta ble 5 3 -4
53–24 d p. 1059 N ephrotic Syndromes
53–25 b p. 1059 Pregna ncy
53–26 a p. 1060 Chronic Rena l Disea se
53–27 c p. 1060 Chronic Rena l Disea se
53–28 c p. 1061 Pregna ncy a nd Chronic Rena l Disea se
53–29 a p. 1062 Ma na gement
53–30 d p. 1062 Dia lysis During Pregna ncy
53–31 d p. 1064 Prevention
53–32 b p. 1064 O bstructive Rena l Fa ilure
53–33 a p. 1064 Urethra l Diverticulum
356

CHAPTER 54

Gastrointestinal Disorders

54–1. Which of the following diagnostic studies are 54–5. What is the approximate incidence of laparotomy
considered safe to use in pregnancy? and laparoscopy for nonobstetrical indications in
a. Cystoscopy pregnancy?
b. Flexible sigmoidoscopy a. 1:50
c. Endoscopic retrograde cholangiopancreatography b. 1:100
d. All of the above c. 1:500
d. 1:1000
54–2. All EXCEPT which of the following are common
indications for surgery during pregnancy? 54–6. Which of the following statements is true regarding
a. Appendicitis nonobstetrical surgery in pregnancy?
b. Adnexal mass a. There is increased risk of stillbirth.
c. Cholecystitis b. There is increased risk of cerebral palsy.
d. Nephrolithiasis c. There is increased risk of preterm delivery.
d. There is no increase in long-term risks to the
54–3. Which of the following is preferred treatment to fetus or the mother.
maintain adequate nutrition in a pregnant patient
with nausea and vomiting? 54–7. In cases of severe hyperemesis gravidarum,
a. Hyperalimentation all EXCEPT which of the following initial
complications are common?
b. Parenteral feeding
a. Acidosis
c. Enteral alimentation
b. Dehydration
d. Dextrose-containing solutions
c. Hypokalemia
54–4. Which of the following is the most common serious d. Mild transaminitis
complication of parenteral feeding?
a. Hemothorax 54–8. What organism has a proposed association with
hyperemesis gravidarum?
b. Pneumothorax
a. Helicobacter pylori
c. Catheter sepsis
b. Bacteroides fragilis
d. Brachial plexus injury
c. Clostridium difficile
d. Streptococcus viridans
G a strointestina l Disorders 357

54–9. A serious complication seen in hyperemesis 54–13. All EXCEPT which of the following medications are
gravidarum is shown here. What is the likely cause safe to use in pregnancy for the treatment of reflux
of this finding? esophagitis?

C
a. Cimetidine

H
A
b. Omeprazole

P
T
c. Misoprostol

E
R
d. Calcium carbonate

5
4
54–14. What is the rate of hospital readmission in patients
with hyperemesis gravidarum?
a. 5%
b. 15%
c. 30%
d. 45%

54–15. What is the main mechanism underlying reflux


esophagitis in pregnancy?
a. Increased gastric emptying
b. Excessive gastric acid production
c. Relaxation of the upper esophageal sphincter
Reproduced with permission from Song LM, Topazian M: Gastrointestinal endoscopy. In
Longo DL, Fauci AS, Kasper DL, et al (eds): Harrison’s Principles of Internal Medicine, d. Relaxation of the lower esophageal sphincter
18th ed. New York, McGraw-Hill, 2012, Figure 291-20.
54–16. A pregnant patient with history of swallowing
a. Dehydration problems presents with dysphagia, chest pain,
b. Helicobacter pylori infection and vomiting. A barium swallow done just prior
to pregnancy revealed these images. What is the
c. Persistent vomiting
patient’s diagnosis?
d. Vitamin K deficiency

54–10. Which vitamin deficiency in hyperemesis gravidarum


could lead to fetal intracranial hemorrhage?
a. Thiamine
b. Vitamin A
c. Vitamin D
d. Vitamin K

54–11. Which vitamin deficiency in hyperemesis gravidarum


is associated with confusion, ocular findings, and
ataxia?
a. Thiamine
b. Vitamin A
Reproduced with permission from Kahrilas PJ, Hirano I: Diseases of the esophagus. In
c. Vitamin D Longo DL, Fauci AS, Kasper DL, et al (eds): Harrison’s Principles of Internal Medicine,
d. Vitamin K 18th ed. New York, McGraw-Hill, 2012, Figure 292-5.

54–12. Which of the following interventions is the preferred a. Achalasia


initial treatment of hyperemesis gravidarum? b. Hiatal hernia
a. Glucocorticoids c. Peptic ulcer disease
b. Enteral nutrition d. Diaphragmatic hernia
c. Hyperalimentation
d. Antiemetics and intravenous hydration
358 Medica l a nd Surgica l Complica tions

54–17. A pregnant patient presents with peptic ulcer 54–23. A patient with Crohn disease presents for
disease. She is symptomatic, and a urea breath test is preconceptional counseling. The patient is currently
positive. Which of the following treatment options in remission, but her surgeon has recommended
S
should be avoided in this patient? bowel resection. She would like to know how
E
C
a. Amoxicillin, 1000 mg twice daily this surgery will affect her future fertility and
T
pregnancies. Which of the following statements
I
b. Metronidazole, 500 mg twice daily
O
should be part of her counseling?
N
c. Clarithromycin, 500 mg twice daily
a. She can expect her fertility to decrease if she has
1
2
d. Tetracycline, 500 mg four times daily surgery.
b. She will need to be delivered by cesarean section
54–18. A pregnant patient presents with diarrhea for 3 days.
if she conceives.
She is not febrile, has not had bloody stools, and is
able to drink liquids without vomiting. Which of the c. Laparoscopic anastomosis compared with
following drugs is indicated? anastomosis via laparotomy has a higher
subsequent fertility rate.
a. Ciprofloxacin
d. None of the above
b. Metronidazole
c. Bismuth subsalicylate 54–24. Why is high-dose folic acid supplementation
d. Trimethoprim-sulfamethoxazole indicated in pregnant patients with inflammatory
bowel disease?
54–19. A pregnant patient is referred to you because of a. To prevent fetal cardiac defects
concurrent Crohn disease. Which of the following is b. To prevent bowel disease relapses
true regarding her disease?
c. To counteract the antifolate actions of
a. It conveys a 3–5% risk of cancer. sulfasalazine
b. A proctocolectomy would be curative. d. To act synergistically with calcium in the
c. It involves the deep layers of small and large prevention of osteoporosis
bowel.
d. It is typically associated with antineutrophil 54–25. A pregnant patient presents in the early first
cytoplasmic antibody. trimester while taking the immune modulator
infliximab to treat her Crohn disease. Which of the
54–20. Options for the treatment of inflammatory bowel following statements is true regarding this drug?
disease in pregnancy include all EXCEPT which of a. It is considered safe in pregnancy.
the following medications? b. It is associated with fetal-growth restriction.
a. Infliximab c. It is associated with higher rates of congenital
b. Mesalamine skeletal defects.
c. Methotrexate d. All of the above
d. Glucocorticoids
54–26. What is the most common cause of intestinal
54–21. If one partner of a couple has inflammatory bowel obstruction in pregnancy and the puerperium?
disease, which of the following is a known potential a. Volvulus
cause of subfertility in this couple? b. Adhesions
a. Development of rectovaginal fistulas c. Carcinoma
b. Use of immune modulators as treatment d. Intussusception
c. Sperm abnormalities caused by sulfasalazine
d. None of the above 54–27. Cases of bowel obstruction in pregnancy are least
likely to occur during which of the following time
54–22. Which of the following statements is true regarding frames?
inflammatory bowel disease in pregnancy? a. Early first trimester
a. Relapses are usually mild. b. Midpregnancy
b. Pregnancy increases the likelihood of a flare. c. Third trimester
c. Most treatments are discontinued during d. Immediately postpartum
pregnancy.
d. Active disease in early pregnancy increases the
likelihood of poor pregnancy outcome.
G a strointestina l Disorders 359

54–28. A patient in the midtrimester presents with 54–32. Which method(s) may be appropriately used in
colicky abdominal pain, nausea, and vomiting. At pregnancy for appendicitis diagnosis?
laparotomy, surgical findings are similar to those a. Transvaginal sonography

C
shown here. What is her likely diagnosis?
b. Computed tomography (CT)

H
A
c. Magnetic resonance (MR) imaging

P
T
d. All of the above

E
R
5
54–33. A second-trimester pregnant patient presents with

4
appendicitis. One of her MR images is shown here
that displays the appendix with surrounding edema
(arrow), the adjacent right ovary (arrowhead), and
fetus (carets). Which of the following statements is
accurate regarding this condition?

Reproduced with permission from Cunningham FG, Leveno KJ, Bloom SL, et al (eds):
Gastrointestinal disorders. In Williams Obstetrics, 24th ed. New York, McGraw-Hill,
2014, Figure 54-4.

a. Perforation
b. Crohn disease
c. Colonic volvulus
d. Ulcerative colitis

54–29. What is the incidence of appendicitis in pregnancy?


Used with permission from Dr. April Bailey.
a. 1:500
b. 1:1500 a. The risk of spontaneous abortion is increased.
c. 1:5000 b. Accuracy of diagnosis improves with increasing
d. 1:10,000 gestational age.
c. Surgical evaluation is best postponed until the
54–30. During what part of pregnancy is a patient most second trimester.
likely to be diagnosed with a ruptured appendix? d. Tocolytics decrease the risk of preterm labor
a. First trimester associated with this complication.
b. Second trimester
c. Third trimester
d. Postpartum

54–31. What are the most reproducible findings in a


pregnant woman with appendicitis?
a. Fever
b. Anorexia
c. Nausea and vomiting
d. Persistent abdominal pain and tenderness
360 Medica l a nd Surgica l Complica tions

CHAPTER 54 ANSw ER KEy

Q uestion Letter Pa ge
S
number a nswer cited Hea der cited
E
C
T
54–1 d p. 1069 Dia gnostic Techniques
I
O
54–2 d p. 1070 La pa rotomy a nd La pa roscopy
N
1
54–3 c p. 1070 N utritiona l Support
2
54–4 c p. 1070 N utritiona l Support
54–5 c p. 1070 La pa rotomy a nd La pa roscopy
54–6 d p. 1070 La pa rotomy a nd La pa roscopy
54–7 a p. 1070 Hyperemesis G ra vida rum
54–8 a p. 1070 Hyperemesis G ra vida rum
54–9 c p. 1071 Complica tions
54–10 d p. 1071 Complica tions
54–11 a p. 1071 Complica tions
54–12 d p. 1072 Ma na gement
54–13 c p. 1072 G a stroesopha gea l Reflux Disea se
54–14 c p. 1072 Ma na gement
54–15 d p. 1072 G a stroesopha gea l Reflux Disea se
54–16 a p. 1073 Acha la sia
54–17 d p. 1073 Peptic Ulcer
54–18 c p. 1074 Acute Dia rrhea
54–19 c p. 1075 Infla mma tory Bowel Disea se, Ta ble 5 4 -4
54–20 c p. 1076 Ulcera tive Colitis a nd Pregna ncy
54–21 c p. 1076 Infla mma tory Bowel Disea se a nd Fertility
54–22 d p. 1076 Infla mma tory Bowel Disea se a nd Pregna ncy
54–23 c p. 1076 Infla mma tory Bowel Disea se a nd Fertility
54–24 c p. 1076 Ulcera tive Colitis a nd Pregna ncy
54–25 a p. 1077 Crohn Disea se a nd Pregna ncy
54–26 b p. 1078 Intestina l O bstruction
54–27 a p. 1078 Intestina l O bstruction
54–28 c p. 1078 Intestina l O bstruction, Figure 5 4 -4
54–29 b p. 1078 Appendicitis
54–30 c p. 1078 Appendicitis
54–31 d p. 1079 Dia gnosis
54–32 d p. 1079 Dia gnosis
54–33 a p. 1079 Pregna ncy O utcomes
361

CHAPTER 55
00

Hepatic, Biliar , and Pancreatic Disorders

55–1. All EXCEPT which o the ollowing liver-related 55–6. Re erring to the patient described in Question
changes are physiologic in pregnancy? 55–5, which o the ollowing medications is most
a. Asterixis appropriate to initiate?
b. Palmar erythema a. Hydroxyzine
c. Spider angiomas b. Cholestyramine
d. Elevated serum alkaline phosphatase levels c. Diphenhydramine
d. Ursodeoxycholic acid
55–2. What is the underlying pathophysiology o
intrahepatic cholestasis o pregnancy? 55–7. he patient rom Question 55–5 reports that she
a. Acute hepatocellular destruction has been reading about intrahepatic cholestasis o
pregnancy and understands that the condition may
b. Incomplete clearance o bile acids
be associated with stillbirth. You explain that the
c. Microvascular thrombus accumulation association is somewhat ambiguous but that one
d. Eosinophil in iltration o the liver plausible reason may involve which o the ollowing
etal complications?
55–3. Which o the ollowing clinical eatures are a. Cardiac arrest
characteristic o intrahepatic cholestasis o
b. Hepatocellular injury
pregnancy?
c. Hypoxic encephalopathy
a. Maculopapular rash
d. Disseminated intravascular coagulation
b. Nausea and vomiting
c. Generalized pruritis 55–8. All EXCEPT which o the ollowing statements
d. Serum transaminase levels > 500 U/L regarding acute atty liver o pregnancy are true?
a. It occurs in about 1 in 10,000 pregnancies.
55–4. Which o the ollowing viral in ections has been
b. Recurrence in subsequent pregnancies is
associated with a marked increase in the risk or
common.
intrahepatic cholestasis o pregnancy?
c. It is the most common cause o liver ailure in
a. Hepatitis C
pregnancy.
b. Hepatitis B
d. All statements are true.
c. Cytomegalovirus
d. Human immunode iciency virus 55–9. Which mutation patterns o the enzymes o atty
acid oxidation have classically been associated with
55–5. A 42-year-old primigravida presents at 35 weeks’ maternal acute atty liver o pregnancy?
gestation with several new complaints. Most a. Homozygous mutation in the etus and mother
bothersome is yellowing o her sclera. A ter
b. Heterozygous mutation in the etus and mother
per orming a thorough history, physical, and
laboratory evaluation, you make a diagnosis o c. Homozygous mutation in the etus; heterozygous
intrahepatic cholestasis o pregnancy. Approximately mutation in the mother
what percentage o women with this condition will d. Heterozygous mutation in the etus; homozygous
develop jaundice? mutation in the mother
a. < 1%
b. 10%
c. 50%
d. > 90%
362 Medica l a nd Surgica l Complica tions

55–10. From an etiopathogenesis perspective, acute atty 55–14. You stabilize the patient described in Question
liver o pregnancy is analogous to what childhood 55–12 and correct her coagulopathy. A ter this, you
illness? induce labor, and she has a vaginal delivery. On
S
a. Reye syndrome postpartum day 2, she appears to be doing well, but
E
C
you notice that her urine output has increased to
b. Biliary atresia
T
approximately 800 mL per hour. What is the most
I
O
c. Autoimmune hepatitis likely cause o this condition?
N
d. Epstein-Barr virus in ection a. Pituitary tumor
1
2
55–11. All EXCEPT which o the ollowing are clinical b. Acute tubular necrosis
characteristics that increase the risk or acute atty c. Hypothalamic dys unction
liver o pregnancy? d. Elevated serum vasopressinase concentrations
a. Nulliparity
b. Female etus 55–15. Although it can be dangerous i coagulopathy is
not adequately addressed, what is the approximate
c. win gestation cesarean delivery rate in women who have acute atty
d. hird trimester liver o pregnancy?
a. 10%
55–12. A 33-year-old G2P1 presents at 35 weeks’ gestation
with complaints o nausea and vomiting. Laboratory b. 30%
evaluation reveals markedly elevated transaminase c. 60%
levels, renal dys unction, and coagulopathy. A d. 90%
peripheral smear is per ormed and is shown here.
What is the underlying etiology o the blood smear 55–16. his image shows a hepatocellular carcinoma illing
indings in this patient? the le t side o this cirrhotic liver. What is the most
common cause o liver cancer?

Reproduced with permission rom Lichtman MA, Beutler E, Kipps J: Color atlas III:
Red cell morphology. In Williams Hematology, 7th ed. New York, McGraw-Hill, 2006,
Plate III-3.

Reproduced with permission rom Kemp WL, Burns DK, Brown G: Pathology o the
a. Autoimmune antibody binding liver, gallbladder, and pancreas. In Pathology: T e Big Picture. New York, McGraw-Hill,
2008, Figure 15-14A.
b. Lowered capillary oxygen tension
c. Decreased cholesterol production
a. Alcohol abuse
d. Increased destruction in the spleen
b. Hemochromatosis
55–13. As a part o the evaluation or the patient described c. Chronic viral hepatitis
in Question 55–12, you want to con irm the d. Primary biliary cirrhosis
suspected diagnosis with imaging. Which o the
ollowing modalities is most appropriate?
a. Sonography
b. Computed tomography
c. Magnetic resonance imaging
d. None o the above
Hepa tic, Bilia ry, a nd Pa ncrea tic Disorders 363

55–17. All EXCEPT which o the ollowing clinical indings 55–22. Chronic hepatitis B in ection is most likely to
would indicate a severe course o acute hepatitis that develop in which o the ollowing patients?
should prompt hospitalization? a. A newborn

C
a. Hyperglycemia b. An 8-year-old child

H
A
b. Hypoalbuminemia c. A 32-year-old healthy woman

P
T
c. Central nervous symptoms d. All are equally likely to develop chronic in ection

E
R
d. Prolonged prothrombin time

5
55–23. Approximately what percentage o patients in ected

5
55–18. Which o the ollowing statements regarding acute with hepatitis C will have no identi iable risk actors?
viral hepatitis is correct? a. 10%
a. Jaundice is usually the presenting symptom. b. 33%
b. Serum transaminase levels correspond with c. 66%
disease severity. d. 90%
c. Low-grade ever is more common with hepatitis
A in ection. 55–24. What is the primary adverse perinatal outcome in
d. Bilirubin levels typically all as transaminase levels women in ected with hepatitis C?
rise. a. Stillbirth
b. Low birthweight
55–19. Complete clinical recovery is LEAST likely to
occur ollowing acute in ection with which o the c. Admission to intensive care unit
ollowing? d. Vertical transmission o hepatitis C virus
a. Hepatitis A
55–25. Which o the ollowing statements regarding breast
b. Hepatitis B eeding in women in ected with hepatitis B (HBV)
c. Hepatitis C and C (HCV) is correct?
d. Hepatitis E a. It is contraindicated in both.
b. It is contraindicated only in HBV in ection.
55–20. Which type o hepatitis virus is represented here?
c. It is contraindicated only in HCV in ection.
Coa t prote in d. Neither is a contraindication.
Core
55–26. Which o the ollowing statements regarding
42 DNA ge nome
nm
hepatitis E is correct?
– +
DNA polyme ra s e a. It is a DNA virus.
b. No e ective vaccine is available.
c. It is transmitted by sexual contact.
Virus particle d. A ected pregnant women are at increased risk or
Modif ed with permission rom Ryan K et al. Sherris Medical Microbiology, 3rd ed.
mortality compared with a ected nonpregnant
Originally published by Appleton & Lange. Copyright 1994 McGraw-Hill. women.

a. Hepatitis A 55–27. Rates o which o the ollowing complications


appear to be increased in pregnant women with
b. Hepatitis B
autoimmune hepatitis?
c. Hepatitis C
a. Preeclampsia
d. Hepatitis E
b. Hepatitis lares
55–21. A ter in ection with hepatitis B, what is the irst c. Maternal mortality
detectable serological marker? d. All o the above
a. HBs antigen
55–28. Steatohepatitis is most common in women o which
b. HBe antigen ethnic group?
c. Anti-HBs antibody a. Asian
d. IgM anti-HB core antibody b. Hispanic
c. White
d. A rican American
364 Medica l a nd Surgica l Complica tions

55–29. he ollowing image depicts the typical nodular, 55–31. Which o the ollowing management strategies
ibrotic appearance o a cirrhotic liver. What is the is recommended or sonographically identi ied
most common cause o this condition in the general asymptomatic gallstones?
S
population? a. Cholecystectomy
E
C
b. Low-cholesterol diet
T
I
O
c. Ursodeoxycholic acid
N
d. None o the above
1
2
55–32. A 31-year-old G2P1 20 weeks’ gestation complains
o vomiting, ever, and right upper quadrant
pain. Examination reveals right upper quadrant
tenderness. One diagnostic sonographic image is
shown here. What is the most appropriate next step
in her management?

Reproduced with permission rom Geller DA, Goss JA, sung A: Liver. In Brunicardi FC,
Andersen DK, Billiar R, et al (eds): Schwartz’s Principles o Surgery, 9th ed. New York,
McGraw-Hill, 2010, Figure 31-16A.

a. Viral hepatitis
b. Alcohol exposure
c. Autoimmune hepatitis
d. Nonalcoholic atty liver disease

55–30. Women with cirrhosis who become pregnant are


at increased risk or all EXCEPT o the ollowing
untoward outcomes? a. Cholecystectomy
a. Preterm birth b. Discharge home with oral antibiotics
b. Gestational diabetes c. Endoscopic retrograde cholangiopancreatography
c. Fetal-growth restriction d. Admission or serial abdominal examinations and
d. Rupture o splenic artery aneurysms intravenous antibiotics
Hepa tic, Bilia ry, a nd Pa ncrea tic Disorders 365

CHAPTER 55 ANSw ER KEy

Q uestion Letter Pa ge

C
number a nswer cited Hea der cited

H
A
P
55–1 a p. 1084 Hepa tic Disorders

T
E
55–2 b p. 1085 Intra hepa tic Cholesta sis of Pregna ncy—Pa thogenesis

R
5
55–3 c p. 1085 Intra hepa tic Cholesta sis of Pregna ncy—Clinica l Presenta tion

5
55–4 a p. 1085 Intra hepa tic Cholesta sis of Pregna ncy—Clinica l Presenta tion
55–5 b p. 1085 Intra hepa tic Cholesta sis of Pregna ncy—Clinica l Presenta tion
55–6 d p. 1085 Intra hepa tic Cholesta sis of Pregna ncy—Ma na gement
55–7 a p. 1086 Cholesta sis a nd Pregna ncy O utcomes
55–8 b p. 1086 Acute Fa tty Liver of Pregna ncy
55–9 c p. 1086 Acute Fa tty Liver of Pregna ncy—Etiopa thogenesis
55–10 a p. 1086 Acute Fa tty Liver of Pregna ncy—Etiopa thogenesis
55–11 b p. 1087 Acute Fa tty Liver of Pregna ncy—Clinica l a nd La bora tory Findings
55–12 c p. 1087 Acute Fa tty Liver of Pregna ncy—Clinica l a nd La bora tory Findings
55–13 d p. 1087 Acute Fa tty Liver of Pregna ncy—Clinica l a nd La bora tory Findings
55–14 d p. 1088 Acute Fa tty Liver of Pregna ncy—Ma na gement
55–15 d p. 1088 Acute Fa tty Liver of Pregna ncy—Ma na gement
55–16 c p. 1088 Vira l Hepa titis
55–17 a p. 1088 Acute Hepa titis
55–18 c p. 1088 Acute Hepa titis
55–19 c p. 1088 Acute Hepa titis
55–20 b p. 1090 Hepa titis B
55–21 a p. 1090 Hepa titis B
55–22 a p. 1090 Hepa titis B
55–23 b p. 1091 Hepa titis C
55–24 d p. 1091 Hepa titis C
55–25 d p. 1090 Hepa titis B a nd Hepa titis C
55–26 d p. 1092 Hepa titis E
55–27 d p. 1092 Autoimmune Hepa titis
55–28 b p. 1092 N ona lcoholic Fa tty Liver Disea se
55–29 b p. 1093 Cirrhosis
55–30 b p. 1093 Cirrhosis
55–31 d p. 1095 Cholelithia sis a nd Cholecystitis
55–32 a p. 1096 Medica l versus Surgica l Ma na gement
366

CHAPTER 56

Hematological Disorders

56–1. A hemoglobin concentration below which of the 56–4. For the patient described in Question 56–3, what is
following thresholds would indicate anemia in the most appropriate initial treatment?
an iron-supplemented pregnant woman in any a. Red cell transfusion
trimester? b. Folic acid, 4 mg orally daily
a. 9.0 g/dL c. Hydroxyurea, 1 g orally daily
b. 9.5 g/dL d. Elemental iron, 200 mg orally daily
c. 10.0 g/dL
d. 10.5 g/dL 56–5. Of medical conditions associated with anemia
of chronic disease, which is most frequently
56–2. What is the most common cause of antepartum encountered in pregnancy?
anemia in pregnant women? a. Crohn disease
a. Thalassemia b. Hodgkin lymphoma
b. Iron deficiency c. Chronic renal insufficiency
c. Folic acid deficiency d. Systemic lupus erythematosus
d. Anemia of chronic disease
56–6. A 36-year-old G3P2 at 18 weeks’ gestation reports
56–3. A 19-year-old primigravida at 29 weeks’ gestation extreme fatigue and is found to be have a hemoglobin
is noted to have anemia with a hemoglobin concentration of 7.5 g/dL. The erythrocyte mean
concentration of 8 g/dL. The peripheral blood smear corpuscular volume is markedly elevated and measures
below is obtained. Which of the following laboratory 124 fL. A peripheral blood smear is obtained and is
findings are likely to accompany this condition? shown here. What is the most likely etiology?

Reproduced with permission from Beutler E: Disorders of iron metabolism. In Lichtman


MA, Kipps TJ, Seligsohn U (eds): Williams Hematology, 8th ed. New York, McGraw-Hill,
2010, Figure 42-10C.
Reproduced with permission from Green R: Folate, cobalamin, and megaloblastic anemia.
In Lichtman MA, Kipps TJ, Seligsohn U (eds): Williams Hematology, 8th ed. New York,
a. Decreased serum ferritin level McGraw-Hill, 2010, Figure 41-12A.
b. Elevated mean corpuscular volume
c. Decreased total iron binding capacity a. Iron deficiency
d. Positive sickle-cell screen (Sickledex) result
b. Vitamin B6 deficiency
c. Folic acid deficiency
d. Vitamin B12 deficiency
Hema tologica l Disorders 367

56–7. All EXCEPT which of the following statements 56–11. Which of the following statements is true regarding
regarding autoimmune hemolytic anemia in women who are heterozygous for the glucose-6-
pregnancy are true? phosphate dehydrogenase mutation?

C
a. Pregnancy can accelerate hemolysis. a. Infections in pregnancy can precipitate hemolysis.

H
A
b. The direct Coombs test is usually positive. b. Some degree of protection against malaria is

P
T
c. The indirect Coombs test is usually positive. conferred.

E
c. Lyonization results in a variable degree of enzyme

R
d. The cause of aberrant antibody production

5
originates from fetal microchimerism. activity.

6
d. All of the above
56–8. Pregnant women with paroxysmal nocturnal
hemoglobinuria are at increased risk for which of the 56–12. Which of the following pregnancy complications has
following? been associated with polycythemia vera?
a. Renal failure a. Stillbirth
b. Venous thrombosis b. Coagulopathy
c. Maternal mortality c. Placenta previa
d. All of the above d. Placental abruption

56–9. What is the typical inheritance pattern of the 56–13. What is the prevalence of sickle-cell trait among
mutation in the spectrin gene that results in African-American women in the United States?
hereditary spherocytosis? a. 1%
a. Mitochondrial b. 4%
b. X-linked dominant c. 8%
c. Autosomal dominant d. 25%
d. Autosomal recessive
56–14. In patients with sickle-cell disease, red cells may
56–10. The following image demonstrates the appearance assume the following configuration under which of
of spherocytes using scanning electron microscopy. the following conditions?
In addition to identifying erythrocytes with this
appearance, what other laboratory finding helps
confirm the diagnosis of hereditary spherocytosis?

Reproduced with permission from Longo DL: Atlas of hematology and analysis of periph-
eral blood smears. In Longo DL, Fauci AS, Kasper DL, et al (eds): Harrison’s Principles of
Internal Medicine, 18th ed. New York, McGraw-Hill, 2012, Figure e17-12.

Reproduced with permission from Cunningham FG, Leveno KJ, Bloom SL, et al (eds):
Hematological disorders. In Williams Obstetrics, 24th ed. New York, McGraw-Hill, a. Hyperglycemia
2014, Figure 56-2B.
b. Low oxygen tension
a. Low serum haptoglobin level c. Dietary protein deficiency
b. Elevated serum bilirubin level d. Administration of certain antibiotics
c. Increased erythrocyte osmotic fragility
d. Increased serum level of fibrin split products
368 Medica l a nd Surgica l Complica tions

56–15. A 22-year-old primigravida with sickle-cell disease 56–18. Which of the following contraceptive choices may
presents with complaints of fever, cough, and help prevent painful crises in women with sickle-cell
increasing dyspnea. The following chest radiograph disease?
S
is obtained. All EXCEPT which of the following are a. Intrauterine device
E
C
precipitants of this condition?
b. Surgical sterilization
T
I
O
c. Depot medroxyprogesterone acetate
N
d. Combination oral contraceptive pills
1
2
56–19. Sickle-cell trait (hemoglobin AS) has been associated
with an increased risk for which of the following?
a. Preeclampsia
b. Placental abruption
c. Gestational diabetes
d. Urinary tract infections

56–20. Women with which of the following α -globin


genotypes are at risk to have offspring with
homozygous α -thalassemia?
a. α α /--
b. α -/ α α
c. α -/ α -
Reproduced with permission from Cunningham FG, Leveno KJ, Bloom SL, et al (eds): d. α α / α α
Pulmonary disorders. In Williams Obstetrics, 22nd ed. New York, McGraw-Hill, 2005,
Figure 46-1.
56–21. Which of the following findings on hemoglobin
electrophoresis would be most consistent with a
a. Infection diagnosis of β -thalassemia minor?
b. Atelectasis a. Hemoglobin A2 less than 1%; normal fetal
c. Coagulopathy hemoglobin level
d. Marrow emboli b. Hemoglobin A2 less than 1%; fetal hemoglobin
greater than 2%
56–16. Pregnant women with sickle-cell anemia are at c. Hemoglobin A2 greater than 3.5%; normal fetal
increased risk for which of the following maternal hemoglobin level
complications?
d. Hemoglobin A2 greater than 3.5%; fetal
a. Renal failure hemoglobin greater than 2%
b. Cardiomyopathy
c. Pulmonary hypertension 56–22. What is the most common cause of
d. All of the above thrombocytopenia in pregnancy?
a. Severe preeclampsia
56–17. Which of the following statements is accurate b. Consumptive coagulopathy
regarding the use of prophylactic red cell transfusions c. Gestational thrombocytopenia
for pregnant women with sickle-cell anemia?
d. Immune thrombocytopenic purpura
a. Gestation duration is increased.
b. Perinatal mortality rate is decreased. 56–23. When indicated in pregnancy, which of the
c. Rate of fetal-growth restriction is decreased. following is the most appropriate initial treatment of
d. Risk of red cell alloimmunization is increased. immune thrombocytopenic purpura?
a. Azathioprine
b. Laparoscopic splenectomy
c. Systemic corticosteroids
d. Intravenous anti-D immunoglobulin G
Hema tologica l Disorders 369

56–24. Which of the following management strategies is 56–29. Hemophilia A, characterized by a severe deficiency
recommended to detect fetal thrombocytopenia in factor VIII, is inherited in what fashion?
in women who have chronic immune a. X-linked dominant

C
thrombocytopenic purpura?
b. X-linked recessive

H
a. Cordocentesis

A
c. Autosomal dominant

P
b. Scalp sampling

T
d. Autosomal recessive

E
R
c. Cesarean delivery

5
d. None of the above 56–30. In women affected by hemophilia A, the risk for

6
excessive hemorrhage at delivery can be reduced by
56–25. Most cases of thrombocytosis in which the platelet all EXCEPT which of the following?
count exceeds 1 million/ µL are caused by which of a. Uterotonics
the following conditions?
b. Desmopressin
a. Malignancy
c. Avoiding episiotomy
b. Iron deficiency
d. Operative vaginal delivery
c. Autoimmune conditions
d. Essential thrombocytosis 56–31. Pregnancy physiology results in which of the
following changes to factor VIII and von Willebrand
56–26. Although there is considerable clinical overlap factor (vWF) levels?
between thrombotic thrombocytopenic purpura a. Increased factor VIII and vWF levels
(TTP) and hemolytic uremic syndrome (HUS),
these entities can be distinguished by all EXCEPT b. Decreased factor VIII and vWF levels
which of the following? c. Increased factor VIII levels; decreased vWF factor
levels
a. HUS has more renal dysfunction.
d. Decreased factor VIII levels; increased vWF factor
b. HUS is seen primarily in adults. levels
c. TTP more frequently has associated neurologic
aberrations. 56–32. Although pregnancy outcomes are generally good in
d. All statements are true. women who have von Willebrand disease, which of
the following pregnancy-related complications may
56–27. What is the cornerstone of treatment for thrombotic be encountered in up to 50 percent of such cases?
thrombocytopenic purpura? a. Preterm birth
a. Anticoagulation b. Placental abruption
b. Plasmapheresis c. Postpartum hemorrhage
c. Platelet transfusion d. Fetal-growth restriction
d. Intravenous immunoglobulin (IVIG)

56–28. Because the treatment of the two conditions is very


different, differentiating severe preeclampsia from
thrombotic thrombocytopenic purpura (TTP) is
important. Which of the following clinical findings
favors TTP?
a. Moderate thrombocytopenia
b. Severe hemolysis
c. Mild disseminated intravascular coagulation
d. Marked transaminase elevation
370 Medica l a nd Surgica l Complica tions

CHAPTER 56 ANSw ER KEy

Q uestion Letter Pa ge
S
number a nswer cited Hea der cited
E
C
T
56–1 d p. 1101 Anemia s
I
O
56–2 b p. 1102 Iron Deficiency Anemia
N
1
56–3 a p. 1103 Iron Deficiency Anemia —Dia gnosis
2
56–4 d p. 1103 Iron Deficiency Anemia —Trea tment
56–5 c p. 1103 Anemia Associa ted with Chronic Disea se—Pregna ncy
56–6 c p. 1104 Folic Acid Deficiency
56–7 b p. 1104 Autoimmune Hemolysis
56–8 d p. 1105 Pa roxysma l N octurna l Hemoglobinuria
56–9 c p. 1106 Heredita ry Spherocytosis
56–10 c p. 1106 Heredita ry Spherocytosis
56–11 d p. 1106 Erythrocyte Enzyme Deficiency
56–12 a p. 1107 Polycythemia Vera
56–13 c p. 1108 Sickle-Cell Hemoglobinopa thies—Inherita nce
56–14 b p. 1108 Sickle-Cell Hemoglobinopa thies—Pa thophysiology
56–15 c p. 1108 Pregna ncy a nd Sickle-Cell Syndromes
56–16 d p. 1108 Pregna ncy a nd Sickle-Cell Syndromes, Ta ble 5 6 -2
56–17 d p. 1110 Prophyla ctic Red Cell Tra nsfusions
56–18 c p. 1111 Contra ception a nd Steriliza tion
56–19 d p. 1111 Sickle-Cell Tra it
56–20 a p. 1112 Alpha Tha la ssemia s
56–21 d p. 1113 Beta Tha la ssemia s
56–22 c p. 1114 Thrombocytopenia
56–23 c p. 1114 Immune Thrombocytopenia —Dia gnosis a nd Ma na gement
56–24 d p. 1115 Detection of Feta l Thrombocytopenia
56–25 d p. 1115 Thrombocytosis
56–26 b p. 1116 Thrombotic Microa ngiopa thies—Etiopa thogenesis
56–27 b p. 1116 Thrombotic Microa ngiopa thies—Trea tment
56–28 b p. 1116 Thrombotic Microa ngiopa thies—Pregna ncy
56–29 b p. 1117 Hemophilia A a nd B
56–30 d p. 1117 Hemophilia A a nd B—Pregna ncy
56–31 a p. 1118 Von W illebra nd Disea se—Pregna ncy
56–32 c p. 1118 Von W illebra nd Disea se—Pregna ncy
371

CHAPTER 57
00

Diabetes Mellitus

57–1. The number of Americans with diabetes is 57–3. The graphic below concerning the age-specific
increasing, in part, because of which of the incidence of gestational diabetes suggests which of
following? the following?
a. An obesity prevalence that has plateaued
b. An aging population that is more likely to 10
develop type 2 diabetes 9 < 25 ye a rs
c. Decreased populations within minority groups, 25–34 ye a rs
8
who are more likely to develop type 1 diabetes ≥ 35 ye a rs
d. All of the above 7
)
t
n
e
c
6
r
57–2. In 2006 in the United States, the rate of diabetes
e
p
(
during pregnancy approximated which of the 5
e
c
following?
n
e
4
d
i
a. 0.5%
c
n
I
b. 4% 3

c. 9% 2
d. 14% 1

0
1989–90 1993–94 1997–98 2001–02 2002–04
Pe riod (ye a rs )

Reproduced with permission from Cunningham FG, Leveno KJ, Bloom SL, et al (eds):
Diabetes mellitus. In Williams Obstetrics, 24th ed. New York, McGraw-Hill, 2014,
Figure 57-1.

a. The greatest increase in incidence was in the


group of women ≥ 35 years.
b. For all maternal age groups, the incidence of
gestational diabetes has increased since 1989.
c. The decreases seen in the group of women
younger than 25 years may be explained by
improved dietary intake.
d. None of the above

57–4. A 27-year-old woman with proliferative retinopathy


would have what diagnosis by the White classification?
a. R diabetes
b. H diabetes
c. RF diabetes
d. A2 gestational diabetes
372 Medica l a nd Surgica l Complica tions

57–5. What fasting plasma glucose level is used as the 57–9. Fetal hyperinsulinemia in the second half of
threshold to diagnose overt diabetes? pregnancy is associated with which of the following?
a. 105 mg/dL a. Altered fetal growth
S
b. 116 mg/dL b. Neonatal hypoglycemia
E
C
T
c. 126 mg/dL c. Maternal hyperglycemia
I
O
d. 140 mg/dL d. All of the above
N
1
2
57–6. Which of the following is a risk factor in pregnant 57–10. What might be said of the pregnancy yielding this
women for impaired carbohydrate metabolism? 6050 g newborn?
a. Family history of diabetes
b. Previous infant with polycystic kidney disease
c. High serum levels of antiphospholipid antibodies
d. All of the above

57–7. Fetuses of overtly diabetic mothers have an increased


risk for which of the following?
a. Preterm delivery
b. Spontaneous abortion
c. Congenital malformation
d. All of the above

57–8. This figure illustrates the frequency of congenital


malformations at given maternal glycohemoglobin
levels early in pregnancy. What can be said regarding
this relationship?

30

25

20
t
n
15.8
e
c
15
r
11.7
e
P
10 Reproduced with permission from Cunningham FG, Leveno KJ, Bloom SL, et al (eds):
3/19 Diabetes mellitus. In Williams Obstetrics, 24th ed. New York, McGraw-Hill, 2014,
5 2/17 Figure 57-3.
5 2.8
2/40
1/35
0 a. The baby was at risk for neonatal hyperglycemia.
<6 6–6.9 7–7.9 ≥8
b. The mother probably had excellent glycemic
Reproduced with permission from Cunningham FG, Leveno KJ, Bloom SL, et al (eds):
control.
Diabetes mellitus. In Williams Obstetrics, 24th ed. New York, McGraw-Hill, 2014, c. The mother had an increased risk for shoulder
Figure 57-2.
dystocia.

a. Gestational diabetes confers an increased risk of d. All of the above


congenital malformation.
b. If the glycohemoglobin A1C is < 6, there is no
risk of congenital malformation.
c. As preconceptional glucose control worsens, the
incidence of congenital malformation increases.
d. The highest risk for congenital malformation is
seen with a glycohemoglobin A1C level between 7
and 8.
Dia betes Mellitus 373

57–11. This graphic comparing birthweight distributions of 57–14. What is the most likely cause for the increased
neonates born to diabetic and nondiabetic mothers incidence of respiratory distress syndrome in the
illustrates which of the following? neonates of diabetic mothers?

C
a. Indicated preterm delivery

H
27 700

A
b. Delayed maturation of type II pneumocytes

P
24

T
600 c. Decreased production of surfactant in a

E
R
21 hyperglycemic environment

5
500
d. All of the above

7
s
18
t
n
N
e
Norma l Dia be tic

o
i
.
t
400
a
o
15 57–15. Hyperbilirubinemia in the newborns of diabetic
p
f
c
c
o
i
mothers is related to which of the following?
t
n
e
12 300

t
b
r
o
a
a. Newborn polycythemia
i
l
s
d
9
f
o
200 b. Relative fetal hypoxia
.
o
N
6
c. Hyperglycemia-mediated increases in maternal
100
3 affinity for oxygen and fetal oxygen consumption
0 0 d. All of the above
4 3 2 1 0 1 2 3 4 5 6
Birthwe ight (s ta nda rd devia tions ) 57–16. What can be said regarding the inheritance of
diabetes?
Reproduced with permission from Cunningham FG, Leveno KJ, Bloom SL, et al (eds):
Diabetes mellitus. In Williams Obstetrics, 24th ed. New York, McGraw-Hill, 2014,
a. Breast feeding increases the inheritance risk.
Figure 57-4. b. There is a genetic component to the inheritance
of type I diabetes.
a. Newborns of diabetic mothers are “growth c. If both parents have type 2 diabetes, the risk of
promoted.” inheritance approximates 20%.
b. The birthweight distribution of newborns of d. None of the above
diabetic mothers is skewed toward consistently
heavier birthweights. 57–17. What is the maternal mortality rate associated with
c. Growth promotion from maternal hyperglycemia class H diabetes?
does not precluded fetal-growth restriction, which a. 10%
is defined as 2 standard deviations below the
mean. b. 25%
d. All of the above c. 50%
d. 75%
57–12. The incidence of stillbirth is highest in pregnancies
complicated by which of the following? 57–18. For women with chronic hypertension and
a. Overt diabetes diabetic nephropathy, what is the associated risk of
developing preeclampsia?
b. Gestational diabetes
a. 20%
c. Overt diabetes and hypertension
b. 40%
d. Gestational diabetes and hypertension
c. 60%
57–13. Which of the following is a reasonable explanation d. 80%
for hydramnios in diabetic pregnancy?
a. Maternal endothelial leak caused by 57–19. The adverse effects of pregnancy on diabetic
hyperglycemia retinopathy may be reduced by which of the
following during pregnancy?
b. Glucose reabsorption by the fetal glomerular
collecting system a. Good glycemic control
c. Osmotic gradient created by high glucose b. Laser retina photocoagulation in pregnancy
concentrations in the amnionic fluid c. Folate, 4 mg orally daily throughout pregnancy
d. All of the above d. A and B
374 Medica l a nd Surgica l Complica tions

57–20. What is the most important component of diabetic 57–26. Concerning labor and delivery in insulin-requiring
ketoacidosis treatment in pregnancy? diabetics, which of the following is true?
a. Restore euglycemia a. The patient should receive insulin as usual and
S
b. Provide intravenous hydration have breakfast prior to induction.
E
C
b. The mother should be adequately hydrated, and
T
c. Provide intravenous potassium repletion
I
euglycemia is maintained by a continuous insulin
O
d. Provide intravenous bicarbonate to correct
N
infusion.
acidosis
1
c. Primary cesarean is preferred for any insulin-
2
57–21. Which of the following infections is increased in requiring diabetic mother whose baby has an
gravidas with overt diabetes? estimated fetal weight ≥ 4000 g.
a. Pyelonephritis d. Euglycemia is best achieved by administering
dextrose-containing intravenous fluids and
b. Respiratory infections regularly scheduled insulin injections.
c. Wound infection after cesarean delivery
d. All of the above 57–27. Which of the following defines gestational diabetes?
a. Any diabetes that is first detected in pregnancy.
57–22. Concerning the preconceptional period, what can be b. Diabetes that does not require insulin during
said of care for the diabetic woman? pregnancy.
a. Should achieve euglycemia c. A glycohemoglobin A1C level < 7 found early in
b. Should begin daily folate pregnancy.
c. Should have an ophthalmological appointment to d. None of the above
screen for retinopathy
d. All of the above 57–28. Concerning the screening of gravidas for gestational
diabetes, which of the following is true?
57–23. Women with type 1 diabetes should achieve a. Approximately 80% of institutions in the United
glycemic control with which of the following during States use universal screening.
pregnancy? b. Universal screening is endorsed by the
a. Insulin International Workshop Conference on
b. Diet alone Gestational Diabetes.
c. Insulin and diet c. The American College of Obstetricians and
Gynecologists (ACOG) endorses one-step glucose
d. Oral hypoglycemic agents screening that uses a 75-g glucose tolerance test.
57–24. During which of the following epochs in pregnancy d. The American College of Obstetricians and
Gynecologists (ACOG) endorses a two-step
is the peak incidence of maternal hypoglycemia
screening process that begins with a 50-g oral
noted?
glucose screen.
a. 10–14 weeks
b. 20–24 weeks 57–29. Reported by Landon (2009), the Maternal-Fetal
c. 28–32 weeks Medicine Units Network evaluated women with
mild gestational diabetes, defined as a fasting blood
d. 34–38 weeks
glucose level > 95 mg/dL. Compared with women
receiving only standard obstetrical care, those
57–25. In pregnant women with overt diabetes, which of
receiving dietary counseling and glucose monitoring
the following statements is true regarding detection
had decreased rates of which of the following
of fetal anomalies?
morbidities?
a. Sonographic anatomical evaluation is best
performed at 18–22 weeks’ gestation. a. Fetal macrosomia
b. The accuracy of sonographic fetal anatomical b. Cesarean delivery
evaluation is not diminished by concurrent c. Shoulder dystocia
maternal obesity. d. All of the above
c. Maternal serum alpha-fetoprotein determination
should be completed early, between 12 and
16 weeks, in mothers with overt diabetes.
d. None of the above
Dia betes Mellitus 375

57–30. In women with gestational diabetes, early fasting 57–35. Concerning glucose monitoring in pregnancy, which
hyperglycemia is associated with increased rates of of the following is true?
which of the following? a. Fasting blood glucose levels < 100 mg/dL should

C
a. Fetal macrosomia be the goal.

H
A
b. Cesarean delivery b. Preprandial surveillance is superior to

P
postprandial testing.

T
c. Maternal hypertension

E
c. Daily monitoring with reflectance meters may

R
d. All of the above

5
reduce fetal macrosomia rates.

7
57–31. Which of the following factors have been implicated d. Weekly monitoring to achieve fasting blood
in fetal macrosomia? glucose levels < 110 mg/dL reduces cesarean
a. Leptin delivery rates.
b. C-peptide
57–36. Which of the following is appropriate initial insulin
c. Insulin-like growth factor dosing for pregnant women who meet criteria for
d. All of the above therapy?
a. 0.5 units/kg NPH, administered daily in two
57–32. As reported by Metzger (2007), the Fifth divided doses
International Working Conference on Gestational
b. 20 units NPH and 10 units regular insulin,
Diabetes recommended fasting blood glucose levels
administered daily each morning
be kept below what value?
c. 0.7–1.0 units/kg insulin, using a combination of
a. 95 mg/dL
NPH and regular insulin, administered daily in
b. 100 mg/dL divided doses
c. 110 mg/dL d. None of the above
d. 120 mg/dL
57–37. Regarding metformin therapy for gestational
57–33. Concerning nutritional instructions for women with diabetes, which of the following is true?
gestational diabetes, which of the following is true? a. It crosses the placenta.
a. Daily caloric intake should range between 30 and b. Compared with insulin therapy, rates of neonatal
35 kcal/kg. hypoglycemia are increased.
b. A carbohydrate-controlled diet should maintain c. Compared with insulin therapy, rates of adverse
mild hyperglycemia and avoid ketosis. perinatal outcomes are increased.
c. The American College of Obstetricians and d. All of the above
Gynecologists (ACOG) recommends that
carbohydrate intake should comprise no more 57–38. Your patient is a 33-year-old G3P2 white female
than 20% of the total daily calories. who presents for her first prenatal visit at 16 weeks’
d. All of the above gestation. She had a newborn weighing 9 lb and
gestational diabetes in her last pregnancy. At this
57–34. Which of the following reflects the American visit, all EXCEPT which of the following are
College of Obstetricians and Gynecologists (ACOG) appropriate prenatal laboratory tests?
recommendations concerning activity in gravidas a. Rapid plasma reagin test
with gestational diabetes?
b. Serum total cholesterol level
a. Modified bed rest near term
c. 50-g oral glucose tolerance test
b. Moderate exercise for those without
contraindications d. Quad screen to assess fetal risk of aneuploidy and
neural-tube defect
c. Maintenance of the same exercise routine
established before conception
d. None of the above
376 Medica l a nd Surgica l Complica tions

57–39. Testing of the patient in Question 57–38 shows 57–41. Ms. Smith is diagnosed with gestational diabetes that
a negative quad screen result, a glucose level of is controlled solely with diet. Which of the following
146 mg/dL, and a negative rapid plasma reagin test are important for the management of her pregnancy
S
result. Which of the following should be ordered at term?
E
C
next? a. Induction at 38 weeks’ gestation
T
a. 3-hour oral glucose tolerance test
I
b. Weekly umbilical artery Doppler studies
O
N
b. Genetic counseling and offer genetic c. Cesarean delivery if the estimated fetal weight is
1
amniocentesis 4250 g
2
c. Microhemagglutination assay Treponema pallidum d. None of the above
(MHA-TP)
d. None of the above References
57–40. Three-hour oral glucose tolerance testing of the HAPO Study Cooperative Research Group: Hyperglycemia and
patient in Question 57–38 yields normal results. adverse pregnancy outcomes. N Engl J Med 358:2061, 2008
Subsequently, she returns to her prenatal clinic at Landon MB, Spong CY, Thom E, et al: A multicenter, random-
ized treatment trial of mild gestational diabetes. N Engl J Med
normal intervals. At 26 weeks’ gestation, what test
361(14):1339, 2009
should be ordered? Metzger BE, Buchanan TA, Coustan DR, et al: Summary and recom-
a. Rapid plasma reagin test mendations of the Fifth International Workshop-Conference on
b. Targeted fetal sonography Gestational Diabetes. Diabetes Care 30(Suppl 2):S251, 2007

c. 3-hour glucose tolerance test


d. 50-g oral glucose tolerance screen
Dia betes Mellitus 377

CHAPTER 57 ANSw ER KEy

Q uestion Letter Pa ge

C
number a nswer cited Hea der cited

H
A
P
57–1 b p. 1125 Introduction

T
E
57–2 b p. 1125 Cla ssifica tion During Pregna ncy

R
5
57–3 b p. 1125 Cla ssifica tion During Pregna ncy

7
57–4 a p. 1126 W hite Cla ssifica tion in Pregna ncy
57–5 c p. 1127 Dia gnosis
57–6 a p. 1127 Dia gnosis
57–7 d p. 1128 Feta l Effects
57–8 c p. 1128 Ma lforma tions
57–9 d p. 1129 Altered Feta l G rowth
57–10 c p. 1129 Altered Feta l G rowth, Hypoglycemia
57–11 d p. 1129 Altered Feta l G rowth
57–12 c p. 1129 Unexpla ined Feta l Demise
57–13 c p. 1130 Hydra mnios
57–14 a p. 1130 Respira tory Distress Syndrome
57–15 d p. 1130 Hyperbilirubinemia a nd Polycythemia
57–16 b p. 1131 Inherita nce of Dia betes
57–17 c p. 1131 Ma terna l Effects
57–18 c p. 1131 Dia betic N ephropa thy
57–19 d p. 1132 Dia betic Retinopa thy
57–20 b p. 1133 Dia betic Ketoa cidosis
57–21 d p. 1133 Infections
57–22 d p. 1134 Preconceptiona l Ca re
57–23 c p. 1134 Insulin Trea tment
57–24 a p. 1135 Hypoglycemia
57–25 a p. 1135 Second Trimester
57–26 b p. 1135 Third Trimester a nd Delivery
57–27 a p. 1136 G esta tiona l Dia betes
57–28 d p. 1136 G esta tiona l Dia betes
57–29 d p. 1136 Screening a nd Dia gnosis
57–30 d p. 1139 Ma terna l a nd Feta l Effects
57–31 d p. 1140 Feta l Ma crosomia
57–32 a p. 1140 Ma na gement
57–33 a p. 1140 Dia betic Diet
57–34 b p. 1141 Exercise
57–35 c p. 1141 G lucose Monitoring
57–36 c p. 1141 Insulin Trea tment
57–37 a p. 1141 O ra l Hypoglycemic Agents
57–38 b p. 1137 Ta ble 5 7 -1 2
57–39 a p. 1136 Screening a nd Dia gnosis
57–40 c p. 1137 Ta ble 5 7 -1 2
57–41 d p. 1142 O bstetrica l Ma na gement
378

CHAPTER 58

Endocrine Disorders

58–1. Which of the following is true of thyroid-stimulating 58–5. The condition shown here is commonly associated
hormone (TSH) during pregnancy? with which of the following maternal signs?
a. Decreased levels are found in early pregnancy.
b. TSH crosses the placenta and stimulates fetal
thyroxine production.
c. TSH levels are increased in early pregnancy
because of the effects of human chorionic
gonadotropin (hCG).
d. None of the above

58–2. Maternal sources account for what percentage of


fetal thyroxine at term?
a. 10%
b. 30%
c. 50%
d. 70%

58–3. In a study of more than 1000 women with thyroid a. Tachycardia


peroxidase (TPO) antibodies, there was an associated b. Enophthalmos
increased risk of which of the following?
c. Maternal androgen insensitivity
a. Placenta previa d. All of the above
b. Preterm delivery
c. Placental abruption
d. Preterm rupture of membranes

58–4. Which clinical symptom is not characteristic of mild


thyrotoxicosis?
a. Thyromegaly
b. Tachycardia
c. Cold intolerance
d. Poor maternal weight
Endocrine Disorders 379

58–6. The following fetal condition is associated with fetal 58–9. Which of the following is the initial treatment
thyrotoxicosis. If identified antenatally, the mother consideration for thyrotoxicosis in the pregnant
may be treated with which of the following agents, woman?

C
which is subsequently transported transplacentally a. Administer sodium iodide

H
to the fetus?

A
b. Initiate corticosteroids

P
T
c. Administer potassium iodide

E
R
d. None of the above

5
8
58–10. Untreated maternal thyrotoxicosis may lead to which
of the following fetal complications?
a. Stillbirth
b. Macrosomia
c. Postterm gestation
d. All of the above

58–11. Subclinical hyperthyroidism is characterized by which


of the following changes in serum hormone levels?
a. High total thyroxine level
b. Low free thyroxine (FT 4) level
c. Low thyroid-stimulating hormone and normal
FT 4 levels
d. High thyroid-stimulating hormone and normal
Reproduced with permission from Cunningham FG, Leveno KJ, Bloom SL, et al (eds): FT 4 levels
Endocrine disorders. In Williams Obstetrics, 24th ed. New York, McGraw-Hill, 2014,
Figure 58-2.
58–12. Which of the following is true of routine thyroid-
a. Iodine stimulating hormone screening in pregnancy?
b. Prednisone a. It is advocated by the American Association of
Clinical Endocrinologists.
c. Propylthiouracil (PTU)
b. It is not advocated by the American College of
d. Intravenous immune globulin G (IVIG)
Obstetricians and Gynecologists (ACOG).
58–7. If a pregnant woman develops fever or sore throat c. It leads to improved outcome in neonates whose
while taking a thionamide, how should this be mothers have subclinical hypothyroidism.
managed? d. All of the above
a. Stop the medication immediately
58–13. Severe hypothyroidism is uncommon in later
b. Increase her dose of propylthiouracil pregnancy for which of the following reasons?
c. Change from propylthiouracil to methimazole a. It is often associated with infertility.
d. None of the above b. It is associated with increased abortion rates.
58–8. Which of the following is an appropriate initial c. Most women with severe hypothyroidism elect
thionamide dose for pregnant women with first-trimester termination.
hyperthyroidism and is the one typically used at d. A and B
Parkland Hospital?
a. 50 mg po tid 58–14. Concerning initial treatment of maternal
hypothyroidism, which of the following is true?
b. 100 mg po tid
a. Approximately 400 µg thyroxine should be given
c. 200 mg po tid orally daily.
d. 300 mg po bid b. Thyroid-stimulating hormone (TSH) levels
should be measured at 2-week intervals.
c. The thyroxine dose should be adjusted in 25- to
50-µg increments to achieve TSH levels between
0.5 and 2.5 mU/L.
d. All of the above
380 Medica l a nd Surgica l Complica tions

58–15. An increased incidence of which of the following 58–21. Which of the following is true of thyroid nodules
pregnancy outcomes have been linked with maternal during pregnancy.
subclinical hypothyroidism? a. Are poorly assessed by fine-needle aspiration
S
a. Stillbirth b. Can be safely removed before 24 weeks’ gestation
E
C
b. Preeclampsia
T
c. When smaller than 0.5 cm, can be reliably
I
O
c. Placenta previa detected sonographically
N
d. None of the above d. If cancerous, confer a worse prognosis than if
1
2
found in nonpregnant controls
58–16. What condition is described by a low serum free
thyroxine level and normal thyroid-stimulating 58–22. Which of the following is true regarding parathyroid
hormone values? hormone activity in pregnancy?
a. Isolated hypothyroxinemia a. It acts directly on the bone and kidney.
b. Isolated hyperthyroxinemia b. It maintains intracellular calcium concentration.
c. Subclinical hypothyroidism c. It acts indirectly on the small intestine through its
effects on vitamin D synthesis and lowers serum
d. Subclinical hyperthyroidism
calcium levels.
58–17. Mild iodine deficiency during pregnancy typically d. None of the above
may cause which of the following?
a. Neonatal goiter 58–23. What is true concerning parathyroid hormone
(PTH) during pregnancy?
b. Endemic cretinism
a. Maternal PTH levels are increased.
c. Placental abruption
b. Maternal PTH leads to total serum calcium levels
d. Neurodevelopmental abnormalities in offspring that are higher than nonpregnant levels.

58–18. What is the incidence of congenital hypothyroidism? c. PTH that provides the greatest clinical effects is
probably of placental and decidual origin.
a. 1/500
d. All of the above
b. 1/1000
c. 1/3000 58–24. Which of the following is true of
d. 1/9000 hyperparathyroidism?
a. It is generally a disease of young females.
58–19. Which of the following is true of postpartum b. It has a reported prevalence of 2–3 per 10,000
thyroiditis? women.
a. It affects 5–10% of women during the first year c. It is caused mainly by hyperfunctioning of all
postpartum. four parathyroid glands.
b. It develops at a higher rate in women with type 1 d. It may be masked by pregnancy due to significant
diabetes mellitus. calcium shunting to the fetus.
c. It is related to increasing serum levels of thyroid
autoantibodies. 58–25. Which management strategies are considered
d. All of the above appropriate for the pregnant patient with
hyperparathyroidism?
58–20. In regard to postpartum thyroiditis, which of the a. With hypercalcemic crisis, intravenous saline
following is true? infusion and diuresis should be implemented.
a. Initially, there is a hypothyroid phase. b. If a gravida is symptomatic, then oral calcium,
b. Initial hyperthyroidism responds well to 1–1.5 g daily in divided doses, is implemented.
thionamides. c. If a gravida is asymptomatic, surgical removal
c. Beta blockers may be of help in the initial phase should be performed early and before the end of
of postpartum thyroiditis. the first trimester.
d. All of the above d. Hyperkalemia and hypermagnesemia are
complications of hypercalcemic crisis, and levels
of these electrolytes should be monitored.
Endocrine Disorders 381

58–26. What is true concerning hypocalcemia in pregnancy? 58–29. Which of the following is preferred medical
a. The most common cause is renal insufficiency. therapy in pregnancy for hypertension with
pheochromocytoma?
b. Maternal treatment consists of large oral doses of

C
phosphate. a. Hydralazine

H
A
c. Associated hypoparathyroidism follows 20% of b. Beta blockers

P
T
thyroidectomy cases. c. Alpha blockers

E
R
d. Hypocalcemic gravidas may have neonates with d. Calcium-channel blockers

5
bone demineralization.

8
58–30. Concerning Cushing syndrome in general, which of
58–27. Which of the following is true of a the following is true?
pheochromocytoma? a. Most cases arise from long-term corticosteroid
a. It is called the 10-percent tumor. treatment.
b. It is found in 0.1% of hypertensive patients b. This syndrome is rare, with an annual incidence
during pregnancy. of 1 in 3000.
c. It is detected by a 24-hour urine collection c. Occasionally, severe Cushing-associated estrogen
for free catecholamines, metanephrines, or excess may lead to severe feminization.
vanillylmandelic acid (VMA).
d. All of the above
d. All of the above
58–31. Complications of Cushing syndrome in pregnancy
58–28. The patient whose magnetic resonance (MR) include which of the following?
imaging is shown here has a history of hypertension,
a. Greater than 50% of mothers will have
palpitations, and frequent flushing episodes. The
hypertension.
most likely diagnosis is which of the following?
b. Greater than 50% of mothers will develop
diabetes mellitus.
c. Maternal Cushing syndrome carries a maternal
mortality rate of 7%.
d. All of the above

58–32. Concerning Addison disease, which of the following


is true?
a. The most frequent cause is tuberculosis.
b. Cortisone therapy may be discontinued
postpartum.
c. Adrenal hypofunction does not usually affect
fertility.
d. Low serum cortisol levels in pregnancy should
prompt adrenocorticotropic hormone (ACTH)
stimulation testing.

58–33. Hyperaldosteronism is pregnancy is most likely


caused by which of the following?
a. Adrenal carcinoma
b. Adrenal aldosteronoma
c. Bilateral adrenal hyperplasia
Reproduced with permission from Cunningham FG, Leveno KJ, Bloom SL, et al (eds):
Endocrine disorders. In Williams Obstetrics, 24th ed. New York, McGraw-Hill, 2014, d. None of the above
Figure 58-6.
58–34. Normal prolactin levels in pregnancy are considered
a. Wilms tumor those below what threshold?
b. Adrenal tumor a. 12 pg/mL
c. Liver hepatoma b. 18 pg/mL
d. Renal medulloblastoma c. 25 pg/mL
d. 30 pg/mL
382 Medica l a nd Surgica l Complica tions

CHAPTER 58 ANSw ER KEy

Q uestion Letter Pa ge
S
number a nswer cited Hea der cited
E
C
T
58–1 a p. 1147 Thyroid Physiology a nd Pregna ncy
I
O
58–2 b p. 1147 Thyroid Physiology a nd Pregna ncy
N
1
58–3 c p. 1148 Autoimmunity a nd Thyroid Disea se
2
58–4 c p. 1148 Hyperthyroidism
58–5 a p. 1148 Hyperthyroidism
58–6 c p. 1149 Trea tment
58–7 a p. 1149 Trea tment
58–8 b p. 1149 Trea tment
58–9 d p. 1151 Ma na gement
58–10 a p. 1151 Thyroid Storm a nd Hea rt Fa ilure
58–11 c p. 1151 Subclinica l Hyperthyroidism
58–12 b p. 1155 TSH Level Screening in Pregna ncy
58–13 d p. 1152 Hypothyroidism
58–14 c p. 1153 Trea tment
58–15 b p. 1154 Subclinica l Hypothyroidism a nd Pregna ncy
58–16 a p. 1155 Isola ted Ma terna l Hypothyroxinemia
58–17 a p. 1155 Iodine Deficiency
58–18 c p. 1156 Congenita l Hypothyroidism
58–19 d p. 1156 Postpa rtum Thyroiditis
58–20 c p. 1156 Clinica l Ma nifesta tions
58–21 b p. 1157 N odula r Thyroid Disea se
58–22 a p. 1157 Pa ra thyroid Disea ses
58–23 c p. 1157 Pa ra thyroid Disea ses
58–24 d p. 1158 Hyperpa ra thyroidism
58–25 a p. 1158 Ma na gement in Pregna ncy
58–26 d p. 1158 Hypopa ra thyroidism
58–27 d p. 1159 Pheochromocytoma
58–28 b p. 1160 Pheochromocytoma Complica ting Pregna ncy
58–29 c p. 1160 Ma na gement
58–30 a p. 1160 Cushing Syndrome
58–31 a p. 1161 Cushing Syndrome a nd Pregna ncy
58–32 d p. 1161 Adrena l Insufficiency—Addison Disea se
58–33 b p. 1162 Prima ry Aldosteronism
58–34 c p. 1162 Prola ctinoma s
383

CHAPTER 59
00

Connective-Tissue Disorders

59–1. All EXCEPT which of the following isor ers are 59–4. A patient presents with photosensitivity an the
acquire connective tissue isor ers? rash seen here. What is the best screening test for
a. Rheumatoi arthritis systemic lupus erythematosus?
b. Osteogenesis imperfecta
c. Systemic lupus erythematosus
d. Antiphospholipi antibo y syn rome

59–2. Which of the following statements is true regar ing


immune-me iate connective tissue isor ers?
a. hey typically have no renal involvement.
b. hey have a clearly eluci ate pathogenesis.
c. hey are always associate with rheumatoi
factor.
d. hey may or may not have an association with
autoantibo y formation.

59–3. How shoul a patient with a connective-tissue


isor er be counsele regar ing isease activity
uring pregnancy? Use with permission from Dr. Martha Rac.

a. It will worsen.
b. It will improve. a. Anti-Smith antibo y

c. It will be unchange . b. Antinuclear antibo y

d. It will be mo ulate by the effect of pregnancy c. Antiphospholipi antibo y


hormones. d. Anti- ouble-stran e -DNA antibo y

59–5. How early can fetal cells an free fetal DNA be


etecte in maternal bloo ?
a. 1st trimester
b. 2n trimester
c. 3r trimester
d. Puerperium

59–6. he “autoimmunity gene,” which pre isposes to


lupus, rheumatoi arthritis, an Crohn isease, is
locate on what chromosome?
a. Chromosome 6
b. Chromosome 16
c. Chromosome 21
d. Chromosome 22
384 Medica l a nd Surgica l Complica tions

59–7. Which systemic lupus erythematosus-specific 59–13. Base on the 1997 Revise Criteria of the American
antibo y correlates with nephritis an vasculitis Rheumatism Association, a patient coul be
activity when seen in high titers? iagnose with systemic lupus erythematosus if she
S
a. Anti-Ro ha which of the following fin ings?
E
C
b. Anti-La a. Diarrhea, arthritis, anemia, weight loss
T
I
b. Malar rash, anemia, oral ulcers, anti-Smith
O
c. Antinuclear
N
antibo ies
d. Anti- ouble-stran e -DNA
1
c. Discoi rash, renal failure, increase antinuclear
2
59–8. Which autoantibo y is associate with thrombosis, antibo y titers
fetal loss, an thrombocytopenia? d. History of preeclampsia, antiphospholipi
a. Anti-Ro antibo ies, fetal loss
b. Antinuclear
59–14. What is the inci ence of systemic lupus
c. Antiplatelet erythematosus in pregnancy?
d. Antiphospholipi a. 1:1200
b. 1:3300
59–9. Ninety-five percent of patients with systemic lupus
erythematosus experience all EXCEPT which of the c. 1:6000
following clinical manifestations? d. 1:10,000
a. Fever
59–15. A patient presents for preconceptional counseling
b. Arthralgias
with a history of systemic lupus erythematosus
c. Proteinuria (SLE). She was iagnose as an a olescent but has
d. Weight loss one well uring the past few years an is currently
in remission. She wants to know what her chances
59–10. Which of the following autoantibo ies are specific are of having an uncomplicate pregnancy. Which
for lupus? of the following oes not factor into the obstetrical
a. Anti- ouble-stran e -DNA an anti-Ro outcome of a patient with SLE?
b. Anti- ouble-stran e -DNA an anti-La a. ime since last flare
c. Anti-Smith an anti-RNP b. Presence of antinuclear antibo ies
d. Anti-Smith an Anti- ouble-stran e -DNA c. Presence of antiphospholipi antibo ies
d. Disease activity at the beginning of pregnancy
59–11. In a ition to systemic lupus erythematosus (SLE),
what other con itions coul present with low titers 59–16. Following counseling, the patient in Question 59–15
of antinuclear antibo y? woul like to pursue pregnancy. What is the most
a. Chronic inflammation common complication that she will likely encounter
uring her pregnancy?
b. Acute viral infection
c. Autoimmune isor er other than SLE a. Anemia
b. Preeclampsia
d. All of the above
c. Deep-vein thrombosis
59–12. All EXCEPT which of the following laboratory d. Fetal-growth restriction
fin ings may be consistent with a iagnosis of
systemic lupus erythematosus? 59–17. A patient with systemic lupus erythematosus presents
a. Anemia for prenatal care. She is currently asymptomatic on
azathioprine. Her 24-hour urine collection shows 5 g
b. Leukopenia
of protein. What can be sai regar ing this patient’s
c. Decrease d- imer levels potential pregnancy outcome?
d. False-positive Venereal Disease Research a. Her fetus has an increase risk of fetal eath.
Laboratory (VDRL) test result
b. Her fetus has a perinatal mortality risk > 50%.
c. Immunosuppressive treatment shoul be
iscontinue uring pregnancy.
d. She will have a poor outcome whether her isease
remains in remission or not.
Connective-Tissue Disorders 385

59–18. he patient in Question 59–17 woul like to 59–21. he chil ren of patients with systemic lupus
know how you will iagnose a pen ing lupus erythematosus may have increase rates of which of
flare. She states that her rheumatologist follows the following complications?

C
her complement levels routinely. What can be sai a. Neonatal lupus

H
regar ing lupus isease activity in pregnancy an

A
b. Learning isor ers

P
serial evaluation of complement levels?

T
c. Congenital heart block

E
a. Disease activity oes not correlate well with

R
complement levels. d. All of the above

5
9
b. Disease activity correlates best with ecreasing
levels of C3 complement. 59–22. A postpartum patient with systemic lupus
erythematosus (SLE) is concerne after her baby was
c. Disease activity correlates best with ecreasing born with a rash. Her pe iatrician says it is relate
levels of C4 complement. to lupus. What can you tell the patient regar ing the
d. Disease activity correlates best with ecreasing cutaneous manifestations of neonatal lupus?
levels of CH 50 complement. a. hey are usually transient.
59–19. Which of the following rugs shoul be avoi e if b. hey never present beyon the first week of life.
at all possible uring the treatment of obstetrical c. he recurrence risk in a future pregnancy
patients with systemic lupus erythematosus? approximates 5%.
a. Aspirin d. All of the above
b. Azathioprine
59–23. A patient with systemic lupus erythematosus has
c. Corticosteroi s anti-SS-A antibo ies. She is worrie about the risk of
d. Mycophenolate mofetil the con ition seen in this M-mo e image. What is
the risk of this complication in her fetus or neonate?
59–20. Which rug is most helpful in managing a. < 1%
ermatological manifestations of systemic lupus
erythematosus? b. 2–3%
a. Gol salts c. 6–7%
b. Azathioprine d. 10–15%
c. Cyclophosphami e
d. Hy roxychloroquine
386 Medica l a nd Surgica l Complica tions

59–24. Which of the following are safe an effective 59–30. Once clinical criteria are present, all EXCEPT
contraceptive metho s for patients with systemic which of the following tests are useful in iagnosing
lupus erythematosus who lack antiphospholipi antiphospholipi antibo y syn rome?
S
antibo ies? a. Lupus anticoagulant
E
C
a. Progesterone only pills b. Antiplatelet antibo ies
T
I
b. Progesterone intrauterine evice
O
c. Anticar iolipin antibo ies
N
c. Combination estrogen-progesterone pills d. Anti-β 2 glycoprotein I antibo ies
1
2
d. All of the above
59–31. Which is the main mechanism by which
59–25. All EXCEPT which of the following are common antiphospholipi antibo ies cause amage?
features of antiphospholipi antibo y syn rome? a. Deactivation of the tissue factor pathway
a. hrombocytosis b. Increase protein C an protein S activity
b. Central nervous system involvement c. Increase eci ual pro uction of prostaglan in E2
c. Recurrent arterial or venous thrombosis d. Exposure of the basement membrane of
d. Fetal loss in the secon half of pregnancy en othelium an syncytiotrophoblast

59–26. Which of the following might be associate with the 59–32. A pregnant patient has known antiphospholipi
presence of anti-β 2 glycoprotein I antibo ies? antibo ies. Last year, she was hospitalize for
a. Neonatal lupus pulmonary embolism. Which of the following
therapies is most effective uring her current
b. Intervillous space thrombosis
pregnancy?
c. Systemic lupus erythematosus flare
a. Glucocorticoi s
d. All of the above
b. Low- ose aspirin
59–27. Nonspecific antiphospholipi antibo ies are present c. Low-molecular-weight heparin
in low titers in what percentage of the normal d. Low- ose aspirin an unfractionate heparin
nonpregnant population?
a. 1%
b. 5%
c. 10%
d. 15%

59–28. Nonspecific antiphospholipi antibo ies are present


in low titers in what percentage of the normal
pregnant population?
a. 1%
b. 5%
c. 10%
d. 15%

59–29. Which of the following tests is least specific for lupus


anticoagulant?
a. Partial thromboplastin time
b. Dilute Russell viper venom time
c. Platelet neutralization proce ure
d. All of the above are highly specific
Connective-Tissue Disorders 387

59–33. Which statement is false regar ing the connective- 59–36. Which connective-tissue isor er is characterize by
tissue isor er in which these joint fin ings are most a necrotizing granulomatous vasculitis affecting the
classically seen? respiratory tract an ki neys?

C
a. Systemic sclerosis

H
A
b. akayasu arteritis

P
T
c. Ehlers-Danlos syn rome

E
R
d. Wegener granulomatosis

5
9
59–37. Which isor er is characterize by chronic
inflammatory arteritis of the great vessels?
a. Systemic sclerosis
b. akayasu arteritis
c. Ehlers-Danlos syn rome
d. Wegener granulomatosis

59–38. A pregnant patient presents with a characteristic


rash, shown here, an asymmetrical muscle
weakness. A muscle biopsy reveals inflammation an
muscle fiber egeneration. Which of the following is
false?

Repro uce with permission from Shah A, St. Clair W: Rheumatoi arthritis. In Longo DL,
Fauci AS, Kasper DL, et al (e s): Harrison’s Principles of Internal Me icine, 18th e . New
York, McGraw-Hill, 2012, Figure 321-1.

a. It is more common in women than in men.


b. It is characterize by chronic polyarthritis.
c. It is more likely to evelop in women who have
been pregnant.
d. Cigarette smoking increases the risk of this
isor er.

59–34. Which of the following factors contributes the


Repro uce with permission from Allre A, Usatine RP: Dermatomyositis. In Usatine
highest score when etermining whether a patient RP, Smith MA, Chumley H, et al (e s): T e Color Atlas of Family Me icine. New York,
meets criteria for a iagnosis of rheumatoi arthritis? McGraw-Hill, 2009, Figure 174-2.

a. Symptom uration > 6 weeks


b. Involvement of two large joints a. She most likely has polymyositis.
c. Abnormal C-reactive protein levels b. An abnormal electromyogram is expecte .
d. High-positive rheumatoi factor level c. She may have an associate malignant tumor.
d. he isease typically respon s to intravenous
59–35. All EXCEPT which of the following rugs may be immune globulin (IVIG) an corticosteroi s.
use safely in early pregnancy to treat rheumatoi
arthritis? 59–39. Which of the following inherite isor ers has been
a. A alimumab associate with spontaneous uterine rupture?
b. Leflunomi e a. Chon ro ysplasia
c. Sulfasalazine b. Huntington isease
d. Cyclooxygenase-2 (COX-2) inhibitors c. Epi ermolysis bulla
d. Ehlers-Danlos syn rome
388 Medica l a nd Surgica l Complica tions

CHAPTER 59 ANSw ER KEy

Q uestion Letter Pa ge
S
number a nswer cited Hea der cited
E
C
T
59–1 b p. 1168 Introduction
I
O
59–2 d p. 1168 Immune-Media ted Connective-Tissue Disea ses
N
1
59–3 d p. 1168 Immune-Media ted Connective-Tissue Disea ses
2
59–4 b p. 1169 Clinica l Ma nifesta tions a nd Dia gnosis
59–5 a p. 1168 Immune-Media ted Connective-Tissue Disea ses
59–6 b p. 1169 Systemic Lupus Erythema tosus
59–7 d p. 1169 Ta ble 5 9 -1
59–8 d p. 1169 Ta ble 5 9 -1
59–9 c p. 1169 Ta ble 5 9 -2
59–10 d p. 1169 Ta ble 5 9 -1 ; Clinica l Ma nifesta tions a nd Dia gnosis
59–11 d p. 1169 Clinica l Ma nifesta tions a nd Dia gnosis
59–12 c p. 1170 Ta ble 5 9 -3 ; Clinica l Ma nifesta tions a nd Dia gnosis
59–13 b p. 1170 Ta ble 5 9 -3
59–14 a p. 1170 Lupus a nd Pregna ncy
59–15 b p. 1170 Lupus a nd Pregna ncy
59–16 b p. 1171 Ta ble 5 9 -4
59–17 a p. 1170 Lupus N ephritis
59–18 a p. 1171 Ma na gement During Pregna ncy
59–19 d p. 1172 Pha rma cologica l Trea tment
59–20 d p. 1172 Pha rma cologica l Trea tment
59–21 d p. 1172 Perina ta l Morta lity a nd Morbidity
59–22 a p. 1172 N eona ta l Lupus Syndrome
59–23 b p. 1172 Congenita l Hea rt Block
59–24 d p. 1173 Long-Term Prognosis a nd Contra ception
59–25 a p. 1173 Ta ble 5 9 -5 ; Antiphospholipid Antibody Syndrome
59–26 b p. 1173 Specific Antiphospholipid Antibodies
59–27 b p. 1174 Pregna ncy a nd Antiphospholipid Antibodies
59–28 b p. 1174 Pregna ncy a nd Antiphospholipid Antibodies
59–29 a p. 1174 Antiphospholipid Antibody Syndrome Dia gnosis
59–30 b p. 1174 Antiphospholipid Antibody Syndrome Dia gnosis
59–31 d p. 1174 Pregna ncy Pa thophysiology
59–32 d p. 1175 Trea tment in Pregna ncy
59–33 c p. 1176 Rheuma toid Arthritis
59–34 d p. 1177 Ta ble 5 9 -6
59–35 b p. 1176 Ma na gement
59–36 d p. 1180 W egener G ra nuloma tosis
59–37 b p. 1180 Ta ka ya su Arteritis
59–38 a p. 1180 Infla mma tory Myopa thies
59–39 d p. 1181 Ehlers Da nlos Syndrome
389

CHAPTER 60
00

Neurological Disorders

60–1. Neurovascular disorders account for what percent of 60–6. A 28-year-old woman who is 8 weeks pregnant
maternal deaths in the United States? presents with symptoms that are consistent with
a. 1% a migraine headache. Which of the following
medications used to treat migraine headaches should
b. 5%
be avoided?
c. 10%
a. Ibuprofen
d. 20% b. Metoprolol
60–2. Why is magnetic resonance imaging a preferred c. Sumatriptan
modality in the diagnosis of neurovascular disorders d. Ergotamine derivatives
in pregnancy?
a. It is the most cost effective. 60–7. A young woman presents for preconceptional
counseling because of her history of seizure disorder.
b. It does not involve ionizing radiation. She currently takes phenytoin and phenobarbital,
c. It is excellent for detecting recent hemorrhage. and her last seizure was 1 year ago. What should
d. None of the above she be told about her medication use and imminent
attempt at conception?
60–3. What is the most common neurologic complaint a. An attempt should be made to reduce her
during pregnancy? medications to a single drug.
a. Seizure b. She should change to valproate to reduce the risk
b. Headache of fetal teratogenicity.
c. Leg numbness c. No change in her medication regimen is indicated
d. Hand weakness if her seizure disorder is stable.
d. Since she has been seizure free for 1 year, she
60–4. Which type of headache is most likely to be affected should withdraw from all medication use.
by pregnancy-induced hormonal changes?
a. Cluster 60–8. Which of the following is not a reason for
subtherapeutic anticonvulsant levels during
b. ension pregnancy?
c. Migraine a. Increased gastric motility
d. Intracranial nonvascular b. Increased nausea and vomiting
60–5. Which of the following obstetrical complications c. Increased glomerular filtration
is increased in women who experience migraine d. Induction of hepatic enzymes during pregnancy
headaches?
a. Preeclampsia
b. Preterm labor
c. Premature rupture of membranes
d. None of the above
390 Medica l a nd Surgica l Complica tions

60–9. Which of the following medications is not associated 60–13. What is the most common etiology for ischemic
with an increased risk of this congenital anomaly stroke in pregnancy?
when taken in early pregnancy? a. Cocaine use
S
b. Hypertension
E
C
T
c. Saccular aneurysm
I
O
d. Arteriovenous malformation
N
1
2
60–14. A 36-year-old African American primigravida who
is 30 weeks pregnant had a headache earlier in the
day. Her husband brings her to the emergency
department after noticing she is unable to use
her right hand and arm normally. Which of the
following laboratory studies is not appropriate in her
evaluation?
a. Serum lipids
b. Alkaline phosphatase
c. Antiphospholipid antibody
d. Hemoglobin electrophoresis
Reproduced with permission from Cunningham FG, Leveno KJ, Bloom SL, et al (eds):
Fetal imaging. In Williams Obstetrics, 23rd ed. New York, McGraw-Hill, 2010, Figure 60–15. he patient in Question 60–14 presents for her
16-9A. postpartum visit and wants to discuss risks and
management during a future pregnancy. Which of
a. Valproate the following statements is most accurate?
b. Phenytoin a. Prophylactic β -blocker therapy during pregnancy
c. Carbamazepine is indicated.
d. None of the above b. here are no firm guidelines for prophylaxis
during a future pregnancy.
60–10. Serum levels of antiepileptic medications are c. he recurrence risk is high, and she should not
unreliable in pregnancy for which of the following pursue another pregnancy.
reasons? d. All women with prior stroke should be given
a. Altered protein binding prophylactic anticoagulation.
b. Increased glomerular filtration
60–16. Which of the following is not an accurate statement
c. Levels are not available for new medications
regarding maternal middle cerebral artery embolism
d. None of the above during pregnancy?
a. May be caused by paradoxical embolism
60–11. Increased rates of which of the following have not
contributed to the increase in stroke prevalence? b. Occurs more commonly in the first trimester
a. Obesity c. Must exclude thrombosis and hemorrhage prior
to diagnosis
b. Heart disease
d. reatment includes antiplatelet therapy during
c. Smoking in women
pregnancy
d. Diabetes mellitus

60–12. When do pregnancy-related strokes most commonly


occur?
a. Postpartum
b. Intrapartum
c. First trimester
d. Second trimester
N eurologica l Disorders 391

60–17. How do the morbidity and mortality rates with the 60–20. During computed tomographic (C ) angiography
following lesion compare with those of subarachnoid performed as part of an evaluation for headache, the
hemorrhage? following was found in your patient. What is the risk

C
of lesion rupture?

H
A
P
T
E
R
6
0
Reproduced with permission from Cowan JA Jr, T ompson BG: Neurosurgery. In Doherty
GM (ed): Current Diagnosis & reatment: Surgery, 13th ed. New York, McGraw-Hill,
2010, Figure 36-6B.

Reproduced with permission from Roberts HR, Key NS, Escobar MA: Hemophilia A and
hemophilia B. In Lichtman MA, Kipps J, Seligsohn U (eds): Williams Hematology, 8th ed. a. 0.1%
New York, McGraw-Hill, 2010, Figure 124-14.
b. 0.5%
c. 1%
a. Increased
d. 5%
b. Decreased
c. No difference 60–21. For a pregnant patient, which of the following
d. No information available statements regarding the lesion seen in Question
60–20 is most accurate?
60–18. What is the most important management strategy a. he cardinal symptom of rupture is hemiparesis.
used to reduce complications associated with
b. Aneurysms are most likely to bleed in the first
Charcot-Bouchard aneurysms?
trimester.
a. Control of systolic hypertension
c. Surgical repair is the preferred treatment for
b. Appropriate prophylactic anticoagulation women remote from term.
c. Prepregnancy closure of a patent foramen ovale d. Cranial computed tomography with contrast is
d. None of the above the preferred imaging modality.

60–19. What is the most common cause of subarachnoid 60–22. Which of the following statements regarding
hemorrhage? arteriovenous malformations and pregnancy is true?
a. rauma a. he mortality rate associated with hemorrhage is
b. Cerebral venous thrombosis 25–50%.
c. Ruptured saccular aneurysm b. Bleeding from this lesion is more frequent during
pregnancy.
d. Ruptured arteriovenous malformation
c. hey are the most common abnormality of
the cerebrovascular system encountered during
pregnancy.
d. None of the above

60–23. Which of the following organisms is suggested to act


as an environmental trigger in the development of
multiple sclerosis?
a. Influenza B
b. Herpesvirus 6
c. Chlamydia trachomatis
d. None of the above
392 Medica l a nd Surgica l Complica tions

60–24. What percent of women will develop multiple 60–27. Acute exacerbations of multiple sclerosis during
sclerosis following an episode of isolated optic pregnancy can safely be treated with which of the
neuritis? following therapies?
S
a. 10% a. Interferon β 1a and β 1b
E
C
b. 25% b. High-dose corticosteroids
T
I
O
c. 50% c. Intravenous immune globulin
N
d. 75% d. All of the above
1
2
60–25. Identification of the lesions shown in this imaging 60–28. A 34-year-old woman who was diagnosed
study is most useful in what capacity? with multiple sclerosis 2 years ago presents for
preconceptional counseling. She currently takes
baclofen and natalizumab. What she should be told
about her condition?
a. Patients with multiple sclerosis have slightly
poorer perinatal outcomes.
b. Intravenous immune globulin may prevent
relapses during the puerperium.
c. Baclofen and natalizumab should both be stopped
before attempting to conceive.
d. Breast feeding has a protective effect on the
postpartum relapse rate.

60–29. Antibodies to which of the following structures are


commonly found in patients with myasthenia gravis?
a. Nonself antigens
b. Acetylcholine receptor
c. Oligodendrocyte glycoprotein
Reproduced with permission from Hauser SL, Goodin DS: Multiple sclerosis. In Longo d. Muscle-specific tyrosine kinase
DL, Fauci AS, Kasper DL, et al (eds): Harrison’s Principles of Internal Medicine, 18th ed.
New York, McGraw-Hill, 2012, Figure 380-3A.
60–30. A 30-year-old white woman with myasthenia
gravis who is currently taking corticosteroids and
a. o determine the likelihood of a relapse
azathioprine presents for prenatal care at 12 weeks’
b. o estimate the risk of obstetric complications gestation. What is the most appropriate treatment to
c. o confirm the initial diagnosis of multiple manage her disease?
sclerosis a. Stop azathioprine
d. o evaluate response following treatment with b. Continue her current medications
interferon β 1a
c. Recommend prophylactic thymectomy
60–26. For patients with multiple sclerosis, which of the d. Start plasmapheresis in the third trimester to
following is not associated with an increased risk of reduce neonatal effects
relapse in the puerperium?
a. Breast feeding
b. Relapses during pregnancy
c. High relapse rate prior to pregnancy
d. High multiple sclerosis disability score
N eurologica l Disorders 393

60–31. he patient in Question 60–30 has a prenatal 60–33. In which trimester do exacerbations of myasthenia
sonographic examination in the third trimester. gravis most commonly occur?
What is the best explanation for the findings shown a. First

C
here?
b. Second

H
A
c. hird

P
A

T
d. Equal rates across all trimesters

E
R
6
60–34. During pregnancy, what is the most appropriate

0
treatment of acute Guillain-Barré syndrome?
a. Cyclosporine
b. Interferon 1β 2
c. Intravenous dexamethasone
d. High-dose intravenous immune globulin

60–35. How is the frequency of the lesion shown here


altered during pregnancy?

a. Doubled
b. No change
a. Her fetus also has a diagnosis of myasthenia c. Quadrupled
gravis. d. Decreased by half
b. he finding is unrelated to her diagnosis of
myasthenia gravis. 60–36. Which of the following is not a recommended
c. She has a higher likelihood of having developed therapy for the condition shown in Question
gestational diabetes. 60–35?
d. his is a transient side effect of transplacental a. Massage
passage of maternal antibodies. b. Prednisone
c. Valacyclovir
60–32. Which of the following medications should be
avoided during labor and delivery in a woman with a d. Artificial tears
diagnosis of myasthenia gravis?
60–37. Which of the following complications is increased in
a. Gentamicin women who experience Bell palsy during pregnancy?
b. Succinylcholine a. Preeclampsia
c. Magnesium sulfate b. Preterm labor
d. All of the above c. Premature rupture of membranes
d. None of the above
394 Medica l a nd Surgica l Complica tions

60–38. Which of the following complications is not 60–40. Which of the following is not a criteria for the
increased in women with spinal cord injuries? diagnosis of idiopathic intracranial hypertension?
a. Preterm labor a. Recent weight gain > 20 lb
S
b. Low birthweight b. Intracranial pressure > 250 mm H 2O
E
C
c. Normal cerebral spinal fluid chemistries
T
c. Gestational diabetes
I
O
d. Asymptomatic bacteriuria d. Normal cerebral magnetic resonance imaging
N
1
60–41. Which of the following is not a common therapy
2
60–39. Which of the following activities does not precipitate
autonomic dysreflexia in parturients with spinal cord used to treat the development of visual field defects
injuries? associated with idiopathic intracranial hypertension
a. Amniotomy in pregnancy?
b. Foley catheter insertion a. Furosemide
c. Epidural catheter placement b. Prednisone
d. Uterine tocodynamometer transducer positioning c. Acetazolamide
d. herapeutic lumbar puncture
N eurologica l Disorders 395

CHAPTER 60 ANSw ER KEy

Q uestion Letter Pa ge

C
number a nswer cited Hea der cited

H
A
P
60–1 c p. 1187 Introduction

T
E
60–2 b p. 1187 Centra l N ervous System Ima ging

R
6
60–3 b p. 1187 Hea da che

0
60–4 c p. 1187 Hea da che
60–5 a p. 1188 Migra ine in Pregna ncy
60–6 d p. 1188 Ma na gement
60–7 a p. 1189 Preconceptiona l Counseling
60–8 a p. 1190 Epilepsy During Pregna ncy
60–9 b p. 1190 Ta ble 6 0 -2
60–10 a p. 1191 Ma na gement in Pregna ncy
60–11 c p. 1191 Cerebrova scula r Disea se
60–12 a p. 1191 Risk Fa ctors
60–13 b p. 1191 Risk Fa ctors
60–14 b p. 1191 Ischemic Stroke
60–15 b p. 1193 Recurrence Risk of Ischemic Stroke
60–16 b p. 1192 Cerebra l Embolism
60–17 a p. 1193 Intra cerebra l Hemorrha ge
60–18 a p. 1193 Intra cerebra l Hemorrha ge
60–19 c p. 1193 Suba ra chnoid Hemorrha ge
60–20 c p. 1194 Intra cra nia l Aneurysm
60–21 c p. 1194 Intra cra nia l Aneurysm
60–22 d p. 1194 Arteriovenous Ma lforma tions
60–23 b p. 1194 Multiple Sclerosis
60–24 d p. 1194 Multiple Sclerosis
60–25 c p. 1194 Multiple Sclerosis
60–26 a p. 1195 Effects of Pregna ncy on MS
60–27 d p. 1195 Ma na gement During Pregna ncy a nd the Puerperium
60–28 b p. 1195 Ma na gement During Pregna ncy a nd the Puerperium
60–29 b p. 1196 Mya sthenia G ra vis
60–30 b p. 1196 Mya sthenia a nd Pregna ncy
60–31 d p. 1197 N eona ta l Effects
60–32 d p. 1196 Mya sthenia a nd Pregna ncy
60–33 d p. 1196 Mya sthenia a nd Pregna ncy
60–34 d p. 1197 Pregna ncy
60–35 c p. 1197 Bell Pa lsy
60–36 c p. 1198 Pregna ncy
60–37 a p. 1198 Pregna ncy
60–38 c p. 1198 Spina l-Cord Injury
60–39 d p. 1198 Spina l-Cord Injury
60–40 a p. 1199 Idiopa thic Intra cra nia l Hypertension
60–41 b p. 1199 Effects of Pregna ncy
396

CHAPTER 61

Ps chiatric Disorders

61–1. Which of the following has been associated with 61–6. A 35-year-old primipara presents to your office for
psychiatric disorders in pregnancy? routine follow-up. The patient reports feeling very
a. Substance abuse sad, anxious, and exhausted. In addition, she has
a great sense of hopelessness. Even when the baby
b. Poor infant outcomes
sleeps, she finds that she cannot get to sleep. This
c. Poor obstetrical outcomes has been going on for at least a month. The patient
d. All of the above has a history of depression for which she took
medication prior to pregnancy. She denies thoughts
61–2. A 20-year-old primipara presents for her 2-week of suicide or infanticide. Which of the following
postpartum appointment. During your conversation, statements is true?
she reports that she experienced some sadness with a. For women who discontinue treatment for
crying and insomnia when she came home from the depression, 70% relapse.
hospital. Overall, she considers herself a fairly happy
b. If this patient goes untreated, there is a 25%
person, and those symptoms have since resolved. She
chance that she will be depressed 1 year from
mentions it only because it has never happened to
now.
her before. What is the most likely diagnosis?
c. Her condition could lead to insecure attachment
a. Postpartum blues
and later behavioral problems in the child.
b. Domestic violence
d. All of the above
c. Postpartum psychosis
d. Postpartum depression 61–7. You diagnose the patient in Question 61–6 with
a major depressive disorder. You think it is severe.
61–3. What is the best management plan for the patient in Your best plan of management is which of the
Question 61–2? following?
a. Emotional support a. Psychotherapy
b. Initiate bupropion b. Antidepressant treatment and psychotherapy
c. Brief course of fluoxetine c. Treatment with a mood stabilizer such as lithium
d. Weekly therapy sessions for 4–6 months d. Hospitalization under the care of a psychiatrist

61–4. Risk factors for depression include which of the 61–8. In a woman with severe depression, which of the
following? following medications might best be tried initially?
a. History of abuse a. Lithium
b. Nicotine dependence b. Citalopram
c. Family history of depression c. Amitriptyline
d. All of the above d. Tranylcypromine

61–5. What is the most common mood disorder? 61–9. Which of the following selective serotonin-reuptake
a. Schizophrenia inhibitors has been most closely associated with fetal
heart defects?
b. Bipolar disorder
a. Citalopram
c. Major depression
b. Fluoxetine
d. Anorexia nervosa
c. Paroxetine
d. Sertraline
Psychia tric Disorders 397

61–10. A 25-year-old G1P0 at 12 weeks’ gestation with a 61–15. Lithium has been linked to which of the following
history of major depression presents for prenatal fetal anomalies?
care. She reports that she was taking paroxetine a. Club foot

C
up until 2 weeks ago when she found out she
b. Omphalocele

H
was pregnant. Her psychiatrist has changed her

A
c. Ebstein anomaly

P
medication, and she is doing well. Based on her

T
history, you should consider offering her which of d. Pulmonary sequestration

E
R
the following?

6
61–16. Which of the following statements regarding

1
a. Amniocentesis
postpartum psychosis is true?
b. Pregnancy termination
a. It is more common in multiparas.
c. Fetal echocardiography
b. Its incidence is 1/10,000 deliveries.
d. Maternal brain magnetic resonance imaging
c. It usually manifests 6–8 weeks after delivery.
61–11. A postpartum patient who is exclusively breast d. It is more common in patients with obstetrical
feeding is diagnosed with major depression. The complications.
decision is made to start a selective serotonin-
reuptake inhibitor. Which of the following has the 61–17. What is the most important risk factor for
highest detectable concentrations in breast milk? postpartum psychosis?
a. Citalopram a. Drug abuse
b. Fluoxetine b. Bipolar disorder
c. Paroxetine c. Major depression
d. Sertraline d. Multifetal gestation

61–12. Pregnant women not adequately prepared for 61–18. What is the recurrence risk of postpartum psychosis?
electroconvulsive therapy are at increased for all a. 5%
EXCEPT which of the following?
b. 10%
a. Aspiration
c. 25%
b. Hypotension
d. 50%
c. Respiratory acidosis
d. Aortocaval compression 61–19. A 32-year-old G4P3 at 39 weeks’ gestation presents
in active labor. The patient had postpartum
61–13. Which of the following statements regarding bipolar psychosis in her last pregnancy. In addition to close
disorder is true? monitoring for recurrence and involvement of the
a. Periods of depression last at least 2 months. family, you plan for her to initiate which of the
following agents in the puerperium?
b. There is no genetic component to the illness.
a. Lithium
c. Up to 20% of patients with this illness commit
suicide. b. Fluoxetine
d. It is more common among pregnant women c. Citalopram
compared with nonpregnant women. d. No pharmacologic agent

61–14. A 25-year-old G1P0 at 18 weeks’ gestation presents 61–20. Which of the following statements regarding anxiety
for prenatal care. The patient has a long history disorders is true?
of debilitating bipolar disorder. She had been a. They are more common in pregnant women than
prescribed lithium and was taking this during the nonpregnant women.
early weeks of her current pregnancy. Based on her
b. The prevalence of these disorders in adults in the
history, you should offer her which of the following?
United States is 1.8%.
a. Amniocentesis c. With generalized anxiety disorder, symptoms and
b. Pregnancy termination severity increase across pregnancy.
c. Fetal echocardiography d. They are often characterized by irrational
d. Maternal brain magnetic resonance imaging fear, nausea, insomnia, dizziness, and frequent
urination.
398 Medica l a nd Surgica l Complica tions

61–21. A 21-year-old G1P0 at 15 weeks’ gestation presents 61–25. A 30-year-old G2P1 at 8 weeks’ gestation with a
for prenatal care. She has a history of anxiety and history of schizophrenia controlled with haloperidol
is concerned for her baby. Which of the following presents for prenatal care. Which medication
S
counseling points are true? regimen do you have planned for her during
E
C
a. No pharmacotherapy agents can be used safely in pregnancy?
T
pregnancy. a. Continue her current medication regimen
I
O
N
b. Pharmacotherapy for anxiety has been strongly b. Stop haloperidol until the second trimester
1
linked to cleft lip and palate. c. Change haloperidol to an atypical antipsychotic
2
c. Benzodiazepines taken during the third trimester d. Stop haloperidol for the remainder of pregnancy
can cause neonatal withdrawal syndrome. and manage her with psychotherapy
d. All studies have shown that her child is at
increased risk for various neuropsychiatric 61–26. Obstetrical complications of eating disorders include
conditions. which of the following?
a. Spontaneous abortion
61–22. Which of the following is not a prominent feature of b. Low neonatal birthweight
schizophrenia spectrum disorders?
c. Poor maternal wound healing
a. Somnolence
d. All of the above
b. Hallucinations
c. Disorganized thinking 61–27. Which of the following statements regarding eating
d. Abnormal motor behavior disorders is true?
a. Lifetime prevalence is 10–20%.
61–23. If one parent has schizophrenia, what is the risk of b. Eating disorder symptoms worsen with
this condition developing in offspring? pregnancy.
a. < 1% c. Binge-eating disorder is associated with low
b. 5–10% neonatal birthweight.
c. 15–20% d. Pregnant women with a history of an eating
d. 50–60% disorder should be closely monitored for weight
gain.
61–24. Within 5 years from the first signs of schizophrenia,
what percent of patients are employed? 61–28. Which of the following is not a personality disorder?
a. 10% a. Paranoid
b. 30% b. Schizoid
c. 50% c. Schizotypal
d. 70% d. Schizoaffective
Psychia tric Disorders 399

CHAPTER 61 ANSw ER KEy

Q uestion Letter Pa ge

C
number a nswer cited Hea der cited

H
A
P
61–1 d p. 1204 Psychia tric Disorders

T
E
61–2 a p. 1205 Ma ternity Blues

R
6
61–3 a p. 1205 Ma ternity Blues

1
61–4 d p. 1205 Prena ta l Eva lua tion
61–5 c p. 1206 Ma jor Depression
61–6 d p. 1206 Ma jor Depression
61–7 b p. 1207 Fig 6 1 -1 Trea tment
61–8 b p. 1207 Trea tment
61–9 c p. 1207 Feta l Effects of Thera py
61–10 c p. 1207 Feta l Effects of Thera py
61–11 b p. 1207 Feta l Effects of Thera py
61–12 c p. 1209 Electroconvulsive Thera py
61–13 c p. 1209 Bipola r a nd Rela ted Disorders
61–14 c p. 1209 Bipola r a nd Rela ted Disorders
61–15 c p. 1209 Bipola r a nd Rela ted Disorders
61–16 d p. 1210 Postpa rtum Psychosis
61–17 b p. 1210 Postpa rtum Psychosis
61–18 d p. 1210 Postpa rtum Psychosis
61–19 a p. 1210 Postpa rtum Psychosis
61–20 d p. 1210 Anxiety Disorders
61–21 c p. 1210 Trea tment
61–22 a p. 1210 Schizophrenia Spectrum Disorders
61–23 b p. 1210 Schizophrenia Spectrum Disorders
61–24 c p. 1210 Schizophrenia Spectrum Disorders
61–25 a p. 1211 Trea tment
61–26 d p. 1211 Feeding a nd Ea ting Disorders—Pregna ncy
61–27 d p. 1211 Feeding a nd Ea ting Disorders
61–28 d p. 1211 Persona lity Disorders
400

CHAPTER 62

Dermatological Disorders

62–1. Which o the ollowing is the most common, 62–5. In pemphigoid gestationis, there is a reaction
occurring in 1 percent o pregnancies? between maternal immunoglobulin G (IgG) and
a. Pemphigoid which o the ollowing?
b. Atopic eruptions a. Collagen V
c. Intrahepatic cholestasis o pregnancy b. Collagen X
d. Pruritic urticarial papules and plaques o c. Collagen XV
pregnancy (PUPPP) d. Collagen XVII

62–2. Which o the ollowing statements regarding 62–6. A 22-year-old G1P0 at 26 weeks’ gestation presents
intrahepatic cholestasis o pregnancy is true? or her prenatal appointment. She complains o an
a. It has not been linked to adverse etal outcome. itchy rash that started 2 to 3 days prior. She denies
taking any medications. A photograph is provided
b. Hepatic transaminase levels are commonly in the
below. What is your treatment plan?
thousands.
c. It is associated with abnormally elevated serum
bile acid levels.
d. An erythematous maculopapular rash precedes
development o pruritus.

62–3. A 25-year-old G1P0 at 35 weeks’ gestation presents


to your o ice complaining o intense pruritus or
one week. On examination, the patient has no
lesions except or impressive excoriations covering a
large portion o her body. She denies rash, ever, or
sick contacts. The most likely diagnosis is which o
the ollowing?
a. Eczema o pregnancy
b. Pemphigoid gestationis
c. Intrahepatic cholestasis o pregnancy
d. Pruritic urticarial papules and plaques o
pregnancy (PUPPP)

62–4. Which o the ollowing statements about


pemphigoid gestationis is true?
Reproduced with permission rom Wolf K, Johnson RA (eds): Bullous diseases. In
a. It is most commonly seen on the ace. Fitzpatrick’s Color Atlas and Synopsis o Clinical Dermatology, 6th ed. New York,
McGraw-Hill, 2009, Figure 6-15.
b. It results rom an in ection with herpesvirus.
c. It typically starts in the irst trimester o
a. 7-day course o cephalexin
pregnancy.
b. 5-day course o oral acyclovir
d. It has been associated with preterm birth and
etal-growth restriction. c. Over-the-counter clotrimazole cream
d. Oral corticosteroids and oral antihistamine
Derma tologica l Disorders 401

62–7. You recommend that the patient in Question 62–6 62–9. An 18-year-old G1P0 at 37 weeks’ gestation presents
take oral diphenhydramine and return in 1 week. to your o ice complaining o rash and itching.
One week later, she complains o vesicular lesions. A photograph is provided. On examination, you

C
A photograph is provided below. What should be note striae and erythematous papules on her gravid

H
included in your counseling? abdomen. The papules extend to her thighs but

A
P
spare the umbilicus. She otherwise eels well, and

T
she denies ever or sick contacts. The most likely

E
R
diagnosis is which o the ollowing?

6
2
Reproduced with permission rom Cunningham FG, Leveno KJ, Bloom SL, et al (eds):
Dermatological disorders. In Williams Obstetrics, 23rd ed. New York, McGraw-Hill,
2010, Figure 56-5.

Used with permission rom Dr. Kathryn Grande.


a. There is a 50-percent chance that the baby will
have similar lesions.
a. Scabies
b. This has never been associated with preterm birth
b. Discom orts o pregnancy
or growth restriction.
c. Intrahepatic cholestasis o pregnancy
c. Although the lesions may heal, there will likely be
signi icant scarring. d. Pruritic urticarial papules and plaques o
pregnancy (PUPPP)
d. I this does not respond to oral corticosteroids,
other treatment modalities including intravenous
62–10. Treatment or the patient in Question 62–9 is which
immunoglobulin or plasmapheresis may be
o the ollowing?
needed.
a. Antipruritics, acyclovir, topical corticosteroids
62–8. Which o the ollowing has been used in the b. Antipruritics, cholestyramine, ursodeoxycholic
treatment o pemphigoid gestationis? acid
a. Oral prednisone c. Antipruritics, skin emollients, topical
b. Oral antihistamines corticosteroids
c. High-dose intravenous immunoglobulin therapy d. Antipruritics and an antibiotic with good
gram-positive coverage
d. All o the above
62–11. Which o the ollowing is a risk actor or PUPPP?
a. Multiparity
b. Female etus
c. Multi etal gestation
d. A rican American ethnicity
402 Medica l a nd Surgica l Complica tions

62–12. Which o the ollowing statements regarding PUPPP 62–15. A 19-year-old G1P0 at 16 weeks’ gestation presents
is true? or prenatal care. She has severe acne, and her
a. It seldom recurs in subsequent pregnancies. dermatologist has resisted treating her because o the
S
pregnancy. The patient would like to know what
b. Lesions originate in the periumbilical area.
E
C
medications she can take or acne while pregnant.
c. Symptoms requently persist or 6 to 9 months
T
Which o the ollowing is/are sa e during pregnancy?
I
O
postpartum.
a. Azelaic acid
N
d. Patients should be counseled about the signi icant
1
b. Benzoyl peroxide
2
scarring that will likely occur.
c. Topical erythromycin
62–13. A 24-year-old G2P0 presents at 28 weeks’ gestation d. All o the above are acceptable
complaining o a 4-week history o pruritic,
erythematous arm papules. A photograph is 62–16. Which o the ollowing is true about the condition
provided. The patient otherwise eels well, and she represented in this photograph?
denies sick contacts. Which o the ollowing is the
likely diagnosis?

Reproduced with permission rom Lawley TJ, Yancey KB: Approach to the patient with
a skin disorder. In Longo DL, Fauci AS, Kasper DL, et al (eds): Harrison’s Principles o
Internal Medicine, 18th ed. New York, McGraw-Hill, 2012, Figure 51-7.

a. Emollients should be avoided.


b. Postpartum lares are uncommon.
c. It has a variable course in pregnancy.
d. It has been associated with postterm pregnancy.
Reproduced with permission rom Karen JK, Pomeranz MK: Skin changes and diseases
in pregnancy. In Goldsmith LA, Katz SI, Gilcrest BA (eds): Fitzpatrick’s Dermatology in 62–17. Which o the ollowing is the most common
General Medicine, 8th ed. New York, McGraw-Hill, 2012, Figure 108-6. pregnancy-speci ic dermatosis?
a. Cutaneous lupus
a. Scabies
b. Erythema nodosum
b. Psoriasis
c. Eczema in pregnancy
c. Prurigo gestationis
d. Pruritic olliculitis o pregnancy
d. Eczema in pregnancy

62–14. Treatment or the patient in Question 62–13 would


include which o the ollowing?
a. Oral luconazole
b. Permethrin 5-percent cream
c. Oral trimethoprim-sul amethoxazole
d. Low- or moderate-potency topical corticosteroids
Derma tologica l Disorders 403

62–18. A 24-year-old G2P1 at 22 weeks’ gestation 62–20. A pregnant patient with a history o sarcoidosis
complaining o a 2-day rash. It started in her axilla reports having red, warm nodules o her legs that
but has now spread to her torso. She reports having have since lattened and now look like bruises. She

C
this rash be ore when she previously used oral denies trauma. A photograph is provided. What is

H
contraceptives. She denies contact with new products the most likely diagnosis?

A
P
or insect bites. A photograph is provided. What is

T
the most likely diagnosis?

E
R
6
2
Used with permission rom Dr. Stephan Shivvers.

Reproduced with permission rom Wolf K, Johnson RA (eds): Psoriasis. In Fitzpatrick’s a. Erythema nodosum
Color Atlas and Synopsis o Clinical Dermatology, 6th ed. New York, McGraw-Hill,
2009, Figure 3-13.
b. Pemphigoid gestationis
c. Intrahepatic cholestasis o pregnancy
a. Acne d. Pruritic urticarial papules and plaques o
b. Pustular psoriasis pregnancy (PUPPP)
c. Superin ected herpes
62–21. The condition in Question 62–20 is also associated
d. Pemphigoid gestationis with which o the ollowing?
a. Medications
62–19. Treatment or the patient in Question 62–18 should
include which o the ollowing? b. Malignancy
a. Acyclovir c. In lammatory bowel disease
b. Acyclovir and antimicrobials or secondary d. All o the above
in ection
c. Systemic corticosteroids and antimicrobials or
secondary in ection
d. Systemic corticosteroids, acyclovir, and
antimicrobials or secondary in ection
404 Medica l a nd Surgica l Complica tions

62–22. A 35-year-old G4P3 at 30 weeks’ gestation presents 62–27. Second-generation antihistamines are considered
complaining o a “lump” on her tongue. It is not better than irst-generation antihistamines or which
pain ul but does bleed easily. A photograph is o the ollowing reasons?
S
provided. What is the most likely diagnosis? a. They are less sedating.
E
C
b. They are much less expensive.
T
I
O
c. They are pregnancy category A.
N
d. They are pregnancy category C.
1
2
62–28. Which o the ollowing is a second-generation
antihistamine?
a. Loratadine
b. Diphenhydramine
c. Chlorpheniramine
d. None o the above

62–29. Regarding the use o high-potency topical


corticosteroids, which o the ollowing statements is
true?
a. Syphilis
a. They should only be used or 2 to 4 days.
b. Condyloma
b. They are best reserved or re ractory disorders.
c. Neuro ibromatosis
c. They should be used over extensive areas or best
d. Pyogenic granuloma e icacy.
62–23. Which o the ollowing statements is true regarding d. They are the pre erred agent or initial treatment
o dermatological disorders.
the condition in Question 62–22?
a. It is contagious. 62–30. Which o the ollowing increases systemic absorption
b. It is slow growing. o topical corticosteroids?
c. It does not typically bleed. a. Occlusive dressings
d. It occurs in response to low-grade local irritation b. Prolonged treatment duration
or trauma. c. Compromised epidermal barrier
62–24. Which o the ollowing conditions is characterized d. All o the above
by acial pustules and coalescing draining sinuses?
62–31. Coverage or which o the ollowing is most
a. Erythema nodosum important when treating skin disorders complicated
b. Rosacea ulminans by bacterial in ections?
c. Neuro ibromatosis a. Anaerobes
d. Pyogenic granuloma b. Gram-positive organisms
c. Gram-negative organisms
62–25. What is the initial choice o treatment or
hidradenitis suppurativa? d. All o the above
a. Coal tar
62–32. Which o the ollowing therapeutic agents should be
b. Oral luconazole avoiding during pregnancy?
c. Oral antimicrobials a. Desonide
d. Oral corticosteroids b. Cetirizine
62–26. Which o the ollowing is true regarding the c. Methotrexate
e ect o pregnancy on the size and number o d. Triamcinolone acetonide
neuro ibromatosis lesions?
a. Pregnancy has no e ect.
b. Lesions may increase in size and number.
c. Lesions become smaller but more numerous.
d. Lesions become smaller and less numerous.
Derma tologica l Disorders 405

CHAPTER 62 ANSw ER KEy

Q uestion Letter Pa ge

C
number a nswer cited Hea der cited

H
A
P
62–1 c p. 1214 Pregna ncy-Specific Derma toses

T
E
62–2 c p. 1214 Intra hepa tic Cholesta sis of Pregna ncy

R
6
62–3 c p. 1215 Ta ble 6 2 -1

2
62–4 d p. 1214 Pemphigoid G esta tionis
62–5 d p. 1214 Pemphigoid G esta tionis
62–6 d p. 1214 Pemphigoid G esta tionis
62–7 d p. 1214 Pemphigoid G esta tionis
62–8 d p. 1214 Pemphigoid G esta tionis
62–9 d p. 1216 Pruritic Urtica ria l Pa pules a nd Pla ques of Pregna ncy (PUPPP)
62–10 c p. 1216 Pruritic Urtica ria l Pa pules a nd Pla ques of Pregna ncy (PUPPP)
62–11 c p. 1216 Pruritic Urtica ria l Pa pules a nd Pla ques of Pregna ncy (PUPPP)
62–12 a p. 1216 Pruritic Urtica ria l Pa pules a nd Pla ques of Pregna ncy (PUPPP)
62–13 c p. 1216 Atopic Eruption of Pregna ncy (AEP)
62–14 d p. 1216 Atopic Eruption of Pregna ncy (AEP)
62–15 d p. 1216 Derma tologica l Conditions N ot Specific to Pregna ncy
62–16 c p. 1216 Derma tologica l Conditions N ot Specific to Pregna ncy
62–17 c p. 1216 Atopic Eruption of Pregna ncy (AEP)
62–18 b p. 1216 Derma tologica l Conditions N ot Specific to Pregna ncy
62–19 c p. 1216 Derma tologica l Conditions N ot Specific To Pregna ncy
62–20 a p. 1216 Derma tologica l Conditions N ot Specific to Pregna ncy
62–21 d p. 1216 Derma tologica l Conditions N ot Specific to Pregna ncy
62–22 d p. 1216 Derma tologica l Conditions N ot Specific to Pregna ncy
62–23 d p. 1216 Derma tologica l Conditions N ot Specific to Pregna ncy
62–24 b p. 1216 Derma tologica l Conditions N ot Specific to Pregna ncy
62–25 c p. 1216 Derma tologica l Conditions N ot Specific to Pregna ncy
62–26 b p. 1216 Derma tologica l Conditions N ot Specific to Pregna ncy
62–27 a p. 1217 Derma tologica l Trea tment
62–28 a p. 1217 Derma tologica l Trea tment
62–29 b p. 1217 Derma tologica l Trea tment
62–30 d p. 1217 Derma tologica l Trea tment
62–31 b p. 1217 Derma tologica l Trea tment
62–32 c p. 1217 Derma tologica l Trea tment
406

CHAPTER 63

Neoplastic Disorders

63–1. Which o the ollowing is the most common benign 63–7. An ovarian cancer survivor presents or
neoplasm in pregnancy? preconceptional counseling. She has had prior pelvic
a. Ovarian cyst irradiation. With this history, she is at increased risk
or all EXCEPT which o the ollowing obstetrical
b. Pyogenic granuloma
complications?
c. Endocervical polyp
a. Stillbirth
d. Breast ibroadenoma
b. Preterm birth
63–2. What are the most common cancers in pregnancy? c. Fetal birthweight < 2500 g
a. Breast, thyroid, cervix d. Fetal congenital mal ormations
b. Breast, cervix, lymphoma
63–8. Published case reports describe rare instances o
c. Breast, thyroid, lymphoma maternal tumor metastasizing to the etus. With
d. hyroid, cervix, melanoma which o the ollowing cancer types does this most
requently occur?
63–3. O the ollowing imaging modalities, which is the a. Melanoma
sa est during pregnancy?
b. Leukemia
a. Sonography
c. Lymphoma
b. Computed tomography
d. Breast cancer
c. Diagnostic radiography
d. Magnetic resonance imaging 63–9. An asymptomatic 19-year-old G1P0 presents or
prenatal care. Her routine obstetrical care should
63–4. When radiation therapy is needed during pregnancy, include which o the ollowing elements?
which o the ollowing potential etal risks should a. Pelvic examination alone
the patient be in ormed o ?
b. Pelvic examination and Pap smear
a. Microcephaly
c. Pelvic examination, Pap smear, and human
b. Mental retardation papillomavirus testing
c. Growth restriction d. Pelvic examination, Pap smear, and human
d. All o the above papillomavirus vaccination

63–5. Embryonic exposure to cytotoxic drugs may cause 63–10. For at-risk women, cervical cytological screening
major congenital mal ormations in what percentage is recommended more requently than the routine
o cases? screening guideline schedule. Which o the ollowing
a. 2% patients should have more requent cervical cancer
screening?
b. 20%
a. 33-year-old with history o diabetes
c. 33%
b. 19-year-old with three li etime sexual partners
d. 50%
c. 22-year-old with in utero diethylstilbestrol
63–6. Why is chemotherapy occasionally withheld in the exposure
last 3 weeks prior to delivery? d. 26-year-old with prior human papillomavirus
a. o allow the patient to breast eed vaccination
b. o decrease the likelihood o maternal neutropenia
c. o decrease the chance o etal-growth restriction
d. o decrease the overall risk o late mutagenic e ects
N eopla stic Disorders 407

63–11. Which o the ollowing serovars are considered high- 63–17. A 29-year-old G2P1 patient at 22 weeks’ gestation
risk human papillomavirus serotypes? is re erred to your clinic or colposcopy due to
a. 6, 11 a Pap smear report noting high-grade squamous

C
intraepithelial lesion (HSIL). What can you tell her
b. 6, 16

H
regarding colposcopy in pregnancy?

A
c. 11, 18

P
a. It is best delayed to the puerperium.

T
d. 16, 18

E
b. It is more likely to be unsatis actory.

R
6
63–12. A 40-year-old pregnant patient calls to discuss her c. She will need a repeat study at 26 weeks’ gestation.

3
Pap smear results. Her Pap smear shows atypical d. It is indicated or the evaluation o HSIL lesions.
squamous cells o undetermined signi icance, and
human papillomavirus testing is negative. Which 63–18. he cervical biopsy rom the patient in Question
o the ollowing is your recommendation or 63–17 shows invasive squamous cell carcinoma.
subsequent evaluation? Which o the ollowing is a part o ormal cervical
a. Colposcopy 6 weeks postpartum staging in pregnancy?
b. Colposcopy in the third trimester a. Chest radiograph
c. Repeat cytology 6 weeks postpartum b. Renal sonography
d. Repeat cytology in the third trimester c. Abdominopelvic computed tomography
d. None o the above
63–13. For a cervical intraepithelial neoplasia (CIN) 1 lesion
diagnosed in early pregnancy, what is the chance 63–19. I microinvasive disease is diagnosed rom a cervical
that repeat cytology at the end o the puerperium conization specimen obtained in the early second
will be normal? trimester, which o the ollowing statements is true
a. 20–30% regarding pregnancy management?
b. 40–50% a. She may deliver vaginally.
c. 60–70% b. She will need cesarean delivery.
d. 80–90% c. She will require adjuvant radiotherapy in the
puerperium.
63–14. Cervical conization during pregnancy is associated d. She will need radical hysterectomy at the time o
with an increased risk o all EXCEPT which o the delivery.
ollowing?
a. Preterm delivery 63–20. All EXCEPT which o the ollowing are reasons
that surgical treatment is pre erred to radiotherapy
b. Membrane rupture
in the treatment o early-stage pregnancy-associated
c. Residual neoplasia invasive cervical cancer?
d. Inadequate diagnostic tissue sample a. Radiation therapy destroys ovarian unction.
b. Radiation therapy can cause intestinal injury.
63–15. What is the incidence o invasive cervical cancer in
the general obstetrical population? c. Complication rates are higher with radiotherapy.
a. 1 in 2500 d. Surgical treatment is more e ective than
radiotherapy.
b. 1 in 5500
c. 1 in 8500 63–21. A 32-year-old nulligravida requests ertility-sparing
d. 1 in 11,500 radical trachelectomy or her small Stage IB1 cervical
cancer. How should you counsel her regarding this
63–16. What percentage o cervical cancers are treatment option and uture pregnancy outcomes?
adenocarcinomas? a. Her risk o premature delivery is decreased.
a. 5% b. She will need cesarean delivery in a uture
b. 20% pregnancy.
c. 33% c. Her ertility remains the same as the general
d. 50% obstetrical population.
d. Her chance o a live birth would be the same as
the general obstetrical population.
408 Medica l a nd Surgica l Complica tions

63–22. You elect to per orm transvaginal sonography 63–24. What is the recommended standard o care or the
or your patient with a uterus that is larger than treatment o endometrial carcinoma?
expected based on her gestational age. Her sonogram a. Curettage with progestational treatment
S
is shown here. Regarding leiomyomas in pregnancy,
b. Curettage without progestational treatment
E
C
which o the ollowing statements is true?
T
c. otal abdominal hysterectomy and bilateral
I
O
salpingoophorectomy
N
d. All o the above
1
2
63–25. A 17-year-old gravida is re erred or evaluation o
an adnexal mass ound at 9 weeks’ gestation during
sonography or irst-trimester bleeding. A sonogram
o her right ovary is shown, and dimensions o
the mass are marked by calipers. Based on its
appearance, what is the most likely diagnosis?

a. hey require sonographic surveillance.


b. heir incidence in pregnancy approximates 2%.
c. hey grow in pregnancy due to human chorionic
gonadotropin stimulation.
d. he vaginal delivery rate is only 30% i
leiomyomas are larger than 10 cm.

63–23. Prior to conception, uterine artery embolization


(UAE) was recommended or your patient with
symptomatic leiomyomas. Rates o which o the a. Serous cystadenoma
ollowing are increased in pregnancies ollowing
b. Corpus luteum cyst
UAE?
c. Mature cystic teratoma
a. Miscarriage
d. Ovarian hyperstimulation syndrome
b. Cesarean delivery
c. Postpartum hemorrhage
d. All o the above
N eopla stic Disorders 409

63–26. A pregnant patient at 11 weeks’ gestation complains 63–27. A patient at 9 weeks’ gestation is treated or
o intermittent le t lower quadrant pain and nausea a ruptured and excised corpus luteum cyst o
and vomiting. She undergoes diagnostic exploratory pregnancy. Which o the ollowing regimens is

C
laparotomy. Which o the ollowing is true regarding recommended?

H
management, in general, o patients with this a. Micronized progesterone, 200 mg orally daily or

A
P
diagnosis? 3 weeks

T
E
b. 17-hydroxyprogesterone caproate, 150 mg

R
6
intramuscularly × 1 dose

3
c. 8% progesterone vaginal gel, one applicator
Fimbria
vaginally daily or 1 week
d. 17-hydroxyprogesterone caproate, 150 mg
intramuscularly weekly or 3 weeks

63–28. Which o the ollowing is a rare, solid benign


ovarian tumor that can virilize a emale etus?
a. Luteoma
b. Brenner tumor
Ova ry c. Mature cystic teratoma
d. Hyperreactio
Dis te ndeluteinalis
d
A B fa llopia n tube
63–29. A pregnant patient presents with bilateral adnexal
masses. She is pregnant with twins. Both masses are
similar to the image shown below. What is the most
likely diagnosis?
Fimbria

Ova ry

Dis te nde d
B fa llopia n tube

Reproduced with permission rom Brooks Heinzman A, Hof man BL: Pelvic mass. In
Hof man BL, Schorge JO, Schaf er JI, et al (eds): Williams Gynecology, 2nd ed. New
York, McGraw-Hill, 2012, Figure 9-22.

a. Excision o the adnexa is always required. a. Luteoma


b. All patients require progesterone replacement. b. Granulosa cell tumor
c. Oophoropexy could prevent a recurrence o this c. Mature cystic teratoma
complication. d. Hyperreactio luteinalis
d. All o the above
410 Medica l a nd Surgica l Complica tions

63–30. Which o these pregnancy-associated adnexal 63–36. All EXCEPT which o the ollowing may be
masses can lead to hypovolemia with ascites, renal indicated in the management o thyroid cancer in
dys unction, and adult respiratory distress syndrome? pregnancy?
S
a. Luteoma a. Radioiodine
E
C
b. Corpus luteum b. Delay surgery until the second trimester
T
I
O
c. Granulosa cell tumor c. Expectant management with thyroxine treatment
N
d. Hyperstimulated ovary d. Surgical treatment with thyroxine replacement
1
2
63–31. What percentage o ovarian cancer ound in 63–37. Reed-Sternberg cells, such as the one shown here, are
pregnancy is early-stage? consistent with which o the ollowing lymphoid cell
a. 25% malignancies?
b. 50%
c. 75%
d. 90%

63–32. All EXCEPT which o the ollowing are considered


risk actors or breast cancer?
a. Nulliparity
b. Advancing age
c. Breast eeding
d. BRCA1 and BRCA2 gene mutation

63–33. A pregnant patient at 26 weeks’ gestation has a


palpable breast mass and undergoes mammography
and needle biopsy o the identi ied lesion. Although Reproduced with permission rom Gascoyne RD, Skinnider BF: Pathology o malig-
nant lymphomas. In Lichtman MA, Kipps J, Seligsohn U (eds): Williams Hematology,
clinical examination and biopsy results suggest 8th ed. New York, McGraw-Hill, 2010, Figure 98-34.
a benign mass, mammogram indings suggest
malignancy. What should be the next management
a. Hodgkin lymphoma
step?
b. Hairy cell leukemia
a. Chemotherapy
c. Acute myeloid leukemia
b. Mass excision
d. cell derived lymphoma
c. Repeat mammography
d. Repeat needle biopsy 63–38. What is the system used or staging Hodgkin and
other lymphomas called?
63–34. Your evaluation o the patient in Question 63–33
a. Clark
con irms breast cancer in pregnancy. For women
with this diagnosis, all EXCEPT which o the b. Breslow
ollowing treatments are associated with improved c. Bethesda
patient survival rates? d. Ann Arbor
a. Chemotherapy
b. otal mastectomy 63–39. What standard single-agent chemotherapy regimen
is recommended or treatment o Hodgkin disease
c. Modi ied mastectomy
during the irst trimester?
d. Pregnancy termination
a. Bleomycin
63–35. he patient in Question 63–33 is now 28 weeks b. Vinblastine
pregnant. Her axillary nodes were positive or cancer, c. Dacarbadine
and chemotherapy is indicated. All EXCEPT which d. Doxorubicin
o the ollowing agents are acceptable in pregnancy?
a. rastuzumab
b. Doxorubicin
c. 5-Fluorouracil
d. Cyclophosphamide
N eopla stic Disorders 411

63–40. Which virus is associated with Burkitt lymphoma? 63–42. A patient presents at 14 weeks’ gestation with the
a. Hepatitis C skin lesion shown below and palpable lymph nodes.
Which o the ollowing dermatological neoplasms is
b. Herpesvirus

C
this is consistent with?

H
c. Epstein-Barr virus

A
P
d. Human immunode iciency virus

T
E
R
63–41. Which o the ollowing statements regarding

6
leukemia in pregnancy is alse?

3
a. Leukemias may arise rom bone marrow.
b. Remission is common during pregnancy.
c. ermination o pregnancy will improve
prognosis.
d. Leukemias are more common in patients older
than 40 years.

Reproduced with permission by Hardin MJ: Cutaneous conditions. In Knoop KJ, Stack
LB, Storrow AB, et al (eds): T e Atlas o Emergency Medicine, 3rd ed. New York,
McGraw-Hill, 2010, Figure 13-80.

a. Stage I melanoma
b. Stage II melanoma
c. Stage I basal cell carcinoma
d. Stage II squamous cell carcinoma
412 Medica l a nd Surgica l Complica tions

CHAPTER 63 ANSw ER KEy

Q uestion Letter Pa ge
S
number a nswer cited Hea der cited
E
C
T
63–1 a p. 1219 Introduction
I
O
63–2 a p. 1219 Introduction
N
1
63–3 a p. 1219 Dia gnostic Ima ging
2
63–4 d p. 1220 Ra dia tion Thera py
63–5 b p. 1220 Chemothera py
63–6 b p. 1220 Chemothera py
63–7 d p. 1220 Fertility a nd Pregna ncy a fter Ca ncer Thera py
63–8 a p. 1220 Pla centa l Meta sta ses
63–9 a p. 1221 Cervix: Epithelia l N eopla sia
63–10 c p. 1221 Cervix: Epithelia l N eopla sia
63–11 d p. 1222 O ncogenic Huma n Pa pilloma viruses
63–12 c p. 1222 Ta ble 6 3 -1
63–13 c p. 1223 Cervica l Intra epithelia l N eopla sia
63–14 d p. 1223 Cervica l Coniza tion
63–15 c p. 1223 Inva sive Cervica l Ca ncer
63–16 b p. 1223 Inva sive Cervica l Ca ncer
63–17 d p. 1222 Ta ble 6 3 -1
63–18 d p. 1223 Inva sive Cervica l Ca ncer
63–19 a p. 1223 Ma na gement a nd Prognosis
63–20 d p. 1223 Ma na gement a nd Prognosis
63–21 b p. 1224 Pregna ncy a fter Ra dica l Tra chelectomy
63–22 b p. 1224 Leiomyoma s
63–23 d p. 1226 Fertility Considera tions
63–24 c p. 1226 Endometria l Ca rcinoma
63–25 c p. 1227 Figure 6 3 -6
63–26 c p. 1227 Complica tions
63–27 b p. 1227 Complica tions
63–28 a p. 1228 Pregna ncy Luteoma
63–29 d p. 1228 Hyperrea ctio Luteina lis
63–30 d p. 1228 O va ria n Hyperstimula tion Syndrome
63–31 c p. 1228 O va ria n Ca ncer
63–32 c p. 1229 Risk Fa ctors
63–33 b p. 1229 Dia gnosis
63–34 d p. 1230 Ma na gement
63–35 a p. 1230 Ma na gement
63–36 a p. 1231 Thyroid Ca ncer
63–37 a p. 1231 Hodgkin Disea se
63–38 d p. 1231 Sta ging a nd Trea tment
63–39 b p. 1231 Hodgkin Disea se
63–40 c p. 1232 N on-Hodgkin Lymphoma
63–41 c p. 1232 Leukemia s
63–42 b p. 1233 Ma ligna nt Mela noma
413

CHAPTER 64
00

Infectious Diseases

64–1. Which o the ollowing is the primary 64–4. A 26-year-old G1P0 at 32 weeks’ gestation is seen in
immunological etal response to in ection? your o ice or ever, myalgias, and a rash o 2 days’
a. IgA duration. She has widely dispersed vesicular lesions
that are pruritic. A picture o her ace is provided
b. IgE
here. Mortality rom this in ection is predominately
c. IgG due to which o the ollowing?
d. IgM

64–2. What percentage o women who have not previously


had varicella will become in ected a ter exposure?
a. 1–2%
b. 5–10%
c. 20–30%
d. 60–95%

64–3. Which o the ollowing clinical intervals best


describes the contagious period or varicella?
a. From rash appearance until lesion crusting
b. From varicella exposure until lesion appearance
c. From 1 day be ore rash onset until lesion crusting Reproduced with permission rom Wolf K, Johnson RA (eds): Viral in ections o skin and
mucosa. In Fitzpatrick’s Color Atlas and Synopsis o Clinical Dermatology, 6th ed. New
d. From rash appearance until complete lesion York, McGraw-Hill, 2009, Figure 27-38.
resolution
a. Pneumonia
b. Meningitis
c. Liver ailure
d. Renal ailure
414 Medica l a nd Surgica l Complica tions

64–5. Which o the ollowing statements regarding herpes 64–7. An 18-year-old G1P0 at 36 weeks’ gestation presents
zoster, shown here, is true? in active labor and appears to have active varicella.
Despite all e orts, the in ant delivers vaginally
S
shortly therea ter. You are very concerned or which
E
C
o the ollowing reasons?
T
a. Maternal antibody has not yet ormed.
I
O
N
b. Mortality rates or the newborn approach 30%.
1
c. The in ant may develop disseminated visceral and
2
central nervous system disease.
d. All o the above

64–8. A 19-year-old G2P0 at 37 weeks’ gestation presents


or her routine prenatal care appointment. While
in the waiting room, she has a conversation with
another patient who is coughing and scratching at a
vesicular rash. The 19-year-old denies a prior history
o chickenpox, and you are now concerned that she
has been exposed to the varicella-zoster virus (VZV).
Used with permission rom Dr. Mary Jane Pearson. What should be done or her now?
a. O er varicella vaccination with Varivax
a. It is painless.
b. De er VZV serology testing and proceed with
b. It is contagious i blisters are broken. varicella zoster immune globulin (VariZIG)
c. It is responsible or many congenital administration
mal ormations. c. Test or VZV serology and administer VariZIG i
d. It is more requent and more severe in pregnant results are negative or antibodies to VZV
women. d. Test or VZV serology and give VariZIG and
Varivax i results are negative or antibodies to VZV
64–6. One mani estation o congenital varicella syndrome
is demonstrated in the photograph below. At which 64–9. Which o the ollowing statements regarding the
o the ollowing gestational ages did in ection most varicella vaccine, Varivax, is alse?
likely occur?
a. Seroconversion is 98%.
b. It is not secreted into breast milk.
c. It is recommended in pregnancy to all women
with no history o varicella.
d. At 10 years postvaccination, the breakthrough
in ection rate approximates 5%.

64–10. During which o the ollowing periods should


in luenza vaccination be avoided?
a. First trimester
b. Postpartum, i breast eeding
c. During pregnancy and or 1 month be ore
becoming pregnant
d. None o the above

Reproduced with permission rom Cunningham FG, Leveno KJ, Bloom SL, et al (eds):
In ectious diseases. In Williams Obstetrics, 24th ed. New York, McGraw-Hill, 2014,
Figure 64-1.

a. < 6 weeks
b. 6–10 weeks
c. 13–20 weeks
d. 28–32 weeks
Infectious Disea ses 415

64–11. A 22-year-old G1P0 at 32 weeks’ gestation presents 64–12. A 31-year-old G1P0 at 8 weeks’ gestation presents
with ever, coryza, conjunctivitis, cough, and rash. complaining o a ever and rash o 5 day’s duration.
The patient was raised in a separatist group that does The rash started on her ace and has spread to her

C
not believe in immunizations. You suspect measles, trunk and extremities. A photograph o her chest

H
and a photograph o her ace is provided here. is provided here. Given her gestational age, which

A
P
Which o the ollowing counseling points is true? o the ollowing in ections would be the most

T
concerning or this patient and her etus?

E
R
6
4
Reproduced with permission rom Ryan KJ, Ray CG (eds): Mumps virus, measles, rubella,
Reproduced with permission rom Wolf K, Johnson RA (eds): Severe and li e-threatening and other childhood exanthems. In Sherris Medical Microbiology, 5th ed. New York,
skin eruptions in the acutely ill patient. In Fitzpatrick’s Color Atlas and Synopsis o Clini- McGraw-Hill, 2010, Figure 10-6.
cal Dermatology, 6th ed. New York, McGraw-Hill, 2009, Figure 8-5.

a. Rubeola
a. Treatment is supportive.
b. Rubella
b. The virus is not teratogenic.
c. Varicella
c. There is an increased incidence o preterm
delivery and low birthweight with maternal d. Toxoplasmosis
in ection.
64–13. I the patient in Question 64–12 has acute rubella,
d. All o the above you would expect serologic testing to most likely
show which o the ollowing results?
a. IgM positive, IgG negative
b. IgM negative, IgG negative
c. IgM negative; IgG positive, high a inity
d. IgM positive; IgG positive, high a inity

64–14. With maternal rubella in ection at 8 weeks’


gestation, the risk o congenital in ection most
closely approximates which o the ollowing?
a. 1%
b. 10%
c. 25%
d. up to 90%
416 Medica l a nd Surgica l Complica tions

64–15. The most common single de ect associated with 64–18. For the patient in Question 64–17, you obtain and
congenital rubella is which o the ollowing? send serological testing or parvovirus B19. Which
a. Glaucoma o the ollowing testing outcomes would satis y you
S
that she is immune and needs no urther evaluation?
b. Microcephaly
E
C
a. IgG positive, IgM negative
T
c. Sensorineural dea ness
I
b. IgG positive, IgM positive
O
d. Pulmonary artery stenosis
N
c. IgG negative, IgM negative
1
2
64–16. Hand oot and mouth disease is caused by which o d. IgG negative, IgM positive
the ollowing viruses?
a. Togavirus 64–19. The serological test results or the patient in
Question 64–17 indicate that she has had a recent
b. Parvovirus
parvovirus B19 in ection. Your management plan at
c. Coxsackievirus this point should be which o the ollowing?
d. None o the above a. O er pregnancy termination
b. Begin serial measurement o maternal viral loads
64–17. A 20-year-old G1P0 at 18 weeks’ gestation presents
every 2 weeks or 10 weeks
to your o ice or prenatal care. The patient just
started working at a day-care center and is concerned c. Begin serial etal sonographic evaluation every
about acquiring in ections rom the children. She 2 weeks or the irst 10 weeks a ter in ection
reports several colds since starting the job. She d. Per orm sonographic etal evaluation now to
wasn’t eeling well last week and noted headache, identi y hydrops and, i the scan results are
nausea, and pharyngitis. Three weeks ago, one normal, resume prenatal care with no urther
child in particular looked rather ill. She shows you sonographic surveillance
a picture o the child on her cell phone, and the
photograph is provided here. Your concern or this 64–20. Four weeks a ter irst presenting or care, the etus
patient is in ection with which o the ollowing? o the patient in Question 64–17 is now hydropic
and shows elevated middle cerebral artery peak
systolic velocities. You counsel her regarding etal
blood sampling and etal trans usion. Which o the
ollowing is a true statement?
a. Most etuses require multiple trans usions.
b. The mortality rate without trans usion is 90%.
c. The overall mortality rate ollowing trans usion is
50%.
d. Reports o the neurodevelopmental outcomes o
etuses trans used or parvovirus-induced anemia
are con licting.

64–21. Which o the ollowing statements regarding


cytomegalovirus (CMV) is true?
a. Pregnancy increases the severity o maternal
CMV in ection.
b. It is the most common perinatal in ection in the
developed world.
c. Most women who contract CMV will have
symptoms that will alert the provider.
d. Once a woman has been in ected with CMV, she
is immune and will never become ill again rom
a. Parvovirus CMV.
b. Rubeola virus
c. Rubella virus
d. Coxsackievirus
Infectious Disea ses 417

64–22. Which o the ollowing is not a risk actor 64–27. A 19-year-old G1P0 at 29 weeks’ gestation presents
or intrapartum group B streptococcus (GBS) to your o ice complaining o lulike symptoms and
transmission? a target-shaped rash on her neck, which is shown in

C
a. Membranes ruptured ≤ 18 hours this photograph. She has just returned rom a hiking

H
trip in New Hampshire. Her history and rash are

A
b. Delivery at < 37 weeks’ gestation

P
concerning or which o the ollowing?

T
c. Intrapartum temperature ≥ 38.0o Celsius

E
R
d. Prior in ant with invasive early-onset GBS disease

6
4
64–23. Which o the ollowing antibiotics is never
an appropriate choice or penicillin-allergic
patients requiring group B streptococcus (GBS)
chemoprophylaxis?
a. Ce azolin
b. Vancomycin
c. Clindamycin
d. Erythromycin

64–24. Which o the ollowing is irst-line inpatient therapy


or methicillin-resistant Staphylococcus aureus Reproduced with permission rom Levinson W: Spirochetes. In Review o Medical Micro-
in ection? biology and Immunology, 12th ed. New York, McGraw-Hill, 2012, Figure 24-6.

a. Ri ampin
a. Lyme disease
b. Vancomycin
b. Toxoplasmosis
c. Clindamycin
c. West Nile virus in ection
d. Trimethoprim-sul amethoxazole
d. Methicillin-resistant Staphylococcus aureus
64–25. A 25-year-old G3P2 at 34 weeks’ gestation presents in ection
with decreased etal movement, ever, myalgias,
and headache or 4 days. Her etus is dead. The 64–28. Prompt treatment o the patient in Question 64–27
patient’s cervix is 3-cm dilated, and with rupture o should prevent most adverse pregnancy outcomes.
membranes, dark-brown luid is seen. Postpartum, Assuming she has no allergies, which o the
the patient is diagnosed with listeriosis. Which o the ollowing is the pre erred treatment?
ollowing is pre erred treatment or her? a. Spiramycin
a. Vancomycin b. Vancomycin
b. Ce triaxone c. Amoxicillin
c. Erythromycin d. Doxycycline
d. Ampicillin and gentamicin
64–29. Which o the ollowing statements regarding
64–26. Which o the ollowing is not a typical sign or toxoplasmosis in ection in pregnancy is true?
symptom o salmonellosis? a. The risk o etal in ection increases with
a. Fever gestational age.
b. Abdominal pain b. The risk o etal in ection decreases with
gestational age.
c. Bloody diarrhea
c. The severity o etal in ection is much greater in
d. Nausea/vomiting late pregnancy.
d. The risk and severity o etal in ection are not
dependent on gestational age.
418 Medica l a nd Surgica l Complica tions

64–30. Which o the ollowing etal outcomes is not 64–32. Which o the ollowing statements regarding
associated with toxoplasmosis in ection during prenatal screening or toxoplasmosis is alse?
pregnancy? a. Screening is recommended in areas o low
S
a. Hydrocephalus prevalence.
E
C
b. Chorioretinitis b. The parasite is rarely detected in tissue or body
T
I
luids.
O
c. Placental microabscesses
N
d. Intracranial calci ications c. IgM antibodies alone should not be used to
1
detect acute toxoplasmosis.
2
64–31. A di erential diagnosis or the indings seen in this d. The presence o high-avidity IgG indicates that
etal sonographic image would include all EXCEPT in ection did not occur in the preceding 3 to 5
which o the ollowing? months.

64–33. For a pregnant woman with chloroquine-resistant


malaria, the current recommended treatment is
which o the ollowing?
a. Me loquine
b. Atovaquone
c. Quinine and erythromycin
d. Doxycycline and primaquine

64–34. Which o the ollowing statements regarding West


Nile virus is true?
a. Treatment is with acyclovir.
b. The incubation period is 3 to 4 weeks.
c. O those who are in ected, 25% develop
meningoencephalitis or acute laccid paralysis.
d. Diagnosis is by detection o viral IgG and IgM in
serum and IgM in cerebrospinal luid.

64–35. I a pregnant woman develops anthrax and has


li e-threatening allergies to both cipro loxacin and
penicillin, which o the ollowing is a recommended
Reproduced with permission rom Cunningham FG, Leveno KJ, Bloom SL, et al (eds):
alternative?
In ectious diseases. In Williams Obstetrics, 24th ed. New York, McGraw-Hill, 2014,
Figure 64-1. a. Ri ampin
b. Vancomycin
a. Rubella
c. Doxycycline
b. Toxoplasmosis
d. Trimethoprim-sul amethoxazole
c. Parvovirus in ection
d. Cytomegalovirus in ection
Infectious Disea ses 419

CHAPTER 64 ANSw ER KEy

Q uestion Letter Pa ge

C
number a nswer cited Hea der cited

H
A
P
64–1 d p. 1239 Feta l a nd N ewborn Immunology

T
E
64–2 d p. 1240 Va ricella -Zoster Virus

R
6
64–3 c p. 1240 Va ricella -Zoster Virus

4
64–4 a p. 1240 Ma terna l Infection
64–5 b p. 1240 Ma terna l Infection
64–6 c p. 1240 Feta l a nd N eona ta l Infection
64–7 d p. 1240 Feta l a nd N eona ta l Infection
64–8 c p. 1241 Ma terna l Vira l Exposure
64–9 c p. 1241 Va ccina tion
64–10 d p. 1241 Va ccina tion
64–11 d p. 1242 Rubeola (Mea sles)
64–12 b p. 1243 Rubella —G erma n Mea sles
64–13 a p. 1243 Rubella —G erma n Mea sles
64–14 d p. 1243 Feta l Effects
64–15 c p. 1243 Feta l Effects
64–16 c p. 1244 Coxsa ckievirus
64–17 a p. 1244 Pa rvovirus
64–18 a p. 1245 Dia gnosis a nd Ma na gement
64–19 c p. 1245 Dia gnosis a nd Ma na gement
64–20 d p. 1245 Dia gnosis a nd Ma na gement
64–21 b p. 1245 Cytomega lovirus
64–22 a p. 1249 Risk-Ba sed Prevention
64–23 d p. 1250 Intra pa rtum Antimicrobia l Prophyla xis
64–24 b p. 1252 Ma na gement
64–25 d p. 1253 Ma terna l a nd Feta l Infection
64–26 c p. 1253 Sa lmonellosis
64–27 a p. 1254 Lyme Disea se
64–28 c p. 1254 Trea tment a nd Prevention
64–29 a p. 1255 Ma terna l a nd Feta l Infection
64–30 c p. 1255 Ma terna l a nd Feta l Infection
64–31 c p. 1255 Ma terna l a nd Feta l Infection
64–32 a p. 1255 Screening a nd Dia gnosis
64–33 a p. 1256 Ma na gement
64–34 d p. 1257 W est N ile Virus
64–35 c p. 1258 Anthra x
420

CHAPTER 65

Sexuall Transmitted Infections

65–1. Which of the following does not increase the risk of 65–4. The best test to perform for the patient in Question
transmission of syphilis? 65–3 to ascertain a definitive diagnosis is which of
a. Cervical inversion the following?
b. Cervical hyperemia a. Rapid plasma reagin (RPR)
c. Cervical friability b. Bacterial culture of lesion exudate
d. Abrasions of the vaginal mucosa c. Dark-field examination of lesion exudate
d. Serum assay for herpes simplex virus 1 and
65–2. The incubation period of syphilis is which of the 2 antibodies
following?
a. 1–7 days 65–5. The lesions in this photograph are most indicative of
which stage of syphilis?
b. 10 days
c. 3–90 days
d. 120–180 days

65–3. A 21-year-old G4P2 at 17 weeks’ gestation presents


for her first prenatal care visit. She has a history
of prostitution, but she denies engaging in such
activities for the past month. During examination, a
painless lesion is noted on the right labia. A picture
of the lesion is provided below. The most likely
diagnosis is which of the following?

Used with permission from Dr. Jonathan Willms.

a. Primary
b. Secondary
c. Late latent
d. Early latent

65–6. Findings of secondary syphilis include all EXCEPT


which of the following?
a. Fever
b. Macular rash
Reproduced with permission from Cunningham FG, Leveno KJ, Bloom SL, et al (eds):
Sexually transmitted infections. In Williams Obstetrics, 24th ed. New York, McGraw-
c. Condylomata lata
Hill, 2014, Figure 65-1. d. Strawberry cervix
a. Chancroid
b. Primary syphilis
c. Bartholin gland duct abscess
d. Herpes simplex virus infection
Sexua lly Tra nsmitted Infections 421

65–7. A 40-year-old G9P8 at 10 weeks’ gestation is 65–12. In women with gonorrhea, what percent also have
found to have a rapid plasma reagin (RPR) of 1:4. chlamydial infection?
Confirmatory testing is positive. She has no signs a. 1%

C
or symptoms of syphilis. The patient’s RPR was
b. 40%

H
nonreactive in all of her prior pregnancies, and she

A
c. 75%

P
had a nonreactive RPR 6 months ago. Her stage is

T
which of the following? d. 100%

E
R
a. Primary

6
65–13. Risk factors for gonorrhea that should prompt

5
b. Secondary
screening in pregnancy include all EXCEPT which
c. Late latent of the following?
d. Early latent a. Drug use
b. Age > 25 years
65–8. A 20-year-old G1P0 at 10 weeks’ gestation presents
to your office for a rash of her palms and soles. She c. New sexual partner
saw an informational poster on syphilis, and she is d. Prior gonococcal infection
concerned that she has it. Her RPR titer is 1:64,
and confirmatory testing is positive. The appropriate 65–14. The current recommendation for the treatment of
treatment is which of the following? uncomplicated gonococcal infection in pregnancy is
a. Benzathine penicillin G, 2.4 million units which of the following?
intramuscularly, weekly for two doses a. Azithromycin 1 g orally as a single dose
b. Benzathine penicillin G, 2.4 million units b. Ceftriaxone 250 mg intramuscularly as a single
intramuscularly, weekly for three doses dose
c. Benzathine penicillin G, 2.4 million units c. Erythromycin ethylsuccinate 400 mg orally four
intramuscularly, weekly for four doses times daily for 14 days
d. Aqueous procaine penicillin, 2.4 million units d. Ceftriaxone 250 mg intramuscularly as a single
intramuscularly, plus probenecid 500 mg orally dose plus azithromycin 1 g orally as a single dose
four times daily, both for 10 days
65–15. A 15-year-old G1P0 presents at 18 weeks’ gestation
65–9. The patient in Question 65–8 does not come to any complaining of pain of her wrists, the tops of her
of her subsequent appointments. Now at 36 weeks’ hands, ankles, and the tops of her feet. She has never
gestation, she represents, you repeat her RPR, and it experienced anything like this previously. She has no
is 1:128. The most likely reason for this is which of medical problems and eight lifetime sexual partners.
the following? A nucleic acid amplification test is positive for
a. Laboratory error gonorrhea, and you suspect disseminated infection.
The treatment for this is which of the following?
b. Successful treatment
a. Azithromycin 1 g orally as a single dose
c. Reinfection or treatment failure
b. Ceftriaxone 250 mg intramuscularly as a single
d. Coexistent systemic lupus erythematosus dose
65–10. A pregnant woman with syphilis and a true c. Ceftriaxone 250 mg intramuscularly as a single
penicillin allergy should be treated with which of the dose plus azithromycin 1 g orally as a single dose
following? d. Ceftriaxone 1000 mg intravenously every
a. Erythromycin 24 hours continued for 24 to 48 hours after
improvement followed by oral therapy to
b. Azithromycin complete a week of treatment
c. Doxycycline, after delivery
d. Benzathine penicillin G, after desensitization 65–16. With 1.4 million cases reported in 2011, which
of the following is the most commonly reported
65–11. All EXCEPT which of the following are infectious disease in the United States?
manifestations of a Jarisch-Herxheimer reaction in a a. Chancroid
pregnant woman treated for syphilis? b. Gonorrhea
a. Contractions c. Genital herpes
b. Night sweats d. Chlamydia trachomatis infection
c. Decreased fetal movement
d. Late fetal heart rate decelerations
422 Medica l a nd Surgica l Complica tions

65–17. The most commonly identifiable infectious cause of 65–22. A 19-year-old G2P0 at 18 weeks’ gestation presents
ophthalmia neonatorum is which of the following? to the emergency room complaining of painful
a. Haemophilus ducreyi lesions of the perineum. She reports fever at home.
S
A photograph of the patient’s perineum is provided
b. Chlamydia trachomatis
E
C
below. The best next step in the management of this
T
c. Trichomonas vaginalis patient is which of the following?
I
O
d. Neisseria gonorrhoeae
N
1
2
65–18. First-line treatment for a chlamydial infection in
pregnancy is which of the following?
a. Azithromycin 1000 mg orally as a single dose
b. Ofloxacin 300 mg orally twice a day for 7 days
c. Doxycycline 100 mg orally twice a day for 7 days
d. Erythromycin base 250 mg orally four times a
day for 14 days

65–19. What percentage of pregnant women acquire


herpes simplex virus-1 (HSV-1) or HSV-2 during
pregnancy?
a. 0–0.1
b. 0.5–2
c. 5–7.5
d. 10–15
Reproduced with permission from Cunningham FG, Leveno KJ, Bloom SL, et al (eds):
65–20. The most common route of neonatal HSV Sexually transmitted infections. In Williams Obstetrics, 24th ed. New York, McGraw-
Hill, 2014, Figure 65-5.
transmission is which of the following?
a. Postnatal
a. Obtain lesion exudate for viral culture
b. Peripartum
b. Send human immunodeficiency virus (HIV)
c. Intrauterine testing
d. Preconceptional c. Diagnose patient with HSV by clinical
presentation alone
65–21. A patient who has HSV-2 isolated from genital
d. Send spinal fluid for HSV polymerase chain
secretions in the absence of HSV-1 or HSV-2
reaction (PCR) assay
antibodies is classified as which of the following?
a. Transient infection 65–23. At 36 weeks’ gestation, you elect to provide the
b. Reactivation disease patient in Question 65–22 acyclovir suppression.
c. First episode primary infection You do this for all EXCEPT which of the following
reasons?
d. First episode nonprimary infection
a. It will decrease viral shedding.
b. It reduces the chances of the newborn acquiring
herpes to zero.
c. It will decrease the chance of her having an
outbreak at term.
d. It will decrease the chances of her requiring a
cesarean delivery because of an outbreak.
Sexua lly Tra nsmitted Infections 423

65–24. A 27-year-old G2P1 at 39 weeks’ gestation presents 65–26. A 40-year-old G3P1 at 18 weeks’ gestation comes
in active labor. The patient is known to have genital to see you for a second opinion. A photograph of
herpes, so a thorough exam is performed for lesions. her perineum is provided below. She would like

C
No lesions are seen on the perineum or cervix, but treatment, but her current doctor refuses because he

H
a suspicious lesion is seen on her lower thigh. It feels the lesions will decrease in size after pregnancy.

A
P
is tender to the touch and suggestive of a herpetic Which of the following treatments would be

T
lesion. Your plan of management is which of the acceptable for use in pregnancy?

E
R
following?

6
5
a. Imiquimod
a. Cesarean delivery b. Podophyllin resin
b. Acyclovir and tocolysis c. Interferon therapy
c. Occlusive dressing over the lesion and vaginal d. Trichloracetic acid
delivery
d. No change in management from any other 65–27. You see a 20-year-old G1P0 at 10 weeks’ gestation
patient in labor preparing for a vaginal delivery in your office for Gardasil exposure. The patient
reports that she received the first dose 1 month ago
65–25. After delivery of the patient in Question 65–24, she and before she knew she was pregnant. Your plan of
asks you if she can breast feed her infant given that care is which of the following?
she has active HSV. You counsel her which of the a. Continue the series as any damage has already
following? been done
a. She should not breast feed. b. Delay the remaining doses until the patient has
b. She can breast feed, but only if she is not taking delivered
acyclovir. c. Delay the remaining doses until the patient has
c. She can breast feed while taking acyclovir but not delivered and completed her breast feeding
valacyclovir. d. Recommend termination of pregnancy because
d. She can breast feed, provided she has no breast of the strong likelihood of a significant fetal
lesions and practices strict hand washing. anomaly

65–28. Risk factors for bacterial vaginosis include all


EXCEPT which of the following?
a. Smoking
b. Douching
c. Vitamin D deficiency
d. Advanced maternal age
424 Medica l a nd Surgica l Complica tions

65–29. A 15-year-old G1P0 at 15 weeks’ presents to the 65–31. Which of the following statements about the
emergency room complaining of foul-smelling, gray, treatment of HIV in pregnancy is true?
watery discharge. She also has pruritus. The results a. Treatment is recommended for all HIV-infected
S
of the wet mount are provided. The diagnosis is pregnant women.
E
C
which of the following?
b. All medications used for the treatment of HIV
T
I
are safe in pregnancy.
O
N
c. Treatment is only required if the HIV-infected
1
woman would qualify for treatment when not
2
pregnant.
d. Zidovudine must be added to whatever regimen
the woman is already taking, even if her viral load
is adequately suppressed.

65–32. Which of the following statements regarding


intrapartum management of HIV is true?
a. If cesarean delivery is planned, it should be
scheduled at 36 weeks’ gestation.
b. Cesarean delivery is recommended for women
with a viral load > 1000 copies/mL.
Used with permission from Dr. Lauri Campagna and Rebecca Winn, WHNP. c. In labor, with a plan for vaginal delivery,
amniotomy should be performed as soon as
a. Gonorrhea possible to hasten delivery.
b. Candidiasis d. In labor, internal monitors should be placed
because fetuses of HIV-infected women are at
c. Trichomoniasis
increased risk for distress.
d. Bacterial vaginosis

65–30. When is transmission of human immunodeficiency


virus (HIV) from a woman to her baby most likely
to occur?
a. Intrapartum
b. Before 36 weeks’ gestation
c. In the days before delivery
d. Transmission occurs equally throughout gestation
Sexua lly Tra nsmitted Infections 425

CHAPTER 65 ANSw ER KEy

Q uestion Letter Pa ge

C
number a nswer cited Hea der cited

H
A
P
65–1 a p. 1265 Pa thogenesis a nd Tra nsmission

T
E
65–2 c p. 1265 Pa thogenesis a nd Tra nsmission

R
6
65–3 b p. 1266 Figure 6 5 -1

5
65–4 c p. 1267 Dia gnosis
65–5 b p. 1266 Clinica l Ma nifesta tions
65–6 d p. 1266 Clinica l Ma nifesta tions
65–7 d p. 1266 Clinica l Ma nifesta tions
65–8 a p. 1268 Trea tment
65–9 c p. 1268 Trea tment
65–10 d p. 1268 Penicillin Allergy
65–11 b p. 1268 Penicillin Allergy
65–12 b p. 1269 G onorrhea
65–13 b p. 1269 G onorrhea
65–14 d p. 1269 Screening a nd Trea tment
65–15 d p. 1270 Dissemina ted G onococca l Infections
65–16 d p. 1270 Chla mydia l Infections
65–17 b p. 1270 Chla mydia l Infections
65–18 a p. 1270 Screening a nd Trea tment
65–19 b p. 1271 Herpes Simplex Virus (HSV)
65–20 b p. 1271 Pa thogenesis a nd Tra nsmission
65–21 c p. 1271 Clinica l Ma nifesta tions
65–22 a p. 1273 Dia gnosis
65–23 b p. 1273 Peripa rtum Shedding Prophyla xis
65–24 c p. 1273 Peripa rtum Shedding Prophyla xis
65–25 d p. 1273 Peripa rtum Shedding Prophyla xis
65–26 d p. 1275 Trea tment
65–27 b p. 1275 Immuniza tion
65–28 d p. 1276 Ba cteria l Va ginosis
65–29 c p. 1276 Trichomonia sis
65–30 c p. 1278 Ma terna l a nd Perina ta l Tra nsmission
65–31 a p. 1279 Antiretrovira l Thera py
65–32 b p. 1281 Prena ta l HIV Tra nsmission
This page intentionally left blank
In d ex

Note: Page numbers ollowed by f represent gures.

A Amnionic uid embolism, 276f Bicornuate uterus


Abdominal pain, 246 Amnionic uid index (AFI), 73 vs. septate uterus, 17
Abdominal pregnancy, 124 borderline, 73 Bilateral oophorectomy, 34
treatment, 125 measurements, 69 Biliary disorders, 361–364
Abdominal wall, 8 Amnionic sheet/band, 38 Bilobate placenta, 36, 36f
Abnormal labor, 150–158 Amnion nodosum, 38 Biophysical pro les, 106, 107
ABO incompatibility, 93 Amniotomy, 177, 292 Bipolar disorder, 397. See also Psychological
Abortion, 112–117 Analgesia, 166–172 disorders
complete, 113 Anal sphincter, 9 Birthing tub, 153
de ned, 112 external, 10 Birth rate, 4
elective, 116 Androgenesis, 129 Birthweight, 294
rst-trimester, 112 Anemia, 89, 93, 296, 366, 367 distribution, 373, 373f
incomplete, 114 Anesthesia, 166–172, 186 neonate, 281
menstrual age and, 116 or breech delivery, 192 placental peptide hormones and, 34
missed, 114 epidural, 170 Bishop score system, 174
recurrent, 114 general, 171 Bladder, 24
second-trimester, 116 regional, 166, 168, 169 caudad separation o , 203
septic, 114 spread o , 170 Bladder exstrophy, 15
threatened, 112 Aneuploidy, 64 Bladder outlet obstruction, 102
Abortus, 3 Angiotensin II, 23 Blastocyst, 32
Achalasia, 357 Anterior cervical lip, 243 Blindness, 231
Achondroplasia, 53 Anthrax, 418 Blood trans usion, 94
Acid-base analysis, 220 Antibiotic prophylaxis, 180, 203 Blood vessels, 65
Acid-base equilibrium, 23 Antibiotics, 76, 246, 284 Body mass index (BMI), 320, 320f, 321, 322.
Acidemia, 223 Antibodies, 92 See also Obesity
Acoustic stimulation testing, 108 Anticonvulsants, 76, 272 Bowel obstruction, 358
Acrocentric chromosome, 82 Anti-D antibodies, 92, 93 Brachial plexus injuries, 182, 226
Acute bilirubin encephalopathy, 224 Anti-D immunoglobulin, 94, 95, 113 Braxton Hicks contractions, 20, 283
Acute peripartum ischemia, 223 Antiphospholipid antibody syndrome, 296, 345 Breast cancer, 256, 410
Addison disease, 381 Antiseizure medications, 50 Breast eeding, 221, 242
Adiana method, 264 Antral ollicles, 30 bene ts o , 241
Adiponectin, 321 Anxiety disorders, 397 contraception in, 244
Adolescent pregnancy, 52 Apgar score, 219, 223 contraindications to, 242
Adrenocorticotropic hormone (AC H) Appendectomy, 308 Breast milk, 241
secretion, 25, 25f Appendicitis, 359 Breast pain, 242
Air travel, in pregnancy, 59 Arrhythmias, 328 Breathing, 105, 105f
Aldosterone, 26 Arteriovenous mal ormations, 391 Breech delivery, 188–193. See also Forceps
Alloimmunization, red blood cells, 92 Asphyxia, 162 delivery; Vaginal delivery
Alobar holoprosencephaly, 64, 82, 82f Assisted reproductive technologies, 53 anesthesia or, 192
Alpha etoprotein (AFP), 87 Asthma, 337, 338 and bone racture in neonates, 189
Alveoli, 229 Attention de cit hyperactivity disorders, 224 cardinal movements o , 189
Ambiguous genitalia, 45, 45f Augmentation, o labor, 173–178 external cephalic version, 192
Amenorrhea, 255 induction vs., 173 etal suitability or, 189
Amino acid, 294 peripartum complications, 174 internal podalic version, 192
Amniocentesis, 83, 90 Autism, 224 maternal morbidity and mortality in, 188
Amnioin usion, 163, 223, 291 Autoimmunity gene, 383 pelvic adequacy or, 189
Amnion, 33 Axillary mass, 242f preterm, 188
Amnionic uid, 43, 66, 68–74 risk actor or, 188
absence/diminution o , 68 B vs. cesarean delivery, 188
absorption o , 218 Bacterial vaginosis, 423 Bupivacaine, 168
categorization, 70 Bandl ring, 139 Burkitt lymphoma, 411
evaluation in multi etal gestations, 69, 69f Bariatric surgery, 323 Butorphanol, 167
normal volume, 68 Bell palsy, 393
resorption, 68 β-hCG, 123, 131, 133, 254 C
sonographic evaluation, 68 β-thalassemia, 368 Cancers, neoplastic diseases and, 406
source or, 68 Beta thalassemia, 54 Carbohydrate metabolism, 372

427
428 Index

Carbon dioxide, trans er across placenta, 42 Cesarean scar pregnancy, 126 myometrial, 140, 141
Carbon monoxide poisoning, 342 Charcot-Bouchard aneurysms, 391 pattern, 164
Cardiac arrest, 318 Chemotherapeutic agents, 134 pelvic outlet, 153
Cardiac rhabdomyomas, 65 Chest radiograph, 131 uterine, 163
Cardiovascular disease, 325–330 Chickenpox, 414 Cornuectomy, 124
antepartum/intrapartum care, 325 Childhood obesity, 322 Coronal suture, 44
arrhythmias, 328 Chlamydia conjunctivitis, 220 Corpus luteum, 30, 122
dyspnea, 328 Chlamydia trachomatis, 317 Corticosteroids, 76, 232, 271, 284
Eisenmenger syndrome, 328 Choanal atresia, 63 Corticotropin-releasing hormone (CRH), 141, 281
heart ailure, 328 Cholecystectomy, 308 Counseling, preconceptual, 50–55
in ective endocarditis, 328 Cholelithiasis, 308 C-peptide levels, 294
Mar an syndrome, 325, 329 Cholestasis, 400 Cranial nerve VII injury, 226
mitral stenosis, 326 Chorioamnionitis, 38, 152, 164, 284 CRH. See Corticotropin-releasing hormone (CRH)
myocardial in arction, 329 Chorioangiomas, 38 Critical care, 314–319
systemic lupus erythematosus, 327 Choriocarcinoma, 134 Crohn disease, 358
valve replacement, 326 Chorion rondosum, 32 Crowning, 180f
Cardiovascular system, etal, 43 Chorionic villus sampling (CVS), 90 Cushing syndrome, 381
Catheters, 176, 244 Chromosomal abnormalities, 72, 88 Cystic brosis, 89, 341, 342
Caudal regression sequence, 64 Chromosomal microarray analysis, 5, 52, 84 Cystic brosis conductance transmembrane
Cavum septum pellucidum, 63 Chronic hypertension, 331–336 regulator (CF R), 89
Central nervous system, 26 antihypertensive medications, 334 Cystotomy, 208
de ects, 79 comorbidities, 332 Cytogenetic analysis, 236
Cephalopelvic disproportion, 150 complication, 333f Cytomegalovirus (CMV), 416
Cerclage, 115 diagnosis, 332
Cerebral artery embolism, 390 preeclampsia, 333, 334–335 D
Cerebral edema, 270 prehypertension, 331 D-antigen erythrocytes, 92
treatment o , 271 risk o stillbirth, 333 Death
Cerebral lesion, 231 treatment, 331, 332 etal (See Fetal death)
Cerebral palsy, 223 Classical (vertical) hysterotomy, 207 in ants, 4, 51, 90
types o , 232 Clindamycin, 247 maternal (See Maternal death)
Cervical cancer, 406, 407 Cloacal exstrophy, 15 in postterm pregnancy, 288
Cervical cerclage, 18 Common iliac artery, 11, 11f, 12 pregnancy-related, 4
Cervical dilation, 151 Complete abortion, 113 Deciduas, 31, 31f
Cervical eversion, 20, 20f Computed tomography (C ), 248f, 310 Decomposition, 192
Cervical incompetence, 115 Condoms, 258 Deep-vein thromboses, 344, 346
Cervical intraepithelial neoplasia (CIN), 407 Congenital adrenal hyperplasia, 100 Delivery
Cervical lacerations, 173 Congenital cystic adenomatoid mal ormation, 100 blood loss at, 274f
Cervical length, 62 Congenital diaphragmatic hernia, 65, 102 breech, 188–193
Cervical mucus, pattern o , 21, 21f Congenital heart block, 100 cesarean (See Cesarean delivery)
Cervical pregnancy, 125 Congenital heart disease, 2 operative vaginal, 194–201
Cervical ripening, 138, 174 Connective tissue disorders, 383–388 placenta, 185f, 206
prostaglandins or, 175, 176 antiphospholipid antibody syndrome, 386 vaginal, 180–187
Cervix, 9, 138, 139 autoantibodies, 384 Delivery date, 57
position o , 148 autoimmunity gene, 383 calculation, device or, 57f
un avorable, 290 immune-mediated, 383 Depot medroxyprogesterone acetate (DMPA),
Cesarean delivery, 139, 154, 168, 181, 202–209, in ammatory arteritis, 387 258
322, 323. See also Vaginal birth a ter lupus, 384–386 Depression, 396, 397. See also Psychological
cesarean (VBAC) rheumatoid arthritis, 387 disorders
breech delivery vs., 188 systemic lupus erythematosus, 384–386 Dermatological disorders, 400–405
complications, 18, 210 uterine rupture, 387 eczema, 402
criteria, 203 Consanguinity, 84 erythema nodosum, 403
de ned, 202 Continuous etal monitoring, 162 hidradenitis suppurativa, 404
with dinoprostone, 175 Contraception, 118, 132, 253–259 intrahepatic cholestasis, 400
election o , 196 in breast eeding, 244 pemphigoid gestationis, 400, 401
elective, 202 depot medroxyprogesterone acetate, 258 pruritic urticarial papules and plaques o
and etal injury, 203 emergency, 258 pregnancy (PUPPP), 401–402
uid sequestration, 208 ailure rate, 253 pruritus, 400
hysterotomy incision during, 205 hormonal, 256, 257 pustular psoriasis, 403
increase in, 5, 5f, 202 implants, 256 Dermatological neoplasms, 411
induction o labor and, 173 intrauterine device, 253–255, 256 Dexamethasone, 100, 271
need or, 195 long-acting reversible, 253 Diabetes, 52
and neonatal morbidity, 210 method o , 257 preconceptual counseling, 50
preoperative preparation or, 203 oral, 256, 257 in pregnancy, 295
primary, 202, 210 transdermal patch, 257 Diabetes mellitus, 371–376
prior, 210–216 vaginal ring, 258 birthweight distribution, 373, 373f
puerperal pelvic in ections ollowing, 247 Contracted pelvic inlet, 153 carbohydrate metabolism, 372
rate o , 202, 211 Contraction-associated proteins (CAPs), 138 diabetic ketoacidosis, 374
repeat, 211 Contractions, 147, 175, 235 asting plasma glucose level, 372
trends, 215f Braxton Hicks, 20, 283 gestational, 371, 374–375
vaginal delivery vs., 180 orces during labor, 151 hydramnios, 373
Index 429

hyperbilirubinemia, 373 pheochromocytoma, 381 tracing, 108


hyperinsulinemia, 372 postpartum thyroiditis, 380 variability, 160
hypoglycemia, 374 prolactin levels, 381 Fetal hemoglobin F, 43
incidence o , 371, 371f thyroid peroxidase ( PO) antibodies, 378 Fetal imaging, 62–67
macrosomia, 375 thyroid-stimulating hormone ( SH), 378 Fetal lung, maturity, 230–231
maternal mortality rate, 373 thyrotoxicosis, 378, 379 Fetal mal ormation, 63
met ormin therapy, 375 thyroxine, 378 Fetal movements, 56, 104
preeclampsia, 373 Endometrial carcinoma, 408 Fetal pelvis, 15
prevalence o , 371 Endometrial layer, shedding o , 30 Fetal skeletal mineralization, 25
rate during pregnancy, 371 Endometrium, 9, 9f Fetal surveillance, 104
renal disease and, 353 Epidural analgesia, 213 Doppler velocimetry, 107
respiratory distress syndrome, 373 Epidural anesthesia, 152, 170, 176 Fetal swallowing, 44
stillbirth, 373 intravenous meperidine vs., 170 Fetal therapy, 98–103
type 1, 374 Episiotomy, 181, 186, 249, 250 Fetal viability, limits o , 44
Diabetic ketoacidosis, 374 Epispadias, 15 Fetomaternal hemorrhage, 95
Diaphragm, 258 Erythema nodosum, 403 Fetoplacental blood volume, 43
Diastolic blood ow, 33 Essure method, 263–264 Fetotoxic agents, 75–80
Diet, 53 Estrogen, 30, 34 Fetus, 32f, 51
Diethylstilbestrol, 76 Exercise, in pregnancy, 59 anomaly, 234
Dif usion, 42 External cephalic version, 192 blood ow rom placenta to, 33, 33f
Dilatation and curettage (D & C), 115, 131 Extrachorial placentation, 37 blood trans usion or, 94
or complete hydatidi orm mole management, Eye movements, 42 breech, 145
131 breech, vaginal delivery o , 188
Dinoprostone, 174, 175 F cardiovascular system, 43
Domestic violence. See Intimate partner violence Face presentation, 145f, 155f congenital mal ormation in, 71
Doppler ultrasound, 159 risk actor or, 154 delivery o , 138
Doppler velocimetry, 107, 108 Factor V Leiden mutation, 345 evaluation o , 72f
Down syndrome, 82, 88, 89 Fallopian tube, 11, 11f, 261 growth-restricted, 295
at gestational age, 88, 88f Fascial closure, 207 heart block in, 100
Ductus arteriosus, 218 Femoral artery, 8 hemopoiesis in, 43
Dührssen incisions, 191f Ferguson re ex, 139 hydrocephalic, 184
Dyspnea, 3, 3f, 328 Fertility rate, 2 nonviable, 45
Dystocia, 150, 156 Fetal acidemia, 169 oxygenation, 288
shoulder (See Shoulder dystocia) Fetal alcohol syndrome, 75 position, 145, 156f, 180, 189
Fetal anemia, 92, 94 presentations, 144, 154, 155
E Fetal arrhythmia, 98 resuscitate, 163
E1 (prostaglandin), 175 Fetal assessment, 104–110 with spina bi da de ects, 63
E2 (prostaglandin), 116, 174 Fetal blood sampling, 89, 90 vesicoamnionic shunt placement in, 102
Eating disorders, 398 Fetal breathing motions, 105, 105f Fish, intake o , 59
Echogenic etal bowel, 89 Fetal cardiac abnormalities, 51 Flexion points, 199
Eclampsia, 271–272 Fetal cells, 33, 218 Fluorescence in-situ hybridization (FISH), 84, 90
Ectopic pregnancy, 118–127 Fetal death, 109 Fluoroscopic procedures, 310
de ned, 118 cause o , 235, 236 Folate, 50, 58
diagnosis o , 121, 121f, 1124 de ned, 2, 234 Folic acid, 87
heterotopic pregnancy, 118 rate, 2, 4, 102 Follicles, 30
methotrexate therapy during, 123 reporting requirements or, 234 Forceps delivery, 191 , 194. See also Breech
ovarian, 125 Fetal disorders, 92–97 delivery; Vaginal delivery
risk or, 118 Fetal distress, 162 ailed, 195
tubal pregnancy, 118 postterm pregnancy and, 289 etal indications or, 194
types, 125 Fetal echocardiography, 99 maternal morbidity with, 194
unicornuate uterus and, 17 Fetal-growth disorders, 294–299 vacuum extraction, 194, 195
Eczema, 402 chromosomal aneuploidy, 296 Fragile-X syndrome, 84
Eisenmenger syndrome, 328 undal height measurements, 297 Fundal height, 58
Elective abortion, 116 oligohydramnios, 297
Embryo, 42 Fetal head, 82f, 180 G
hemopoiesis in, 43 biparietal diameter o , 12 Gallbladder, 24
Embryogenesis, 42–47 bones o , 44 Gangrene, 316
Embryonic period, 42 circum erence o , 63 Gastrointestinal disorders, 356–360
Encephalopathy, 224 delivery, 140 achalasia, 357
Endocardial cushion de ects, 65 dimensions, 44 appendicitis, 359
Endocarditis, 328 engagement o , 146 bowel obstruction, 358
Endocrine disorders, 378–382 palpation o , 146 Crohn disease, 358
Addison disease, 381 position, 144 Helicobacter pylori, 356
Cushing syndrome, 381 transverse position, 182 hyperemesis gravidarum, 257f, 356–327
hyperaldosteronism, 381 Fetal heart, 65f in ammatory bowel disease, 358
hyperparathyroidism, 380 Fetal heart rate, 106, 106f, 108, 159, 223 nonobstetrical surgery, 356
hyperthyroidism, 379 acceleration, 159 re ux esophagitis, 357
hypocalcemia, 381 deceleration, 160, 161, 164 surgeries, 356
hypothyroidism, 379, 380 monitoring, 163 ulcerative colitis, 359
parathyroid hormone (P H), 380 pattern, 160, 162 vitamin de ciency, 357
430 Index

Gastrointestinal tract, 24 sickle-cell anemia, 367–368 Hyperthyroidism, 101


evaluation, 309 spherocytosis, 367 Hypocalcemia, 381
Gastroschisis, 66 thrombocytopenia, 368, 369 Hypogastric artery, 12
General anesthesia, 171 thrombotic thrombocytopenic purpura Hypoglycemia, 374
Genetics, 81–85 ( P), 369 Hypothyroidism, 101, 379, 380
Genital herpes, 423 von Willebrand disease, 369 Hypovolemia, 279
Gentamicin, 247 Hemoglobin electrophoresis, 54 treatment o , 279
Gestation, 53, 282 Hemoglobinopathies, 89 Hypoxemia, 338
multi etal, 53 Hemolysis, 92 Hypoxia, 295
singleton, 212 Hemophilia, 369 Hypoxic-ischemic encephalopathy, 223
sonographic evaluation, 51 Hemopoiesis, in embryo/ etus, 43 Hysterectomy, 203, 207
survival rate, 42 Hemorrhage, 232, 315 peripartum, 202–209
Gestational age, 231, 234, 236, 287 intracranial, 225, 231 Hysterosalpingography
Down syndrome at, 88, 88f intraventricular, 232, 285 disadvantages o , 16
estimation, 63 postpartum, 185, 240, 275 Hysterotomy, 205, 206
etal growth at, 294f Hemorrhage, obstetrical, 274–280 classical/vertical, 207
and etal mortality rate, 234 coagulation, 279
neonatal mortality rate, 281 midtrimester bleeding, 275 I
predictor o , 62 placental abruption, 277 Ichthyosis, 34
risk o preterm delivery, 230 placenta previa, 277 Idiopathic intracranial hypertension, 394
Gestational diabetes, 371, 374–375. See also rapid blood trans usion, 279 Ilioinguinal nerve, 170f
Diabetes mellitus vital signs, 279 Immunoglobulins, 43
Gestational trophoblastic disease, 128–135. Hepatic disorders, 361–364 Imper orate hymen, 15
See also Molar pregnancy Hepatitis B, 220 Implantation, 30–35
treatment o , 134 Hepatitis B in ection, 363 Incarcerated uterus, symptoms o , 18
Gestational trophoblastic neoplasia, 128 Hepatitis C in ection, 363 Incomplete abortion, 114
development o , 133 Herbal remedies, 79 Indomethacin, 76, 141, 285
diagnosis o , 133 Herpes simplex virus (HSV), 422, 423 Induction, o labor, 173–178, 213, 271
risk or, 132 Herpes zoster, 414 augmentation vs., 173
sign o , 133 Heterotopic pregnancy, 118 and cesarean delivery, 173
staging o , 133 Hidradenitis suppurativa, 404 contraindications to, 173
treatment o , 134 HIV, 424 misoprostol and, 175
Gigantomastia, 21, 21f Hodgkin disease, 410 peripartum complications, 174
Glucose-6-phosphate dehydrogenase de ciency, Home births, 5, 218 In ants. See also Neonates
367 Hormone-eluting devices, 253 with achondroplasia, 53
Glucosuria, 50 Hormones, 34, 256 adult levels o IgM, 43
Gonadal dysgenesis, 45, 45f Human chorionic gonadotropin (hCG), 33, 34, death, 4, 51, 90
Gonadal mosaicism, 83 56, 123, 131, 133, 141 with Down syndrome, 82
Gonorrhea, 421 Human papillomavirus, 407 In ection, 169
Graa an ollicle, 30 Human placental lactogen, 43 intrauterine devices, 255
Graves disease, 310 Human platelet antigen (HPA), 95 and pregnancy-related deaths, 246
Greenstick racture, 227f Hydatidi orm moles, 131 puerperal, 246–252
Group B streptococcus (GBS), 417 characteristic o , 129 uterine, 246
Gunshot, 318 complete, 130 wound, risk actor or, 247
diagnosis o , 132 In ectious diseases, 413–419
H partial, 129 anthrax, 418
Hand oot and mouth disease, 416 treatment or, 129, 131 chickenpox, 414
Headache, 3, 3f, 389 types o , 129 cytomegalovirus (CMV), 416
postdural, 169 Hydramnios, 69, 373 etal response, 413
Health care, or women, 5 etiology o , 71 etal transmission, 415
Heart, partitioning o , 42 idiopathic, 72 group B streptococcus (GBS), 417
Heart ailure, 328. See also Cardiovascular Hydrops etalis, 102, 102f hand oot and mouth disease, 416
disease Hydroxyethyl starch, 272 herpes zoster, 414
Heart rate, 23, 106, 106f 17-Hydroxyprogesterone caproate, 284 listeriosis, 417
Hegar sign, de ned, 12 Hyperaldosteronism, 381 malaria, 418
Helicobacter pylori, 356 Hyperbilirubinemia, 373 measles, 415
HELLP (hemolysis, elevated liver enzymes, low neonatal, 224 rubella, 415, 416
platelet count) syndrome, 271 Hyperemesis gravidarum, 257f, 356–327 salmonellosis, 417
Hemabate, 275 Hyperhomocysteinemia, 345 toxoplasmosis, 417, 418
Hematological disorders, 366–370 Hyperinsulinemia, 22, 22f, 372 varicella, 413, 414
anemia, 366, 367 Hypermagnesemia, 272 varicella-zoster virus (VZV), 414
β -thalassemia, 368 Hyperoxia, 230 In ective endocarditis, 180
olic acid de ciency, 366 Hyperreactio luteinalis, 409 In erior epigastric artery, 8, 8f
glucose-6-phosphate dehydrogenase de ciency, Hypertension, 268, 271 location o , 8
367 Hypertensive disorders, 268–273 In ammatory arteritis, 387
hemophilia A, 369 and maternal death, 268 In ammatory bowel disease, 358
paroxysmal nocturnal hemoglobinuria, preeclampsia, 268–269, 270 In uenza, 340
367 Hyperthermia, 84 In undibulopelvic ligament, 10f, 11
Index 431

Intellectual disability, 224 Late-preterm births, 281 Metritis, 246


Intelligence quotient (IQ) scores, 309 Lead poisoning, 59 Microcytic anemia, 89
Internal podalic version, 192 Leiomyomas, 408 Middle cerebral artery
Internal pudendal artery, 8 Leopold maneuvers, 146 Doppler velocimetry, 108
Internal venous plexus, 169f Leukemia, 411 Midpelvis, 12, 12f
Interval tubal sterilization, 261 Liley curve, 94 Midtrimester bleeding, 275
Intervillous space Lisinopril, 75 Mi epristone, 140
blood, oxygen saturation o , 42 Listeriosis, 417 Migraine, 389
Intimate partner violence, 58, 317 Lithium, 77 Mineralization, etal skeletal, 25
Intracranial hemorrhage, 95, 96, 225, 231 Live birth Miscarriage, 112, 113
Intrahepatic cholestasis, 400 percentage o , 4 rst-trimester, 112
Intrapartum assessment, 159–165 registration, 2 recurrent, 114
Intrapartum etal stimulation tests, 162 Lochia, 240 septate uterus and, 18
Intrauterine devices (IUDs), 253–255, 256 Long-acting reversible contraception (LARC), Misoprostol, 174, 175, 185
timing o insertion, 256 253 Missed abortion, 114
Intrauterine in ection, 255 Lungs, etal Mitral stenosis, 326
Intrauterine pregnancy, 56, 120 maturity, 230–231 Molar pregnancy. See also Gestational
Intraventricular hemorrhage, 232, 285 Lung-to-head ratio, 102 trophoblastic disease
Invasive moles Lupus, 384–386. See also Connective tissue evacuation, 130, 131, 132
characteristics, 133 disorders risk actor or, 129
Iodide, 43 treatment o , 131
Iodine, 25 M Monochorionic twins, 301, 303
Ionizing radiation, 309 Macrophages, 269 Monozygotic twinning, 300, 301
Iron supplementation, 58 Macrosomia, 290, 297–298, 375 Montevideo units, 152
Isochromosome, 83 Magnesium sul ate, 160, 272, 285 Mood disorder, 396. See also Psychological
Isthmus, 12 Magnetic resonance imaging, 124f, 310–311 disorders
Malaria, 418 Motor vehicle accident, 317
J Mal ormation, congenital, 78 Müllerian ducts, 14
Jarisch-Herxheimer reaction, 421 Mar an syndrome, 325, 329. See also anomalies, 15, 16, 17, 17f
Cardiovascular disease Multi actorial inheritance, 84
K Mastitis, 250 Multi etal gestation
Karyotypes, 65, 81, 84, 236 Maternal death, 3, 3f, 4, 4f chorionicity, 301
Kell-sensitized cases, 92 breech delivery and, 188 complications, 303, 304
Ketamine, 171 due to anesthetic complications, 166 death o co-twin, 304
Kielland orceps, 196 hypertensive disorders and, 268 dif erential diagnosis, 302
rate, 4 rst-trimester sonographic image, 301f
L rom uterine rupture, 215 in ant mortality rate, 300
Labetalol, 271 Maternal evaluation, general considerations, maternal adaptation, 302
Labia minora, 8 308–313 monochorionic twins, 301, 303
Labor Maternal oor in arction, 37 monozygotic twinning, 300, 301
abnormal, 150–158 Maternal morbidity, 194 morbidity/mortality, neonatal, 302
active-phase abnormalities, 151 Maternal plasma hypo-osmolality, 21, 21f placental examination, 301, 301f
augmentation o , 173–178 Mayer-Rokitansky-Küster-Hauser (MRKH) presentation, 304
cardinal movements o , 146, 181 syndrome, 16 preterm birth, 304
clinical stages o , 138 McRoberts maneuver, 183f risks, 300, 301
contraction orces during, 151 Mean cell volume (MCV), 43 super ecundation, 300
induction o , 173–178, 213 Measles, 415 twin-reversed-arterial-per usion ( RAP),
latent phase o , 147 Meckel diverticulum, 33 303
in multiparas, 148 Meconium, 163, 221 twin-twin trans usion syndrome, 303
normal, 144–149 Meconium aspiration syndrome (MAS), 163, vaginal trial o labor, 304
in nulliparas, 148, 152 289 Multi etal gestations, 39, 53
obstacle to, 150 Melanoma, 406 amnionic uid evaluation in, 69, 69f
physiology o , 138–143 Membrane stripping, 177, 290 Multiple sclerosis, 391, 392
placental separation, 140 Menstrual cycle Myasthenia gravis, 392, 393
positions during, 152 duration o , 30 Mycophenolate mo etil, 76
precipitous, 153 endometrial layer, shedding o , 30 Myocardial in arction, 329
preparatory division o , 147 ollicular phase o , 30 Myometrial contractions, 140, 141
preterm, 281–286 Menstrual period, 42, 244
prolonged, 156 Mental retardation, 309 N
second-stage, 151, 152, 180 Meperidine, 166 Naegele rule, 42
stage o , 147 intravenous, vs. epidural anesthesia, NAFLD. See Nonalcoholic atty liver disease
third stage o , 184 170 (NAFLD)
trial o , 211 Mesonephric duct, 15 Naloxone, 166
Lacerations, 186, 194 Metabolic syndrome, 321 Nasal hypoplasia, 79
Laminaria, 115 Metanephros, 14 Nasopharyngeal suctioning, 181
Laparoscopic surgery, 311, 311f Metastatic disease, 134 Necrotizing enterocolitis, 231
Laparotomy, 275 Methergine, 185 Neonatal morbidity, cesarean delivery and,
Last menstrual period, 42 Methotrexate, 123, 134 210
432 Index

Neonates, 3, 218–222. See also In ants Nitroglycerin, 272 Perineal body, 9


acidosis, 223 Nonalcoholic atty liver disease (NAFLD), 321 Peripartum hysterectomy, 202–209
bilirubin levels in, 224 Nonimmune hydrops, 96 Periventricular leukomalacia, 232
birthweight, 281 Nonpuerperal tubal sterilization, 261, 262 risks or, 232
bone ractured in, 189 Nonsteroidal antiin ammatory drug (NSAID), Personality disorder, 398
chest compressions in, 219 141 P annenstiel incision, 203–204
circumcision, 221 Nonstress test (NS ), 109, 292 Phenotype, 83
death, 2, 213 Nonviable etus, 45 Phenylketonuria, 52
endotracheal tube placement in, 218 Nulligravida, 57, 119 Pheochromocytoma, 381
racture in, 227 Nulliparous pregnancy, 291, 292 Phlegmon, 248
hemorrhagic disease o , 225 Pituitary gland, 24
injury with shoulder dystocia, 182 O Placenta
intubation o , 218 Obesity, 320–324 abnormalities, 36–41
mortality rate, 281 adiponectin, 321 blood ow to etus, 33, 33f
preterm (See Preterm newborn) bariatric surgery, 323 calcium deposits, 38
respiratory distress syndrome in, 230 body mass index (BMI), 320, 320f, 321, 322 delivery o , 185f, 206
respiratory tract, 218 cesarean delivery, 322, 323 development o , 30–35
resuscitation, 218 childhood, 322 disk, measurement o , 36
screening, 221 insulin resistance, 321 unctions, 32
secondary apnea vs. primary apnea in, 218 maternal, vaginal birth a ter cesarean and, iodide transport across, 43
term, diseases and injuries o , 223–228 213 low-lying, 277
Neoplastic diseases, 406–412 metabolic syndrome, 321 metastases to, 38
benign neoplasm, common, 406 nonalcoholic atty liver disease (NAFLD), 321 pathological examination o , 36
breast cancer, 410 race/ethnicity and, 321 sur ace o , 36f
Burkitt lymphoma, 411 risk actor or stillbirth, 322 thickness, increased, 37
cancers, common, 406 Obstetrical complications, 53, 87 trans er o carbon dioxide, 42
cervical cancer, 406, 407 Occiput posterior position, 146, 182, 198 Placenta accreta, 278f
cervical intraepithelial neoplasia (CIN), 407 Oligohydramnios, 69, 72, 297 management o , 278
dermatological neoplasms, 411 medications, 73 Placenta increta, 278f
endometrial carcinoma, 408 Oocytes, 30 Placental abruption, 38, 277
Hodgkin disease, 410 Oophorectomy, bilateral, 34 Placental peptide hormones, 34
human papillomavirus, 407 Operative vaginal delivery, 194–201 Placental tumor, 38, 38f
hyperreactio luteinalis, 409 criteria, 194 Placental villi, 278
imaging modalities, 406 long-term risks, 195 Placenta previa, 40, 40f, 277
incidence o , 407 maternal indications or, 194 Pneumonia, 339, 340
leiomyomas in pregnancy, 408 and neurodevelopmental disorders, 195 Polycystic kidney disease (PKD), 66
leukemia, 411 Ophthalmia neonatorum, 422 Polycythemia, 224
melanoma, 406 Opiate, 170 Polyhydramnios, 66, 71
ovarian cancer, 406, 410 Ovarian cancer, 406, 410 clinical sign o , 69
Reed-Sternberg cells, 410f Ovarian pregnancy, 125 placental abnormality associated with, 71, 71f
serovars, 407 Ovary, oocytes, 30 sonographic evidence, 63
thyroid cancer, 410 Ovulation, histological sign o , 30 Pomeroy procedure, 261
uterine artery embolization, 408 Oxytocin, 140, 141, 173, 176, 185, 275 Postdural headache, 169
Nephrolithiasis, 352 Postpartum, 248
Nephrotic syndrome, 352–353 P urinary retention, 243
Nerve root, 226 Pancreatic disorders, 361–364 Postpartum hemorrhage, 185, 240, 275
Neural-tube de ect Parathyroid hormone (P H), 380 secondary, 241
open, 86, 87 Parietal peritoneum closure, 207 Postpartum thyroiditis, 380
screening, 57 Parkland method, 261 Postpartum tubal sterilization, 9
Neuroimaging, 224 Paroxysmal nocturnal hemoglobinuria, 367 Postterm pregnancy, 287–293
Neurological disorders, 389–395 Parturition death in, 288
arteriovenous mal ormations, 391 abnormalities o , 142 and etal distress, 289
Bell palsy, 393 cervical unctions/events, 138 etal-placental actors, 287
cerebral artery embolism, 390 phases o , 138, 140, 142 membrane stripping, 290
Charcot-Bouchard aneurysms, 391 Peak systolic velocity, 93 Precipitous labor and delivery, 153
headache, 389 Pelvic racture, 153 Preconceptual counseling, 50–55
idiopathic intracranial hypertension, 394 Pelvic inlet, 12, 153 Preeclampsia, 268–269, 333, 334–335
migraine, 389 Pelvic joint, relaxation o , 11 clinical sign o , 270
multiple sclerosis, 391, 392 Pelvic outlet, 12, 153 diabetes mellitus, 373
myasthenia gravis, 392, 393 Pelvis treatment o , 271
pregnancy-related strokes, 390 bones o , 11 uric acid levels, 270
seizure, 389 C scan o , 248f Pregnancy
spinal cord injuries, 394 etal, 15 abdominal, 124
subarachnoid hemorrhage, 391 puerperal in ection, 246, 247 acid-base equilibrium, 23
Ni edipine, 285 shapes, 198 adolescent, 52
Nipple stimulation, 105 Pemphigoid gestationis, 400, 401 air travel in, 59
Nitabuch layer, 32 Perinatal mortality, 4, 294f alcohol during, usage o , 58
Nitrazine, 147 Perinatal mortality rates (PMR), 287f, 288f aldosterone during, 26
Nitric oxide (NO), 175 Perinatal period, 2 arteriovenous oxygen, 24
Index 433

binge drinking during, 75 Psychological disorders, 396–399 Schizophrenia, 398. See also Psychological
bladder during, 24 anxiety, 397, 398 disorders
blood volume, 22 bipolar disorder, 397 Secondary postpartum hemorrhage, 241
blood volume during, 274 depression, 396, 397 Seizure, 389
calcium metabolism during, 22 eating disorders, 398 Seizure disorder, 50
central nervous system during, 26 mood disorder, 396 Selective serotonin-reuptake inhibitors
coagulation system in, 23 personality disorder, 398 (SSRIs), 77
and diabetes, 52 postpartum psychosis, 397 Sepsis syndrome, 316
diabetes in, 295 schizophrenia, 398 Septate uterus
due date, 57 Psychosocial screening, 57 bicornuate uterus vs., 17
ectopic (See Ectopic pregnancy) Pudendal nerve, 167 and miscarriage, 18
elevated estrogen levels in, 34 Puerperal complications, 246–252 Septic abortion, 114
exercise in, 59 Puerperal sterilization, 260–262, 263 Septic pelvic thrombophlebitis, 249
gallbladder during, 24 Puerperium, 240–245 Septic shock, 316, 351
gastrointestinal tract during, 24 sterilization in (See Puerperal sterilization) Sex chromosomes, 81
immunological unction, 23 Pulmonary disorders, 337–343 abnormality, 83
increase in protein during, 22 asthma, 337, 338 Sexual assault, 317
intrauterine, 56, 120 cystic brosis, 341, 342 Sexually transmitted in ections, 420–424
iodine during, 25 hypoxemia, 338 bacterial vaginosis, 423
iron supplement during, 58 in uenza, 340 genital herpes, 423
lung volume, 24, 24f pneumonia, 339, 340 gonorrhea, 421
maternal nutrition during, 295 poisoning, 342 herpes simplex virus (HSV), 422, 423
medications during, 75 sarcoidosis, 341 HIV, 424
mortality rates related to, 52 tuberculosis, 340 Jarisch-Herxheimer reaction, 421
nulliparous, 291, 292 Pulmonary edema, 314 ophthalmia neonatorum, 422
ovarian, 125 Pulmonary embolism, 347 perineum lesion, 422, 422f
pituitary gland during, 24 Pulmonary lobe, 171 rapid plasma reagin (RPR), 421
postterm, 287–293 PUPPP. See Pruritic urticarial papules and syphilis, 420, 420f, 421
prolonged, 287 plaques o pregnancy (PUPPP) treatment, 423
red cell alloimmunization in, 92 Pustular psoriasis, 403 trichomoniasis, 424
smoking in, 79 Pyelonephritis, 316, 351 Shoulder dystocia, 182, 184
thyroid gland during, 25, 25f Sickle cell anemia, 84, 367–368
trimesters, 42 R Simpson orceps, 196
o unknown location, 120 Racial dif erences, 56 Single-gene disorders, 52
vaccinations in, 59 Radiation exposure, 309 Sinus tachycardia, 99
venous pressure, 23 Rapid plasma reagin (RPR), 421 Skeletal dysplasia, 66
violence during, 54 Recreational drugs, 79 Skull racture, 226
weight gain, 58 Recurrent abortion, 114 Sleep-wake cycles, 104
Preimplantation genetic diagnosis, 90 Recurrent miscarriage, 114 Small- or-gestational age, 281
Premature rupture o the etal membranes Red blood cells, 279 Smoking, 53, 57, 79
(PROM), 282, 284 alloimmunization, 92 Sonography, 62, 63, 98. See also Ultrasound
Prenatal care, 56–60 characteristics, 94 Spherocytosis, 367
Prenatal diagnosis, 86–91 Reed-Sternberg cells, 410f Spinal blockade, 167, 168, 169
Preterm, 2 Re ux esophagitis, 357 Spinal cord injuries, 394
Preterm labor, 281–286 Regional anesthesia, 166, 168 Spiral arteries, 31f
antimicrobials and, 284 contraindications to, 169 remodeling o , 32
li estyle actors, 282 Renal anomaly, 15 vasoconstriction o , 31
rates, in United States, 281 Renal disorders, 350–355 Spiral blood vessels, 33
and respiratory distress syndrome, 284 Renal pelvis dilatation, 66 Staphylococcus aureus, 417. See also In ectious
risk or, 282 Respiratory distress syndrome, 223, 230, 315, diseases
Preterm newborn, 229–233 373 Steatohepatitis, 363
brain abnormalities in, 231 clinical signs o , 229 Stem-cell transplantation, 101
intraventricular hemorrhage in, 232 in neonates, 230 Sterilization, 260–265
with necrotizing enterocolitis, 231 preterm birth and, 284 puerperal, 260–262, 263
respiratory distress syndrome in, 229–230 treatment o , 230 tubal (See ubal sterilization)
Preterm premature rupture o membranes, 153 Respiratory tract, neonates, 218 vasectomy, 264
Primary pulmonary ef usion, 102 Rhabdomyomas, cardiac, 65 Stillbirths, 2, 52, 234–237
Primigravida, 53, 56, 64, 68, 86, 89, 98, 100, Ribavirin, 76 antepartum, risk actors or, 235
139, 174, 175, 218, 270 Right ventricular hypertrophy, 4, 4f cause o , 235
Primipara, 249, 250 Robertsonian translocation, 83 postterm, 290
Progesterone, 140 Rubella, 415, 416 Stress testing, 105
peak production, 30 Subarachnoid hemorrhage, 391
Prostaglandins, 116, 139f, 142, 174, 175, 176 S Subchorionic brin deposition, 37, 37f
Protein, increase in, 22 Sacrococcygeal teratoma, 102f Succenturiate lobe, 36, 36f, 37
Proteinuria, 268 Sacrospinous ligament, 167 Succinylcholine, 168
Pruritic urticarial papules and plaques o Salmonellosis, 417 Super ecundation, 300
pregnancy (PUPPP), 401–402 Salpingostomy, 123 Suppurative mastitis, 250
Pruritus, 400 Sampson artery, 10, 10f Suprapubic pressure, 183f
Pseudogestational sac, 120 Sarcoidosis, 341 Supraventricular tachycardia, 99
434 Index

Sur actants, 230 ubal pregnancy, 118 Uteroovarian ligament, 10f, 11


Symmetrical growth restriction, 295 ubal sterilization, 260 Uterotonics, 131
Syphilis, 420, 420f, 421 Adiana method o , 264 Uterus, 15, 248f
Systemic lupus erythematosus, 327, 384–386 Essure method o , 263–264 atonic, 275
interval, 261 contraction pain, 139
T nonpuerperal, 261, 262 incarcerated, 18
achyarrhythmia, 99 postpartum, 9 smooth muscles, 140
ay-Sachs disease, 89 uberculosis, 340
V
eratogens, 75–80 urner syndrome, 83
Vaccinations, in pregnancy, 59
eratology, 75–80 win-reversed-arterial-per usion ( RAP), 303
Vacuum extraction, 195, 199–200
erbutaline, 141 win-twin-trans usion syndrome ( S), 72,
Vagina
erminal cardiotocogram, 106 102, 303
anomaly, 16, 16f
T ermal injuries, 318
arteries, 9, 9f
T reatened abortion, 112 U
bleeding, 95, 113, 132, 133, 175, 212, 213,
T rombocytopenia, 95, 96, 170, 225, 368, 369 Ulcerative colitis, 359
235, 241, 275
T romboembolic disorders, 344–349 Ultrasound, 62. See also Sonography
Vaginal birth a ter cesarean (VBAC), 202. See also
antiphospholipid syndrome, 345 Doppler, 159
Cesarean delivery
deep-vein thromboses, 344, 346 Umbilical arteries
complications associated with, 211
actor V Leiden mutation, 345 Doppler interrogation, 109
and maternal obesity, 213
homocysteine levels, 345 velocimetry wave orms, 107, 107f
Vaginal delivery, 180–187, 271. See also Breech
hyperhomocysteinemia, 345 Umbilical cord
delivery; Forceps delivery
incidence o , 344 avulsion, 39
blood loss or, 23
pulmonary embolism, 347 blood gas, 163, 220
o breech etus, 188
thrombolytics, 348 blood sample, 220
cesarean delivery vs., 180
thrombophilia, 344 coiling index, 39
complications, 156
T rombolytics, 348 cyst, 39
episiotomy during, 181
T rombophilia, 344 delayed clamping, 181
indicator, 212
T rombotic thrombocytopenic purpura ( P), 369 knot in, 40
nerves injuries during, 157
T yroid cancer, 77, 410 long, 39
spontaneous, 244
T yroid disorders, 101 sectioning o , 39
Vaginal septum, 17
T yroid gland, 25, 25f short, 38
Vaginosis, 423
T yroid peroxidase ( PO) antibodies, 378 stump, 221f
Valve replacement, 326
T yroid-stimulating hormone ( SH), 378 Umbilical vein, 43
Varicella, 413, 414
T yrotoxicosis, 20, 378, 379 Unicornuate uterus, 17
Varicella-zoster virus (VZV), 50, 414
T yroxine, 378 and ectopic pregnancy, 17
Vasa previa, 40
itle V Maternal and Child Health Services Ureter, 10f, 11
Vasectomy, 264
Block Grants, 5 Uric acid
Vestibule, 9
ocolysis, 163 levels, preeclampsia and, 270
Villous trophoblast, 32
opiramate, 76 Urinary retention, 148, 243
Violence, during pregnancy, 54
oxoplasmosis, 417, 418 Urinary tract disorders, 350–355
Vitamin A, 59
ranscervical catheter, 176 Urogenital sinus, 15
Vitamin A-derived compounds, 78
ransvaginal ultrasound, 62, 212 Uterine artery, 10, 10f, 11
Vitamin B6, 59
ransverse etal lie, 144, 146f Doppler velocimetry, 108
Vitamin D, de ciency o , 59
causes o , 156 embolization, 408
Vitamin K, 221, 225
ransverse uterine incision, 204 Uterine atony, 275
Von Willebrand disease, 369
ransverse vaginal septum, 16 Uterine blood ow, 20
VZV. See Varicella-zoster virus (VZV)
RAP. See win-reversed-arterial-per usion Uterine dehiscence, 213
( RAP) Uterine didelphys, 17, 17f W
rauma, 314–319 Uterine undus, 115f Weight gain, 58
richomoniasis, 424 Uterine incision closure, 210, 212 West Nile virus, 418
risomy, 82, 88 Uterine in ection, 246 Withdrawal method, amily planning,
rophoblast, 32, 123 Uterine inversion, 276 258
rophogen, de ned, 75 Uterine rupture, 156, 211, 213, 214, 277, 387 Wound in ection, risk actor or, 247
rue hermaphroditism, 46, 46f incomplete, 213
SH. See T yroid-stimulating hormone ( SH) maternal deaths rom, 215 Z
P. See T rombotic thrombocytopenic purpura and neonatal death, 213 Zavanelli maneuver, 184, 191
( P) Uterine tachysystole, 175 Zygote, 83

You might also like